chemistry question bank for hiighschool

221
(SOLVED PAPER DCE - 2005* 1. How many number of atoms are present in F.C.C. unit cell? (a) 4 (b) 3 (c) 2 (d) 1. 2. The H-spectrum show (a) Heisenberg's uncertainty principle (b) diffraction (c) polarisation (d) presence of quantised energy level. 3. The sublimation energy of I 2 ( s) is 57.3 kJ/mol and the enthalpy of fusion is 15.5 kJ/mol. The enthalpy of vaporisation of I 2 is (a) 41.8 kJ/mol (b) - 4 1 . 8 kJ/mol (c) 72.8 kJ/mol (d) -72.8 kJ/mol. 4. The correct order of pseudohalide, polyhalide and interhalogen are (a) BrV, OCN , IF 5 (b) IF 5 , Brl 2 ", OCN" (c) OCN~, IF 5 , Brlj (d) OCN", Brl 2 ", IF S . 5. When 2-butyne is treated with Pd-BaS0 4 ; the product formed will be (a) c/s-2-butene (b) /ra«s-2-butene (c) 1-butene (d) 2-hydroxy butane. 6. The rate constant for a chemical reaction has units L mol~' s -1 , order of the reaction will be (a) 0 (c) 2 (b) 1 (d) 3. 7. Buna-S is a polymer of (a) butadiene only (b) butadiene and styrene (c) styrene only (d) butadiene and nitryl. 8. Least paramagnetic property is shown by (a) Fe (b) Mn (c) Ni (d) Cu. 9. Maximum enol content is in O (a) ^ o ("» s K H O O o o H 20. [Fe(N0 2 ) 3 Cl 3 ] and [Fe(0 - NO) 3 Cl 3 ] shows (a) linkage isomerism (b) geometrical isomerism (c) optical isomerism (d) none of these. 11. A certain compound gives negative test with ninhydrin and positive test with Benedict's solution. The compound is (a) a protein (b) a monosaccharide (c) a lipid (d) an amino acid. 12. Which of the following gives iodoform test? (a) CH 3 - CH 2 (OH) (b) C 2 H 5 CHO (c) (CH 2 OH) 2 (d) none of these. 13. For equilibrium reaction, value of Gibbs free energy change is (a) > 0 (b) < 0 (c) = 0 (d) * 0. 14. In the reaction, CH 3 -C = C-CH 3 (OX (ii) H 2 0/Zn > CH, - C - C - CH, 3 llll o o (b) 0 2 (d) KMn0 4 . X is (a) HN0 3 (c) 0 3 15. Xis dissolved in water. Maximum boiling point is observed when Jfis .... (0.1 M each) (a) CaS0 4 (b) BaCl 2 (c) NaCl (d) urea. 16. On adding 0.1M solution each of [Ag + ], [Ba + ], [Ca + ] in a Na 2 S0 4 solution, species first precipitated is [K v , BaS0 4 =10"", K, p CaS0 4 = 10" 6 , K,„ Ag 2 S0 4 = lO" 5 ] (a) Ag 2 S0 4 (c) CaS0 4 (b) BaS0 4 (d) all of these. 17. Penetrating power of waves are (a) y > a > 0 (b) y < « > P (c) y > X-rays > a (d) y < a < p. 18. Which of the following statements does not form a part of Bohr's model of hydrogen atom? * based on memory CHEMISTRY TODAY | AUGUST '05 35

Upload: bansalcomics

Post on 20-Nov-2014

1.060 views

Category:

Documents


27 download

TRANSCRIPT

(SOLVED PAPER

DCE - 2005* 1. How many number of atoms are present in F.C.C. unit cell? (a) 4 (b) 3 (c) 2 (d) 1.

2. The H-spectrum show (a) Heisenberg's uncertainty principle (b) diffraction (c) polarisation (d) presence of quantised energy level.

3. The sublimation energy of I2(s) is 57.3 kJ/mol and the enthalpy of fusion is 15.5 kJ/mol. The enthalpy of vaporisation of I2 is (a) 41.8 kJ/mol (b) - 4 1 . 8 kJ/mol (c) 72.8 kJ/mol (d) -72.8 kJ/mol.

4. The correct order of pseudohalide, polyhalide and interhalogen are (a) BrV, OCN , IF5 (b) IF5, Brl2", OCN" (c) OCN~, IF5, Brlj (d) OCN", Brl2", IFS.

5. When 2-butyne is treated with Pd-BaS04 ; the product formed will be (a) c/s-2-butene (b) /ra«s-2-butene (c) 1-butene (d) 2-hydroxy butane.

6. The rate constant for a chemical reaction has units L mol~' s -1 , order of the reaction will be (a) 0 (c) 2

(b) 1 (d) 3.

7. Buna-S is a polymer of (a) butadiene only (b) butadiene and styrene (c) styrene only (d) butadiene and nitryl.

8. Least paramagnetic property is shown by (a) Fe (b) Mn (c) Ni (d) Cu.

9. Maximum enol content is in O

(a) ^

o

("» s K H

O O

o o

H

20. [Fe(N02)3Cl3] and [Fe(0 - NO)3Cl3] shows (a) linkage isomerism (b) geometrical isomerism (c) optical isomerism (d) none of these.

11. A certain compound gives negative test with ninhydrin and positive test with Benedict's solution. The compound is (a) a protein (b) a monosaccharide (c) a lipid (d) an amino acid.

12. Which of the following gives iodoform test? (a) CH3 - CH2(OH) (b) C2H5CHO (c) (CH2OH)2 (d) none of these.

13. For equilibrium reaction, value of Gibbs free energy change is (a) > 0 (b) < 0 (c) = 0 (d) * 0.

14. In the reaction, C H 3 - C = C - C H 3

(OX (ii) H20/Zn

> CH, - C - C - CH, 3 l l l l

o o

(b) 0 2

(d) KMn04 .

X is (a) HN03

(c) 0 3

15. Xis dissolved in water. Maximum boiling point is observed when J f i s .... (0.1 M each) (a) CaS04 (b) BaCl2

(c) NaCl (d) urea.

16. On adding 0.1M solution each of [Ag+], [Ba+], [Ca+] in a Na2S04 solution, species first precipitated is

[Kv, BaS04 = 1 0 " " , K,p CaS04 = 10"6, K,„ Ag2S04 = lO"5]

(a) Ag2S04

(c) CaS04

(b) BaS04

(d) all of these.

17. Penetrating power of waves are (a) y > a > 0 (b) y < « > P (c) y > X-rays > a (d) y < a < p.

18. Which of the following statements does not form a part of Bohr's model of hydrogen atom?

* based on memory

CHEMISTRY TODAY | AUGUST '05 35

(a) energy of the electrons in the orbit is quantised (b) the electron in the orbit nearest the nucleus has

the lowest energy (c) electrons revolve in different orbits around the

nucleus (d) the position and velocity of the electrons in the

orbit cannot be determined simultaneously.

19. Electrons will 1st enter into the set of quantum numbers n = 5, I = 0 or n = 3, I = 2 (a) n = 5, / = 0 (b) both possible (c) n = 3, / = 2 (d) data insufficient.

20. Which of the following is most reactive towards nucleophilic substitution reaction? (a) CH2 = CH - CI (b) C6H5C1 (c) CH3CH = CHC1 (d) C1CH2 - CH = CH2

21. For the reaction 2H2 + 0 2 -> 2HzO, AH = -571 . Bond energy of H - H = 435, 0 = 0 = 498, then calculate the average bond energy of O - H bond using the above data. (a) 484 (b) - 484 (c) 271 (d) -271 .

22. Calculate the volume of H2 gas at NTP obtained by passing 4 amperes through acidified H 2 0 for 30 minutes is (a) 0.0836 L (b) 0.0432 L (c) 0.1672 L ' (d) 0.836 L.

23. Propyne and propene can be distinguished by (a) conc. H 2 S0 4 (b) Br2 in CC14

(c) alk. KMn0 4 (d) AgN0 3 in NH3.

24. IA]<-Lindlar's C H 3 - C = C - C H 3

Na in liq. NH3 catalyst

[A] and [5] are respectively (a) cis, trans-2-butene (b) both trans-2-butene (c) trans-cis-2-butene (d) both cw-2-butene.

25. How many bonds are there in

(a) 14 G, 8 n (c) 9 a , 4 n

(b) 18 a , 8 71 (d) 14 o, 2 t i

26. In an antifluorite structure, cations occupy (a) octahedral voids (b) centre of cube (c) tetrahedral voids (d) corners of cube.

27. Which is false about H 2 0 2 ?

(a) act as both oxidising and reducing agent (b) two OH bonds lies in the same plane (c) pale blue liquid (d) can be oxidised by ozone.

28. Which of the following is non-permissible? (a) n = 4, I = 3, m = 0 (b) n = 4, I = 2, m = 1 (c) n = 4, I = 4, m = 1 (d) n = 4, I = 0, m = 0.

29. Which has minimum osmotic pressure? (a) 200 ml of 2 M NaCl solution (b) 200 ml of 1 M glucose solution (c) 200 ml of 2 M urea solution (d) all have same.

30. Solution A contains 7 g/L of MgCl2 and solution B contains 7 g/L of NaCl. At room temperature, the osmotic pressure of (a) solution A is greater than B (b) both have same osmotic pressure (c) solution B is greater than A (d) can't determine.

31. Partial pressure of 0 2 in the reaction 2Ag20(.v)

(a) KP

(c) 3yfKp

4 A g w + 0 2 ( g ) i s

(b)

(d) 2KP.

32. Pair of species having identical shapes for molecules is (a) CF4, SF4 (b) BF3, PC13

(c) XeF2, C 0 2 (d) PF5, IF5.

33. Strength of acidity is in order

OH OH OH

(II) (a) II > I > III > IV (c) I > IV > III > II

(HI) (IV)

(b) III > IV > I > II (d) IV > III > I > II.

34. Metallic lusture is explained by (a) diffusion of metal ions (b) oscillation of loose electrons (c) excitation of free protons (d) existence of b.c.c. lattice.

35. Which one is true peroxide?

36 CHEMISTRY TODAY | AUGUST'05

(a) N0 2

(c) Ba02

(b) Mn02

•(d) S02.

SOLUTIONS

1. (a) : In face centred cubic structure since it possesses 8 corner atoms and six at the centre of each face, the contribution will be

= 8 x ^ + 6 x ^ = 1 + 3 = 4 atoms per unit cell. 8 2

2. (d) 3. (a) : I2 w -» I2 ; AH] = + 57.3 kJ/mol

I2(J) —> I 2 W ; AH2 = + 15.5 kJ/mol The enthalpy of vaporisation of I2

= AHi - AH2 = 57.3 - 15.5 = + 41.8 kJ/mol. 4. (d) : A few ions, consisting of two or more electronegative atoms of which at least one is nitrogen, that have properties similar to those of halide ions, are called pseudohalide ions. e.g. OCN~. Halide ions often react with molecules of halogens or interhalogen and form polyhalide ions. e.g. Brl2". The halogens, on account of difference in their electronegativities, combine with each-other and form binary covalent compounds of AB„ type which are called interhalogen compounds. A is always bigger atom and B is smaller atom and n may have values 1, 3, 5 and 7 corresponding to oxidation states of halogens, e.g. IF5.

5. (a) : H,C - C = C - CH, P d - B a S % 2-butyne

CH,

H' / / C H j \ H

c/s-2-butene

Selective reduction from ( = ) bond to ( = ) bond which is syn type. 6. (c) : Litre mol-1 time-1 is the unit for second order reactions. 7. (b) : Buna-S rubber is a co-polymer of three moles of butadiene and one mole of styrene.

CH -- CH,

nCII, = CH - C H = CH2 + n [ Q 1,3-butadiene

styrene

• CH2 - CH = CH - CH2 - CH - CH, -

buna-S

8. (d) : 25Mn —» 3ids, 4s2, number of unpaired electrons = 5 26Fe —> 3d6, 4s2, number of unpaired electrons = 4 28Ni —> 3 cP, 4s2, number of unpaired electrons = 2 29Cu —> 3 d w , 4s1, number of unpaired electrons = 1 Paramagnetic character depends upon number of unpaired electrons.

9. (c ) : o o O t H v - O

CH3 — C — CH, — C — CH3

acetylacetone (keto form, 24%)

U T H-i U i ^ - V j i CH3-C = CH-C-CH 3

(enolic form, 76%)

A V

O H - O

If the enolic form besides H-bonding is also stabilised by conjugation (i.e. enolic double bond is in conjugation with a multiple bond or a benzene ring), the enol content further increases.

10. (a) : This type of isomerism occurs in complex compounds which contain ambidentate ligands like N02", SCN", CN", S203

2- and CO. These ligands have two donor atoms but at a time only one atom is directly linked to the central metal atom of the complex, e.g. [Fe(N02)3Cl3] and [Fe(ONO)3Cl3] are linkage isomers as N02" is linked through O. 11. (b) : Since it gives a negative test with ninhydrin, it cannot be a protein or an amino acid. Since it gives a positive test with Benedict's solution, it must be a monosaccharide but not a lipid. 12. (a) : Iodoform test is exhibited by ethyl alcohol, acetaldehyde, acetone, methyl ketones and those alcohols which possess CH3CH(OH) - groups.

OH

CH3CH - H + 41, + 6NaOH ethyl alcohol

C H I 3 + 5NaI iodoform

+ HCOONa + 5H20 13. (c) :At equilibrium,

AH = positive and TAS = positive and AH = TAS.

:. AG = AH - TAS = 0.

14. (c) : 0 3 converts alkynes to 1,2-diketones. 15. (b) : Elevation in boiling point is a colligative property which depends upon the number of

CHEMISTRY TODAY AUGUST '05 37

molecules/ions. Higher the number of ions, greater will be elevation in boiling point.

-5

23. (d)

16. (a) : [S042"] in Ag2S04

.-11

10"

[S042~] in BaS04 =

[S042~] in CaS04 =

1 0 "

0.1 = 10

0.1x0.1

- 1 0

- = 1 0 "

10 - 6

0.1 - = 10 i-5

As the ionic product of BaS04 is less i.e. Ba2+ will be precipitated first. 17. (c) : Absorption increases with increase in wavelength very fastly. y-rays have very higher energy and very short X, i.e. penetration is very higher. It passes through concrete. X-rays will pass through metal sheets, a-rays are just the nuclei of helium atom that can be stopped by even a paper. (3-rays are stopped by Al foils.

Y > A'-ray > (3 > a 18. (d) : (d) is uncertainty principle and not Bohr's postulate. 19. (c): The order of energies and hence that of filling of orbitals can be calculated by (n +1) rule which states as follows: (i) Orbitals are filled in order of increasing (n + I) values. (ii) If two orbitals have the same {n + I) value, the orbital with lower value of n is filled first. Thus, n = 3 will be filled first. 20. (d): Ally I chloride > vinyl chloride > chlorobenzene 21. (a) : A//reaction = 2 * B.E. (H2) + B.E. (02)

- 2 x B.E. (H20) - 571 = 2 x 435 + 498 - 2 x B.E. (H20) - 571 = 870 + 498 - 2 x B.E. (H20) - 2 x B.E. (H20) = -571 - 1368 = - 1939. B.E. (H20) = 1939/2 = 969.5

969.5 B.E. (O - H) = —^— = 484.75

22. (d) : O = 4 x 30 x 60 C 1

H ,0 H, + - O , 2 2 2

2H+ + 1e -> H2 (1 mole) (i.e. 22400 cc at NTP) IF = 2 x 96500 C

4 x 30 x60 C will give H2 22400x4x30x60

2x96500 = 835.64 cc = 0.836 L.

Reaction with Propene Propyne

(a) conc. H2S04 addition addition (b) Br2 in CCl4 decolourises decolourises (c) alk. KMn04 decolourises decolourises

(glycol is (oxalic acid is formed) formed)

(d) AgN03 no reaction 2CH3C = CH in NH3 + AgN03 + 2NH40H

—> 2CH3C = CAg + 2NH4N03 + 2H20

Price : Rs. 495 1 /

Chapte rw ise Index • • •

5 M o d e l Test Papers

Solved Papers - 1994 - 2005

Po in tw ise Tex t for Phys ics ,Chemis t ry and Ma thema t i c s

(Based on latest sy l labus) WcbGr III •Of

B O O K S

Available at ail leading bookshops.

42 CHEMISTRY TODAY | AUGUST '05 38

24. (a) : Alkynes give different products with different reducing agents. With Lindlar's catalyst (i.e. Pd deposited over BaS04

or CaC03 and partially poisoned by lead acetate, sulphur or quinoline) or nickel boride (also called P-2 catalyst), cw-alkenes are formed but with Na in liquid NH3 at 200 K (Birch reduction) /nmy-alkenes are formed.

CH, V / C = c x

ew-butene (A)

.CH, / Lindlar's

catalyst CH, - C = C - CH3

Na in liq. NH 3

CH,

H > c = c x

/ H

C H , <ra /7 i -bu tene

(B)

25. (c) :

9a , 4n

26. (c) : In antifluorite structure, the oxide ions form a face-centered cubic array and the metal ions fill half the tetrahedral holes. This is considered to be the inverse of the fluorite structure, where the cations form the face-centered cubic array and the anions fit into the tetrahedral holes.

27. (b) : The value of dipole moment of H202 is 2.1 D. This suggests that all the four atoms do not lie in the same plane.

o o

°x ° H OO

H-^l KOl48AO~0.^

O o o

o II

Structure of H202

28. (c) : When n = 4, I * 4.

29. (b): Osmotic pressure is a colligative property which depends upon the number of particle. NaCl is ionic compound hence it has high osmotic pressure in comparison to glucose and urea. Concentration of particles is less in 1 M glucose solution

in comparison to 2 M urea solution. So 1 M glucose solution has minimum osmotic pressure.

30. (a) : The osmotic pressure of solution A is greater than B due to greater number of particles. 31. (a) : [Ag2Ow] = [Ag(s)] = 1 Hence Kp = P0y

32. (c) : Both have linear structure. F - X e - F , O = C = 0

33. (b) : Electron withdrawing groups increase the acidity while electron-donating groups decrease the acidity of phenols.

OH OH OH OH

NO,

' 3

34. (b): The presence of mobile electrons or oscillation of loose electrons are responsible for metallic lusture.

35. (c): Ba02 is a true peroxide as it contains peroxide ion, 02

2". For complete solved paper, refer MTG's DCE Guide

^ How many physical chemists does it take to change a light bulb? Only one, but he'll change it three times, plot a straight line through the data, and then extrapolate to zero concentration.

^ How many physical chemists does it take to wash a beaker? None. That 's what organic chemists are for!

> What is the dieter's element? Nobelium

> If a bear in Yosemite, and one in Alaska fall into water, which one would dissolve faster? The bear in Alaska because it is polar.

'P Where does one put the dishes? In the zinc.

> What do you call a tooth in a glass of water? One molar solution.

y Why does hamburger have lower energy than steak? Because it 's in the ground state.

CHEMISTRY TODAY | AUGUST '05 39

SOLVED PAPER

KERALA PET - 2005 1. 100 g CaC03 is treated with 1 litre of 1 N HC1. What would be the weight of C0 2 liberated after the completion of the reaction? (a) 5.5 g (b) 11 g (c) 22 g (d) 33 g (e) 44 g.

2. The relationship between the energy of the radiation with a wavelength 8000 A and the energy E2

of the radiation with a wavelength 16000 A is (a) £ , = 6E2 (b) E} = 2E2

(c) = 4E2

(e) = E2. (d) £, = 1/2 E2

3. If the molecule of HC1 were totally polar, the expected value of dipole moment is 6.12 D (debye), but the experimental value of dipole moment was 1.03 D. Calculate the percentage ionic character. (a) 17 (b) 83 (c) 50 (d) zero (e) 90.

4. Which one of the following molecules has the smallest bond angle? (a) NH3 (b) PH3

(c) H 2 0 (d) H2Se (e) H2S.

5. If the absolute temperature of a gas is doubled and the pressure is reduced to one half, the volume of the gas will be (a) remain unchanged (b) be doubled (c) increase fourfold (d) be halved (e) be reduced to one-fourth.

6. To what temperature must a neon gas sample be heated to double pressure, if the initial volume of gas at 75°C is decreased by 15.0%? (a) 319°C (b) 592°C (c) 128°C (d) 60°C (e) 90°C.

7. When electric current is passed through an ionic hydride in molten state

(a) hydrogen is liberated at anode (b) hydrogen is liberated at cathode (c) no change takes place (d) hydride ion migrates towards cathode (e) hydride ion remains in solution.

8. The order of first ionization energies of the elements Li, Be, B, Na is (a) Li > Be > B > Na (b) Be > B > Li > Na (c) Na > Li > B > Be (d) Be > Li > B > Na (e) B > Be > Li > Na.

9. The froth-floatation process is based upon (a) the difference in the specific gravity of ore and

gangue particles (b) the magnetic properties of gangue and ore (c) preferential wetting of gangue particles by oil (d) preferential wetting of ore particles by oil (e) the solubility of ore particles in suitable reagent.

10. Which one of the following statements is true for all the alkali metals? (a) their nitrates decompose on heating to give N0 2

and 0 2

(b) their carbonates decompose on heating to give C0 2

and the metal oxide (c) they react with oxygen to give mainly the oxide

M 2 0 (d) they react with halogens to give the halides MX (e) they react with nitrogen to give nitrides.

11. The order of acidic strength of boron trihalides is (a) BF3 < BC13 < BBr3 < BI3

(b) BI3 < BBr3 < BC13 < BF3

(c) BC13 < BBr3 < BI3 < BF3

(d) BBr3 < BC13 < BF3 < BI3

(e) BF3 < BI3 < BC13 < BBr3.

12. The carbide which reacts with water to form propyne is (a) CaC2 (b) SiC (c) Mg2C3 (d) A14C3

(e) Be2C.

42 CHEMISTRY TODAY | AUGUST '05 40

13. Effective magnetic moment of Sc3+ ion is (a) 1.73 (b) 0 (c) 5.92 (d) 2.83 (e) 3.87.

14. Potassium permanganate acts as an oxidant in alkaline and acidic media/The final products formed from KMn04 in the two conditions are respectively (a) Mn02~ and Mn3+ (b) Mn3+ and Mn2+

(c) Mn2+ and Mn3+ (d) Mn02 and Mn2+

(e) Mn2+ and Mn02 .

15. Calculate the mass loss in the following.

,2H + -> 2He + \n (Given the masses : 2H = 2.014; 3H = 3.016, He = 4.004, n = 1.008 amu) (a) 0.018 amu -(b) 0.18 amu (c) 0.0018 amu (d) 1.8 amu (e) 18 amu.

16. Match list I and list II and select the correct answer using the code given below the lists.

List I List II Nuclear reactor Substance used components 1. Moderator A. Uranium 2. Control rods B. Graphite 3. Fuel rods C. Boron 4. Coolant D. Lead

E. Sodium. Codes : (a) 1-B, 2-A, 3-C, 4-E (b) 1-B, 2-C, 3-A, 4-E (c) 1-C, 2-B, 3-A, 4-E (d) 1-C, 2-D, 3-A, 4-B (e) 1-D, 2-C, 3-B, 4-A.

17. AH and AS for a reaction are +30.558 kJ mol"1

and 0.066 k JKr1 moL1 at 1 atm pressure. The temperature at which free energy change is equal to zero and the nature of the reaction below this temperature are (a) 483 K, spontaneous (b) 443 K, non-spontaneous (c) 443 K, spontaneous (d) 463 K, non-spontaneous (e) 463 K, spontaneous.

18. What would be the heat released when an aqueous solution containing 0.5 mole of HN03 is mixed with 0.3 mole of OH~ (enthalpy of neutralisation is -57.1 kJ)? (a) 28.5 kJ (b) 17.1 kJ (c) 45.7 kJ (d) 1.7 kJ (e) 2.85 kJ.

19. Am + 3 BQ,) ^ 4C(i,). Initial concentration of A '(g)

is equal to that of B. The equilibrium concentrations of A and C are equal. Kc = (a) 0.08 (b) 0.8 (c) 8 (d) 80 (e) 1/8.

20. Two moles of PCI5 is heated in a closed vessel of 2 litre capacity. When the equilibrium is attained 40% of it has been found to be dissociated. What is the Kc. in mol/dm3? (a) 0.532 (b) 0.266 (c) 0.133 (d) 0.174 (e) 0.25

21. Dry air is passed through a solution containing 10 g of a solute in 90 g of water and then through pure water. The loss in weight of solution is 2.5 g and that of pure solvent is 0.05 g. Calculate the molecular weight of the solute? (a) 50 (b) 180 (c) 100 (d) 25 (e) 51.

22. The van ' t Hoff factor for BaCl2 at 0.01 M concentration is 1.98. The percentage of dissociation of BaCl2 at this concentration is (a) 49 (b) 69 (c) 89 (d) 98 (e) 100.

23. The standard electrode potentials of Ag+/Ag is + 0.80 V and Cu+/Cu is +0.34 V. These electrodes are connected through a salt bridge and if (a) copper electrode acts as a cathode, then E°ce]] is

+ 0.46 V (b) silver electrode acts as anode, then £°cen is - 0.34 V (c) copper electrode acts as anode then £°cdl is +0.46 V (d) silver electrode acts as a cathode then E°ceu is

- 0.34 V (e) silver electrode acts as anode and £°cel! is +1.14 V.

24. In alkaline medium C102 oxidises H 20 2 to 0 2 and itself gets reduced to CI". How many moles of H 20 2

are oxidised by 1 mole of C102? (a) 1.0 (b) 1.5 (c) 2.5 (d) 3.5 (e) 5.0

25. For the reaction : 2N205 4N02 fc) + 0 2 if the concentration of NO, increases by 5.2 x 10~3 M in 100 seconds then the rate of the reaction is

CHEMISTRY TODAY | AUGUST '05 41

(a) 1.3 x 10"5 Ms"1 (b) 5 x 10"4 Ms"1

(c) 7.6 x 10"4 Ms"1 (d) 2 x 10-3 Ms"1

(e) 2.5 x 10"5 Ms"1

26. A first order reaction is 10% complete in 20 min. The time taken for 19% completion is (a) 30 min (b) 40 min (c) 50 min (d) 38 min (e) 45 min.

27. Lyophilic sols are more stable than lyophobic sols because the particles (a) are positively charged (b) are negatively charged (c) are solvated (d) repel each other (e) are heavy.

28. Potassium stearate is obtained by the saponification of an oil or fat. It has the formula CH3 - (CH2)16 - COO"K+. The molecule has a lyophobic end (CH3) and a lyophilic end COO~K+. Potassium stearate is an example for (a) lyophobic colloid (b) lyophilic colloid (c) multimolecuiar colloid (d) macromolecular colloid (e) associated colloid or micelle.

2 9 . ( i ) K 4 [ F e ( C N ) 6 ] ( i i ) K 3 [ C r ( C N ) 6 ] ( i i i ) K 3 [ C O ( C N ) 6 ] ( iv ) K 2 [ N i ( C N ) 4 ]

Select the complexes which are diamagnetic? (a) (i), (ii) and (iii) (b) (ii), (iii) and (iv) (c) (i), (iii) and (iv) (d) (i), (ii) and (iv) (e) (ii) and (iv).

30. Which is not true of the coordination compound [Co(en)2Cl2]Cl? (a) exhibits geometrical isomerism (b) exhibits optical isomerism (c) exhibits ionisation isomerism (d) is an octahedral complex (e) is a cationic complex.

31. The IUPAC name of the compound is H O O C - C H , ~ C H - CH 2 - CH 2 - C O O H

C O O H (a) 2(carboxymethyl)-pentane-l,5-dioic acid (b) 3-carboxyhexane-l,6-dioic acid (c) butane-1,2,4-tricarboxyIic acid (d) 4-carboxyhexane-l,6-dioic acid (e) 1,2-dicarboxy-pentanoic acid.

32. How much of sulphur is present in an organic compound, if 0.53 g compound gave 1.158 g of BaS04

on analysis? (a) 10% (b) 15% (c) 20% (d) 25% (e) 30%.

33. An alkene having the molecular formula C9H18 on ozonolysis gives 2,2-dimethyl propanal and 2-butanone. The alkene is (a) 2,2,4-trimethyl-3-hexene (b) 2,2,6-trimethyI-3-hexane (c) 2,3,4-trimethyl-2-hexene (d) 2,2,4-trimethyl-2-hexene (e) 2,2-dimethyl-3-heptene.

34. Observe the following reactions and predict the nature of A and B.

35. Nitration of aniline in strongly acidic medium, results in the formation of w-nitroaniline also. This is because (a) amino group is meta orienting during electrophilic

substitution reaction (b) nitro group goes always to the meta position

irrespective of the substituents (c) nitration of aniline is a nucleophilic substitution

reaction in strongly acidic medium (d) in strongly acidic conditions aniline is present as

anilinium ion

42 CHEMISTRY TODAY | AUGUST '05 42

(e) strong acids generate nitrite anion which can attack only the'meta position.

36. How many sigma and pi bonds are present in toluene? (a) 3n + 8CT (b) 3% + 6a (c) 37t + 10a (d) 6n + 3 a (e) 6n + 6g.

37. Which of the following Fischer's projection formula is identical to D-glyceraldehyde?

CH,OH

(a) OH-

CH2OH

-CHO (b) H- OH

H CHO

CHO

(c) HO-

(d) H-

• CHzOH

H CHO H

•CH2OH (e) HO- • CH2OH

HO CHO

38. The name of the compound is

(a) (2Z, 4Z)-2,4-hexadiene (b) (2Z, 4£)-2,4-hexadiene (c) (2E, 4Z)-2,4-hexadiene (d) (4E, 4Z)-2,4-hexadiene (e) (2E, 4£)-2,4-hexadiene.

39. When 32.25 g of ethylchloride is subjected to dehydrohalogenation reaction the yield of the alkene formed is 50%. The mass of the product formed is (atomic mass of chlorine is 35.5). (a) 14 g (b) 28 g (c) 64.5 g (d) 56 g (e) 7 g.

40. Chlorination of toluene in presence of light and heat followed by treatment with aqueous NaOH gives (a) o-cresol (b) p-cresol (c) mixture of o-cresol and p-cresol (d) benzoic acid (e) 1,3,5-trihydroxy toluene.

41. CH3 - CHO + HCN A. Compound A on hydrolysis gives

(a) CH3 - C H 2 - C O O H (b) CH3 - CH2 - CH2 - NH2

(c) CH3 - CO - COOH (d) CH3 CO - CH = NOH (e) CH 3 -CH-COOH

OH

42. Which of the fol lowing does not undergo Cannizzaro's reaction? (a) benzaldehyde (b) 2-methylpropanal (c) jD-methoxybenzaldehyde (d) 2,2-dimethylpropanal (e) formaldehyde.

43. Identify the product in the following sequence.

. . . (i) diazotization „ 3,4,5-tribromoanihne — ^ ? ( i i ) H 3 P 0 2

(a) 3,4,5-tribromobenzene (b) 1,2,3-tribromobenzene (c) 2,4,6-tribromobenzene (d) 3,4,5-tribromonitrobenzene (e) 3,4,5-tribromophenol.

44. Among the amines (i) C6H5NH2 (ii) CH3NH2

(iii) (CH3)2NH (iv) (CH3)3N the order of basicity is (a) (i) < (iv) < (ii) < (iii) (b) (iv) < (iii) < (ii) < (i) (c) (i) < (ii) < (iii) < (iv) (d) (ii) < (iii) < (iv) < (i) (e) (iv) < (iii) < (ii) < (i).

45. The number average molecular mass and mass average molecular mass of a polymer are respectively 30,000 and 40,000. The poly dispersity index of the polymer is (a) < 1 (b) > 1 (c) 1 (d) 0 (e) - 1 .

46. In biological systems, the RNA molecules direct the synthesis of specific proteins which are characteristics of each kind of organism. This process is known as (a) transcription (b) mutation (c) replication (d) translation (e) flocculation.

CHEMISTRY TODAY | AUGUST '05 43

47. Pick up the correct statement. (a) CO which is major pollutant resulting from the

combustion of fuels in automobiles plays a major role in photochemical smog

(b) classical smog has an oxidising character while the photochemical smog is reducing in character

(c) photochemical smog occurs in day time whereas the classical smog occurs in early morning hours

(d) during formation of smog the level of ozone in the atmosphere goes down

(e) classical smog is good for health but not photochemical smog.

48. In Antarctica ozone depletion is due to the formation of following compound. (a) acrolein (b) peroxy acetyl nitrate (c) S0 2 and S0 3 (d) chlorine nitrate (e) formaldehyde.

ANSWERS

1. (c) : CaC03 + 2HC1 CaCl2 + C0 2 + H 2 0 100 g 2 N 44

100 g CaC03 with 2 N HC1 gives 44 g C0 2

100 g CaC03 with 1 N HC1 will give 22 g C02 . 2. (b) : E e= l/X

8000 _ 16000 ,,

16000

El=2E2.

3.

XLOO

E2 8000 (a) : Percentage of ionic character

experimental value of dipole moment theoretical value of dipole moment

= — x 100 = 16.83% « 17%. 6.12

4. (d) : NH3 = 107°, PH3 = 93°, H 2 0 = 104.5C

H2Se = 91°, H2S = 92.5°.

5. (c) :Pl-P,Vl = V,T1 = T P2 = P/2, V2 = ?. T2 = 2T Applying gas equation,

W _ PjV2 or, PV T

PR 2 T

F, = 4 V. 71 T2

6. (a) : P{ = P, K, = V, Tx = 273 + 75 = 348 K 85V 100

P2 = 2P, V2 = r , = ?

PxV _2Px85V 348 ~~ r 2 x l 0 0

r2 = 348x2x85

100

r 2 = 591.6 K = 318.6°C. 7. (a) : The electrolysis of the molten ionic hydride (LiH) yields hydrogen at anode.

MH + H 2 0 MOH + H2 T 8. (b) : In a period, the value of ionisation potential increases from left to right with breaks where the atoms have somewhat stable configurations. However, in a group, the ionisation potential decreases from top to bottom.

Be B Li Na eV 9.3 8.3 5.4 5.1

9. (d) : Froth floatation process is based on the preferential wetting properties with the frothing agent and water. 10. (d) : (a) With the exception of LiN03 , the other nitrates decompose to nitrites and oxygen.

2MN03 2MN02 + 0 2

2LiN03 Li20 + 2N0 2 + 1/2 0 2

(b) Since the alkali metals are highly electropositive, their carbonates are highly stable towards heat and readily soluble in water. Li2C03 decomposes on heating and insoluble in water.

Li2C03 Li20 + CO, (c) When heated in atmosphere of oxygen, the alkali metals ignite and form oxides.

M - M20 normal oxide

(Li 20)

O, • M202 —

peroxide (Nap,)

-> MO,

(d) The alkali metals directly react with halogens forming the halides of the type MX.

2M + X2 —» 2MX (e) Of the alkali metals, only lithium reacts with nitrogen and forms a nitride.

6Li + N2 2Li3N

11. (a) : The relative strength of Lewis acids of boron trihalides increases in the order BF3 < BC13 < BBr3 < BI3. This order can be explained on the basis of the tendency of the halogen atom to back donate its electrons to the boron atom and makes it less electron deficient.

12. (c) : Mg2C3 + 4H20 CH3C = CH + 2Mg(OH)2

13. (b) : n e f f = > ( « + 2)B.M. where n = number of unpaired electrons. For Sc3+ = 3d0, n = 0. n = 0

42 CHEMISTRY TODAY | AUGUST '05 44

14. (d) : In alkaline medium, KMn0 4 is first reduced to manganate and then to insoluble manganese dioxide.

2 M n ( V + H 2 0 2Mn0 2 + 20H" + 3[0] In acidic medium, manganous sulphate is formed.

Mn<V + 8H+ + 5e~ Mn2+ + 4H 2 0 15. (a) : Mass loss = mass of reactants - mass of products

= (2.014 + 3.016) - (4.004 + 1.008)

= 5.030 - 5.012 = 0.018 amu.

16. (b)

17. (d) : AG = AH- TAS 0 = +30.558 - T x 0.066

30.558 „ „ or, T= = 463 K

0.066

If (dG) r P = 0, the sign '= ' refers to a reversible process i.e. non-spontaneous.

18. ( b ) : 0.3 mole of OH" will neutralise only 0.3 mole of HN0 3 . Heat evolved = 57.1 x 0.3 = 17.13 kJ.

19. (c) : A + 3 5 4C a b 0

a - x (b - 3x) 4x

Kc = [C]4 4x • 4x • 4jc • 4x [A][Bf (a - x){b - 3x)

Given a = b, a - x = 4x => a = 5x = b 4x•4x•4x•4x 4x•4x-4x-4x „

Kc = (5x-x)(5x-3 xy 4x-2x-2x-2x

20. (b) : According to Ostwald's dilution law,

a 2 Kc =

( l - a ) F

a = 0.40, K = —= — = 1 litre. C Z / 2

Kc = 0.40x0.40

( l - 0 . 4 0 ) x l = 0.266.

21. (c) : Loss in weight of solution pressure of solution Loss in weight of solvent « p° - ps

(•_• p° = vapour pressure of pure solvent)

o P -Ps loss in weight of solvent

ps loss in weight of solution

P -Ps wxM mxW

0.05

22. (a) :

10x18 90 xm

BaCl2

1

m -- 2x2.5 2x250 = 100.

0.05 Ba2+ + 2C1"

Initial 1 0 0 After disso. 1 - a a 2 a Total = 1 - a + a + 2 a = 1 + 2 a . i = 1 + 2 a

1.98-1 0.98 a = = = 0.49

2 2 For 1 mole, a = 0.49

0.49 For 0.01 mole, a = = 49.

0.01 23. (c) : The cell reaction is

Cu w + 2Ag+ Cu2+ + 2Ag The two half cell reactions are

Cu —» Cu2+ + 2e~ Oxidation (anode) Ag+ + e —> Ag Reduction (cathode)

Ecew = Eox - £ red = 0.80 - 0.34 = +0.46 V. 24. (c) : C102 CL

C102 + 2H 2 0 + 5e CI" + 40H" h 2 o 2 o 2

H 2 0 2 + 2 0 H - 0 2 + 2H 2 0 + 2e C102 + 2HzO + 5e CI- + 40H~] x 2 H2Q2 + 2QH- 0 2 + 2H2Q + 2e] x 5 2C102 + 5H 2 0 2 + 20H" 2C1" + 50 2 + 6H 2 0

2C102 = 5H 20 2

C102 = 2.5 H 20 2 .

25. (a) : 2N 2 0 5 ^ 4 N 0 2 (s) + 0 2 fe)

Rate of the reaction with respect to N 0 2

i -3 _ 1 4 N Q 2 ] _ 1 5.2x10' ~ 4 dt ~ 4 X 100

26. (b) : For 1st case: , 2.303, 90 k = log

20 100 For Ilnd case :

, 2.303, 81 k = log t 100

From eqn. (i), 2.303, .90 2.303, 81

log = log 20 100 t 100

- = 1.3xl0~5 Ms

(0

(ii)

t = 20( log81- log l00 ) 20(1 .908-2)

(log 9 0 - l o g 100)

20 x (-0.092)

( -0 .046)

(1 .954-2 )

= 40 min.

CHEMISTRY TODAY | AUGUST '05 4 5

27. (c ) : Lyophilic sols are more stable than lyophobic sols because the residue left on evaporation can be readily transformed back into colloidal state simply by adding solvent.

28. (e) : Substances whose molecules aggregate spontaneously in a given solvent to form particles of colloidal dimensions are called associated colloids. The molecules of soaps and detergents are usually smaller than the colloidal particles. However, in concentrated solutions, these molecules associate and form aggregates of colloidal size. These aggregates of soaps or detergent molecules are called miscelles.

29. (c) : 45 4p 3 d

[Fe(CN)6]4'

[Co(CN)6p-

[Ni(CN)4p-

11 11 11 XX X X xx

3 d cP-sp3, diamagnetic

45 4p

11 11 11 XX X X xx

3d cP-sp3, diamagnetic

45 4 p

11 11 11 11 XX x x XX

dsp1, diamagnetic

30. (c) : Ionisat ion isomerism arises when the coordination compounds give different ions in solution, this condition is not satisfied with [Co(en)2Cl2]Cl.

ci -1 CI c/s-form trans-form

Geometrical isomers

«5-/-isomer

optical isomers

1 2 3 4 5 6 31. (b) : HOOC - CH2 - CH - CH2 - CH2 - COOH

COOH 3-carboxyhexane-l,6-dioic acid

32. (e) : Percentage of sulphur

mass ofBaSO, — X

233 mass of organic compound

32 1.158

-x 100

233 0.53

33. (a) : C9H18 + 0 3 -

-x 100 = 30%.

CH,

• H,C - C - CHO 3 I

CH3

2,2-dimethylpropanal

+ CH3CH2COCH3

2-butanone

On the basis of product formation, the alkene must be

CH, CH, I

CH3 - C - H C = C - CH2CH3

CH3

2,2,4-trimethyl-3-hexene

This is further confirmed by the reaction CH, CH,

CH3 - C - H C = C - CH2CH3 — C H 3 C - HC ; C C H , C H , I I l i l C H I •. i

CH 3

':; CC

CH, o i l o

CH, O II

-> CH 3C-CHO + CH3C-CH2CH3

CH,

34. (c) : HBr > A

Formation of A shows electrophilic addition reaction, (i) HBr —> H+ + B r

r \ . .CH = CH

(») ©f CH J

[ o f N C H 3 + B r - _ > ( o T ^

+ H+ > C6H5CHCH3

Br

(A)

46 CHEMISTRY TODAY | AUGUST '05

tor HBr M)

> B

Formation of B follows free radical addition reaction.

(i) HBr hv -> H + Br

( i i ) ( g p Br

(iii) [ O j ^ Br

NH2

35. (d ) : [ Q ]

aniline

+ HBr

HNO,

©a. NH,

p-nitroaniline (51%)

NH, NFL

NO,

NO,

m-nitroaniline o-nitroaniline (47%) (2%)

This is due to the reason that under acidic conditions, protonation of the - NH2 group gives anilinium cation

+

(NH3) which is deactivating and /w-directing.

H

36. (c) C - H I H

IOct + 371

37. (c) : The configuration, in which OH group lies towards right and the H-atom towards left, the CHO lies at the top and CH2OH lies at the bottom of the Fischer projection, is called D-configuration.

CHO

H- •OH

CH2OH D-glyceraldehyde

(2) CHO

(2) CHO

( 4 ) H -O interchange „

• OH — , „.. „ . > Cf 1,011-(1)

-OH

(3) C H , O H H (4)

CHO

or, HO- • CH,OH

H 38. (e) : If the atoms or groups of higher priority on each carbon atom of the double bond are on the opposite sides of the double bond, the configuration is designated as E and if they are on the same side, the configuration is designated as Z.

(2E, 4£)-2,4-hexadiene

-HC1 > C2H4

28

39. (e) : C2H5C1 64.5 32.25

64.5 g C2H5C1 yields 28 g C2H4

28x32.25 32.25 g C2H5C1 yields = — — —

As the yield is 50%, so the mass of alkene formed = 14/2 = 7 g.

= 14gC 2 H 4

CH,

40. (d) : [ O

CH2C1 CI,

high temp.

CI,

toluene benzyl chloride

CHC1,

benzal chloride

CC1, CI, NaOH

-> CsHsCOOH benzoic acid

benzo trichloride

OH r 41. (e) : CHjCHO + HCN > CH 3C-CN

H (A)

OH

C H 3 - C - C O O H

H

CHEMISTRY TODAY | AUGUST '05 47

42. (b): Aldehydes containing no a - H atoms on heating with concentrated alkali solution (50%) undergo Cannizzaro's reaction. In this reaction one molecule is oxidised at the expense of other which is reduced to alcohol.

C6H5CHO no a -H atom

CH, I

CH3-CHCHO a-H atom

OCH,

CHO no a -H atom

CH, I

CH,C - CHO I CH3

no a -H atom

HCHO no a -H atom

43. (b)

44. (a) : (CH3)2NH > CH3NH2 > (CH3)3N > C6HSNH2

iii > ii > iv > i This anomalous behaviour of tertiary amines is due to steric factors, i.e. crowding of alkyl groups cover nitrogen atom from all sides and thus makes the approach and bonding by a proton relatively difficult which results the maximum steric strain in tertiary amines. The electrons are there but the path is blocked, resulting the reduced in its basicity. In case of aniline, the phenyl group exerts - I (inductive) effect, i.e. it withdraws electrons. This results to the lower availability of electrons on nitrogen for protonation.

45. (b) : Number average molecular mass, M„ =30,000 Mass average molecular mass, Mw = 40,000

Mw __ 40,000 Poly dispersity index (PDI) = ^ ~ 3 Q Q()0

- = 1.33

46. (d) : Translation is the RNA directed synthesis of polypeptides. This process requires all three classes of RNA. Although the chemistry of peptide bond formation is relatively simple, the processes leading

to the ability to form a peptide bond are exceedingly complex. The template for correct addition of individual amino acids is the wRNA, yet both /RNAs and rRNAs are involved in the process. The /RNAs carry activated amino acids into the ribosome which is composed of rRNA and ribosomal proteins. The ribosome is associated with the /kRNA ensuring correct access of activated /RNAs and containing the necessary enzymatic activities to catalyze peptide bond formation. 47. (c) : Classical or London-type or sulphurous smog-results from a high concentration of sulphur oxides in the air and is caused by the use of sulphur bearing fossil fuels, particularly coal.

Photochemical or Los Angeles-type smog - has its origin in the nitrogen oxides and hydrocarbon vapours emitted by automobiles and other sources which then undergo photochemical reactions in the lower atmosphere. Characteristics Classical

(London-type) Photochemical (Los Angeles-type)

Primary pollutants SOj, soot particles

NO*, organics

Secondary pollutants H 2 S 0 4 , sulphate aerosol etc.

0 3 , PAN, organics, acids, aerosols etc.

Temperature cool (< 35°F) hot (> 75°F)

Relative humidity high, usually

foggy

low, usually hot and dry

Type of temperature inversion

radiation inversion

subsidence inversion

Time air pollution peaks

early morning noon-evening

48. (d) : At higher altitudes inorganic chlorine is abundant, most of it in the form of HC1 or of chlorine nitrate, C10N02. These are called "chlorine reservoirs"; they do not themselves react with ozone, but they generate a small amount of chlorine-containing radicals - CI, CIO, C102 and related species, referred to collectively as the 'C10 r family' - which do. An increase in the concentration of chlorine reservoirs leads to an increase in the concentration of the ozone - destroying radicals. Polar stratospheric clouds play a major role in the depletion of ozone layer, Antarctica by promoting the release of photolytically active chlorine from its reservoir species.

HC1 + C10N02

42 CHEMISTRY TODAY | AUGUST '05 48

SOLVED PAPER

DPMT - 2005 1. Which of the following is not an actinide? (a) curium (b) californium (c) uranium (d) terbium

2. Europium is (a) s-block element (c) <i-block element

(b) p-block element (d) /-block element.

3. For an electron if the uncertainty in velocity is Av, the uncertainty in its position (Ax) is given by , . hm 47t (a) T-r- (b)

(c)

47tAv h

47twAv (d)

hm&v 4nm h-Av

4. The reagent in Friedel Craft's reaction is (a) pyridine (b) RCOC1 (c) RCOOH (d) HC1.

5. TheK s pofMg(OH) 2 i s 1 x 10"12. 0.01 MMg(OH)2

will precipitate at the limiting pH. (a) 3 (b) 9 (c) 5 (d) 8

6. Equation of Boyle's law is dP dV dP

(a) "1T= 7T (b) dV

V dV_

P dT d2P „. d2p d2V

(d) = + y ' P dT

7. A radioactive sample is emitting 64 times radiations than non hazardous limit. If its half life is 2 hours, after what time it becomes non-hazardous? (a) 16 hr (b) 12 hr (c) 8 hr (d) 4 hr.

8. A metal surface is exposed to solar radiations (a) the emitted electrons have energy less than a

maximum value of energy depending upon frequency of incident radiations

(b) the emitted electrons have energy less than maximum value of energy depending upon intensity of incident radiation

(c) the emitted electrons have zero energy

(d) the emitted electrons have energy equal to energy of photons of incident light.

9. Which of the following transitions have minimum wavelength? (a) «4 —» n, (b) n2 —> (c) /?4 n2 (d) «3 ->

10. Orbital is (a) circular path around the nucleus in which the electron

revolves (b) space around the nucleus where the probability of

finding the electron is maximum (c) amplitude of electron wave (d) none of the above.

11. Number of unpaired electrons in Mn4+ is (a) 3 (b) 5 (c) 6 (d) 4.

12. Which of the following sequence is correct as per Aufbau principle? (a) 3s < 3d < 45 < 4p (b) Is < 2p < 45 < 3d (c) 25 < 5s < 4p < 5d (d) 2s < 2p < 3d < 3p.

13. Ionic compounds are formed most easily with (a) low electron affinity, high ionisation energy (b) high electron affinity, low ionisation energy (c) low electron affinity, low ionisation energy (d) high electron affinity, high ionisation energy.

14. The enthalpy change (ATP) for the neutralisation of 1 M HC1 by caustic potash in dilute solution at 298 K is (a) 68 kJ (b) 65 kJ (c) 57.3 kJ (d) 50 kJ.

15. Which of the following is not hydrolysed? ( a ) A S C I 3 ( b ) P F 3

(c) SbCl3 (d) NF3

16. Which of the following gases is linear? (a) C0 2 (b) SOz

(c) NO, (d) S03 .

* based on memory

18 CHEMISTRY TODAY | SEPTEMBER '05 17

17. N H 4 C O O N H 2 W 2NH3fe) + C02(x ) if equilibrium pressure is 3 atm for the above reaction Kp for the reaction is (a) 4 (b) 27 (c) 4/27 (d) 1/27

18. Number of isomeric primary amines obtained from C 4 H n N are (a) 3 (b) 4 (c) 5 (d) 6.

19. If hydrogen electrode dipped in 2 solution of pH = 3 and pH = 6 and salt bridge is connected the e.m.f. of resulting cell is (a) 0.177 V (b) 0.3 V (c) 0.052 V (d) 0.104 V.

20. A radioactive nucleus will not emit (a) alpha and beta rays simultaneously (b) beta and gamma rays simultaneously (c) gamma and alpha rays (d) gamma rays only.

21. In face centred cubic unit cell edge length is

(a) V T

(c) 2r

(b)

(d)

Tl

22. If the Zn2+/Zn electrode is diluted to 100 times then the change in e.m.f. (a) increase of 59 mV (b) decrease of 59 mV (c) increase of 29.5 mV (d) decrease of 29.5 mV

23. Which of the following reactions end in finite time? (a) 0 order (b) 1st order (c) 2nd order (d) 3rd order.

24. If equivalent conductance of 1 M benzoic acid is 12.8 ohm-1 cm2 and if the conductance of benzoate ion and H+ ion are 42 and 288.42 ohm-1 cm2 respectively. Its degree of dissociation is (a) 39% (b) 3.9% (c) 0.35% (d) 0.039%.

25. In which of the following reactions carbon-carbon bond formation takes place? (a) Cannizzaro (b) Reimer-Tiemann (c) HVZ reaction (d) Schmidt reaction.

26. Which gives only monosubstituted product? (a) o-dinitrobenzene (b) /w-dinitrobenzene (c) p-dinitrobenzene (d) nitrobenzene.

27. Which of the following is most polarised? (a) Kr (b) He (c) Ar (d) Xe.

28. Order of boiling point is (a) HF > HI > HBr > HC1 (b) HF > HBr > HI > HC1 (c) HC1 > HBr > HI > HF (d) HC1 > HI > HBr > HF

29. If two substances A and B have P°A : P°B = 1 : 2 and have mole fraction in solution 1 :2 then mole fraction of A in vapours (a) 0.33 (b) 0.25 (c) 0.52 (d) 0.2

30. Order of hydrolysis for the following (I) RCOC1 (II) RCOOR (III)RCONH2 (IV)(RC0)20. (a) I > IV > II > III (b) I > II > III > IV (c) I > III > II > IV (d) IV > III > II > I

31. If an aqueous solution of glucose is allowed to freeze than crystal of which will be separated out first? (a) glucose (b) water (c) both of these (d) none of these.

32. CH = CH HgS0 4 CHjMgBr P/Br2

(a) CH3CH(Br)CH3

(c) CH2 = CH - Br (b) CH3CH2CH2Br (d) BrCH = CH - CH3

33. Number of bonds in benzene (a) 6a and (b) 12a and 3n (c) 3a and 12rc (d) 6a and 6m.

34. If the vrms is 30/?I/2 at 27°C then calculate the molar mass of gas in kilogram. (a) 1 (b) 2 (c) 4 (d) 0.001

35. If the enolate ion combines with carbonyl group of ester, we get

(b) a,p-unsaturated ester (d) acid.

(a) aldol (c) (3-keto aldehyde

COOH

H-36. H-

COOMe

OH -OH

HO-HO-

H -H

COOMe I

I and II are

COOH II

18 CHEMISTRY TODAY | SEPTEMBER '05 18

(a) enantiomers (c) meso compound

(b) diastereomers (d) identical.

37. The standard molar heat of formation of ethane C0 2 and water (1) are respectively -21.1, -94.1 and -68.3 kcal. The standard molar heat of combustion of ether " will be (a) - 172 kcal (b) 162 kcal (c) - 140 kcal (d) 183.5 kcal.

38. (a) 69, 172 (c) 180, 70

z*' • Z and A are (b) 172, 69 (d) 182, 68.

39. A + 2B —* C + D. If - ^ = 5xl0" 4 m o i r V 1 , dt

d [ B ] • then :— is

dt (a) 2.5 x io-4 mol I"1 s"1

(b) 5.0 x 10^ mol 1-' s"> (c) 2.5 x io-3 mol I"1 s-1

(d) 1.0 x IO"3 mol l"1 s"1.

40. Which of the following is paramagnetic? (a) N2 (b) C2

(c) N2+ (d) 02

2".

41. Which of the following compounds will react with NaHC03 solution to give sodium salt and carbon dioxide? (a) acetic acid (b) w-hexanol

(c) phenol (d) both (b) and (c).

42. Which will give chiral molecule?

(a) CH3COCI LiA 'H< > ( b ) C 2 H 5 c h o c ^ M g B r ,

h + / h 2 o

(c) (CH3)2CHC2H5

H \ (d) / C = C

Cu

/ C H 3 cu CH3- ^CH3

43. Which is not a polymer? (a) sucrose (b) enzyme (c) starch (d) teflon.

44. Which statement is wrong for NO? (a) it is anhydride of nitrous acid (b) its dipole moment is 0.22 D (c) it forms dimer (d) it is paramagnetic

45. Which of the following reactions will not give propane?

(a) CH3CH2CH2C1 Mg/ether, h 2 O '

(b) CH3COCI < W > /

(c) CH3CH = CH2 B2H6

C H X O O H

(d) C H 3 C H - C H 3 P / H 1 >

OH

46. A compound A —> C5H,0CI2 on hydrolysis gives C5H l0O which reacts with NH2OH, forms iodoform but does not give Fehling test. A is

CI I

(a) CH3 - C - CH2 - CH2 - CH3

CI CI CI I I

(b) CH3CH2-C-CH2CH3 (c) CH3CH2CH2CH,CH

CI CI CI CI I I

(d) C H 3 - C H - C H - C H 2 - C H 3

47. In photography sodium thiosulphate is used as (a) complexing agent (b) oxidising agent (c) reducing agent (d) none of these.

. - base 48. H O - ^ C ) > + < Q ^ N 2+ C l

(a) < ^ N = N - ^ C ^ O H

(b) < Q ^ - < Q > (o < § K O >

(d) < Q > - < 0 ^ OH

49. There is no S - S bond in (a) S204

2" (c) S 2 0 3

2-(b) S2Os

2-(d) s2o7

2-. 50. Which of the following is not a broad spectrum antibiotic? (a) tetracycline (b) Chloromycetin (c) penicillin (d) none of these.

SOLUTIONS

1. (d) : Terbium is lanthanide as it belongs to 4/-series having configuration [Xe] 4 f 9 6s2. However

18 CHEMISTRY TODAY | SEPTEMBER '05 19

the remaining members belong to 5/-series (actinides). 2. (d) : Europium is a/-block element as it follows the general electronic configuration of the/block element ( 4 / , - 1 4 5 d°>'6s2).

Eu = [Xe] 4 / 7 6s2.

h 3. (c) : Ax •Ap = 4ti

Ax • m • Av = — 471

Ax = - h 47t • mAv

4. (b ) : The reaction of an alkyl halide or aryl halide with benzene in the presence of a Lewis acid, generally A1C13 is known as Friedel Craft's reaction. 5. (b) : Mg(OH), ^ Mg2+ + 20H-Ksp = [Mg2+] [OH-]2

1 X IO"12 = 0.01 [OH-]2

[OH-]2 = 1 x 10-10 [OH] = 10"5. [H+] = 10- |4/10-5 = IO"9. pH = -log[H*] = —log[10 9] = 9. 6. (a) : Boyle's law - PV = constant On differentiating the equation,

PdV + VdP = 0 dP _ dV P ~ V

\_

2

d(PV) = d(C)

VdP=-PdV

7. (b) : N = N0 ^ — =

or, 64

i T ^ N 2) " Vo

/• - \ 6 > -u -" U J n = 6.

T = tV2 * n = 2 x 6 = 1 2 hours. After 12 hours, sample became non-hazardous. 8. (a) 9. (a) : As we know, E = hdX

MX For «4 greater transition, greater the energy difference, lesser will be the wavelength. 10. (b) : The well defined circular path around the nucleus in which the electron revolves is called orbit however the region in space around the nucleus, where the probability of finding electron is maximum is called orbital.

11. (a) : Mn —» [Ar] 3d5 4s2

Mn4+ -» [Ar] 3cP 4i'Q

3d 4s • 4p

1 1 1

12. (b) : According to Aufbau principle, electrons are filled into different subshells of an atom in order of their increasing energy, which means the subshell with lower energy is filled first. I s < 2s < 2p < 3s < 3p < 4s < 3d < 4p < 5s < 4d. 13. (b) : The formation of ionic bond depends upon easy formation of cation and anion. Therefore the ionisation energy value of the metal atom should be low, so that it can easily form cation. On the other hand, the electron affinity value of the non-metal atom should be high so that it can easily form anion.

14. (c) : HC1 and KOH both are strong. 15. (d) : Due to absence of ^-orbitals in N atom, it cannot accept electrons from H20 for hydrolysis of NF3. 16. (a) : C02 is linear due to s/>-hybridisation.

o = c — O 17. (b) : Equilibrium pressure = 3 atm

NH4COONH 2 (.«) 2NH3(K) + C02 ( S )

K p = Pnh3 ' Pco2 = 3" - 3 = 27.

18. (b) : Four 1° amines are possible. CH3CH2CH2CH2NH2, (CH3)2CHCH2NH2, CH3CH(NH2)CH2CH3, (CH3)3CNH2

I f T 6 , 19. (a) : Ecell = - 0 . 0 5 9 1 o g ^ Z J = -0.0591ogl0"3

= - 0.059 x (-3) = 0.177 V. 20. (d) : A radioisotope first emits a or P particles, then it becomes unstable and emits y-rays. 21. (b) : In face centred cubic structure (fee) radius

a of the atom r = , as the atoms touch each other

along the face diagonal of the cube. a

~ 2>/2

a (edge length)

22. ( a ) : =

2 V 2 = 2V2 • /• • ^Zr = —/" . V2 V2

1 0.059, 1 0.059, — l o g ? , = — 2 - l o g —

Number of unpaired electrons = 3

0.059 (-2) = 0.059 V = 59 mV.

(increase) 23. (a): In case of zero order reaction, the concentration of reactant decreases linearly with time, as its rate is independent of the concentration of the reactants.

18 CHEMISTRY TODAY | SEPTEMBER '05 20

24. ( b ) : A°m ( c 6 h 5 cooh) - a 0( c 6 h 5 coo- ) + aC '(h+)

= 42 + 288.42 = 330.42

a = A0 330.42 1 ym

25. (b ) : Reimer-Tiemann.reaction involves the carbon-carbon bond formation.

OH ONa PHD NaOH, 340 K

O J +CHCl3

dil. HCl -NaCl

CHO

salicylaldehyde

NO,

26. (b) gives only monosubstitution NO,

product as - N 0 2 group is meta directing and only one m-position is possible in w-dinitrobenzene.

27. (d ) : Xe is highly polar since the ionisation potential of xenon is quite close to the ionisation potential of oxygen.

28. ( a ) : Because of high electronegativity, H - F forms strong H-bonds. As a result, it exists as an associated molecule (HF)„. Thus, the boiling point (293 K) of HF is abnormally high. The boiling points of the hydrogen halides of other elements are much lower than that of H - F. However, after a sudden drop in boiling point from HF to HCl, the boiling points of other hydrogen halides gradually increase from HCl to HI because of increase in size of the halogen atom CI to I. In other words, volatility decreases in the order : HCl > HBr > HI > HF as the boiling points increase in the order: HX HCl HBr HI HF Boiling pt. 1 8 9 K 206 K 238 K 293 K

29. (d) : Relationship between mole fraction of a component in the vapour phase and total vapour pressure of an ideal solution.

y - Ed _ xa'PA A /'total XA-Pa + x b ' Pb

• 1 x 1 = — = 1 = 0.2 1 x 1 + 2 x 2 1+4 5

30. (a) : Of all the acid derivatives, acid chlorides, i.e. CH3COCI is most reactive. The order of reactivity of acid derivatives decreases in the following order:

RCOC1 > ( R C 0 ) 2 0 > RCOOR > RCONH2.

31. (b) : Freezing point is the tempera ture at which the liquid and the solid form of the same substance are in equilibrium and hence have the same vapour pressure. Due to lower vapour pressure of the solution, solid form of a solution separates out at a lower temperature as shown in figure. The decrease is called depression in freezing point. When solid is the solute, it is solvent that freezes. Hence in the given question water will be separated out first.

32. (a) : CH = CH — — C H ^ B r •

OMgBr

t r„ temperature

CH3CHO

CH3 - C - CH3

OH

CH, - C - H

Br P/Br,

CH, - C - H

H

33. (b)

CH, CH,

H i

H - C C - H I II

H - C ^ C - H

I H

12a and 3Ji-bonds

34. (d) : v m s :

yl302R =

3 RT

M 3 RT M

30x307? = 3i?x300

M

=> M = ^ ^ = l g m = 0.001 kg. 30x30 6 6

35. (c) : Reaction can be explained as follows. O 0

(i) / x

H CH3

C / \ r .

H CH2

enolate ion

18 CHEMISTRY TODAY | SEPTEMBER '05 22

( i i )

O II

A=R H CH,

a o

CH. 7s O II

A r H CH2 - C - OCHJ

CH,

OCH,

O

O -OCH,- "

A 11 H C H 2 - C - C H 3

(5-ketoaldehyde

36. (d) : Both are mirror images of each-other, I and II are identical. 37. (a) : Aim : C2H6 + 7/2 0 2 -> 2C02 + 3H20 AH = [2 A// / (C0 2) + 3AHf (H20)]

- [AHf (C2H6) + 7/2 AHf(02)] = [2 (-94.1) + 3(-68.3)] - [-21 + 0] = -372 kcal.

38. (a) : *f2X 2a m 68 1 i - H2 n — • 69 y -

172 69

39. (a) : A + 25 C + D

Z f M = 5 xl (T 4

dt

i M . ^ = 2.5x10- mol"' sec"'. 2 dt 2

40. (c): N2 : KK a (2 s f a*(2s)1 n(2px)2 n(2py)2 a(2p.)2

(diamagnetic) C2 : KK o(2s)2 a*(2s)2 n(2px)2 n(2py)2 (diamagnetic) N2

+ : KK CT(2s)2 a*(2s)2 n(2px)2 n(2py)2 a(2pz)1

(paramagnetic) 02

2" : KK a(2s)2 a*(2s)2 a(2pz)2 n(2px)2 n(2py)2

n*(2px)2 n*(2py)2 (diamagnetic) 41. (a) : CH3COOH + NaHC03 ->

CH3COONa + C0 2 + H 20

42. (b) : C2H5CHO CHjMgBr

H L

C 2 H 5 - C - O H H+/H2O 2 5 I

CH, C*-chiral carbon as all the four valencies are attached with different substituents or groups.

43. (a) : Sucrose is a disaccharides which upon acid or enzymatic hydrolysis gives only two molecules of monosaccharides. „ H+ or invertase „ , , , Sucrose > D (+)-glucose + D (-) -fructose 44. (a) : Anhydride of nitrous acid is N203.

45. (b) : With calculated amount of Grignard reagent, acetyl chloride forms ketones.

CI CH3COCI + J f M g C H , > CH3COCH3 + M g \ X

46. (a) : As A reacts with hydroxylamine it means A may be aldehyde or ketone. But it does not react with Fehling solution hence A must be a ketone. Secondly it forms iodoform which is a characteristic reaction of methyl ketone.

NOH O

CH3C — CH2CH2CH3 NH,OH II

OH I

CH3 — C — CH2CH2CH3 4

OH

CH3C-CH2CH2CH3

I2/NaOH

' CHI,

CI I

CHj(p — CH0CH7CH3

CI (4)

H , 0

47. (a) : In order to make the image permanent, it is necessary to remove the unreduced silver bromide from the surface of the developed film. This operation is called fixing of image. Fixing is done by dipping the developed film or plate in sodium thiosulphate (hypo) solution. The hypo solution dissolves the unreduced silver bromide by forming a complex.

AgBr + 2Na2S203 Na3[Ag(S203)2] + NaBr sodium argentothiosulphate

(soluble)

Thus sodium thiosulphate acts as a complexing agent.

48. (a) : < ^ 0 ) - N t c r + H - ^ ) - OH base..

< 0 > - N = N - < 0 > - O H /)-hydroxy azobenzene

(orange dye) This is an example of coupling reaction. 49. (d) : In S207

2 - ion, the two S atoms are linked through an oxygen atom forming one S - O - S bond. Each sulphur atom is further connected to three oxygen atoms (forming two S O and one S - O -bonds). Thus, there is no S - S bond. But in all other ions there is a S - S bond. 50. (c) : Penicillinis not a broad spectrum antibiotic since it is active against infections caused by gram positive bacteria only.

CHEMISTRY TODAY | SEPTEMBER '05 23

SOLVED PAPER

J a m m u & K a s h m i r CET - 2 0 0 5

1. Who used the quantum theory for the first time to explain the structure of atom? (a) de Broglie (b) Bohr (c) Heisenberg (d) Einstein.

2. The boiling point of water decreases at high altitudes because (a) the atmospheric pressure is low (b) the temperature is low (c) the atmospheric pressure is high (d) the temperature is high.

3. In a solid lattice, the cation has left a lattice site and is located at interstitial position, the lattice defect is (a) interstitial defect (b) vacancy defect (c) Frenkel defect (d) Schottky defect.

4. The entropy of crystalline substances at absolute zero going by the third law of thermodynamics should be taken as (a) 100 (b) 50 (c) zero (d) different for different substances.

5. AG for a spontaneous reaction is (a) zero (b) negative (c) positive (d) could be positive or negative.

6. The IUPAC name for CH3CO - CH3 is (a) dimethyl ketone (b) acetone (c) propanal (d) propanone.

7. Which of the following is not an endothermic reaction? (a) dehydrogenation (b) ethane to ethene (c) combustion of propane (d) change of chlorine molecule into chlorine atoms.

8. A process in which the system does not exchange heat with the surroundings is known as (a) isothermal (b) isobaric (c) isochoric (d) adiabatic.

9. Which of the following is not a non-electrolyte?

(a) acetic acid (c) ethanol

(b) glucose (d) urea.

10. The unit ohm -1 is used for (a) molar conductivity (b) equivalent conductivity (c) specific conductivity (d) conductivity.

11. The tendency of an electrode to lose electrons is known as (a) electrode potential (b) reduction potential (c) oxidation potential (d) e.m.f.

12. For the feasibility of a redox reaction in a cell, the e.m.f. should be (a) positive (b) fixed (c) zero (d) negative.

13. If the rate of reaction A B doubles on increasing the concentration of A by 4 times, the order of the reaction is (a) 2 (b) 1 (c) 1/2 (d) 4.

14. Fog is a colloidal solution of (a) solid in gas (b) liquid in gas (c) gas in liquid (d) gas in solid.

15. Muddy water can be purified through coagulation using (a) common salt (b) alums (c) sand (d) lime.

16. Formation of ammonia from H2 and N2 by Haber's process using Fe is an example of (a) heterogeneous catalysis (b) homogeneous catalysis (c) enzyme catalysis (d) non-catalytic process.

17. The reason for almost doubling the rate of reaction on increasing the temperature of the reaction system by 10°C is (a) the value of threshold energy increases (b) collision frequency increases (c) the fraction of the molecule having energy equal

to threshold energy or more increases (d) activation energy decreases.

34 CHEMISTRY TODAY SEPTEMBER '05

18. The pH of an aqueous solution having hydroxide ion concentration as 1 x IO-5 is (a) 5 (b) 9 (c) 4.5 (d) 11.

19. Which of the following is not a Lewis acid? (a) BF3 (b) A1C13

(c) HCl (d) LiAlH4.

20. The precipitation takes place only when the product of concentrations of ions (a) exceeds the solubility product (b) is less than the solubility product (c) is negligible (d) is equal to the solubility products.

21. Which of the following has lowest electron affinity? (a) CI (b) 1 (c) F (d) Br.

22. In the calcium fluoride structure the co-ordination number of the cation and anions are respectively (a) 6, 6 (b) 8, 4 (c) 4, 4 (d) 4, 8

23. The total number of orbitals possible for principal quantum number n is (a) n (b) rr (c) In (d) 2rr.

24. The pair having similar geometry is (a) PCI3, NH3 (b) BeCL, H 2 0 (c) CH4, CC14 (d) IFS, PF5.

25. The (/-orbital involved in sp3d hybridisation is (a) rf 2_ 2 (b) dxy

(c) d_2 (d) dxx.

26. If 8.0 g of a radioactive substance has a half-life of 10 hrs., the half life of 2.0 g of the same substance is (a) 2.6 hr (b) 5 hr (c) 10 hr (d) 40 hr.

27. Loss of a beta particle is equivalent to (a) increase of one neutron only (b) decrease of one neutron only (c) both (i) and (ii) (d) none of these.

28. Which of the following is incorrect? (a) Relative lowering of vapour pressure is independent

of the nature of the solute and the solvent. (b) The vapour pressure is a colligative property. (c) Vapour pressure of a solution is lower than the

vapour pressure of the solvent. (d) The relative lowering of vapour pressure is directly

proportional to the original pressure.

29. The presence of the chlorine atom on benzene ring makes the second substituent enter at a position (a) ortho (b) meta (c) para (d) ortho/para.

30. Hydrogen is not obtained when zinc reacts with (a) cold water (b) hot NaOH solution (c) conc. sulphuric acid (d) dilute HCl.

31. The oxidation number of xenon in XeOF2 is (a) zero (b) 2 (c) 4 (d) 3.

32. Which of the following is most volatile? (a) HF (b) HCl (c) HBr (d) HI.

33. The form of iron having the highest carbon content is (a) cast iron (b) wrought iron (c) strain steel (d) mild steel.

34. Which belongs to the actinides series? (a) Ce (b) Cf (c) Ca (d) Cs.

35. Which of the following is planar? (a) XeF2 (b) Xe03F (c) Xe02F2 (d) XeF4.

36. Cr03 dissolves in aqueous NaOH to give (a) Cr04

2- (b) Cr(OH)3-(c) Cr207

2" (d) Cr(OH),.

37. Which of the following is a tribasic acid? (a) H3P04 (b) H4P207

(c) H 4 P 2 0 7 (d) H 4 P 2 O 6 .

38. The total number of possible isomers for the complex compound [Cun(NH3)4][Pt"Cl4] are (a) 3 (b) 6 (c) 5 (d) 4.

39. The molecule having a pyramidal shape out of the following is (a) CO, (b) PC13 (c) SF4 (d) NH3.

40. Which of the following doesn't give a ppt. with silver nitrate solution? (a) ethyl bromide (b) sodium bromide (c) calcium chloride (d) sodium chloride.

41. Which is the most stable carbocation? (a) wo-propyl cation (b) triphenylmethyl cation (c) ethyl cation (d) «-propyl cation.

42. Which is most acidic of the following? (a) methane (b) acetylene (c) 1-butene (d) neo-pentane.

18 CHEMISTRY TODAY | SEPTEMBER '05 35

43. Which is most reactive of the following? (a) ethyl acetate (b) acetic anhydride (c) acetamide (d) acetyl chloride.

44. Which of the following will be chiral? (a) CH3CHC12 (b) CH3CHBrCl2

(c) CD2C12 (d) CH2ClBr.

45. Electronic configuration of deuterium atom is (a) Is1 (b) 2s2 (c) 2s1 (d) Is2.

46. Which of the following is a phenol? (a) pentanoic acid (b) phthalic acid (c) picric acid (d) phosphoric acid.

47. Which of the following is not an organometallic compound? (a) C2H5ONa (b) CH3MgI (c) tetraethyl tin (d) KC4Hc,.

48. The correct order of electron affinity is (a) B < C < O > N (b) B > C > N > O (c) O > C > B > N (d) O < C < B < N.

49. ZieglerNatta catalyst is an organometallic compound containing (a) iron (b) titanium (c) rhodium (d) zirconium.

50. Which of the following cannot undergo nucleophilic substitution under ordinary conditions? (a) chlorobenzene (b) /er/-butylchloride (c) isopropyl chloride (d) none of these.

51. The compound which contains all the four 1°, 2°, 3° and 4° carbon atoms is (a) 2,3-dimethyI pentane (b) 3-chloro-2,3-dimethylpentane (c) 2,3,4-trimethylpentane (d) 3,3-dimethylpentane.

52. Brass, bronze and german silver have one metal in common. This is (a) Zn (b) Fe (c) A1 (d) Cu.

53. Which of the following is the green coloured powder produced when ammonium dichromate is used in fire works? (a) Cr (b) Cr03

(c) Cr203 (d) Cr0(02) .

54. The 7t-bonded organo metallic compound which has ethene as one of its component is

18 (a) Zeise's salt (b) ferrocene (c) dibenzene chromium (d) tetraethyl tin.

55. Malachite is an ore of (a) Fe (b) Ag (c) Cr (d) Cu.

56. The most acidic of the following is (a) ClCH2COOH (b) C6H5COOH (c) CD3COOH (d) CH3CH2COOH.

57. Which of the following is an electrophile? (a) H 2 0 (b) S03 (c) NH3 (d) ROR.

58. An aromatic compound among other things should have a 7T-electron cloud containing (4n + 2)ji electrons where n can't be (a) 1/2 (b) 3 (c) 2 (d) 1.

59. Glycerol is an alcohol which can be classified as (a) trihydric (b) monohydric (c) dihydric (d) hexahydric.

60. The oxidation number of cobalt in K[Co(CO)4] is (a) +1 (b) +3 (c) - 1 (d) - 3 .

61. Bromination of alkanes involves (a) carbanions (b) carbocations (c) .carbenes (d) free radicals.

62. Methylphenyl ether can be obtained by reacting (a) phenolate ions and methyl iodide (b) methoxide ions and bromobenzene (c) methanol and phenol (d) bromo benzene and methyl bromide.

63. Which is not correct? (a) Phenol is more acidic than acetic acid. (b) Ethanol is less acidic than phenol. (c) Ethanol has lower boiling point than ethane. (d) Ethyne is a non-linear molecule.

64. Ascorbic acid is also known as (a) vitamin A (b) vitamin B (c) vitamin C (d) vitamin D.

65. Reaction of phenol with chloroform/sodium hydroxide to give o-hydroxy benzaldehyde involves the formation of (a) dichloro carbene (b) trichloro carbene (c) chlorine atoms (d) chlorine molecules.

66. One of the following that cannot undergo dehydro-halogenation is

CHEMISTRY TODAY | SEPTEMBER '05 36

(a) iso-propyl bromide (b) ethanol (c) ethyl bromide (d) none of these.

67. Which requires catalyst? (a) S + 0 2 S0 2 (b) 2S0 2 + 0 2 2S0 3

(c) C + 0 2 -> C0 2 (d) all of these.

68. Acetic acid will be obtained on oxidation of (a) ethanol (b) propanal (c) methanal (d) glyoxal.

69. A carboxylic acid is converted into its anhydride using (a) thionyl chloride (b) sulphur chloride (c) sulphuric acid (d) phosphorus pentoxide.

70. Which is false? (a) Glucose is a disaccharide. (b) Starch is a polysaccharide. (c) Glucose and fructose are not anomers. (d) Invert sugar consists of glucose and fructose.

71. What kind of isomerism is possible for 1-chloro-2-nitroethene? (a) functional group isomerism (b) position isomerism (c) E/Z isomerism (d) optical isomerism.

72. KCN reacts readily to give a cyanide with (a) ethyl alcohol (b) ethyl bromide (c) bromobenzene (d) chlorobenzene.

73. Peptides are formed from (a) aliphatic amines (b) carbohydrates (c) a-amino acids (d) aromatic amines.

74. Calcium carbide on reaction with water yields (a) methane (b) ethane (c) ethene (d) ethyne.

75. The correct set of the four quantum numbers of a 4d electron is (a) 4, 2, 1 , - 1 / 2 (b) 4, 2, 1 ,0 (c) 4, 3, 2, +1/2 (d) 4, 3, -2 , +1/2.

1. (b) : To overcome the objections of Rutherford's model and to explain the hydrogen spectrum, Bohr proposed a quantum mechanical model of the atom, which is based on the quantum theory of radiation and the classical laws of physics.

2. ( a ) : The normal boiling point is the temperature

at which the vapour pressure of a liquid is equal to exactly one standard atmospheric pressure (760 mm of Hg). Thus, at high altitudes where the atmospheric pressure is less than 760 mm, water boils at temperatures below its normal boiling point of 100°C.

3. (c) : Frenkel defect is caused if some of the ions of the lattice occupy interstitial sites leaving a corresponding number of normal lattice sites vacant.

4. (c) : According to third law of thermodynamics, "every substance has a finite positive entropy but at the absolute zero temperature the entropy may become zero and does so become in the case of perfectly crystalline solids".

5. (b) : The free energy of a reaction is the chemical analogue of potential energy of mechanical systems. A body moves in the direction in which there is a decrease in potential energy. Similarly, in a chemical system, the substance moves in a direction in which there is decrease in free energy, i.e. AG is negative. In a chemical reaction, AG = Gproducls - Greactants. Thus spontaneous changes occur with a decrease in free energy, i.e. AG is negative.

6. (d) : CH3-CO-CH3

Ketones are named by adding the suffix '-one' in place o f ' - e ' of alkane. Thus I.U.P.A.C. name is propanone.

7. (c) : The enthalpy of combustion i.e. AH is always negative. It means combustion is an exothermic reaction.

8. (d) : If a process is carried out under such condition that no exchange of heat takes place between the systems and surroundings, the process is termed adiabatic. The system is thermally isolated, i.e. dQ = 0 and the temperature of the system varies.

9. (a) : The substances whose aqueous solutions allow the passage of electric current and are chemically decomposed, are termed electrolytes. Electrolytic substances are classified as strong or weak according to how readily they dissociate into conducting ions. Acetic acid is a weak electrolyte. Glucose, ethanol and urea are non-electrolytes.

10. (d) : Conductance = — r -resistance

= 4 - = o h m " or mho ohm

11. (c) : The magnitude of the electrode potential of a metal is a measure of its relative tendency to lose or

ANSWERS

18 CHEMISTRY TODAY | SEPTEMBER '05 37

gain electrons, i.e. it is a measure of the relative tendency to undergo oxidation (loss of electrons) or reduction (gain of electrons).

M —» M"+ + ne~ (oxidation potential) M"+ + ne~ —> M (reduction potential)

12. (a): Any redox reaction would occur spontaneously if the free energy change (AG) is negative.

AG° = - nFE° where n is the number of electrons involved, F is the value of Faraday and E° is the cell emf. AG° can be negative if E° is positive.

13. (c) : On increasing the concentration of reactant by 4 times, rate of reaction doubles, i.e. order = 1/2. 14. (b) : Fog is an example of aerosol where the dispersed phase is liquid and dispersion medium is gas. 15. (b) : Alum is added to muddy water so as to destroy the bacteria as well as to coagulate the suspended impurities. 16. (a) : The catalytic process in which the reactants and the catalyst are in different phases is known as heterogenous catalysis.

N2 0 t ) + 3 H 2 t e ) - ^ - * 2 N H 3 t e )

The reactants are in gaseous state while the catalyst is in solid state. 17. (b): When the temperature is increased, heat energy is supplied which increases the kinetic energy of the reacting molecules. This will increase the number of collisions and ultimately the rate of reaction will be enhanced.

18. (b) : [OH"] = 1 x 10"5

pOH = - log[OH"] = 5 pH + pOH = 1 4 => pH = 14 - 5 = 9.

19. (d) : LiAlH4 is a nucleophilic and capable of donating electron pair, thus acts as a Lewis base. 20. (a ) : When the ionic product exceeds the solubility product, the equilibrium shifts towards left hand side, i.e. increasing the concentration of undissociated molecules of the electrolyte. As the solvent can hold a fixed amount of electrolyte at a definite temperature, the excess of the electrolyte is thrown out from the solution as precipitate. Thus, for the precipitation of an electrolyte, it is necessary that the ionic product must exceed its solubility product.

21. (b) : In a group, electron affinity decreases as the size increases.

18

F CI Br I EA (eV) -3.6 -3 .8 -3 .5 -3 .2 22. (b): The Ca2+ ions are arranged in (ccp) arrangement, i.e. Ca2+ ions are present at all corners and at the centre of each face of the cube. The fluoride ions occupy all the tetrahedral sites. This is 8 : 4 arrangement i.e, each Ca2+ ion is surrounded by 8 F~ ions and each F~ ion by four Ca2+ ions.

23. (b) : Total number of orbitals in a principal or main energy shell is equal to n2. 24. (a, c) : PC13, NH3 pyramidal

CH4, CC14 tetrahedral

25. (a) : dsp3 or sp3d : one s + three p + one d (da) 26. (c) : Half-life period is defined as the time during which half the amount of a given sample of the radioactive substance disintegrates. This time period does not depend upon the initial amount of the radioactive substance.

27. (b) : Loss of beta particle is equivalent to decrease of one neutron only.

n —> p + e~ + v

28. (d) : According to Raoult's law, the relative lowering in vapour pressure of a dilute solution is equal to mole fraction of the solute present in the solution.

29. (d) : Chlorine atom is ortho-para directing group. 30. (c) : Zn + H 2 0 ZnO + H2

Zn + 2NaOH Na2Zn02 + H2

Zn + 2HC1 -> ZnCl2 + H2

Zn + 2H2S04 ZnS04 + S0 2 + 2H20. 31. (c) : Suppose the oxidation state of Xe is x. XeOF2

x + (-2) + 2(—1) = 0 =» x - 2 — 2 = 0 => JC - 4 = 0 => x = +4.

32. (d) : The bond strength H - X decreases from HF to HI. Thus, HF is most stable while HI is least stable. The decrease in stability is due to decrease in electronegativity from F to I. This is also observed in the values of dissociation energy of H - X bond.

H - F H - CI H - Br H - I Dissociation energy 1 3 6 1 0 5 8 6 7 0

(kcal mol"1)

HF and HCl are stable upto 1200°C, HBr dissociates slightly and HI dissociates considerably (20%) at 440°C. 33. (a) : Cast iron : iron - 93-95%

carbon - 2.5-5%, impurities about 2%.

CHEMISTRY TODAY | SEPTEMBER '05 38

34. (b) : Ce - lanthanide, Cs - alkali metal, Cf - actinide, Ca - alkaline earth metal.

35. (d) : In the formation of XeF4, sp2cP hybridisation occurs which gives t h e ^ molecule an octahedral structure. The xenon and four fluorine atoms are co-planar while the two equatorial positions are occupied by the two lone pairs of electrons.

36. (a) : Cr03 + 2NaOH Na2Cr04 + H 2 0 yellow solution

37. (a) : H3P04 ^ H+ + H2P04-H2PO4~ ^ H+ + HP04

2" HP04

2" ^ H+ + P043"

H 3P0 4 is a tribasic acid due to presence of three replaceable hydrogen atoms. 38. (d)

39. (b) : In PC13, P atom undergoes sp3 hybridisation. In the tetrahedral configuration, one of the position is occupied by a lone pair. Thus it is pyramidal in shape.

40. (a) : It doesn't provide free Br" ions to react with AgN03 . 41. (b) : Stability of carbonium ions: tertiary alky 1 > secondary alkyl > primary alkyl > methyl. More the number of alkyl groups, the greater the dispersal of positive charge and therefore, more the stability of carbonium ion is observed.

triphenylmethyl cation Dispersal of positive charge increases with the increase in the number of benzene ring.

42. (b) : The s-character of C - H bond of acetylene is higher in comparison to C - H bond of ethene and ethane. The electrons of the C - H bond in acetylene are strongly held by carbon nuclei. This facilitates the removal of hydrogen as proton.

43. (d) : The order of reactivity of acid derivatives towards different reactions decreases in the order:

RCOC1 > (RC0) 20 > RCOOR' > RCONH2

In other words, the reactivity decreases as the basicity of the leaving group increases i.e.

C P < RCOCT < RO- < NH 2 -

CH3

U 44. (b) : Br - C — CI * asymmetric or chiral carbon

H All the four valencies of carbon are satisfied with different atoms/substituents. 45. (a) : Atomic number of deuterium = 1

]D2 W

46. (c) : Picric acid is 2,4,6-trinitrophenol. 47. (a ) : C2H5ONa is not an organometallic compound as there is no carbon-carbon metal bond.

48. (a) : As we go from B —¥ C —¥ O, EA increases due to decreasing size and increasing nuclear charge. The EA of N is however, zero because of the extra stability of the exactly half-filled 2p orbitals. Thus, the overall sequence is B < C < O > N.

49. (b) : A Zeigler-Natta catalyst is composed of at least two parts: a transition metal component and a main group metal alkyl compound. The transition metal component is usually either titanium or vanadium. The main group metal alkyl compound is usually an aluminium alkyl. 50. (d) : All the given species undergo nucleophilic substitution reaction. This reactivity can be explained in terms of the nature of C - X bond which is highly polarised covalent bond due to large difference in the electronegativities of carbon and halogen atoms.

8+ 8-- C - X

'"CH, 'CH3 1= I 3 I 2° 1°

51. (b) : C H 3 - C H - C - C H , - C H , 3 0 4 o |

CI 3-chloro-2,3-dimethylpentane

52. (d) : Brass - Cu = 60-80%, Zn = 40-20% Bronze - Cu = 75-90%, Sn = 25-10% German silver - Cu = 56%, Zn = 24%, Ni = 20% Common metal is copper.

53. (c): (NH4)2Cr207 2K2Cr04 + Cr203 + 1 0 2

18 CHEMISTRY TODAY | SEPTEMBER '05 39

54. (a) :

o, Fe

K

ferrocene

(C2H5)4Sn tetraethyl tin

dibenzene chromium

55. (d) : Malachite : CuC03-Cu(0H),_ 56. (a ) : Any electron withdrawing substituent (having -I-effect) stabilises the anion by dispersing the negative charge and therefore, increases the acidity. Chlorine is an electron withdrawing group.

57. (b) : H - O - H , :NH3 , R - O - R - nucleophiles S03 has electron deficient centre (a reagent which can accept an electron pair in a reaction, is called an electrophile). 58. (a) : According to Huckel rule, all aromatic compounds must have (4n + 2)n electrons where n is an integer, i.e. n = 0, 1, 2, 3,. . . and possesses unusual stability due to the complete delocalisation ofm-electrons. 59. (a) : Glycerol (CH2OH - CHOH - CH2OH) is a trihydric alcohol, which may be considered as derivative of propane, obtained by replacement of three hydrogen atoms from different carbon atoms by three hydroxyl groups. 60. (c) : Let the oxidation number of Co is x.

1 + x + 0 = 0 => x = -l. 61. (d): Halogenation of alkanes takes place in presence of light (sunlight or UV) or at elevated temperature via free radical. 62. (a) : C6H50" + C2H5I C6H5OC2H5 + I" 63. (a) : Phenols are much more acidic than alcohols but less so than carboxylic acids or even carbonic acid. This is indicated by the values of ionisation constants. The relative acidity follows the order:

18

Ka 10-5 > IO"7 > RCOOH H2C03

HO

64. (c) :

IO"10 > C6HsOH

1 0 - 1 4

HOH J 0-18

ROH

Ascorbic acid (vitamin C)

65. (a) : This is Reimer-Tiemann reaction where the electrophile is dichlorocarbene (: CC12) generated from chloroform by the action of a base. OH" + CHC13 — HOH + : CClf —> CI" + : CC12

66. (b): Ethanol cannot undergo dehydrohalogenation. 67. (b) : The reaction 2S0 2 + 0 2 2S0 3 requires V,0 5 or platinised asbestos as a catalyst in contact process. 68. (a) : C2H5OH

RCOOH 69. (d) :

[O] > CH,COOH

RCO - > / O + H 2 0

RCOOH RCO' P205 acts as a dehydrating agent. 70. (a ) : Glucose is a monosaccharide having chemical composition C6H1206. 71. (c) : 0 2 N H C = CHC1

1 -chloro-2-nitroethene

For highly substituted alkenes E and Z system of nomenclature is used which is based on a priority system developed by Cahn, Ingold and Prelog. 72. (b) : C2H5Br + KCN a l c o h o l> C2H5CN + KBr 73. (c) : Peptides are formed by condensation of a-amino acids.

H2N-CH,COOH + H 2 N - C H 2 - C 0 0 H

H2N - CH, - C O M ! - CH2COOH peptide bond

V and so on.

74. (d) : CaC2 + H , 0 C2H2 + Ca(OH)2. ethyne

75. (a) : ; n = 4, / = 2, m = -2 , - 1 , 0, +1, +2

J = ± I > ± I , + I , ± i j ± I 2 2 2 ' 2 2

Questions sent by : Mohd. Abass Padder, Anantnag, J & K

CHEMISTRY TODAY | SEPTEMBER '05 40

SOLVED PAPER

BIHAR C EC E (MAINS 1. One mole each of H 2 0 , CO, H2 and C0 2 are mixed in a 10 litre flask and heated to 1476 K. 20% of water reacts with CO according to the equation:

H 2 O w + C O w / H 2 (G) + C O ,

Calculate the equilibrium constant of the reaction.

2. What is the per cent dissociation of acetic acid in a solution of 0.5 M acetic acid and 0.1 N HCl? [Ka for CH3COOH is 1.8 x 10"5].

3. 1.355 g of a substance dissolved in 55 g of CH3COOH produced a depression in the freezing point of 0.618°C. Calculate the molecular weight of the substance. [K,= 3.85],

4. How long has a current of 3 amperes to be passed through a solution of silver nitrate to coat a metal surface of 80 cm2 with a 0.005 mm thick layer? Density of Ag is 10.5 g cm"3. Atomic weight of Ag = 108.

5. Name the following complex salts: (a) K2[Ni(CN)4] (b) [Co(NH3)3(NO,)3] (c) [Pt(NH3)6]Cl4 (d) [Cr(H,0)3(NH3)3]Cl3

(e) Ni(CO)4.

6. On the basis of MO theory, explain why: (a) bond order of 0 2

+ is greater than that of O, (b) He2

+ is paramagnetic in nature

7. A 0.62 g .sample of N a 2 C 0 3 - H 2 0 is added to 100 ml of 0.1 N H,S04 . Will the resulting solution be acidic, basic or neutral? [At. weight of Na = 23, S = 32],

8. Write balanced equation for the following. (a) P4 + concentrated H N 0 3 —> (b) KMn0 4 + H 2 0 , (in acidic medium) —> (c) A1 + NaOH (aqueous) —> (d) Zn + dilute HN0 3

(e) H 2 S0 4 + HI - »

9. The reaction 2N,0 5 —» 4N0 2 + 0 2 is carried out at constant volume. The concentration o f N 0 2 increases by 1.6 x 10-2 mole in 4 seconds. Calculate the rate of appearance of O,.

10. The standard heats of formation at 298 K for

CC14 H 2 0 (k), C 0 2 (j;) and HCl fe) are -25.5 , -57 .8 , -94.1 and -22.1 kcal/mole respectively. Calculate AH°29S

for the reaction CC14 + 2H 2 0 w - » CO, fe) + 4HC1 (K).

11. Complete the following equations: (C ,H 5 ) ,S0 4

(a) Phenol

( b ) Benzo i c acid

NaOH 9

HCN, HCl PhNHNH, ^ - > 9 9

' A1C1,

> C 6 H s C O C l NH

?

12. An unknown compound with molecular formula C5H l 0O does not reduce Fehling's solution but forms a bisulphite addition compound. It gives a positive iodoform test. What are its possible structures?

13. Electromagnetic radiation of X = 242 nm is just sufficient to ionize Na atom. Calculate the ionization energy of Na atom in kJ/mole.

c = 3 x 108 ms"1, h = 6.626 x IO"34 Js.

14. 0.50 g of an organic compound in Kjeldahl's method gave NH3, which was absorbed in 50 ml of 1 N H2S04 . The neutralisation of remaining acid required 60 ml of 0.5 N NaOH. Find the % N in the compound.

15. Give the name of the monomers of the following, (a) Natural rubber (b) Polypropylene (c) Teflon (d) Neoprene (e) Plexiglass.

16. Write chemical equations for (a) Leaching of bauxite to obtain A1203. (b) Roasting of copper pyrites to obtain Cu 2 0.

17. ^ T h disintegrates to give Pb . How many a and P particles are emitted?

18. Classify the following as Lewis acids and Lewis bases. (a) CH3NH, (b) CN" (c) AICI3 (d) Fe3+

(e) CO,.

19. Convert (in two steps only).

36 CHEMISTRY TODAY | OCTOBER '05

(a) Methane to ethane (b) Ethane to ethylene (c) Acetylene to acetaldehyde (d) Acetylene to but-2-yne. (e) Acetic acid to methane.

20. The ionization constant of NH4+ in water is

5.6 x 10"10. The rate constant for the reaction of NH4+ and

OH" to form NH3 and H 2 0 is 3.4 x IO10 I mol"1 sec"1. Calculate the rate constant for proton transfer from H 2 0 to NH3. [K

W = 1014].

SOLUTIONS

1. H 2 0 ( , ) + CO ( g ) — H

2 (g) + C 0

2 (g) Ini t ial conc . 1 1 1 1 A t eqm. (1 - 0 .2) (1 - 0 .2) (1 + 0.2) (1 + 0.2)

[ H 2 ] [ C Q 2 ] 1 .2X1.2 _ 2 , 5

[ H 2 0 ] [ C 0 ] 0 . 8 x 0 . 8

2. CH3COOH ^ CH3COO" + H+

0.5 0 0 0.5 - x x x

HCl H+ + Cl" 0.1 0.1 0.1

x0c + 0T) = L 8 x 1 0 _ 5

( 0 . 5 - x ) • x < < < 1, the value of x can be negligible.

= ^ = 1.8x10-*

• 1.8x10~5 x 0.5 _ g x j Q-S 0.1

% dissociation = 9 x 10"5 x 100 = 9 x 10"3.

3. 1000 x K j - x w

WxA T 1000x3.85xl.355

55x0.618 = 153.47

4. Mass of silver to be deposited = volume x density = area x thickness x density = 80 x 0.0005 x 10.5 = 0.42 g.

Applying to silver, £ = Z * 96500. 108

Z = - g 96500

W = Z-i-t i.e. 0.42 = x3 xt

t = -96500x0.42 108x3

108 96500

= 125.09 seconds = 2.08 minutes.

5. (a) K2[Ni(CN)4] po tas s ium te t racyanon icke la te ( I I )

(b) [Co(NH3)3(N02)3] t r i ammine t r in i t ro cobal t (III)

(c) [Pt(NH3)6]Cl4

H e x a a m m i n e p l a t i n u m ( I V ) chlor ide

(d) [Cr(H20)3(NH3)3]Cl3

Tr iammine t r i aquochromium(I I I ) ch lor ide

(e) Ni(CO)4

Tetracarbonylnickel(O)

6. (a) 0 2 : KK (ct2.v)2 ( g*2 s f ( o 2 p x f ( n 2 P y f (n2Pzf (n*2pyY (71*2/7,)'

8 - 4 4 „ B . O = — = r 2

0 2+ : KK (o2s)2 (a*2s)2 (o2Px)2 (n2Py)2 (n2p,)2 (n*2Py?

= l = Bond order of 0 2

+ is greater due to absence of one electron in antibonding molecular orbital. (b) He2

+ : o( ls ) 2 a* (Is)1

2 - 1 _ 1 2 ~ 2

Due to presence of one unpaired electron, He2+ is

paramagnetic in nature.

B.O. =

0.62 62

x1000 = 10 7. Meq. of Na 2C0 3 H 2 0

Meq. of H 2S0 4 = 100 x 0.1 = 10 Na 2C0 3 + H 2S0 4 Na 2S0 4 + H , 0 + C 0 2

Meq. added 10 10 0 0 0 Meq . le f t 0 0 10

10 10 10

N, Na,S04 100 = 0 . 1

Solu t ion becomes neutral s ince bo th acid and base are used up

and N a 2 S 0 4 does no t show hydro lys i s .

8. (a) P4 + conc. HN0 3

[2HN03 -> H 2 0 + 2N0 2 + O] x IO P4 + 100 + 6H2Q 4H3PQ4

P4 + 20HN03 4H 3P0 4 + 20N0 2 + 4H 2 0 (b) KMn0 4 + H 2 0 2 (in acidic medium) —> 2KMn04 + 3H2S04 K2S04 + 2MnS04 + 3H,0 + 5 0

5H2Q2+ 5 0 -> 5H2Q + 5Q2

2KMn04 + 3H2S04 + 5H 20 2 A K 2S0 4 + 2MnS0 4

+ 8H 2 0 + 50 2

(c) A1 + NaOH (aqueous) —>

2A1 + 2NaOH + 2H,0 -> 2NaA10, + 3H, T I I2 2 2

sodium meta-aluminate

solution

CHEMISTRY TODAY | OCTOBER '05 37

f

(d) Zn + dilute HN0 3

Zinc reacts with very dilute nitric acid (6%) to form ammonium nitrate.

[Zn + 2HN0 3 Zn(N03)2 + 2H] x 4 H N 0 3 + 8H NH3 + 3 H , 0 NH3 + HNQ3 -> NH4NQ3

4Zn + 10HN03 4Zn(N03)2 + NH 4 N0 3 + 3HzO very dilute

Zn reacts with dilute HN0 3 (20%) to form nitrous oxide (N 20).

[Zn + 2HN0 3 Zn(N03)2 + 2H] x 4 2HNQ3 + 8H - » N2Q + 5H2Q 4Zn + 10HN03 4Zn(N03)2 + N 2 0 + 5H 2 0

dilute (e) H 2 S0 4 + HI

H2SQ4 + 2HI -> SP 2 + I2 + 2H2Q

9. 2N 20 5 -> 4N0 2 + 0 2

1 4 N 2 Q 5 ] 1 d[ NQ2 ] d[ o 2 ] Rate = —— : = — ; = :

2 dt 4 dt dt

= 0.1x10" Rate = —x-4 4

= l x ] 0~3 mol litre-1 sec"1. Rate of appearance of 0 2 :

d[ P 2 ] dt

= rate of reaction = 1 x 10 3 mol. litre 'sec

10. AH°=AH f -AH, ' (products) ' (reactants)

A H f (C02) + 4 A H ° f (HO)] - [ ^ f (CC14) + (H20)

= [-94.1 + 4(-22.1)] - [-25.5 + 2(-57.8)] = -94 .1 - 88.4 + 25.5 + 115.6 = -182.5 + 141.1 = - 4 1 . 4 kcal.

(C2H5)2SO„

NaOH HCN, H C l .

A1C1,

C H = N H CHO CH—NNHPh

(b) C6H5COOH C6H5C0C1 c6H5CONH,

12. Compound C5H10O forms a bisulphite addition compound. It means it has a carbonyl group. As it does not reduce Fehling's solution i.e., it is a

ketone. It undergoes iodoform test, means it must be a methylketone.

O

CH3CCH2CH0CH3 O

(a) CH3CCH2CH2CH3 + NaHSO. CH 3 V .OH

/ C < CH3CH2CH2 S03Na

O II

(b) CH3CCH2CH2CH3 + Ag20 — > no reaction O II

(c) CH3CCH2CH2CH3 + 3I2 + NaOH > CHI3 + CH3CH2CH2COONa

Hence the given compound is pentanone-2.

13. X = 242 nm = 242 x 10~9 m c = 3 x 108 ms"1

E = ho- hc _ 6.626X10"34X3X10;

X 242 xlO"9

= 0.082 x IO""17 J = 0.082 x IO"20 kJ. Energy per mole for ionisation

= 0.082 x io- 2 0 x 6 .02 x 1023

= 493.6 kJ mol-'.

14. Mass of organic compound = 0.50 g Unused acid required = 60 ml of N/2 NaOH 60 ml N/2 NaOH = 60'ml N/2 H 2 S0 4

= 30 ml N H 2 S0 4

Acid used for absorption of ammonia = (50 - 30) = 20 ml N H 2 S0 4

0/ f -t 1.4 x TV] x F 1.4x1x20 % of nitrogen = ttt = — = 36%.

W 0.50

15. (a) Natural rubber :

Monomer - isoprene

(b) Polypropylene :

Monomer - propylene

(c) Teflon: Monomer - tetrafluoroethene CF2 = CF2

(d) Neoprene :

CH3 I 3

C H , = C H - C = C H 2

2-methyl-1,3 -butad iene

CH3 — CH — CH, propene-1

CI I

Monomer - chloroprene C H 2 = CH - C = CH2

-'38 CHEMISTRY TODAY | OCTOBER '05

(e) Plexiglass : Monomer - methylmethylacrylate

C H , I 3

C H 2 = C - C O O C H 3

16. (a) Bauxite ore contains ferric oxide, titanium oxide and silica as impurities. When the powdered ore is digested with an aqueous solution of sodium hydroxide at about 150°C under pressure, the alumina dissolves forming soluble sodium meta-aluminate while ferric oxide (Fe203), Ti02 and silica remain as insoluble part.

A1203 + 2NaOH -> 2NaA102 + H 2 0 NaA102 + 2H 2 0 Al(OH)3 + NaOH

2Al(OH)3 igni ted A1203 + 3H20

a u t o c l a v e

(b) The concentrated ore is heated strongly in a current of air on the hearth of the reverberatory furnace. During roasting the following changes take place. (i) S + 0 2 -> S0 2

(ii) 4As + 30 2 2As203

4Sb + 3 0 2 2Sb203

(iii) 2CuFeS2 + 0 2 Cu2S + 2FeS + S0 2

(iv) 2FeS + 30 2 -> 2FeO + 2S0 2

2Cu2S + 3 0 2 2Cu 2 0 + 2S0 2

17. 234 T , 90 l h

206 82 Pb

28 Decrease in mass = 234 - 206 Mass of a-particle = 4 So number of a-particles emitted = 28/4 = 7 Number of (J-particles emitted = 2 x number of a-particles - (at. no. of pa ren t -a t . no. of end product)

= 2 * 7 - (90 - 82) = 14 - 8 = 6.

18. CH3NH2 - Lewis base (due to presence of lone pair of electrons)

CN _- Lewis acid (due to electron deficient nature) A1C13 - Lewis acid (due to electron deficient nature) Fe3+ - Lewis acid (due to electron deficient nature) C 0 2 - Lewis acid (due to presence of a multiple bond between atoms of dissimilar electronegativity)

19. (a) Methane to ethane

CH4

m e t h a n e

C I ,

uv CH3C1 W u r t z reac t ion

hea t wi th N a in e t h e r CH, - CH, 3 3

e t h a n e

(b) Ethane to ethylene

CH, - CH, CH,CH,Br alc- K 0 H > CH, = CH. 3 3 e t h a n e

to) 2 2 e thy lene

(c) Acetylene to acetaldehyde

CH = CH + 2CH3C00H H g" > CH3CH(OOCCH3)2 ace ty lene e thy l i dene ace t a t e

CH3CO s ^ >CH 3 CHO+ _ O

CH3CO-

(d) Acetylene to but-2-yne

CH = CH 2Na

liq. N H . NaC = CNa •

2CH,I

CH3C — CCH3

b u t - 2 - y n e

(e) Acetic acid to methane

CH3COOH N a 0 H > CH3COONa CH4

acet ic ac id m e t h a n e

20. NH3 + H 2 0 ^ = ^ N H 4 + O H " \ K h = 3.4x10 Kh

,10

- 1 0 N H j + H 2 0 NH4OH + H+; Ka = 5.6x10

K Kf

(base)NH3 K h ^ c . d ( N H : )

(^•acid x -^base = ^ w )

K f IO"14

m = i o . K f = 6.07 x 1 0 s 3.4x10 5.6x10 f

Questions sent by : Pooja Singh

CHEMISTRY TODAY

HOW TO CONTACT US Address: MTG Learning Media (P) Ltd., 406. Taj Apt., Ring Road, Near Safdarjung Hospital, New Delhi -110 029 Tel : (011) - 26191599, 26191601 e-mail: [email protected] website : www.mtg.in

LETTERS Send your comments / suggestions to us. Please include your address and phone number. Only selected letters will be published. Send to the address given above.

QUESTIONS FOR CHEMISTRY CLINIC We answer your chemistry questions in Chemistry Clinic. We regret that only letters considered for publication can be answered. Write to at the address given above, e-mail

CHEMISTRY TODAY ONLINE Visit our website at www.mtg.in

NEW SUBSCRIPTIONS To subscribe to Mathematics Today / Chemistry Today / Physics For You / Biology Today see Super Subscription Offer in the magazine. Or contact us at the address given above. Tel: (011 )-26191601, 26191599 website: www.mtg.in

SUBSCRIPTION ENQUIRIES Far subscription or delivery problems, or to report a change of address, please contact at the address given above. Tel: (0111-26191599, 26191601 e-mail: [email protected] website: www.mtg.in

-'38 C H E M I S T R Y T O D A Y | OCTOBER '05

SOLVE & SEND

Get All India R a n k Very Similar MODEL TEST PAPER for CBSE-AIEEE 2006

Time : 4 hrs. Maximum Marks : 675 Instructions: 1. This question paper contains 225 questions: 1-75 Physics, 76-150 Chemistry, 151-225 Mathematics 2. For every correct answer 3 marks will be credited to your account, 1 mark will be deducted for every wrong answer and no marks

will be awarded for unattempted questions. 3. Use of Trigonometric table, Calculator or any other helping device is restricted. 4. Useful data : At.wt./ Mol.wt. : K M n 0 4 : 158; Cu : 63.5; NaOH : 40; HCl : 36.5; Mg : 24; H : 1; He : 4; C : 12; N : 14; O : 16;

Na : 23; P : 31; S : 32; CI : 35.5; Ca : 40; Zn : 65; Ag : 108. Atomic No : H : 1; He : 2; Li : 3; Be : 4; B : 5, C : 6; N : 7; O : 8; F : 9; Na : 11; Mg : 12; A1 : 13; Si : 14; Fe : 26; Co : 27; Ni : 28; Cu : 29; Zn : 30; Rb : 37 C o n s t a n t s : g = 10 m/s2 , R = 8.3 J K - ' m o H or 0 .0821 atm li tre K 'mol - ' , e = 1.6 x 10"" C, N„ = 6 .023 * 1023, mp = 1.6 x 10-27 kg, me = 9.11 * 10"31 kg, h - 6.6 x IO"34 Js.

PHYS ICS

1. The dimensions of permittivity (e0) of vacuum are (a) M-'L-3T4I2 (b) ML"3T2I2

(c) M"'L3T4I2 (d) ML3T2I2

2. IfP represents radiation pressure, C represents speed of light and Q represents radiation energy striking a unit area per second, then the non-zero integers x,y and z such that PxQyCz is dimensionless are (a) x= ],y= \,z= 1 (b) * = 1, y = -1, z = 1 (c) x = -l,y= 1 , 2 = 1 (d) x = l,y= l , z = - l .

3. Which of the following displacement-time (x - t) graphs shown in figure cannot possibly represent one-dimensional motion of a particle?

a parachutist bails out from it. After 3 s he opens his parachute and decelerates at a constant rate of 5 ms"J. What was the height of the parachutist above the ground when he opened his parachute? Talce g = 10 ms~2. (a) 15 m (c) 45 m

5. In vector diagram shown in figure, R is the rpsultapt of vectors A and 5 . If tf = B/Jl,

the value of angle 9 is (a) 30° (b) 45° (c) 60°

6. A null vector has (a) zero magnitude and a specific direction (b) a finite magnitude and no specific direction (c) a finite magnitude and no specific direction (d) zero magnitude and no specific direction.

7. The maximum height attained by a projectile is increased by 10% by changing the angle of projection, without changing the speed of projection. The percentage increase in the time of flight will be (a) 20% (b) 15% (c) 10% (d) 5%.

8. A car, moving at a speed of 54 km h~', is to go round a curved road of radius 30 m. If the curved road is not

(b) 30 m (d) 60 m.

B /

9 0 ° r

A

(d) 75°.

18 CHEMISTRY TODAY | SEPTEMBER '05 14

banked, what must be the coefficient of friction between the tyres and the road for the car to negotiate the curve? Take g = 10 ms~2. (a) zero (b) 0.25 (c) 0.50 (d) 0.75

9. Two masses m and 2m are joined to each other by means of a frictionless pulley as shown in figure. When the mass 2m is released, the mass m will ascend with an acceleration of (a) g/3 (b) g/2

r \

(C) g

F=2mg (d) 2g.

10. A block of mass M is lying on a horizontal frictionless surface. One end of a rope mass m is fixed to the block and a force F is applied at the free end parallel to the surface. The force acting on the block will be

FM (a) F (b)

(c) Fm

(M + m) (d)

(M + m) Fm

(M - m)

11. An engine pulls a car of mass 1500 kg on a level road at a constant speed of 5 ms~'. If the frictional force is 500 N, what power does the engine generate? (a) 5.0 kW (b) (c) 10 kW (d)

12. A particle of mass 0.1 kg is subjected to a force which varies with distance as shown in figure. If it starts its journey from rest at x = 0, its velocity at x = 12 m is (a) zero (b) (c) 20^3 m/s (d)

7.5 kW 12.5 kW.

F (N)

x ( m ) •

20V2 m/s 40 m/s.

13. A shell explodes into three fragments of equal masses. Two fragments fly off at right angles to each other with speeds of 9 ms-1 and 12 ms-1. What is the speed of the third fragment? (a) 9 ms"1 (b) 12 ms"1

(c) 15 ms"1 (d) 18 ms-1.

14. A rubber ball is dropped from a height of 5 m on a

planet where the acceleration due to gravity is not known. On bouncing it rises to 1.8 m. The ball loses its velocity on bouncing a factor of (a) 16/25 (b) 2/5 (c) 3/5 (d) 9/2.

15. A circular disc is rolling down an inclined plane without slipping. If the angle of inclination is 30°, the acceleration of the disc down the inclined plane is (a) g

(c) g/3

16. A thin uniform metallic triangular sheet of mass A/has sides AB = BC - L. What is its moment of inertia about axis AC lying in the plane of the sheet?

(b) g/2 V2 (d) T g.

IK Axis of j v rotation

i l k , I C'*

(a)

(b)

(d)

MU_ 12

Ml}

2ML

(c) ML

17. If both the mass and the radius of earth decrease by 1%, the value of the acceleration due to gravity will (a) decrease by 1% (b) increase by 1% (c) increase by 2% (d) remain unchanged.

18. A satellite of mass m is orbiting the earth at a height h from its surface. If M is the mass of the earth and R its radius, the kinetic energy of the satellite is

GmM „ GmM (a) 7~ . ( b )

(c)

(R + hy GmM (.R + h) (d)

2(R+hy GmM

2 (R + h)

19. The normal density of gold is p and its bulk modulus is K. The increase in density of a lump of gold when a pressure P is applied uniformly on all sides is (a) pP/K (b) pK/P (c) P/pK (d) K/pP.

20. A steel wire of cross-sectional area 3 x 10"6 m2 can withstand a maximum strain of 10"3. Young's modulus of steel is 2 x i o " Nm~2. The maximum mass of the wire can hold is (take g = 10 ms-2). (a) 40 kg (b) 60 kg (c) 80 kg (d) 100 kg.

-'38 CHEMISTRY TODAY | OCTOBER '05

21. A cube of ice is floating in a liquid of relative density 1.25 contained in a beaker. When the ice melts, the level of the liquid in the beaker (a) rises (b) falls (c) remains unchanged (d) falls at first and then rises to the same height as

before.

22. Two blocks A and B are made of different kinds of wood. Block A floats in water with ] /4th of its above the surface of water. Block B floats in water with 2/3 rds of its volume below the surface of water. The ratio of densities of A and B is (a) 3 : 2 (b) 5 : 3 (c) 9 : 8 (d) 4 : 3.

25. A spring of force constant k is cut into three equal pieces. If these three pieces are connected in parallel, the force constant of the combination will be (a) k/3 (b) k/9 (c) 3k (d) 9k.

26. A small spherical steel ball is placed a little away from the centre of a large concave mirror of radius of curvature R = 2.5 m. The ball is then released. What is the time period of the motion? Neglect friction and take g = 10 ms-2. (a) n/4 sec (b) jt/2 sec (c) 7t sec (d) 2n sec

27. Transverse waves of the same frequency are generated in two steel wires A and B. The diameter of A is twice that of B and the tension in A is half that in B. The ratio of velocities of waves in A and B is

-'38

(a) 1 : 2 (c) 1 : 2V2

(b) 1 : V2

(d) 2V2

28. An ambulance blowing a siren of frequency 700 Hz is travelling slowly towards a vertical reflecting wall with a speed of 2 ms~'. The speed of sound is 350 ms-1. How many beats are heard per second? (a) 4 (b) 6 (c) 8 (d) 10.

29. When a metallic bar is heated from 0°C to 100°C, its length increases by 0.05%. What is the coefficient of linear expansion of the metal? (a) 5 x IO"3 °C~' (b) 5 x IO"4 °Cr[

(c) 5 x 10"5 °C-' (d) 5 x IO"6 °C-'.

30. When a solid metallic sphere is heated, the largest percentage increase occurs in its (a) diameter (b) surface area (c) volume (d) density.

31. The temperature coefficient of resistance of the material of a wire is 0.00125 per°C. Its resistance at 300 K is 1 ohm. At what temperature will the resistance of the wire be 2 ohm? (a) 1154 K (b) HOOK (c) 400 K (d) 1127 K.

32. Cooking vegetables and other food in a pressure cooker saves time and fuel because (a) under increased pressure, water can be made to boil

at a temperature much higher than 100°C (b) under increased pressure, water can be made to boil

at a temperature much lower than 100°C (c) heat losses are reduced to a minimum (d) condensation of steam is prevented.

33. Figure shows the P-V diagram for a Camot cycle. In this diagram (a)

A

(b)

(c) (d)

curve AB represents isothermal process and BC adiabatic process curve AB represents adiabatic process and BC. isothermal process curves CD and DA represent isothermal process curves CD and DA represent adiabatic process.

34. One mole of a monoatomic gas (y = 5/3) is mixed with one mole of a diatomic gas (y = 7/5). What will be the value of y for the mixture? (a) 1.5 (b) 1.54 (c) 1.4 (d) 1.45

CHEMISTRY TODAY | OCTOBER '05

24. Two spheres of equal masses but radii R and 2R are allowed to fall in a liquid. The ratio of their terminal velocities is (a) 1 : 4 (b) 1 : 2 (c) 1 : 32 (d) 2 : 1.

23. Which one of the graphs shown in figure represents the velocity-time (v-t) graph of a small spherical body falling in a viscous fluid?

35. If a gas .has / degrees of freedom, the ratio of C /C v of the gas is

(a) 1 + / (b) l + {

(c) (d) 1 + /

36. A vessel contains a mixture of 1 mole of oxygen and two moles of nitrogen at 300 K. The ratio of the rotational kinetic energy per 0 2 molecule to that per N2

molecule is (a) 1 : 1 (b) 1 : 2 (c) 2 : 1 (d) depends on the moment of inertia of the two

molecules.

37. If the temperature of a black body increases from 7°C to 287°C, then the rate of energy radiation increases by

(a)

(c) 4

(b) 16

(d) 2.

38. A black body is at a temperature of 2880 K. The energy of radiation emitted by this body between wavelengths 499 nm and 500 nrn is V\, between 999 nm and 1000 nm is /A and between 1499 nm and 1500 nm is U3. The Wien's constant b = 2.88 x 106 nm K. Then (a) ( 7 , = 0 (c) U\ > U2

(b) (d)

U3 = 0 U2>U].

39. Three point charges 4q, Q and q are placed in a straight line of length / at points distant 0, //2 and I respectively. The net force on charge q is zero. The value of Q is

(a) - q (b) - 2 q

40. A metall ic solid sphere is placed in a uniform electric field. In figure, which path will the lines of force follow? (a) 1 (b) 2

(d) 4q.

(C) 3 (d) 4.

41. If n drops, each of capacitance C and charged to a potential V, coalesce to form a big drop, the ratio of the energy stored in the big drop to that in each small drop will be (a) n : 1 (b) n m : 1

,5/3 . | M l - 2 (c) n (d) 1.

42. The plates of a parallel plate capacitor are charged to 100 V. A 2 mm thick plate is inserted between the plates. Then to maintain the same potential difference, the distance between the capacitor plates is increased by 1.6 mm. The dielectric constant of the plate is (a) 5 (b) 1.25 (c) 4 (d) 2.5

43. Three resistances of 4 Q each are connected as shown in figure. If the point D divides the resistance into two equal halves, the resistance between points A and D will be (a) 12 Q (b) 6 Q (c) 3 Q

4 a

(d) 1/3 Q.

44. A steady current flows in a metallic conductor of non-uniform cross-section. The quantity/quantities that remains/remain constant along the length of the conductor is/are (a) current, electric field and drift speed (b) drift speed only (c) current and drift speed (d) current only.

45. A constant voltage is applied between the two ends of a uniform metallic wire. Some heat is developed in it. The heat developed is doubled if (a) both the length and radius of the wire are halved (b) both the length and radius of the wire are doubled (c) the radius of the wire is doubled (d) the length of the wire is doubled.

46. A battery of emf E and internal resistance r is connected across a pure resistive device (such as an electric heater) of resistance R. The power output of the device will be maximum if (a) R = r (b) R = j2r (c) R = 2r (d) R = 4 r.

47. A conducting circular loop of radius r carries a constant i. It is placed in a uniform magnetic field B such that B is perpendicular to the plane of the loop. The magnetic force acting on the loop is (a) irB (b) 2 nirB (c) zero (d) nirB.

48. A current I flows along the length of an infinitely long, straight, thin walled pipe. Then (a) the magnetic field at all points inside the pipe is the

same but not zero

-'38 CHEMISTRY TODAY | OCTOBER '05

(b) the magnetic field at any point inside the pipe is zero (c) the magnetic field is zero only on the axis of the

pipe (d) the magnetic field is different at different points

inside the pipe.

49. A rectangular loop of sides 8 cm and 2 cm having resistance of 1.6 £2 is placed in a magnetic field of 0.3 T directed normal to the loop. The magnetic field is gradually reduced at the rate of 0.02 T s-1. How much power is dissipated by the loop as heat? (a) 1.6 x 10"10 W (b) 3 . 2 x l O - , 0 W (c) 6.4 x io -10 W (d) 12.8 x 10~'° W. 5 0" F i g u r e d ^ c n

shows two coils P and Q placed close to each other. When the circuit of coil P is suddenly broken by lifting the key K, (a) a current flows from X to Y in coil O (b) a current flows from Y to X in coil Q (c) no current flows in coil Q (d) an alternating current flows in coil Q.

51. A convex lens is placed between an object and a screen which are fixed distance apart. For one position of the lens the magnification of the image obtained on the screen is mv When the lens is moved by a distance d, the magnification of the image obtained on the same screen is m2. The focal length of the lens is (m] > m2)

d d ^ (

m ]- m

2) ( b )

w m r n s 0000000000 p 0

L p t L

{mx +m2)

( 0 ,m\

(d) d-

52. Monochromatic light is refracted from air into glass of refractive index The ratio of the wavelengths of the incident and refracted wave is (a) 1 : 1 (b) 1 : (X (c) \ i : 1 (d) tf : 1.

53. In Young's double slit experiment the distance d between the slits and S2 is 1.0 mm. What should the width of each slit be so as to obtain 10 maxima of the two slit interference pattern within the central maximum of the single slit diffraction pattern? (a) 0.1 mm (b) 0.2 mm (c) 0.3 mm (d) 0.4 mm.

54. The speed of a wave in a medium is 760 ms-1. If

3600 waves pass through a point in a medium in 2 minutes, its wavelength is

(a) 13.8 m (b) 25.3 m (c) 41.5 m (d) 57.2 m.

55. No photoelectrons are emitted from a metal if the wavelength of light exceeds 6000 A. The work function of the metal is approximately equal to (a) 3 x 10-,s J (b) 3 x 10"19 J (c) 3 x IO"20 J (d) 3 x 10-22 J.

56. Which energy state of doubly ionized lithium (Li^) has the same energy as that of the ground state of hydrogen? Given Z for lithium is 3. (a) n = 1 (b) n = 2 (c) n = 3 (d) n = 4.

57. Choose the wrong statement. A thermonuclear fusion reactor is better than a fission reactor for the following reasons: (a) for the same mass of substances involved, a fusion

reaction releases much more energy than a fission reaction

(b) a fusion reaction can be much more easily controlled than a fission reaction

(c) a fusion reaction produces almost no radioactive waste

(d) the fuel required for fusion is readily available in abundance from sea-water.

58. The radioactivity of an old sample of whisky due to tritium (half-life 12.5 years) was found to be only about 3% of that measured in a recently purchased bottle marked '7 years old'. The sample must have been prepared about (a) 70 years ago (b) 220 years ago (c) 420 years ago (d) 300 years ago.

59. If nc and nh respectively represent the electron and hole concentrations in a semiconductor, then it is (a) an n-type semiconductor if «,, >> nh

(b) a p-type semiconductor if ne < < nh

(c) a pure semiconductor if ne = nh

(d) a good conductor if nc = nh.

60. In figure the input is across the terminals A and C and the output is across B and D. Then the output is (a) zero (b) the same as the input (c) full wave rectified (d) half wave rectified.

-'38 CHEMISTRY TODAY | OCTOBER '05

61. Starting with the same initial conditions, an ideal gas expands from volume F, to F2 in three different ways. The work done by the gas is W{ if the process is purely isothermal, W2 if purely isobaric and if purely adiabatic. Then

(b) (d)

(a) W2> Wx> W.^ (c) Wx > W2 > W3

w2 > rv3 > w, iv l > w\ > w2

:R \ / R 1

\R J / r ^ J

c }

62. Eight equal resistances, each of magnitude R, are connected as shown in the figure. The resultant resistance between P and Q is (a) 4K/5 (b) 8/?/15 (c) 3/?/8 (d) 5R/Z

63. Periodic time of a particle executing SHM is T. 3T

After a time — from equilibrium position, the ratio

of potential and kinetic energies of the particle is (a) 3 : 1 (b) 1 : 3 (c) 2 : 1 (d) 1 : 2

64. The potential difference F of an electric field

E = (yi + x j ) is

(a) V = (x + y) + constant (b) V = ( x - y ) + constant (c) V = xy + constant (d) F = - xy + constant

65. Time-constant of an L-R circuit is T0. A current /0

flows through it. The source of current is switched off

f d') at time t = 0 I I when is represented by r. Assuming this rate to be constant, the current will reduce to zero in a time interval of (a) eT0 (b) TJe (c) T0 (d) e/T,

66. Water rises to a height h in a vertical capillary tube. The tube is inclined at 60° with the vertical. Length of water column will be (a) h (b) 2h (c) h sin 60° (d) h cos 60°

67. An object is placed at 15 cm from first focus of an equiconvex lens. Real image is formed at 60 cm from second focus of the lens. The focal length of the lens is (a) 75 cm (b) 60 cm (c) 45 cm (d) 30 cm

68. In a compound microscope fo= 1.5 cm and fe = 2.5 cm. Length of tube = 25 cm. The magnifying power when final image is formed at infinity is approximately equal to (a) 25 (b) 60 (c) 110 (d) 140

69. A particle is projected from earth's surface such that it escapes. Total mechanical energy of the particle will be (a) infinite (b) positive finite (c) negative finite (d) zero

70. A person covers first half of a linear journey with velocity Vj and the second half with velocity v2. The mean v is given by

(a) v, + v.

1 1 1 (c) - = - + —

V VI V-

(b)

(d)

2 1 1 — = — + — V v2

vi ~ v 2 Vj v2

71. The angular speed of earth at which a body lying on equator loses its weight is (Given g = 10 m/s2, Radius of earth = 6,400 km)

1 radian , , , 1 radian (a)

(c)

80 sec

1 radian

(b)

(d)

800 sec

1 radian

40 sec 400 sec 72. An engine accepts compressed steam at 127°C and rejects it at 47°C. Efficiency of the engine is (a) 40% (b) 30% (c) 20% (d) 10%

73. The colour of bright fringe nearest to the central achromatic fringe in the interference pattern with white light will be (a) violet (b) red (c) yellow (d) green

74. The maximum speed of a particle executing SHM is 10 m/s and maximum acceleration is 31.4 m/s2. Its periodic time is (a) 2 s (b) 4 s (c) 6 s (d) 1 s

75. The relation between magnetic susceptibility % and absolute temperature T, for a paramagnetic material, is (a) X ~ T (b) x « V77

1 ^ 1 ( d ) (c) X

-'38 CHEMISTRY TODAY | OCTOBER '05

CHEMISTRY

76. In a reaction, 4 mol of electrons are transferred to one mol of HN03. The possible product obtained due to reduction is (a) 0.5 mol o f N 2

(c) 1 mol of N0 2

(b) 0.5 mol of N20 (d) I m o l o f N H j

77. If 0.5 mol of BaCl2 is mixed with 0.2 mol ofNa3P04, the maximum number of mol of Ba3(P04)2 that can be formed is (a) 0.7 (b) 0.5 (c) 0.2 (d) 0.1

78. 100 mL of 1 M KMn04 oxidised 100 mL H202 in acidic medium. Volume of same KMn04 required in alkaline medium to oxidise 100 mL of H202 will be

100 , 500 T (a) mL (b) — mL

300 , (c) — mL (d) None.

79. Which combination is odd w.r.t. oxidation number per atom of the underlined? ( a ) H 2 S O S , H 2 S 2 0 8 , K 2 C R 2 0 7

(b) Cr03, Cr<V" SOf (c) both (d) none of these.

80. When 1 * 10~3 mol of the chloride of an element Y was completely hydrolysed, it was found that the resulting solution required 20 ml of 0.1 M aqueous silver nitrate for complete precipitation of the chloride ion. Element Y could be (a) aluminium (b) phosphorus (c) silicon (d) sulphur.

81. [Cr(NH3)5Br] CI and [Cr(NH3)5Cl] Br can be distinguished by X and isomerism shown is Y. X and Y are : (a) BaCl2, ionisation (b) AgN03, ionisation (c) AgN03, coordinate (d) BaCl2, linkage.

82. Which gives maximum number of ions due to ionisation? (a) PtCl4 • 6NH3 (b) PtCl4-5NH3

(c) PtCl4 • 4NH3 (d) equal. 83. Among the following species, identify the isostructural pairs : NF3, NOJ, BF3, H 30+ , HN3

(a) [NF3, NOJ] and [BF3, H 3 0 + ]

(b) [NF3, HN3] and [NOJ, BF3]

(c) [NF3, H 3 0 + ] and [NOJ, BF3]

(d) [NF3, H 3 0 + ] and [HN3, BF3]

84. In the mixture of (NaHC03 + Na2C03), vol. of HCl required is x mL with phenolphthalein indicator and y mL with methyl orange indicator in same titration. Hence volume of HCl for complete reaction of Na2C03 is (a) 2x (b) y

x ( 0 2 (d) ( y - x )

85. When 10 mL of ethyl alcohol (density = 0.7893 g/mL) is mixed with 20 ml of water (density 0.7893 g/mL) is mixed with 20 mL of water (density 0.9971 g/mL) at 25°C, the final solution has a density of 0.9571 g/mL. The percentage change in total volume on mixing is (a) 3.1% (b) 2.4% (c) 1% (d) none of these.

86. For the redox reaction,

Cr 2 0^"+H + +Ni->Cr 3 +

the correct coefficients of the reactants for the balanced reaction are :

+ N i 2 + + H 2 0

(a) (b) (c) (d)

Cr2Oy~ 1

2 1

3

Ni 3 3 1

3

H 14 14 16

12

87. 100 ml of H202 is oxidised by 100 mL of 1M KMn04

in acidic medium (Mn0 4 reduced to Mn2+) 100 mL of same H2Oz is oxidised by Fml of 1M KMn04 in basic medium (Mn0 4 reduced to MnO,). Hence V is (a) 500 (b) 100 (c) 100/3 (d) 500/3

88. N2+ 3H2 —> 2NH3

Molecular weight of NH3 and N2 are x, and x2 their equivalent weight are and y2. Then (yj -y2) is :

2X] (a)

(c) (3x] - x2)

(b)

(d)

(x, - x2)

(x, - 3x2)

89. A rad ioactive element X, decays by the sequence and with half lives, given below X [half Life = 30 min] k< > Y + a

Y [half Life = 20 days] — + 20 which of the following statements is correct ? (a) disintegration constant k2 > (b) atomic number of X and Z are same (c) the mass number of Y is greater than X (d) Y and Z are isotopes

46 CHEMISTRY TODAY | OCTOBER 05

90. The radiation is emitted when a hydrogen atom goes from a high energy state to a lower energy state. The wavelength of one line in visible region of atomic spectrum of hydrogen is 6.5 x 10_7m. Energy difference between the two states is : (a) 3.0 x IO"19 J (b) 1 .0x lO" l 8 J (c) 5.0 x 10~10 J (d) 6.5 x 10"7 J

91. In an experiment, 50 ml of 0.1 M solution of a metallic salt reacted exactly with 25 ml of 0.1 M solution of sodium sulphite. In the reaction SO2" is oxidised to SC>4~. If the original oxidation number of the metal in the salt was 3, what would be the new oxidation number of the metal? (a) 0 (b) 1 (c) 2 (d) 4

92. In centro-symmetric system, the orbital angular momentum, a measure of the momentum of a particular travelling around the nucleus, is quantised. Its magnitude is

(a)

(c) 77 (7+1)

h_ 271

Jl 2%

(b)

(d)

J n r T ) _

271

93. If n and / are respectively the principal and azimuthal quantum numbers, then the expression for calculating the total number of electrons in any energy level is

1 2 ( 2 / + 1) ( b ) /"i" ,2(2/ + l) (a)

(c)

/=1 /=//+]

I 2(2/+ 1) 1=0

(d)

1=1 /=»-1

I 2(2/+ 1) /=0

94. Gas o2 N2 NH3 CH4

a 1.360 1.390 4.170 2.253

The table indicates the values of van der Waal's constant a. (The unit of a is L2 atrn mol-2). The gas which can most easily be liquefied is (a) 0 2 (b) N2 (C) NH, (d) CH4

95. Mixture of volatile components A and B has total V.P. (P = 2 5 4 - 119x) {x = mol fraction of B) Hence pA° and pls° are (a) 135,254 (b) 119,254 (c) 254,135 (d) 135,119 96. 25 mL of an aqueous solution of KC1 was found to require 20 mL of 1 M AgN03 solution when titrated using a K.2Cr04 as indicator. Depression in freezing point of

KC1 with 100% ionisation will be : [Kf= 2.0 mol"' kg and molarity = molality]

(a) 5.0° (b) 3.2° (c) 1.6° (d) 0.8°

97. For the second order reaction

rate constant k is k = 1 1

1

7" (sec)

and half life period is 7— where ka

{ a - x ) is the concentration of reactant after time t, starting with initial concentration a (all concentration in mol L_i ar.i.; time in second). A graph between (a - x)_1 and time 1 is of the type. If OA = 50 mof 1 L and 9 = tan 1 (0.1) then half life period is (a) 500 sec (b) 6.93 sec (c) 50 sec (d) none.

98. Rate constant of a reaction, k is 3.0 x 10-4 s_1, energy of activation E„ = 104.4 kJ mol1 and Arrhenius constant

as T —¥ °° is (a) 2.0 x 1018 s"1

(c) infinite (b) 6.0 x 1014 s~' (d) 3.6 x io30 s"1

99. The resulting of iron-pipe exposed to atmosphere follows the reaction, 2Fe (s) + 30 2 (g) —> Fe203 (s). This is a (a) spontaneous reaction (b) non-spontaneous reaction (c) neither spontaneous reaction nor non-spontaneous

reaction (d) may be spontaneous or non-spontaneous depending

on conditions

100. The dissociation energy of CH4 and C2H6 are respectively 360 and 620 kcal/mol. The bond energy of C—C bond is (a) 260 kcal m o f (b) 180 kcal moM (c) 130 kcal mol"1 (d) 80 kcal mol 1

101. Given Na (s) s > Na (g), Na (g) —^

CI (g) + e~

~> Na+ (g) + e", Cl2 (g) 2C1 (g)

-> cr (g), Na+(g) + CI" (g) > NaCl (s) AHy of NaCl (s) is

(a) - U (b) S + J + D- E- U (c) S + 1+2D-E-U

2S + 2I + D-2(E + U)

-'38 CHEMISTRY TODAY | OCTOBER '05

102. Ky, of CaS04 is 4 x IO12. CaS04 is precipitated on mixing equal volumes of the following solutions (a) 3 x IO"6 M CaCl2 and 3 x 10"6 M (NH4), S0 4

(b) 4 x 10"6 M CaCl2 and 3 x IO"6 M (NH4)2 S04

(c) 6 x -10"6 M CaCl2 and 3 x 10"6 M (NH4)2 S04

(d) In all cases.

103. pH of 0.01 M HS" will be pKu logc

(a) pH = 7 +

(c) pH =

2 2 pKx - pK2

(b) P H = 7 -pKh logc

(d) PH = 7 + pKu - pKh

I Zn2+ | |Cd2+ |Cd 104. Zn | If Zn —> Zn2+ + 2e~ 0.760 V Cd -> Cd2+ + 2e~ 0.495 V

and E,a = -Ercd then [Zn2+]/[Cd2+] is (a) 1 (b) 20 (c) 1010 (d) none.

105. 100 m L o f a buffer of 1MNH3 and 1 NH4 are placed in two voltaic cells separately. A current of 1.5 A is passed through both cells for 20 minutes. If electrolysis of water only takes place

2H 2 0 + 0 2 + 4e~ 40H" (RHS) 2HzO - » 4H+ + 0 2 + 4e~ (LHS), then pH of the

(a) L.H.S. will increase (b) R.H.S. will increase (c) both side will increase (d) both side will decrease

106. Frenkel defect is the (a) Schottky defect (b) interstitial defect (c) combination of a, b (d) none of these.

107. r B a C l 2 = 1 0 0 , r H 2 s o 4 = 2 5 0 , r H C 1 = 1 2 0 . Specific conductance (BaS04) = 22 x 10-4. Hence /Q7,(BaS04) is (a) 4 x 10-4 (b) 10-* (c) 16 x 10"4 (d) none of these.

108. In a packing of identical spheres (of radius r), the distance of 4r is equal to the length of (a) face diagonal in fee (b) face diagonal in bcc (c) body diagonal in bcc and face diagonal in fee (d) face diagonal in bcc and body diagonal in fee

109. In octahedral void and tetrahedral void, radius ratio is (in closed packing)

0.414 0.225 (a)

-'38

0.225 (b) 0.414

(c) 0.225 x 0.414 (d) none.

110. pK/, of CHjCOCT ion is 9.26. pH of a solution when 0.01 M CH3COOH is neutralised 50% and at equivalent point using 0.01 M NaOH are respectively (a) 4.63, 8.22 (b) 4.74, 8.22 (c) 2.37,4.11 (d) 4.74,8.37

111. On heating colourless solid A, gas B (liquid at room temp.) and gas C are formed. A is decomposed by NaOH on heating to form gas D giving white fumes with HCl. Gas C occupies 800 ml/g at NTP. Hence A is (a) NH4N03 (b) NaNO, (c) NH4C1 (d) NH4N02 .

112. Following gives yellow ppt. on boiling with conc. HNO3 + Amm. Molybdate :

(a) AsO;

(c) P0 4 ,3-

(b) M0O4

(d) both (a) and (c).

-» X (green) 113. Black residue

(Mn02) + K,C0 3 + air A" + CI, Y (pink). X and Y are

(a) K,Cr207, KMn04 (b) K,Mn04 , KMn04

(c) KMn04 , K2Mn04 (d) none.

114. Cr04~ (yellow) changes to Cr 2 0 2 - (orange) in pH = x and vice versa in pH = y.x and y can be (a) 6 , 5 (b) 6 , 8 (c) 8 , 9 (d) 7 , 7

115. The sulphur present in an organic compound is oxidised by fuming nitric acid into (a) H,S (b) SO, (c) S0 3 (d) H2S04

116. Which of the following statements are correct for all three halogens (X), CI, Br and I ? (a) they all form oxyacids that are strong acid in aqueous

solution (b) they all react with hot. conc. NaOH (aq) to give

X04~ ions (c) they all need to gain one electron to acquire stable

configuration (d) they all form acidic hydrides.

117. 20 mL of oxygen contracts to 17 mL when subjected to silent electric discharge in an ozoniser. What further contraction would be observed if the partially ozonised oxygen is treated with oil of cinnamon (which absorbs ozone)?

CHEMISTRY TODAY | OCTOBER '05

(a) 2 mL (c) 6 mL

(b) 8 mL (d) 4 mL

118. Which of the following reaction(s) give (s) coloured gases? (a) K2Cr207 + KC1 + H2S04 (conc.) A >

(b) NaN0 3 + H 2S0 4 (conc.) A >

(c) CuS04 + HN0 3 (dil.) A > (d) Electrolysis of brine using mercury cathode and

graphite anode

119. For the molecule PF4CH3, which of the following structures is the most stable, considering that CH3~ is more electropositive than F ?

F CH,

P —CH3 I F CH,

(c) j - p r .

P - F I F F

(d) J P ^ F ^ ^ F

120. Which of the following changes is not realised in the laboratory? (a) Absorption of NO by alkaline sodium sulphite to

form a compound (b) Combustion of metallic Mg in C0 2

(c) Heating hydrated magnesium chloride to get the anhydrous salt

(d) Displacement of chlorine from KC103 by iodine to form KI03

121. Consider the isoelectronic series K+, S2", CI" and Ca2+

The radii of these ionic species follows the order (a) Ca2+ > K+ > CI" > S2" (b) CP > S2" > K+ > Ca2+

(c) S2" > CC > K+ > Ca2+

(d) K+ > Ca2+ > S2" > CI"

122. There is loss in weight when mixture of Li2C03 and Na2C03 .10H20 is heated strongly. This loss is due to (a) Li2C03 (b) Na2C03-10H20 (c) both the above. (d) none of these.

123. Solute (X) is highly soluble in water. On adding conc. H2S04 , there is no reaction and a clear solution is obtained. (A) can be (a) NaN0 3 (b)' Na2HP04

(c) KI (d) KC1

124. Green coloured solute (A) AgNO,

H 2O 2 ,KOH,A ) yellow solution (C)

Yellow ppt. (D). Which is correct? A B C

(a) AgCl KC1 Pbl2

(b) AgCl KC1 PbCr04

(c) AgCl CrCl2

(d) CrCl3 AgCl PbCr04

K2Cr04

-> white ppt (B) (CH3COO)2Pb

CH3COOH

D

Pbl2

Pb2Cr207

Pbl, PbCr04

125. Nq 12 atoms of X (g) are converted into X~ (g) by energy EX.NJ2 atoms of X (g) are converted into X~ (g) by energy E2. Hence ionisation potential and electron affinity of X (g) are

(a)

(c)

2Ei 2(EX-E2)

N n (b)

2 £ ] 2 En

— E2 2E2

Nn (d) none of these.

126. For the elementX, student^ measured its radius as 102 nm, student B as 103 nm and C as 100 nm using same apparatus. Their teacher explained that measurements were correct by saying that recorded values by A, B, and C are (a) crystal, van der Waal and covalent radii (b) covalent, crystal and van der Waal radii (c) van der Waal, ionK .ind covalent (d) none is correct

127. Following graph shows variation of I .P. with atomic number in second period (Li-Ne). Value of I.P. of Na (11) will be (a) above Ne (b) below Ne but above O (c) below Li (d) between N and O.

Ne

N F

c / L / 0

j b " L i i i . I i i i i i i

7

Z • 8 9 10 11

128. Following triads have approximately equal size. (a) Na+, Mg2+, A F (isoelectronic) (b) F", Ne, 02~ (isoelectronic) (c) Fe, Co, Ni (d) Mn+, Fe2+, Cr (isoelectronic).

129. Match list I with list II and select the correct answer

-'49 CHEMISTRY TODAY | OCTOBER '05

using the codes given below the lists. List I List II

(manufacturing process) (catalyst used) A. Deacon's process for 1. Finely divided iron with

chlorine molybdenum as promoter

B. Hydrogenation of 2. Copper(II) chloride vegetable oils

C. Ostwald's process 3. Finely divided nickel for nitric acid powder

D. Haber's process for 4. Platinum gauze, ammonia

Codes: (a) A-4, B-l, C-3, D-2 (b) A-2, B-3, C-l, D-4 (c) A-l, B-4, C-2, D-3 (d) A-2, B-3, C-4, D-l.

130. 1 mole of a substance (X) was treated with an excess of water. 2 moles of readily combustible gas were produced along with solution which when reacted with C02 gas produced a white turbidity. The substance (X) could be (a) Ca (b) CaH2

(c) Ca(OH)2 (d) Ca(NO,)r

O O

131. CH3 - C - CH2 - C - OCH2CH3

(D* is catalyst). A is

OH O I II

(a) C H 3 - C = C H - C - O C 2 H 5

D+

A.

o OH I

(b) CH3 - C - CH — C - OC,H5

OD O I II

(c) C H 3 - C = C H - C - O C 2 H 5

(d) formation of A is not possible.

132. Which will give colour with FeCl3?

rVoH (a) V _ y

OH I

O

(b) C H 3 - C = C H - C - O C 2 H 5

(c)

-'38

OH (d) NH,

133. Isopropyl chloride + KCN — > A H i ° +

> B . IUPAC names of A and B are (a) isopropyl cyanide, isobutyric acid (b) 2-methy 1-1 -propanonitrile, 2-methylpropanoic acid (c) both are correct (d) none is correct.

134. CH, - CH2 - CH2 - CH r There is free rotation about (C, — C3) bond. The same most stable form is repeated after rotation of (a) 60° (b) 120° (c) 240° (d) 360°

135. Least reactive is (a) CH2N2 (b) CH2 = C = O

(c) : CH2 (d) None of these.

136. Which is/are less acidic than phenol?

(a) CH3OH (b) \O/~0H

CH,

(c) ( O h OH (d) H20.

NO,

137. Dehydration of the following in increasing order is

< ^ > - O H O H ^ — O H

1 II III IV

(a) I < II < III < IV (b) II < III < IV < I (c) I < I I I < I V < I I (d) I < IV <11 = 111.

138. C4H6-A

0 , / H , 0 C H 0 3 2 | + 2HCHO . A a n d B CHO

CI, + H 2 0

(a) C H 2 = C H - C H = CH2, CH2 = C H - C H - C H ,

OH (b) CH,=CH - C H = C H , , CH3 - CH - CH - CH3

OH CI (c) CH, = C H - C H = C H 2 , C H 3 - C H - C H - C H 3

OH CI (d) none of these.

139. Chloropicrin is obtained when chloroform reacts with and reaction is (a) nitrous acid, SN (b) nitric acid, SE

(c) picric acid, SN (d) nitric acid, SN.

CHEMISTRY TODAY | OCTOBER '05

140. CH, - CH, — COOH NaN, + conc. H ,SO.

Br2/P

^ A

by reaction Rl

-> B by reaction R2 Which is correct alternate? (a) A - CH3CH2NH,, B - CH3CHCOOH

Br Rt - Schmidt, R2 - HVZ A - CH3CH2CONH2, B - CH3CH2COBr (b) Rl - HVZ, R2 - Schmidt

(c) A - CH,CH2NH2, B - CH3CH2COBr Rl - HVZ, R2 - Schmidt

(d) none is correct.

141. Aldehyde A (C4H80) A1;F B"T0^DE > 5 (ester) Tischenko

H3O+

A and B are

CH,

CH 3 \ CH.

CH CHCOOH CH

3 ^ : C H C H 2 O H

(a)

(b)

(c)

C H

CH3

CH.

CHCHO, CH3.

CH, : CHCOOCHjCH^

^ C H 3

CH,

CHCHO, CH,CHCOOCH / C H 3

• CH,

both are correct (d) none is correct.

142. Stability order of the following is

CH 2 , CH 2 = C H - CH 2 , (CH 3 ) 3C , CH 2 = CH

II III IV

(a) IV < III < II < I (c) I < II < III < IV

(b) IV < II < I < III (d) I V < I < I I I < I I .

143. The monomer that is used to produce orlon is (a) CH, = CHCN (b) CH, = CH - CI (c) CH, = CHF (d) CH, = CC1,.

144. Which of the following cannot undergo nucleophilic substitution under ordinary condition? (a) allyl chloride (b) benzyl chloride (c) n-propyl chloride (d) vinyl chloride.

145. Which of the following is incorrect?

CH2CH3 CHCH,

<a> C o j + HCl

y'cr rO o o~ V I H I II I

(b) H O - C - H + C - H — > H O - C - H + H - C - H r \ 41 I VH- yH H

CH3

H~ sh ift + (c) C H 3 - C + > C H 3 - C H - C H 2

CH, CH,

HO-(d) CH3CH2CHO > CH3 CHCHO + H20

146. An organic compound of molecular formula C4HS

(A), forms precipitates with ammoniacal silver nitrate and ammoniacal cuprous chloride. A has an isomer B, one mole of which reacts with one mole of Br2 to form 1,4-dibromo-2-butene. A and B are (a) CH3 - CH2 - C = CH and

CH2 = CH -CH = CH2

(b) C H , - C = C - CH, and CH, - CH = C = CH2

(c) 1 > C = CH2 and C H 0

CH2 - CH II

CH2 - CH

(d) CH, - C = C - CH3 and CH. / C H ^

^ C H / CH,

147. The Ka values of alcohols, water and phenol are of order IO"17, IO"14 and 10"10 respectively. Which of the following reactions is possible? (a) C6H50" + H20 C6H,OH + OH" (b) C2H.O- + H20 C2H5OH + OH-(c) both (a) and (b) (d) neither of two.

148. B (mix) g c o n ^ H I (CH3)3C - O - CH3

anhydrous HI > A

ether 1

(a) A is a mixture of CH3I and (CH3)3C - OH (b) S is a mixture of CH3OH and (CH3)3C - I. (c) A and B are identical mixture of CH3I and

(CH3)3C - OH (d) both a & b

v V/° 149. H - C . H - C

3 O - H 8 X 0 U

(C - O) bond lengths designated by a , (3, y and 5 are in order (a) a = y < P = 5 (b) a < ( 3 < y = 5 (c) a < y = 5 < (3 (d) all are equal.

-'38 CHEMISTRY TODAY | OCTOBER '05

OCH,

150. O NaNH,

-Br product A by reaction R •

A and R are OCH,

(a) NH ' e l ' m i n a t ' o n addition

OCH3

(b) , elimination addition

OCH3

( c) O , cine substitution

(d) none of these.

M A T H E M A T I C S

(151yA square and a parallelogram have equal bases an3 equal areas. If the perimeter of the parallelogram

is - j times the perimeter of the square, then one angle

of the parallelogram must be (a) 30° (b) 45° (c) 60° (d) 62° 30'.

152. Perpendiculars are drawn on the sides of an equilateral triangle from any point within the triangle, if the lengths of these perpendiculars be 6 cm, 7 cm, 9 cm, then the length of a side of the triangle is

cm (h) (a)

(c) 22 p; — V 3 cm

(b)

(d) l l V J

cm

cm

153'. If the lengths of the parallel sides of an isosceles trapezium are 20 cm and 30 cm and its area is 100 cm2, then the length of the non-parallel side is (a) 41 cm (b) 9 cm (c) V4I c m (d) 3 cm.

154. (1101101), is (a) 81 (b) 121 (c) 109 (d) 92.

155. Volume of the parallelopiped whose sides are

OA = 2i - 3j; OB = i + j-k;OC = 3 i - k is (a) 4/13 (b) 4 (c) 2/7 (d) 3/7.

156. 74.1875 in binary is (a) 10010010.0011 (b) 1001010.0011 (5)^1001010.0101 (d) 1001100.0101.

^157, I f f ( a ) = 2, f'(a) = 1; g(a) = -1, g'(a) = 2, then value of

g ( x ) f ( a ) - f ( x ) g ( a ) x - a

lim

(a) 0 (b) 1 (c) 5 (d) 3.

158. Consider an infinite GP with first term a and common ratio r. Its sum is 4 and the second term is 3/4, then:

(a) a = 2,r = ~

2, r= -

,. f l + 5x2

159. The value of I'm , , , *->o\. 1 + 3x (a) £

3 1 (b) a = j , r = j

(d) y / = 3 , r = 4-

160. If A =

(c) - 1 (d) 0.

then which of the following cos 0 - sin 0 s in0 cos©

statements is not correct (a) A is orthogonal matrix (b) A' is orthogonal matrix (c) determinant A = 1 (d) A is not invertible.

161. If a * b * c, one value of x which satisfies the 0 x-a x-b

x + a 0 x — c • • . equation is given by x+b x+c 0

(a) x = a (b) x = b (c) x = c (d) x = 0.

162. The probability that at least one of the events A and B occurs is 0.6. If A and B occur simultaneously with probability 0.2, then P(A)+ P(B) is (a) 0.4 (b) 0.8 (c) 1.2 (d) 1.4

163. I f f { x ) = x + 2 then / ' ( / ( x ) ) at x = 4 is (a) 8 \ j > f 1 (c) 4 (d) 5.

164. Given A = sin20 + cos40, then for all real value of 0

3 (a)

(c)

1 <A < 2

13

A < 1

16 <A< 1 3 13

(d) - A < A < l - f 4 16

52 CHEMISTRY TODAY | OCTOBER '05

165. Let a, b and c be vectors with magnitudes 3, 4 ancl 5 respectively and a + b + c = 0 then the value of a-b + b- c + c- a- 0 is (a) 47 (b) 25 (c) 50 (d) -25. eIf the second, third and fourth terms in the expression

a)n are 240, 720 and 1080 respectively then the value of n is (a) 15 (b) 20 (c) 10 (d) 5.

(A&Dren different letters of an alphabet are given. Words "with five letters are formed from these given letters. Then the number of words which have at least one letter repeated is (a) 69, 760 (b) 30, 240 (c) 99, 784 (d) none of these.

v168. A lady gives a dinner party for six guests. The number of ways in which they may be selected from among ten friends, if two of the friends will not attend-the party together is (a) 112 (b) 140 (c) 164 (d) none of these. 169. The number of all possible words that can be formed using the letters of word MATHEMATICS is

11! (b)

,177. The determinant

(c) 2! 2! 2!

C, ii 1 1 !

(d) none of these.

170. 7 men and 7 women are to sit around a table so that there is a man on either side of a women, the number of seating arrangements is (a) (7!)2 y t f (6!)2 (c) 6! 7! (d) 7!.

171. The real roots of the equation 7U>8 7 (X 2 - ' ! j c+5) _

(a) 1 and 2 (b) 2 and 3 (c) 3 and 4 (d) 4 and 5.

172. The sum of the series 2, 5, 8, 11, ... n is 60100, then n is (a) 100 (b) 200 (c) 150 (d) 250.

173. Three of six vertices of a rectangular hexagon are chosen at random. The possibility that the triangle with three vertices is equilateral, equals (a) 1/2 (b) 1/5 (c) 1/10 (d) 1/20.

\14:. At what value of x, will X + co2 1

CO

1

CO

X + CO

CO

1 + X

CO2

= 0

(b) x = \ (d) none.

1 — iz then |co| = 1 implies

(a) x = 0 (c) x = - l

\15.\iz - x + iy and co = ^ z -i that in the complex plane (a) z lies on the imaginary axis

z lies on the real axis (c) z lies on the unit circle (d) none of these.

176. The locus of the midpoints of the chords of a circle x2 + y2 = 4 which subtend a right angle at the centre is (a) 2 (b) x2 + y2 = 1

A = b aa + b

b c

ba + c

aa + b ba+c

0 is equal to zero if

(a) a, b, c are in A.P. (c) a, b, c are in H.P.

(b) a, b, c are in G.P. (d) none of the above.

' 178. The equation of circle whose centre lies on 3x-y - 4 = 0 and x + 3y + 2 = 0 and has an area 154 square units is (a) x2 + y2 - 2x + 2y - 47 = 0 (1, - a )

'XpT x2+y2-2x+2y + 47 = 0 Qv,-*) (c) x2 + y2 + 2x - 2y - 47 = 0 V) . J .d) none of the above.

179. If z,, z2, z3 are in A.P. (z,, z2, z3 are complex numbers), then they lie on a (a) circle (b) straight line (c) parabola (d) none of the above.

2z + 1 iz + 1 is -2 , then the 180. If the imaginary part of

locus of the point represented by z is a (a) circle (b) straight line (c) parabola (d) none of the above.

- -

181, The domain of definition of the function y (JC) given by equation 2X + 2y = 2 is (a) 0 < x < 1 (b) 0 < x < 1 (e)--^oc < x < 0 -mdc < x < L

\

182. If the vectors ai+j + k, i+bj + k, i + j + ck

(a 1, b 1, c * 1) are coplanar, then the value of

1 1 1 - a

(a) 0 1 •b 1 - c

(b) 1

is

(c) - 1 (d) 2.

-'53 C H E M I S T R Y T O D A Y | OCTOBER '05

183. If 1, co, co2, co3,...., co" - ' are the wth roots of unity, then (1 - en) (1 - co2)(l - co3) .... (1 - co""1) is « 0 (b) 1 (c) n (d) -n .

(184. The value of (1 - co + w2)(l - co2+ co)6, where co, CoWare the cube roots of unity, (a) 128co (b) -128co2

(c) -128co (d) 28co2.

185. The unit vector perpendicular to both the vectors

i - 2 j + 3k and i+2j-k is

(a)

(c)

J = ( . - ! +j + k)

(i + j - b VJ

(b) ( - i + j + k)

(d) none of the above.

( 186. In a triangle ABC, angle A is greater than angle B. H4fie values of angles A and B satisfy the equation 3sinx - 4sin3x - & = 0 ; 0 < A : < 1 then the measure of the angle C is (a) TI/3 (b) 7i/2 (c) 2tc/3 (d) 5ti/6.

187. If the sum of distance of a point from two perpendicular lines is unity. Then its locus is (a) circle (b) ellipse (c) hyperbola (d) none of the above.

^18>8. The equation of a circle which passes through the intersection of x2 + y2 + I3x - 3y = 0 and 2x2 + 2y2 + %x - ly - 25 = 0 and whose centre lies on \3x + 30y = 0 (a) 4x2 + 4y2 - 30x - 10^ - 25 = 0 (b) 4x2 + 4y2 + 30x - 13>- - 25 = 0 (c) 4x2+ 4f - \lx - lOy + 25 = 0 (d) none of the above.

189. If two circles (x - l ) 2 + (y - 3)2 = r2 and x2 + y2- 8* + 2y + 8 = 0 intersect in two distinct points, then (a) 2 < r < 8 (b) - 7 < r < 2 (c) „ r = 2 (d) r > 2.

^190. A square is inscribed in the circle x2 + y2- 2x + 4y + 3 = 0 and its sides are parallel to the coordinate axes then one vertex of the square is (a) (1 + V2, - 2 ) (b) (1 - V2, - 2 ) (c) ( 1 . - 2 + V2) (d) none of the above.

i l91.Jl im x - » 0

(1 + xy*-e + f

. , l i e (a) 24- ( b ) ~24~

(d) none of these.

192. The integral \[x2]dx where [x] denotes the — ' o

greatest integer less than or equal to x, equals (a) 2 + V2 (b) 2 - V 2 (c) 1.5 (d) none of the above.

193.-If / ( l ) = 1 and / ' ( 1 ) = 4, then the value of

V / 0 0 - 1 • lim is y/x-l

(a) 9 (c) 12

(b) 4 (d) 1.

194. Solution of the differential equation dy 2 2 - r - + y s e c jc = t a n x s e c x is dx

(a) y = tanx - 1 + Ce"ta,u

(b) y2 = taruc - 1 + ceta™ (c) yetanx= tanx - 1 + C (d) ye-tawc= tanx - 1 + C.

195. For positive number*, y, z the numerical value of

1 log xy log \ogyx 1 log^z \og.x log ,y 1

is

dx

(a) 0 (c) - 1

zn 196. J 7 —

)R/6 I + V C O T X

(a) ti/3 (c) 7C/12

" n dx

IS

(b) 1 (d) 2.

(b) 7t/6 (d) ji/2.

197. J . t 3 0 1 + tan x (a) 7t/2 (c) 7t

is

(b) jt/4 (d) 0.

198. I f / : R -> R and g \ R R are one to one, real valued functions, then the value of the integral

] [ f ( x ) + f(-x)][g{x)-g(-x)]dx is -7T

(a) 0 (b) 7t (c) 1 (d) none of these.

-'38 CHEMISTRY TODAY | OCTOBER '05

199. Locus of the point of intersection of the tangents drawn on an ellipse at the points whose eccentric angles differes by a constant is a (a) circle (b) parabola (c) hyperbola (d) ellipse.

200. If vectors a and b are non-collinear and

vectors d = (x-2)a + b and p = (2x + l~)a-b are collinear, then x = (a) 1/3 (c) 0

201. If the roots of b (c - a)x2 + c (a - b)x + a (b

are equal, then a, b, c are in (a) A.P. (b) G.P. (c) H.P. (d) none of these.

202. Among 3n consecutive natural numbers, 3 numbers e chosen at random. The probability that the sum of

(b) 2/3 (d) none of these.

c) = 0

three numbers is a multiple of 3 is

3nJ -3nz -In W (3n - l)(3w - 2)

3n-2 (c) 3 / 7 - 1

'(b)

(d)

3n2-3n + 2 (3« - l)(3n - 2)

3n3 -3n2 + 2n (3n - l)(3n - 2)

203. The function f (x) = 1 + |sin*| is 4a)-^continuous everywhere (b) not defined at x = n (c) not continuous at x = 0 (d) not differentiable at x = 7t/2. 204. From the point P (4, 3) tangents PA and PB are

-drawn to the circle x2 + y2 = 9. Then area of the triangle PAB is , ^ 1 9 2 / i \ 16 (a) -25- (b) T

, , 21^3 . . . 25>/3 (c) — (d)

C u t a n d s e n d to:

The Editor, Chemistry Today, 406, Taj Apartment, Ring Road, Adjacent Safdarjung Hospital, New Delhi • 110 029.

Attach this form along with your response sheet. (Photocopy allowed). Please send a self addressed envelope along with postal stamps worth Rs.20. Do not fix the stamps on the envelope. The despatch will include ranking cum analysis sheet and solutions (detailed). Response sheet will not be entertained without the stamps. The iast date of receipt of Response Sheet is 10th of the next month.

Name

Board Passed/Appearing (Percentage & year).

Correspondence address

Pin Phone :

No. of Quest ions Unattempted No. of Quest ions Attempted

Have you enrol led to any

1 Cor respondence course Q N o [ J Yes, If Yes, please specify .•

2 Test Series Q N o Q Yes, If Yes, please specify

Do you subscr ibe to Mathemat ics Today, Physics For You, Chemistry Today and Biology Today. If Yes Subscript ion No. .

If N o , how you obtained this issue EH From school library QH Magazine Agent /Book Shop

Others (please speci fy)

What you like mos t in Chemis t ry Today?

What suggest ion would you like to give for its improvement?

-'38 CHEMISTRY TODAY | OCTOBER '05

Response Sheet

Ns

1.

i m e

® ® © © 46. ® ® © © 91. ®

Enr<

® © © 2. ® ® © © 47. ® ® © © 92. @ ® © © 3. ® ® © © 48. ® ® © © 93. ® ® © © 4. ® ® © © 49. ® ® © © 94. @ ® © © 5. ® ® © © 50. ® ® © © 95. ® ® © © 6. ® ® © © 51. ® ® © © 96. @ ® © © 7. ® ® © © 52. ® ® © © 97. ® ® © © 8. ® ® © © 53. ® ® © ©• 98. ® ® © © 9. ® ® © © 54. < s > ® © © 99. @ ® © © 10. ® ® © © 55. ® ® © © 100. @ ® © © 11. ® ® © © 56. ® ® © © 101. @ ® © © 12. ® ® © © 57. ® ® © © 102. @ ® © © 13. ® ® © © 58. ® ® © © 103. @ ® © © 14. ® ® © © 59. ® ® © © 104. @ ® © © 15. ® ® © © 60. ® ® © © 105. ® ® © © 16. ® ® © © 61. ® ® © © 106. ® ® © © 17. ® ® © © 62. ® ® © © 107. @ ® © © 18. ® ® © © 63. ® ® © © 108. ® ® © © 19. ® ® © © 64. ® ® © © 109. @ ® © © 20. ® ® © © 65. ® ® © © 110. @ ® © © 21. ® ® © © 66. ® ® © © 111. ® ® © © 22. ® ® © © 67. ® ® © © 112. ® ® © © 23. ® ® © © 68. ® ® © © 113. ® ® © © 24. ® ® © © 69. ® ® © © 114. ® ® © © 25. ® ® © © 70. ® ® © © 115. ® ® © © 26. ® ® © © 71. ® ® © © 116. ® ® © © 27. ® ® © © 72. ® ® © © 117. @ ® © © 28. ® ® © © 73. ® ® © © 118. ® ® © © 29. ® ® © © 74. ® ® © © 119. @ ® © © 30. ® ® © . © 75. ® ® © © 120. ® ® © © 31. ® ® © © 76. ® ® © © 121. ® ® © © 32. ® ® © © 77. ® ® © © 122. @ ® © © 33. ® ® © © 78. ® ® © © 123. ® ® © © 34. ® ® © © 79. ® ® © © 124. @ ® © © 35. ® ® © © 80. ® ® © © 125. ® ® © © 36. ® ® © © 81. ® ® © © 126. ® ® © © 37. ® ® © © 82. ® ® © © 127. @ ® © © 38. ® ® © © 83. ® ® © © 128. ® ® © © 39. ® ® © © 84. ® ® © © 129. @ ® © © 40. ® ® © © 85. ® ® © © 130. ® ® © © 41. ® ® © © 86. ® ® © © 131. ® ® © © 42. ® ® © © 87. ® ® © © 132. ® ® © © 43. ® ® © © 88. ® ® © © 133. ® ® © © 44. ® ® © © 89. ® ® © © 134. @ ® © © 45. ® ® © © 90. ® ® © © 135. ® ® © ©

136. ® 137. @

138. ® 139. ® 140. @ 141. ® 142. @ 143. ® 144. ® 145. @ 146. ® 147. ® 148. ® 149. ® 150. ®

(D © © ® © © ® © © ® © © ® © © ® © © ® © © ® © © ® © © ® © © ® © © ® © © ® © © ® © © ® © ©

MATHEMATICS 151. @ ® © © 152. @ ® © © 153. ® ® © © 154. @ ® © © 155. @ ® © © 156. @ ® © © 157. @ ® © © 158. @ ® © © 159. @ ® © © 160. @ ® © © 161. ® ® © © 162. ® ® © © 163. @ ® © © 164. ® ® © © 165. @ ® © © 166. ® ® © © 167. ® ® © © 168. @ ® © © 169. @ ® © © 170. @ ® © © 171. ® ® © © 172. ® ® © © 173. @ ® © © 174. @ ® © © 175. @ ® © © 176. @ ® © © 177. ® ® © © 178. @ ® © © 179. ® ® © © 180. ® ® © ©

181. ® 182. @

183. ®

184. @

185. ®

186. ® 187. ®

188. ® 189. @

190. ®

191. @

192. ®

193. ®

194. @

195. ®

196. @

197. ®

198. ®

199. @

200. ® 201. @ 202. ® 203. ®

204. ®

205. ®

206. @

207. ®

208. ® 209. @

210. ® 211. @ 212. ® 213. ®

214. @

215. ®

216. ® 217. ®

218. ® 219. ®

220. ® 221. @

222. ® 223. ®

224. ®

225. ®

®

© © © © © © © © © © © © © © © © © © © © © © © © © © © © © © © © © © © © © © © © © © © ©

© © © © © © © © © © © © © ©

® © © ® © © ® © © ® © © ® © © ® © © ® © © ® © © ® © © ® © © ® © © ® © ©

-'38 CHEMISTRY TODAY | OCTOBER '05

205. The value of z satisfying the equation z2 + |z| = 0 is (a) 0, ± 1 , ± / (b) 0, ±i (c) 1 + i (d) 1 - i.

206. The cube root of ( 7 2 - 3 2 ^ 5 ) is 3 + V5 (b) 3 - A/J

6 + 3V? (d) 6 - 3 V I .

(a)

(c)

207. If (1 + 0" = (1 - 0"> then the value of n is (a) 3k (b) 4 k (c) 5k (d) Ik.

(where k is an integer) [» = A/-1 ]

V l + V 2 + A/3 + . . . + \fn n\fn

is 208. The value of Hm

(a) 1 (b) 2/3 (c) 1/4 (d) 0. 209. The value of 1 + co" + co2", where co is the cube root of unity and n is a multiple of 3, is (a) 1 (b) 0 (c) 3 (d) co.

210. How many numbers less than 40,000 can be formed from the digits 1,2,3,4,5 where repetition is not allowed? (a) 24 (b) 72 (c) 48 (d) 96. 211. If for a triangle ABC,

cos A + 2cosB + cosC = 2 then the relation which holds true is

1 1 (b) - + j - = v ' a b c (d) a = b = c.

(a) a + c = 2b

(c) ab + be = ca

212. The lines a^x + />!>> + c, = 0 and a2x + b^y + c2 = 0

cuts the co-ordinate axes in concyclic points if (a) = a2b2 (b) axa2 = bxb2

ai _ c, (c) <2i/>|C| = a2b2c2 (d)

213. If P (x, y) is equidistant from (a + 1, b - 1) and (a - 1, b + 1), then which of the following relation holds true? (a) ay = bx (b) x - y = a - b

a b y

= 1 (d) - + f = i . a b

214. If 8C, - 7C3 = 7C2, then the value of r is (a) 2 or 6 (b) 3 or 5 (c) 3 or 4 (d) 4 or 5.

215. If the perimeter of a rectangle is 100 cm, what will be its sides such that its area is maximum? (a) 25 cm, 25 cm (b) 10 cm, 40 cm (c) 15 cm, 35 cm (d) 20 cm, 30 cm.

216. The value of lim ( s ec6 - ta n 9 ) is 8-H1/2

(a) 0 (b) 1 (c) 1 / S (d) 00.

217. The area bounded by the curves y2 = 4ax and x2 = 4ay is (a) 4 a2 (b) 16a2/3 (c) 16a2 (d) 2a2.

218. If m and n are integers and m = n, then the value j t /2

(d) JI.

of j sin nix • sin nx dx is 0

(a) 7t/4 (b) 0 (c) 7t/2

219. If y = sin-1* + sin"1 V 1-x2 > then dy/dx (a) 0 (b) x

(c) \ll-x (d) A/tt:

220. The value of , ' ° g o " is log ah"

(a) 1 + log ab (b) 1 + log,a (c) 1 (d) 0. 221. If coefficients of 2nd, 3rd and 4th terms in the expansion of (1 + xy are in A.P., then value of n is (a) 2 (b) 5 (c) 7 (d) 10.

222. The inverse of the function / (x) = logI0x is (a) KP (b) JC10

(c) e'° (d) e*.

223. If a2, b2, c2 are in A.P., then a b c -, , , r are in b + c c + a a + b

(a) A.P. ' (b) G.P. (c) H.P. (d) none of these.

224. Which is not the solution of sin79 + sin28 + sin9 = 0

(a) 0 (b) 7t/9 (c) 71/4 (d) 7i/2.

225. If 2tana + cot(3 = tan(3, then find the value of tan(P - a) . (a) tana (b) cota (c) cotp (d) tanp.

-'38 CHEMISTRY TODAY | OCTOBER '05

( A I I M S ) F C B S E ) ( P P M F ) ( A F M C ) F V M M C ) ( E A M C E T ) ( W B - J E E I ( B H U ) ( C M C )

( U P C P M T H

2006 Medical Entrance Exam

Practice Test Paper ( M a n i p a i P M T )

(CET Karnataka • l A i t t U A B i S t f U i i S A B f l i ( J I P M E R

( M P P M t ) ( M G I M S ) [PMDT Bihar ) (PMT HaryanjT) ( K e r a l a P M T ) ( R a j . P M T ) ( T N P C E E )

1. Which element is a gas at 25°C and 1 atm pressure? (a) chlorine (b) phosphorus (c) silicon (d) sulphur

2. Which combustion product is produced the least by gasoline-powered vehicles? (a) C 0 2 (b) H 2 0 (c) N 0 2 (d) S 0 2

3. Which element has the highest electrical conductivity at room temperature? (a) Ge (b) Se (c) Sn (d) Te

4. How should a student prepare 100 ml of a 1.0 M H 2 S0 4 solution from a 10 MH 2S0 4 solution? (a) Add 90 mL of H 2 0 to 10 mL of 10 MH 2S0 4

(b) Add 10 mL of 10 MH 2 S0 4 to 90 mL of HzO (c) Add 10 mL of 10 M H 2 S0 4 to 80 mL of H 20, stir

and dilute to 100 mL after allowing to cool (d) Add 80 mL of H 2 0 lOmL of 10 MH2S04 , stir and

dilute to 100 mL after allowing to cool.

5. What volume of liquid A has the same mass as 80.0 cm3

of liquid B? (a) 40.0 cm3

(b) 97.0 cm3

(c) 160 cm3 (d) 193 cm3

Density (g/cm3)

Liquid A

Liquid B

0.660

1.59

Molar Mass (g/mol)

Mg(OH)2 58.33

6. Mg(OH)2 in the form of Milk of Magnesia is used to neutralize excess stomach acid. How many moles of stomach acid can be neutralized by 1.00 g of Mg(OH)2? (a) 0.0171 (b) 0.0343 (c) 0.686 (d) 1.25

7. A 25.00 mL sample of 0.1050 M H 2S0 4 is titrated with a NaOH solution of unknown concentration. The phenolphthalein endpoint was reached when 17.23 mL

-'38

of the NaOH solution had been added. What is the concentration of the NaOH? (a) 0.07617 M (b) 0.1447 M (c) 0.1524 M (d) 0.3047 M

8. A sample of oxygen gas and a sample of an unknown gas are weighed separately in the same evacuated flask. Use the data given to find the molar mass of the unknown gas (assume experiments are carried out at the same pressure and temperature), (a) 22 g/mol (b) 38 g/mol (c) 44 g/mol (d) 84 g/mol

9. Which pair of gases has the same average rate of diffusion at 25°C ? (a) He and Ne (b) N2 and 0 2

(c) N 2 0 and C 0 2 (d) NH3 and HCl

Mass of evacuated flask 124.46 g Mass of flask 125.10 g 125.10 g + oxygen

125.10 g

Mass of flask + unknown gas 125.34 g

P/at

m M

b

b

1 i 5 10 T/°C

10. According to the phase diagram shown, in what state does the represented substance exist at 1.0 atm and 0.0°C? (a) solid only (b) liquid only (c) gas only (d) solid and liquid only

11. Which liquid has the highest vapour pressure at 25°C? (a) butane, C4H10 (b) glycerol, C3H5(OH)3

(c) octane, C8H18 (d) propanol, C3H7OH

12. Which oxide has the highest melting point? (a) H 2 0 (b) N 0 2

(c) S0 2 (d) Si0 2

13. The enthalpy change of which reaction corresponds to AFF°F for Na 2C0 3 (s) at 298 K?

CHEMISTRY TODAY | OCTOBER '05

AH°f (kJ-mol1)

Cr3+ (aq) Ni2+ (aq)

-143 -54

(a) 2Na (s) + C(s) + 3/20 2 (g) Na2C03(s) (b) N a 2 0 (s) + C02(g) Na2C03(s) (c) 2Na+ (aq) + C03

2" (aq) Na2C03(s) (d) 2Na+ (aq) + 20H" (aq) + C02(aq)

Na2C03(s) + H 2 0

14. Which applies to any endothermic reaction? (a) AH < 0 (b) AH > 0 (c) AG < 0 (d) AG > 0

15. When a bomb calorimeter is used to determine the heat of reaction, which property of the system under investigation is most likely to remain constant? (a) number of molecules (b) pressure (c) temperature (d) volume

16. For the reaction shown, which is closest to the value of AH? 2 C r 3 V ) + 3 N i w

2 C r w + 3Ni2+ {aq)

(a) 124 kJ (b) 89 kJ (c) - 89 kJ (d) -124 kJ

17. An ice cube at 0.00°C is placed in 200 g of distilled water at 25.00°C. The final temperature after the ice is completely melted is 5.00°C. What is the mass of the ice cube ?

(AH/„, = 340 J• g"1, Cp =4.18 J-g"1 • °C~l) (a) 23.6 g (b) 46.3 g (c) 50.0 g (d) 800 g

18. Which reaction occurs with the greatest increase in entropy? (a) 2HzO —> 2H2 + 0 2 (g) (b) 2 N O t e ) - > N 2 0 ; ) + O 2 f e )

(c) C(s) + 02(g)^C02(g)

(d) Br2(p) + C l 2 t e ) - > 2 B r C l t e )

19. For a rate law of the form; Rate = k[A]m[Bf, the exponents m and n are obtained from (a) changes in rate with changing temperature (b) the coefficients of A and B in the balanced equation (c) the concentrations of A and B in single experiment (d) changes in the reaction rate for different

concentrations of A and B.

20. What is the order of a reaction for which the units of k are L-mof'-s"1 and the units of the rate are mol-L~' s""1? (a) zero order (b) first order

(c) second order (d) some other order

21. For the reaction A + B —» C, the rate law is: Rate = k[Af. Which change (s) will increase the rate of the reaction? I. Increasing the concentration of A. II. Increasing the concentration of B. (a) I only (b) II only (c) Both I and II (d) Neither I nor II

22. Which does NOT change with time for a first order reaction? (a) the amount of reactant that disappears in each half life (b) the concentration of the reactant (c) the length of each half-life (d) the rate of the reaction

23. The rates of which reactions are increased when the temperatures is raised? (a) I only (c) Both I and II

I endothermic reactions

II exothermic reactions

(b) II only (d) Neither I nor II

24. When a catalyst is added to the system represented by this energy reaction coordinate diagram, which dimensions in the diagram are changed? (a) (b) (c)

1 and 2 only 1 and 3 only 2 and 3 only (d) 1, 2, 3

25. Which statement is true for a reaction at equilibrium? (a) All reaction ceases (b) The reaction has gone to completion (c) The rates of the forward and reverse reactions are

equal. (d) The amount of product equal s the amount of reactant

26. For the hypothetical reaction, 2A (s) +B(g) ^ 3C (g)

what is the equilibrium expression?

(a)

(c)

K =

K = -

[ c r

[Af[B] [C]3

[AY + [B]

(b)

(d)

K =

K =

3[Cj 2[A][B] [Cf [5]

27. Acetyl salicylic acid (aspirin) behaves as an acid

-'59 CHEMISTRY TODAY | OCTOBER '05

according to the equation shown. Calculate Kh for the C9H704- (aq) ion. (Ka = 3.0 x 10"4)

HC9H7O4 {aq) + H 2 0 / H30+(O?) + C9H704~ {aq)

(a) 3.0 x 10"17 (b) 3.3 x 10"11

(c) 9.0 x 10~8 (d) 3.3 x io3

28. What will happen to the pH of a buffer solution when a small amount of a strong base is added? The pH will (a) increase slightly (b) decrease slightly (c) remain exactly the same (d) become 7.0

29. When a solution of NH3 (Kh =1.8 x 10"5) is titrated with a strong acid the indicator used should change colour near a pH of (a) 1 (b) 5 (c) 9 (d) 13

30. When solid silver chloride (MM = 143.4) is added to 100 mL of H20, 1.9 x 10~4 grams dissolves. What is the K,„ for silver chloride? (a) 1.3 x 10"5

(c) 3.7 x 10"8 (b) 3.7 x 10-6

(d) 1.8 x 10~10

31. In which species does the underlined element have an oxidation number of +2? (a) S02C12 (b) Fe(CN)6

4~ (c) HN02 (d) Ni(CO)4

32. Which transformation is an oxidation? (a) V03r V02

+ (b) Cr02 -> Cr042+

(c) S03 S042" (d) N0 3 - -> N0 2 -

33. _Sn2+ (aq) + _N03" (aq) + _H+(aq) _Sn4+ (aq) + _NO(g) + _H20

What is the coefficient for H+ (aq) when the equation above is balanced correctly with the smallest integer coefficients? (a) 2 (b) 4 (c) 6 (d) 8

34. In electrochemical cells the cathode is always the electrode where (a) oxidation occurs (b) reduction occurs (c) positive ions are formed (d) negative ions are formed

(uq) 2Ga3+ 3H 2(g) 35. 2Ga w + 6H The potential of the cell for the reaction given is 0.54 V. If the concentrations of the ions are 1.0 M and

60

the pressure of H2 is 1 atm, what is E° for the half-reaction : Ga3+

iail) + 3e~ —> Ga w ? (a) - 0.54 V (c) 0.27 V

(b) - 0.27 V (d) 0.54 V

36. All of the following affect the number of moles of metal deposited during electrolysis except the (a) current used (b) electrolysis time (c) charge on the ion (d) molar mass

37. The emission spectrum of hydrogen in the visible region consists of (a) a continuous band of light (b) a series of equally spaced lines (c) a series of lines that are closer at low energies (d) a series of lines that are closer at high energies

38. Which atom in its ground state has the most unpaired electrons? (a) Ge (b) As (c) Se (d) Br

39. An monoatomic ion that has 18 electrons and a +2 charge (a) has 16 protons (b) has the symbol Ar2+

(c) has 18 neutrons (d) is isoelectronic with Ar.

40. Which atom has the largest atomic radius? (a) Li (b) K (c) As (d) Br

41. What is the maximum number of electrons that occupy the n = 3 level? (a) 6 (b) 8 (c) 10 (d) 18

42. How does the reducing ability of the elements vary across the period from Na to Ar? It (a) decreases steadily (b) increases steadily (c) decreases then increases (d) increases then decreases

43. Which species contains only covalent bonds? (a) H2S04 ' (b) NH4N03

(c) NaOCl (d) K2Cr04

44. How many valence electrons are in the pyrophosphate ion, P207

4"? (a) 48 (b) 52

CHEMISTRY TODAY I OCTOBER '05

(c) 54 (d) 56

45. Which species has the largest F-A-F bond angle where A is the central atom? (a) BF3 (b) CF4

(c) NF3 (d) OF2

46. The triple bond in carbon monoxide consists of (a) 3 sigma bonds (b) 2 sigma bonds and 1 pi bond (c) 1 sigma bond and 2 pi bonds (d) 3 pi bonds

47. The boiling points of the halogens, F2, Cl2, Br2 and I2, increase in that order. This is best attributed to differences in (a) covalent bond strengths (b) dipole forces (c) London dispersion forces (d) colligative forces

48. How many structural isomers have the formula C3H6CI2? (a) 1 (b) 2 (c) 3 (d) 4

49. A reaction in which a carboxylic acid reacts with an alcohol to form an organic compound and-water is called (a) esterification (b) hydrolysis (c) neutralization (d) saponification

50. What substance is formed when CF2—CF2 is polymerized? (a) polyethylene (b) polyurethane (c) PVC (d) teflon

ANSWERS

1. (a) 2. (d) 3. (c) 4. (c) 5. (d) 6. (b) 7. (d) 8. (c) 9. (c) 10. (a) 11. (a) 12. (d) 13. (a) 14. (b) 15. (d) 16. (a) 17. (b) 18. (a) 19. (d) 20. (c) 21. (a) 22. (c) 23. (c) 24. (b) 25. (c) 26. (d) 27. (b) 28. (a) 29. (b) 30. (d) 31. (b) 32. (b) 33. (d) 34. (b) 35. (a) 36. (d) 37. (d) 38. (b) 39. (d) 40. (b) 41. (d) 42. (a) 43. (a) 44. (d) 45. (a) 46. (c) 47. (c) 48. (d) 49. (a) 50. (d)

tmrnm® ¥ @ u r Kmmhik Q. Is it safe to leave the aluminium lining of an old swimming pool in the ground ? A. Aluminium rapidly forms a coating of oxide when exposed to air. This is fairly unreactive but if it is exposed to moderate acid or basic conditions then it could dissolve and be mobilised in water. This aluminium could get into the local water table although it's unlikely that much if any of the aluminium would be dissolved. However, if the aluminium is in thick sheets, it is valuable - hundreds of pounds sterling per tonne possibly, so it might actually pay to get it removed.

• • •

Q. If you add just one electron to a chlorine atom, it doubles in size. Why the dramatic increase ? A. The chloride ion, CI" is larger than the chlorine atom, CI since it has one more electron but no greater nuclear charge. This is the case with all the elements in group VII of the periodic table. The radius to which the electrons extend depends on the balance of the attractive force from the nucleus and the repulsive force from all the other electrons. Probably the easiest way of thinking of this is that in a chlorine atom you have 17 protons which are equally matched by 17 electrons. When an extra electrons is added to form a chloride ion this extra electron is not counter balanced by an extra proton and so does not feel a very strong attachment to the nucleus. This makes it circulate at a much larger distance. So in the cases of the halogens, the greater radius of the anion is due to the greater electron-electron repulsion created when a further electron is added to the atom.

• • *

Q. What are the commercial uses of diatomaceous earths ? A. Diatomaceous earths are whitish, powdery silicon deposits consisting essentially of fresh water and marine plankton. They are resistant to heat and chemical reactions and are used in fireproof cements, insulating materials, as an absorbent in the manufacture of explosives and as a filter.

-'38 CHEMISTRY TODAY | OCTOBER '05

PRACTICE PROBLEMS "Enm By: Er. Arvind Tripathi (B.Tech, IT-BHU), Momentum, Gorakhpur

Ionisation constant of a weak acid (fi-4) in terms of and Am is

rA®5 CA2

(b) "

1 .

A *

(a) Ka =

(c) K„=-A : , ( A : , - A J

Ka = a:<A:-A„,)

(d) None of these.

2. The potential of a Ag/AgCI electrode measured with respect to a saturated calomel electrode is-0.022 V. What is the value of the standard reduction potential for the Ag/AgCI electrode. [Given : Hg2Cl2 + 2e -> 2Hg(.s) + 2 C P (sat. KC1); E° = 0.244 V] (a) 0.266 V (b) - 0.266 V (c) - 0.222V (d) 0.222 V.

3. Ag ,Ag- (1M)|| Ag+ (2M)| Ag 1L solution 1 L solution

0.5 F of electricity in the LHS (anode) and IF of electricity in the RHS (cathode) is first passed making them independent electrolytic cells at 298 K. EMF of the cell after electrolysis will be (a) increased (b) decreased (c) no change (d) time is also required.

4. Cost of electricity for the production of xL H2 at NTP at cathode is Rs. x, then cost of electricity for the production of xh 0 2 gas at NTP at anode will be (assume 1 mol of electrons as one unit of electricity) (a) 2x (b) 4x (c) 16A- (d) 32x.

5. For the following cell reaction Pb (5) + Hg2S04 (j) ->> PbS04 + 2Hg (/) E = 0.92 V K,„ (PbS04) = 2 x io-8, Kw (Hg2S04) = 1 x 10"6

Hence, E is (a) 0.92 V (b) 0.89 V (c) 1.04 V (b) 0.95 V

6. The wavelengths of two photons are 2000 A and 4000A respectively. What is the ratio of their energies? (a) 1/4 (b) 4 (c) 1/2 (d) 2.

7. Benzoic ,-idd solution is titrated with NaOH conducto-metricaly, graphical representation of the titration is :

(a) (b)

NaOH N a O H

(c) O (d) o

NaOH NaOH

8. An electron, a proton and an alpha particle have kinetic energies of 16£, 4E and E respectively.- What is the qualitative order of their de Broglie wavelengths ? (a) K >Xp = Xa (b) Xp = Xa> Xe

(c) Xp>Xe>Xa (d) Xa<Xe»Xp

9. The nucleus of an atom is located at x = y = z = 0. If the probability of finding an .y-orbital electron in a tiny volume around x = a, ^ = z = 0 i s l x IO^5, what is the probability of finding the electron in the same sized volume around x = z = 0, y = a? (a) 1 x 10~5 (b) 1 x io-5 x a

(c) 1 x io 5 x a2 (d) 1 x 10~5 x a~]

10. Suppose a particle has four quantum numbers such that permitted value are : n : 1, 2, 3 I : ( « — ! ) , ( « — 3), (n — 5) but no negative number

J <4M4 the latter is not negative

m : - / in integral steps to + / What are other permitted values for n =

(a)

(c) 2 —

m 1 1 2 2

2 " + 2

t J

(b)

(d)

1

in

' 2 " + 2

0 —

-'38 CHEMISTRY TODAY | OCTOBER '05

11. When the value of azimuthal quantum number is 3, the maximum and the minimum values of spin multiplicities are (a) 4, 3 (b) 8, 1 (c) 1, 3 (d) 8, 2 12. The radii of two of the first four Bohr orbits of the hydrogen atom are in the ratio 1: 4. The energy difference between them may be : (a) either 12.09 eV or 3.4 eV (b) either 2.55 eV or 10.2 eV (c) either 13.6 eV or 3.4 eV (d) either 3.4 eV or 0.85 eV 13. Equilibrium is established by introducing an equal number of moles of N2 and H2 into a 1-L flask. At equilibrium, which of the following statements is/are true?

( b ) [HJ < [NJ [H2] = [NH3]

in ; n t

log S

VT-

(a) [H 2]>[N 2] (c) [H2] > [NH3] (-14. Solubility (S) of a solu* a solvent (say H20) is depen on temperature as given by

S = Ae^'KT where AH is he; ^f reaction

solute + HzO —> solution 1 ' ± x For a given solution va . of log S with temp, is shown rraphically. Hence, solute is (a) CuS04 • 5H20 (b) NaCl (c) Sucrose (d) CaO 15. Before equilibrium is set up for the chemical reaction N204 —>2N02, vapour density d of the gaseous mixture was measured. If D is the theoretical value of vapour density, variation of with Did is by the graph. What is value of D/d at point A? (a) 0 (b) 0.5 (c)

16. For the equilibrium LiC1.3NH3 (s) -> LiCl.NH3 <» + 2NH3

Kp = 9 atm2, at 40°C a 5 litre contains 0.1 mole of

LiCl.NH3. How many moles of NH3 should be added to the flask at the temperature to drive the backward reaction for completion. (a) 0.7837 moles (b) 0.8837 moles (c) 0.9837 moles (d) 1.0837 moles 17. 40% of a mixture of 0.2 mole of N2 and 0.6 mole of H2 react to give NH3 according to the equation,

(g) + 3 H2(g) —> 2NH3 (g) at constant temperature and pressure. Then the ratio of the final volume to the initial volume of gases is as-

1

D/d-

(d) 1.5

(a) 4:5 (b) 5:4 (c) 7:10 (d) 8:5 18. CuS04 5H20 CuSO4.3H2O0) + 2H20 (g), Kp for this equilibrium is 1.086 * 1 a t m 2 at 25°C. What is the maximum pressure of water vapour (moisture) in the atmosphere, below which the pentahydrate is efflorescent? (a) 7.92 mm (b) 8.21 mm (c) 6.68 mm (d) 5.69 mm 19. Which of the following buffers containing NH4OH and NH4C1 has the highest pH value? conc. of NH4OH(mol L"1) conc. ofNH4Cl (mol L"1) (a) 0.50 (a) 0.50 (b) 0.10 (b) 0.50 (c) 0.50 (c) 1.50 (d) 0.50 (b) 1.10 20. In an experiment to determine the enthalpy of neutralization of sodium hydroxide with sulphuric acid, 50 cm3 of 0.4 M sodium hydroxide were titrated thermometrically with 0.25 M sulphuric acid. Which of the following plots gives the correct representation?

(a) (b)

10 30 50 Vol. of H,SO„

!0 30 50 Vol. of H,SO,

(C) (d)

10 30 50 Vol. of H,SO,

J 1 L 10 30 50 Vol. of H,SO,

21. In an aqueous solution of 1 M NH4HS the expression for [H+] will be

(a) + ] =

( b ) [ H + ] = X . -K„

(c) [H+] = j A ' a , | ^ + A',

(d) None of these.

-'38 CHEMISTRY TODAY | OCTOBER '05

22. Consider the following vapour pressure composition graph : SP is equal to -(a) PQ + RS (b) PO + QR + RS (c) SR + SQ (d) PQ + QR 00 02 04 06 08 10

XB •

23. Consider the following graph pertaining to distillation. The distillate to residue ratio is

(a) +1 Boiling

point

£x2

ilx

U 2

SF — — J 'l 'l

T, X, X

(d)

Weight percent

U2 U x

0.1 M K4Fe(CN)6

0.1 U FeCU

Stcfe* 1 Side-)--

B

blue colour formation in side X blue colour formation in side Y blue colour formation in both of the side X and Y no blue colour formation

(b)

(c)

24. with K4 [Fe(CN)s] in aq. solution gives blue colour. These are separated by a semipermeable membrane AB as shown. Due to osmosis there is : (a) (b) (c) (d) 25. lmol of N2 and 4 mol of H, are allowed to react in a vessel and after reaction, H 20 is added. Aqueous solution required 1 mol of HCl. Mol fraction of H2 in the mixture after reaction is (a) 1/6 (b) 5/6 (c) 1/3 (d) none of these.

26. 50.0 mL of a gaseous mixture of H, and HCl is exposed to a sodium amalgam, the volume decreases to 42.5 mL. If 100.0 mL of the same mixture is added to 50.0 mL to gaseous ammonia and then exposed to water, what will be the volume of the final mixture ? (a) 35.0 mL (b) 50.0 mL (c) 15.0 mL (d) 70.0 mL

27. A mixture of Na2C204 (A) and KHC204. H2C204. (B) required equal volumes of 0.1 M KMn04 and 0.1 M NaOH separately. Molar ratio of A and B in the mixture is

(a) 1 : 1 (c) 5.5 : 1

(b) (d)

1 : 5.5 3.1 : 1

28. 2 moles of X"+ is oxidized by 5.6 moles of Mn02 into Xm+, 12 moles X"+ is reduced into X"'- by 8.4 moles of H2S in acidic medium (when H2S is oxidised into S02) (a) m = 4.9, n = - 0 . 7 (b) m = 5,n = 2 (c) m = 10, n = 4 (d) m = 7, n = -1

ANSWERS

1. (b) 2. (a) 3. (c) 4. (a) 5. (d) 6. (d) 7. (d) 8. (a) 9. (a) 10. (a)

11. (d) 12. (b) 13. (b) 14. (d) 15. (c) 16. (a) 17. (a) 18. (a) 19. (d) 20. (d) 21. (b) 22. (c) 23. (b) 24. (d) 25. (d) 26. (d) 27. (c) 28. (a)

— - Detailed solutions will be published in next issue. •

Solved Papers 2005 in

j j V C h e m i s t r y T o d a y ^ S

> IIT-JEE Screening May 2005

> CBSE-PMT (Prelims) May 2005

> AIEEE June 2005

> AFMC June 2005

> WB-JEE June 2005 > IIT-JEE (Mains) June 2005

> BHU (Prelims) July 2005 > CBSE-PMT (Mains) July 2005

> Karnataka CET July 2005

> AIIMS July 2005

> CBSE-PMT (Mains) August 2005

> DCE August 2005

> Kerala - PET August 2005

> DPMT September 2005

> J & K CET September 2005 j

> Orissa JEE September 2005 i > Bihar CECE (Mains) October 2005

-'38 CHEMISTRY TODAY | OCTOBER '05

Very Similar MODEL TEST PAPER for AIEEE-2006

1. The number of molecules present in a drop of water weighing 0.06 g is approximately (a) 1021 (b) 2 x 1021

(c) 3 x 1021 (d) 4 x 1021.

2. Helium atom is two times heavier than a hydrogen molecule at 298 K, the average kinetic energy of helium is (a) two times that of a hydrogen molecule (b) same as that of a hydrogen molecule (c) four times that of a hydrogen molecule (d) half that of a hydrogen molecule.

3. Which of the following are the correct axial distances and axial angles for rhombohedral system? (a) a = b = c, a = P = y * 90° (b) a = b * c, a = P = y = 90° (c) a ± b = c, a = P = Y = 90° (d) a * b * c\ a * P * Y * 90°.

4. To which electronic transition between Bohr orbits in hydrogen, the second line in the Balmer series belongs? (a) 3 —> 2 (b) 4 2 (c) 5 —> 2 (d) 6 —> 2.

5. In which of the following compounds does the ratio of the anion size to the cation size have the lowest value? (a) NaCl (b) KC1 (c) MgCl2 (d) NaBr.

6. The frequency of a radiation having a wave number of 2 x 1014 cm"1 will be (a) 6.0 x 10 1 4 s-1 (b) 6.0 x 1024 s"1

(c) 6.6 x 103 s"1 (d) 6.6 x 10-3 s"1.

7. A molecule may be represented by three structures having energies £, , £ , and £3, respectively. The energies of these structures follow the order £ 3 < £2 < £ , , respectively. If the experimental bond energy of the molecule is E0, the resonance energy is (a) (£, +E2 + E3) - E0 (b) E0 - £3

(c) £ 0 - £ , (d) E0-E2.

8. An element {X) forms compounds of the formula XCl,, X2OS and CaX2 but does not form XC\S. Which of the following is the element XI (a) B (b) A1 (c) N (d) P.

9. Polarisation is the distortion of the shape of an anion by an adjacently placed cation. Which of the following statements is correct? (a) Maximum polarisation is brought about by a cation

of high charge. (b) Minimum polarisation is brought about by a cation

of low radius. (c) A large cation is likely to bring about a large degree

of polarisation. (d) Polarising power of a cation is less than that of

anion.

10. The sum of mole fractions A, £ and C in a solution containing 0.1 mole each of A, B and C is (a) 0.1 (b) 0.3 (c) 1.0 (d) 1/3.

11. A solution contains non-volatile solute of molecular mass M2. Which of the following can be used to calculate the molecular mass of solute in terms of osmotic pressure?

^ r rr,T / i \ j / ( m l \ ^

V (a) M2 = VRT (b) M2 =

(c) M2 = •KRT (d) M2 :

/ 7t

I " IRT 12. The heat of neutralisation of aqueous hydrochloric acid by NaOH isx kcal/mole of HC1. Calculate the heat of neutralisation per mole of aqueous acetic acid? (a) 0.5* kcal (b) * kcal (c) 2x kcal (d) cannot be calculated from the given data.

13. Under the same conditions how many ml of 1 M KOH and 0.5 H2S04 solutions, respectively when mixed to form a total volume of 100 ml produce the highest rise in temperature?

-'38 CHEMISTRY TODAY | OCTOBER '05

(a) 67. 33 (c) 40. 60

(b) 33, 67 (d) 50, 50.

14. Which one of the following will favour the reverse reaction in a chemical equilibrium? (a) removal of one of the products regularly (b) increasing the concentration of one of the reactants (c) increasing the concentration of one of the products (d) adding negative catalyst.

15. Which of the following oxides of nitrogen will be the most stable one? ( a ) 2 N 0 2 ( K ) ,

(b) 2NO t e ) ^ (c) 2N205 fe)

N 2(jr) + 202(K); A' = 6.7 x 10 lsmol L" N 2(g)

2N-O

2(g) ' 2 Or)

50 = 2.2 x 1030 molL-'

2(g)

(d"> 2N-0 (s)

K = 1.2 x 1024 mol L"1

2N2fe ) + 0 2 W ; K = 3.5 x 10" mol L"'.

16. Which of the following will occur if a 0.1 M solution of a weak acid is diluted to 0.01 M at constant temperature? (a) [FT] will decrease to 0.01 M (b) pH will decrease (c) percentage ionisation will increase (d) K„ will increase.

17. A certain sample of beer has a pH of 10. The concentration of hydrogen ions in the beer is (a) 10'° M (b) 10-2M (c) 10-4 M (d) 10~'° M.

18. The decomposition of KCIO, to KC1 and 0 2 on heating is an example of (a) intermolecular redox reaction (b) intramolecular redox reaction (c) auto redox reaction (d) none of these.

19. The number of electrons involved in the reduction of nitrate ion to hydrazine is (a) 8 (b) 7 (c) 5 (d) 3.

20. Two platinum wires were dipped in CuS04 solution taken in a beaker and then connected to a battery. The blue colour of copper sulphate disappeared and a gas was found to be evolved at one of the electrodes. The solution in the beaker now contains (a) platinum sulphate (b) cuprous sulphate (c) sulphuric acid fd) copper hydroxide.

21. Which of the following plots represents correctly variation of equivalent conductance (A) with dilution for a strong electrolyte?

(a)

dilution dilution

(C) (d)

dilution dilution

22. The equivalent conductance at infinite dilution of the salt MX is 160.84 ohm"1 cm2 eq_1. If the transport number of M+ is 0.40, the ionic mobility of A'" in cm2 s 1 V~] is

| (a) 3.9 x 1(1 5 (b) 6.7 x 10 ;

• (c) 2.5 x 10~3 (d) 1.0 x 10"3.

23. The reactions : 2NO + 0 2 ^ 2N0 2

2CO + 0 : ^ 2C02

look to be identical, yet the first is faster than the second. The reason is that (a) the first reaction has lower enthalpy change than

the second j (b) the first reaction has lower internal energy change

than the second (c) the first reaction has lower activation energy than

the second (d) the first reaction has higher activation energy than

the second.

24. While studying the decomposition of gaseous N205 , I it is observed that a plot of logarithm of its partial pressure I versus time is linear. The kinetic parameter obtained | from this observation is | (a) specific reaction rate (b) reaction rate

(c) energy of activation (d) molecularity.

25. A first order reaction is half-completed in 45 minutes. How long does it needed for 99.9% of the reaction to be completed?

! (a) 20 hours (b) 10 hours

(c) 7— hours. (d) 5 hours.

18 CHEMISTRY TODAY | SEPTEMBER '05 40

26. The triad of nuclei is isotonic in

(a) 'g C. !7

5R 'gF (b) '62C, '74N, '98F

(c) 'g C, '97F (d) '64C, '74N.

27. Atomic weight of thorium is 232 and its atomic number 90. The number of a-particles and P-particles which-will be lost so that an isotope of lead (atomic weight 208 and atomic number 82) is produced is (a) 4a + 6P (b) 6a + 4P (c) 8a + 2P (d) 10a + 23.

28. A radioactive sample has initial activity of 28 dpm. Half hour later its activity is 14 dpm. How many atoms of nuclide were present initially? (a) 200 (b) 400 (c) 600 (d) 1200.

29. Gelatin is mostly used in making ice cream in order t o

(a) prevent making of a colloid (b) stabilize the colloid and prevent crystallization (c) stabilise the mixture (d) enrich the aroma.

30. ZSM-5 converts (a) alcohol to petrol (b) benzene to toluene (c) toluene to benzene (d) heptane to toluene.

31. Generally the ionization enthalpy in a period increases but there are some exceptions. The one which is not an exception is (a) Be and B (b) N and O (c) Mg and A1 (d) Na and Mg.

32. If the ionization enthalpy and electron gain enthalpy of an element are 275 and 86 kcal mol~' respectively, then the electronegativity of the element on the Mulliken scale is (a) 2.8 (b) 0.0 (c) 4.0 (d) 2.6

33. When an aqueous solution of sodium chloride is electrolysed using platinum electrodes, the ions discharged at the electrodes are (a) sodium and hydrogen (b) sodium and chloride (c) hydrogen and chloride (d) hydroxy! and chloride.

34. Pick up the incorrect statement? (a) asbestos and willemite are silicate minerals (b) anglesite and barytes are sulphate minerals

(c) sylvine and fluorspar are halide minerals (d) calamine and zincite are the minerals of calcium.

35. The first ionisation enthalpy (energy) (kJ mol~') for H, Li, F, Na has one of the following values 1681, 520, 1312, 495. Which of these values corresponds to that of hydrogen? (a) 1681 (b) 1312 (c) 520 (d) 495.

36. When zeolite, which is hydrated sodium aluminium silicate, is treated with hard water the sodium ions are exchanged with (a) H+ ions (b) Ca2+ ions (c) OH- ions (d) S04

2~ ions.

i 37. In electrolvsis of NaCI when Pt electrode is taken ; then H2 is liberated at cathode while with Hg cathode

it forms sodium amalgam. The reason for this is (a) Hg is more inert than Pt (b) more voltage is required to reduce H+ at Hg than

at Pt (c) Na is dissolved in Hg while it does not dissolve

in Pt (d) concentration of H+ ions is larger when Pt electrode

is taken.

38. Which of the following alkaline earth metal oxides shows a coordination number four? (a) BeO (b) MgO (c) SrO (d) CaO.

j | 39. A certain metal is used to prepare an antacid, this

metal accidentlv catches fire which cannot be put out by using C0 2 based extinguishers. The metal is (a) C 7b) Ca (c) Mg (d) Na.

40. Which of the following reactions is employed to produce ozone in the laboratory? (a) exposure of air to UV light (b) reaction of F2 with H 2 0 at low temperature (c) reaction of S0 2 with H202

(d) passage of silent electric discharge through oxygen.

41. Identify the correct order of acidic strength of CO;., | CuO, CaO, H20. j (a) CaO < CuO < HzO < CO :

| (b) H 2 0 < CuO < CaO < C0 2

j (c) CaO < H 2 0 < CuO < C0 2

| (d) H 2 0 < C0 2 < CaO < CuO.

18 C H E M I S T R Y T O D A Y | SEPTEMBER '05 41

42. Thermite is a mixture of (a) 3 parts of powdered A1 and 1 part of Fe203

(b) 1 part of powdered A1 and 3 parts of Fe203

(c) 1 part of powdered A1 and 1 part of Fe203

(d) 2 parts of powdered AI and 1 part of Fe203.

43. Hydrated aluminium chloride is ionic and soluble in water giving (a) Al3+ and CI- ions (b) [A1(H206]3+ and CI" ions (c) [A1C12(H20)4]+ and [A1C14(H20)2]" ions (d) none of these.

44. Water transported through lead pipes becomes poisonous due to the formation of (a) PbO (b) Pb02

(c) Pb(OH)2 (d) Pb304.

45. A solid element (symbol Y) conducts electricity and forms two chlorides PC1„ (a colourless volatile liquid) and TC1„ _ 2 (a colourless solid). To which one of the following groups of the periodic table does Y belong? (a) 13 (b) 14 (c) 15 (d) 16.

46. The correct increasing order of extent of hydrolysis in the following is (a) CC14 < MgCl2 < A1C13 < SiCl4 < PC15

(b) CC14 < A1C13 < MgCl2 < PC15 < SiCl4

(c) A1C13 < MgCl2 < CC14 < PC15 < SiCl4

(d) SiCl4 < MgCl2 < A1C13 < PC15 < CC14.

47. Both boron and aluminium show difference in properties from the remaining members of group 13. This is due to the fact that (a) both B and AI have smaller size as compared to

other members of the family (b) both B and AI have high values of ionization enthalpy (c) both B and AI have 2 and 8 electrons in their

penultimate shell respectively but the remaining elements of this group have 18 electrons

(d) none of these.

48. Among the elements of group 15 which of the following properties shows an increase as we move down from nitrogen to bismuth? (a) stability of - 3 oxidation states (b) reducing character of hydrides (c) electronegativity (d) acidic nature of oxides.

49. A certain compound when burnt gives three oxides. The first turned lime water milky. The second turned anhydrous CuS04 blue and the third formed an aqueous solution of low pH. The elements present in the compound are (a) C, O and S (b) C, H and Ca (c) C, H and Na (d) C, H and S.

50. The shape of molecule SF3C13 is (a) trigonal bipyramidal (b) cubic (c) octahedral (d) tetrahedral.

51. The lowest boiling point of helium is due to its (a) inertness (b) gaseous nature (c) high polarisability (d) weak van der Waals forces between them.

52. Which of the following is not an electronic configuration of noble gas? (a) \s22s22p6 (b) \s2

(c) \s2 2s2 2p6 3s2 (d) Is2 2s2 2p6 3s2 3p6.

53. K2Cr207 is preferred to Na2Cr207 for use in volumetric analysis as a primary standard because (a) Na2Cr207 is hygroscopic while K2Cr207 is not (b) K2Cr207 is hygroscopic while Na2Cr207 is not (c) K2Cr207 is pure while Na2Cr207 is impure (d) none of these.

54. K3[A1(C204)3] is called (a) potassiumalumino oxalate (b) potassiumtrioxalato aluminate(III) (c) potassiumaluminium(III) oxalate (d) potassiumtrioxalato aluminate(VI).

55. A gasJfis passed through water to form a saturated solution. The aqueous solution on treatment with the AgN03 gives a white precipitate. The saturated aqueous solution also dissolves magnesium ribbon with evolution of a colourless gas Y. Identify X and Y. (a) X = C02, Y = Cl2 (b) Jf = Cl2, Y = C0 2

(c) X = Cl2, Y = H2 (d) X = H2, Y = Cl2.

56. Which of the following is a biodegradable pollutant? (a) plastic (b) sewage (c) asbestos (d) mercury.

57. The IUPAC name of the spiro compound,

O C ™ ' -18

CHEMISTRY TODAY | SEPTEMBER '05 42

(a) 2-methylspiro[5.4]deca-1,6-diene (b) 2-methylspiro[4.5]deca-l,6-dier!e (c) 8-methylspiro[4.5]deca-l,7-diene (d) 3-methylspiro[5.4]deca-3,7-diene.

58. Two isomeric alkenes A and B having molecular formula C5H9C1 on adding H2, A gives optically inactive compound while B gives a chiral compound. The two isomers are (a) A is 3-chloro-1 -pentene and B is 1 -chloro-2-pentene (b) A is 2-chloro-3-methyl-2-butene and B is 1-chloro-

3 -methyl-1 -butene (c) A is 3-chloro-2-pentene and B is 2-chloro-2-pentene (d) A is 4-chloro-2-pentene and B. is 4-chloro-2-pentene.

59. When the hybridisation state of carbon atom changes from sp3 to sp2 and finally to sp, the angle between the hybridised orbitals (a) decreases gradually (b) decreases considerably (c) is not affected (d) increases progressively.

60. The energy of the orbitals decreases in the order (a) s > p > sp3 > sp> sp2 (b) p > sp3 > sp2 > sp > s (c) sp3 > sp2> sp> s> p (d) s> sp> sp2 > sp3 > p.

61. When an alkyl iodide (RI) is treated with Na in dry ether, a symmetrical alkane (R2) is obtained. The reaction is called (a) Grignard reaction (b) Fischer-Tropsch reaction (c) Berguis reaction (d) Wurtz reaction.

62. An alkene with molecular formula C4H8 upon ozonolysis gives one mole of propanone and one mole of methanol. The alkene is (a) 1-butene (b) 2-methylpropene (c) 2-butene (d) 2-methyl-2-butene.

63. The treatment of benzene with isobutene in the presence of sulphuric acid gives (a) isobutylbenzene (b) /er/-butylbenzene (c) «-butylbenzene (d) no reaction.

j2

The above transformation proceeds through (a) electrophilic addition (b) benzyne intermediate (c) activated nucleophilic substitution (d) oxirane.

65. Which of the following forms potassium carbonate on boiling with alcoholic KOH solution? (a) dichloromethane (b) tetrachloromethane (c) perchloromethane (d) chloroform.

66. An organic compound Xon treatment with acidified K2Cr207 gives a compound Y which reacts with I2 and sodium carbonate to form tri-iodomethane. The compound X\s (a) CH3OH (b) CH3COCH3

(c) CH3CHO (d) CH3CHOHCH,.

67. Methyl «-propyl ketone on oxidation with K2Cr207

and H2S04 gives mainly (a) CH3CH2COOH + CH3COOH (b) CH3CH2CH2COOH + HCOOH (c) CH3COOH + HCOOH (d) CH3CH2COOH + C0 2 + H20.

68. Oxidation of benzyl chloride with an aqueous solution of lead nitrate in a current of CO, gives (a) benzaldehyde (b) benzyl alcohol (c) p-nitrobenzyl chloride (d) benzyl nitrate.

69. A compound with molecular formula C4H,0O4 on acylation with acetic anhydride gives a compound with molecular formula C,2H18Og. How many hydroxyl groups are present in the compound? (a) one (b) two (c) three (d) four.

70. Which of the following is the weakest acid? OH

(c) HCOOH

71. Diazonium salts are the reaction products between nitrous acids and (a) primary aliphatic amine (b) N-alkyl substituted aromatic amines (c) primary aromatic amines (d) secondary amines.

72. An optically active compound with molecular formula C6H, ,N dissolves in dilute aqueous hydrochloric

18 CHEMISTRY TODAY | SEPTEMBER '05 43

acid and releases N2 gas on treatment with nitrous acid. The compound is

(a)

NHCH,

(b)

.(c) C6H5CH(NH2)CH3 (d) C6H5 - NH - C2H5.

73. The abbreviation PDI refers to (a) name of the polymer (b) poly dispersity index (c) Planck's disposal index (d) poly diagonal index.

74. Match list I (name of vitamin) with list II (deficiency result/disease) and select the correct answer using the codes given below the lists.

List I List II I. Ascorbic acid A. Beri-beri II. Retinol B. Cracked lips III. Riboflavin C. Scurvy IV. Thiamine D. Night blindness. Codes : (a) I-B, II-A, III-C, IV-D (b) I-A, II-B, III-C, IV-D (c) I-D, II-C, III-B, IV-A (d) I-C, II-D, III-B, IV-A.

75. A drug which is structurally related to adrenaline is (a) salbutamol (b) salvarsan (c) diazepam (d) LSD.

ANSWERS

1. (b) : 1 mol of H 2 0 = 18 g = 6.02 x 1023 molecules 0-06 , „„ ,n?3

Hence 0.06 e H ,0 = * 6 .02x10" molecules s 2 18 = 2 x io21 molecules.

2. (b) : Average kinetic energy depends only on temperature and does not depend upon the nature of the gas. 3. (a) : For rhombohedral system,

a = b = c, a = 3 = v * 90°. Rhombohedral system is an alternative representation of the trigonal or hexagonal crystal systems. 4. (b): 1 st line is for 3 —> 2 and 2nd line is for 4 —> 2.

5. (b) : CI" ion is smaller than B r and K+ is larger than Na4. Hence ratio of CP : K+ is smallest.

6. (b) : u = 2xl0 1 4 cm~\ c = v*. = v / o

or, v = cu = (3 x 10IO)(2 x 1014) = 6 x 1024 s"1.

7. (b) : The difference in energy of the actual molecule (experimental value) and energy of the most stable resonating structure is called resonance energy, i.e. E0 — £3. 8. (c) : Since the element X forms XC13, X205 and Ca3X2, therefore, it must be N or P. Since it does form JVCI5, therefore, it must be N since it has no rf-orbitals to expand its covalency from 3 to 5. Therefore, A" must be N.

9. (a) 10. (c) : The sum of mole fractions of all the components = 1.

, , M7RT (M7\RT

12. (d) : AHNMT cannot be predicted unless we know i heat of dissociation of acetic acid.

13. (d) : 50 ml of 1 M KOH + 50 ml of 0.5 M H2S04

will involve complete neutralisation. 14. (c): Increasing concentration of a product favours reverse reaction.

15. (a) : Lower the equilibrium constant, greater is the stability. 16. (c) : On dilution, ionisation increases. 17. (d) : pH = 10 means [H+] = IO"10 M.

18. (b) : Here one element of KC103 is oxidised - 2 0 +5 - }

(O —» O,) while the other is reduced (CI —» CI) and hence it is an example of intramolecular redox reaction. 19. (b) : NO, N2H4

To balance N atoms, multiply NO," by 2, we have +5 - 2

2N03- N2H4

Total O.N. = 2 x +5 Total O.N. = 2 x -2 = +10 = - 4

To balance O.N., add \4e~ to L.H.S., we have 2N03- + 14e~ —» N2H4

or, N0 3 - + 7e~ -> 1/2 N2H4

Therefore, number of electrons involved in the reduction of N03" ions is 7. 20. (c) : At anode (made of Pt), H : 0 is oxidised more easily than S04

2~ ions. H+ ions produced combine with

18 CHEMISTRY TODAY | SEPTEMBER '05 44

SO„2 ions to produce H2S04 . Cu2+ ions are deposited on cathode. 0 2 is evolved at anode. 21. (b) : Equivalent conductance increases with dilution and ultimately becomes constant.

22. (d) : Ionic conductance of ion = transport number of that ion x Aeq° of strong electrolyte containing that ion = (1 - 0.40) x 160.84 = 96.504

ionic conductance _ 96.504 _ 3 Ionic mobility = — - 1 0 96,500 96,500

23. (c): Lower the activation energy, faster is the reaction.

24. (b) : As it is a reaction of 1st order,

t [A] , 2.303, i.e.. k = log

0 p N2O5

PNjOJ

° f ' 2.303 From slope, k can be calculated.

, , , 0.693 . 2.303, 25. (c) : k = — m i n = log

-t + l o g p N 2 O 5

a

o r , '99.9%

45 2.303x45

0.693

'99.9% a -0.999a

log 10 = 448 min

,1 = 7— hours.

2

26. (a) : (a) has all atoms with A — Z - N same.

27. (b) : 2H Th -> 2£f Pb + * \ a + y p 232 = 208 + 4x or, x = 6 90 = 82 + 2 x - y or, y = 4.

28. (d) : x = ^ ^ l o g — = 2 . 3 x l 0 - 2 m i n - 1 . 30 14

Initially, activity = \N i.e. 28 = 2.3 x 10-2 x or, N = 1217 = 1200. 29. (b) : Gelatin stabilises the colloids and prevents crystallisation.

30. (a) : ZSM-5 (size selective catalyst) converts alcohol to petrol.

31. (d) : IE, of B is lower than that of Be because in B, a 2p-electron is to be removed while in Be it is the 2\-electron. Similarly, IE, of AI is lower than that of Mg because in Al, a 3/7-electron is to be removed while in Mg it is the 3,v electron. IE, of N is higher than that

of O because of extra stability of exactly half-filled 2p-orbitals in N. IE, of Mg is higher than that of Na because of higher nuclear charge and completely filled 3.?-orbital in Mg.

32. (a) : IE + EA = 275 + 86 = 361 k cal mol"1

= 361 x 4.184 = 1510.42 kJ mol"1

1510.42 O D Electronegativity = — — = 2.797 = 2.8

33. (c) : As the reduction potential of hydrogen is lower than that of sodium, it will be discharged at the cathode and chlorine will be discharged at the anode.

34. (d) : Calamine is ZnC0 3 and zincite is ZnO. Both are minerals of zinc.

35. (b) : IE of H atom is 1312 kJ mol"1. 36. (b) : Na,ALSi208 • *H 2 0 + Ca2+ . , , >

- z from hard water zeolite

CaAI2Si208 • JCH20 + 2Na" 37. (a) : H+ ions are discharged at a higher potential when Hg cathode is used than that using Pt cathode.

38. (a) : Be shows coordination number of four in BeO which is covalent and has zinc sulphide (wurtzite) structure while all others show a coordination number of six. (NaCl structure) and are ionic.

39. ( c ) : Generally Mg(OH)2 is used as an antacid. The metal is Mg which burns in C0 2 atmosphere.

2Mg + C 0 2 -> 2MgO + C.

40. (d) : 3 0 2 S l ' e n t f c t n c > 2 0 ,

A discharge

41. (a) : Alkaline earth metal oxides are more basic than transition metal oxides which in turn are more basic than non-metal oxides and thus the acidic character increases from left to right in a period.

42. (b) : A mixture of one part of powdered Al and three parts of Fe203 is called thermite.

43. (c) : A i d , • 6H 2 0 d l s s o c i a t i o" > [A1C12(H20)4]+ + [AIC14(H20)J-

44. (c) : Water in presence of oxygen reacts with Pb (pipes) to form soluble Pb(OH)2 which gives poisonous Pb2+ ions in solution.

Pb + 0 2 + H 2 0 -> Pb(OH)2

45. (b) : Element Y belongs to group 14 of the periodic table which forms two chlorides y d 4 (a colourless, volatile liquid) and KC12 (a colourless solid). 46. (a) : Extent of hydrolysis increases in the order:

CC14 < MgCl2 < A1C13 < SiCl4 < PCI5.

C H E M I S T R Y T O D A Y | SEPTEMBER '05 4 5

47. (c) : Both B and AI have 2 (Is2) and 8 (2s2 2p6) electrons in their penultimate (last but one) shell respectively but the remaining elements of group 13 have 18 electrons in their penultimate shell and hence B and AI show different properties from those of remaining members of group 13. 48. (b) : Reducing character of hydrides of nitrogen family increases as the size of the central atom increases down the group. All other properties stated here decrease down the group. 49. (d) : The compound contains C, H and S as the elements which on burning give C02 , H 2 0 and S02 as oxides respectively. First oxide (C02) turns lime water milky, the second oxide (H20) turns anhydrous (white) CuS04 blue due to hydration and the third oxide (S02) dissolves in H 2 0 giving an acid (H2S03) which lowers the pH of the solution. C, H, S are the elements in a compound.

C + 0 2 — C 0 2 ; 2H2 + 0 2 — 2 H 2 0 (1) (II)

s + o2 so2 (III)

C0 2 + Mg(OH)2 -4 MgCOj + H 2 0 (I) lime water insoluble

(milkiness)

5H20 + CuS04 CuS04 • 5H20 (II) colourless blue

S02 + HzO H2S03

(III) acid, lowers pH of the solution

50. (c) : The shape of SF3C13 molecule is octahedral. 51. ( d ) : He has lowest boiling point due to weak van der Waals forces between its atoms. 52. (c) : All noble gases except He (which has Is2) have ns2 np6 electronic configuration of the valence shell. 53. (a) : Na2Cr207 is hygroscopic and cannot be used as a primary standard in volumetric analysis. 54. (b) : IUPAC name is potassiumtrioxalato aluminate(III). 55. (c) : Cl2 + H 2 0 HCl + HCIO;

X AgN03 + HCl AgCl + HN03

white ppt.

2HC1 + Mg MgCl2 + H2 T Y

56. (b) : Sewage is a biodegradable pollutant. 9 io l , r u

»••«. ' O C t 7 6 4 3

2-methylspiro[4.5]deca-l,6-diene

58. (c) : Only option (c) is correct as shown below. CI CI I H2 I

CH3CH2 - C — CH - CH3 —-

3-chloro-2-pentene (A)

CI

2-chloro-2-pentene (B)

• CH3CH2 — CH — CH2CH3

optically inactive

CI

CH optically active

59. (d) : Angle increases progressively. sp3 (109° 28'), sp2 (120°), sp( 180°)

60. (b) : As the s-character increases or the ^-character decreases, the energy of the orbital decreases. Thus the order is p > sp3 > sp2 > sp > s.

61. (d) : Wurtz reaction.

62. (b) : Ozonolysis since all the three alkenes will give different products as shown below.

(•)03 CH3CH2CH — C H 2

1-buterie

CH3CH=CHCH3

(ii) Zn/H20

(>)o3

» CH3CH2CHO + HCHO

(ii) Zn/H20 > 2CH3CHO

(CH3)2C = CH2 J> (CH3)2CO + HCHO (ii) Zn/H20

CH,

63. (b) : C H 3 - C = CH2 + C6H6 isobutene

(CH3)3C-C6H5

ferf-butylbenzene

64. (c)

0 _ N a +

NaOH^ rj- ^ r " N 0 2

The reaction occurs by activated nucleophile substitution.

65. (b) : CC14 + 4KOH -4KC1

> C(OH). 4 - H 2 0

HO - CO - OH > K2C03 carbonic acid 2

18 CHEMISTRY TODAY | SEPTEMBER '05 46

66. (d) : Since compound Y reacts with I2 and Na2C03

to form triiodomethane, therefore, Y must be a methyl ketone. Since Y is obtained by oxidation of X with K.2Cr207, therefore, Xmust be a methylcarbinol, thus X is CH3CHOHCH3. 67. (a) : During oxidation of unsymmetrical ketones, i.e. /7-propyImethy I ketone, the keto group stays with the smaller alkyl group (Popoff's rule) i.e. CH,CH2COOH and CH3COOH are formed.

CH3COCH,CH,CH3 [O] CH3COOH + CH3CH2COOH.

68. (a) : C6H5CH2C1 - ^ ^ [ C 6 H 5 C H 2 - 0 N 0 2 ]

C6HSCHO + HNO, 69. (d) : Mol. wt. of C4H10O4

= 4 x 12 + 10 x l + 4 x 16 = 122 Mol. wt. of acetylated product = C|2H|808

= 12 x 12 + 18 x 1 + 8 x 16 = 290. Increase in mass due to acetylation = 290 - 122

= 168 amu Increase in mass due to acetylation of one OH group

= 42 a.m.u.

Total number of OH groups 168 42

= 4.

70. (a) : Phenol is a weaker acid than carboxylic acid. H N O ,

A r - N = N C P diazonium salt

71. (c) : Ar-NH, - j ^ p aromatic 1° amine

72. (a) : Of all the amines listed, only (a) has the right molecular formula of C6HnN. Since it also contains a chiral carbon, therefore, it is optically active.

NH,

M.F. C6H,,N 73. (b) : Poly dispersity index. 74. (d) : Ascorbic acid - scurvy

Retinol - night blindness Riboflavin - cracked lips Thiamine - beri beri

75. (a) : Salbutamol is structurally related to adrenaline.

Dear Students, W e know that its too difficult to remember the elements in their correct order in the first three periods of the table. So we are presenting some mneumonics to leam few very difficult things in Chemistry in a very easy way.

1.

2.

The order of hardness (Mohs' scale of hardness) Tall gyroscopes can fly apart, orbiting quickly to complete disintegration or, Those girls can fly and other queer things can do. Talc, gypsum or rock salt, calcite, fluorite, apatite, orthaclase, quartz, topaz, corundum, diamond.

The molecular shape of fructose and glucose There are two simple sugars (monosaccharides) - fructose and glucose - which combine in a specific way to form sucrose. Remember the molecular shape of fructose has five carbons (letter

MEMORY Contributed by Neha Chandak, Nagpur

3.

f starts both words) and hexagon is the shape of glucose (letter g in both words).

The elements in their correct order in the first three periods of the table (Hydrogen being omitted and potassium included). Here lies Benjamin bold cry not old friend needlessly. Nature magnifies all simple people sometimes, clots and kings. Hydrogen, helium, lithium, beryllium, boron, carbon, nitrogen, oxygen, fluorine, neon, sodium, magnesium, aluminium, silicon, phosphorus, sulphur, chlorine, argon, potassium (H, He, Li, Be, B, C, N, O, F, Ne, Na, Mg, Al, Si, P, S, CI, Al, K)

If you have such mneumonics in Physics, Chemistry, Botany or Zoology which can he proved as aid to memory then send them to us. On selection we will publish them with your name in our monthly magazines Physics For You, Chemistry Today and Biology Today.

CHEMISTRY TODAY | SEPTEMBER '05 47

ORGANIC

CH2NH2

-NH 2 NaN02/HCl. » Product, Product is

C H 2 N j

(a)

(c) >N = N

Compound

VT(C10HnN) dil.H2S04

N H — N H

MQ

ro CH2N2/sunlighil

> Product, followed by steam catalyst' Product is

(a)

( 0 [ O

(b)

(d)

ro NH.

Which will not perform iodoform reaction with I2/OH ? (a) CH3COCH2CH3 (b) CH,CONH2

(c) C H.COCH, 3 . C H ,

(d) ( C H 3 ) 2 C H C H 2 C O C H ^ C H 2 C H 2 C H 3

In which of the following reactions the product written is not the main product ? (a) CH2 = CHCH2Cl(excess) + Mg >

CH2 = CHCH2 - CH2CH = CH2

(b) C 1 C H 2 - C = CH2 + Mg > C 1 C H 2 - C = CH2

Br MgBr

anhyd. (c) PhH + Ph3C - COC1 A1(;1 > Ph4C

(d) PhCH = O + ND NHCONH > PhCH = NNHCONH,

C " = N

C - = N

(a)

(c)

O H "

0 1 8 - CH,

NH,

NH.

O- (b)

(d)

A compound of formula C6HeO on reaction with excess Na produces 238 litres H2 (at N.T.P) per Kg of the compound. The compound will be (a) PhOH (b) CH2C = C - 0 - C H , C = C - H (c) CH — C - C EE C - CH2CH2OH (d) H - C = C - CH(OH) - CH2C = CH

Each of the following compound undergoes reaction with bleaching powder. The yield of chloroform is minimum in which case ?

CH2COCH3

CH2COCH3

OMe

CH2CHOHCH3

CH,CHOHCH,

Contributed by : Nirmal Singh, Chemistry Classes for IIT-JEE, Kanpur (Mob. : 09839105810)

CHEMISTRY TODAY I FEBRUARY '08

Starch is a polymer of (A) and cellulose is a polymer of (B). (A) and (B) are (a) a-glucose, (3-fructose (b) p-glucose, a-fructose (c) a-glucose, a-fructose (d) a-glucose, P-glucose

For coupling reaction of phenol the best medium is (a) a buffer containing 0.1 M BC1 and 0.1 M BOH

(ptfbBOH = 9) (b) a buffer containing 0.1 M HA and 0.1 M NaA

( p ^ = A- = 9) (c) 0.1 M NaCl (d) a buffer containing 0.1 M HA and 0.1 M NaA

(P* =9) In which of the following the formation of product

in first reaction is faster than that in second reaction ? (a) (i) PhH + P h C l - *

A1C1,

(ii) PhH + CH, anhyd.

: C H C H 2 C l ^ r >

(b) (i) P h N 0 2 " ^ >

(ii) PhNHAc Br, - Fe

(c) (i) p-Cresol + sodamide — > (ii) Benzyl alcohol + sodamide >

(d) (i) HCHO + H 2 0 > (ii) CCI3COCCI3 + H 20 >

I OH ^ k + s heat Product. Product is

Me Ph (a) MeOMe

(c) HO

•Me (b) PhOH

(d)

Me Ph

Me

Me Ph O

(a)

» ( A ) , (A) is

(b)

(c) (d) None of these

, (A) is

(a)

(c)

Br / ^ / O C H O

There are following four statements regarding hydrolysis of acid derivatives :

M e O - ( ^ ^ ) - C O O E t is less reactive than I.

NO r ® -

COOMe

O

II. N Q 2 - < 0 ) - NH - C - Me is more reactive than

MeO

0 II

NH - C - Me.

f ^ O III. L ^ ^ ^ - Q is less reactive than

O

1 0

Q* 0

XV. M e - C - O - C H j js j e s s reactive than O II

Me - C - S - CH3. Of these the correct statements are (a) i,n,m (b) n,m,iv (c) 1, iv (d) i, 11, in, iv

An organic compound (A) {C10H12} on treatment with hot acidic KMn04 gives a dibasic acid which on monosubstitution gives a mixture of three isomeric product. (A) is

Pr(«)

•Vinyl

-Vinyl

B C H E M I S T R Y TODAY I FEBRUARY '08

Consider four reactants : (I) PhCH3 (II) PhCH2Cl (m)PhCHCi2 ( rv)Phcci3 Consider bromination of each compound in presence of Fe catalyst. The ratio of w-product to p-product in four

m(A) m(5) m(C) m(D) cases is represented by ^ J y ^ J y p ( C ) and p ( D ) .

Which of the following is correct ?

(a) m(A) m(D) P(A)

(c) — >

P(D) P(D)

(b) m(C) > m(5)

m(A) p(A) 17. Column (A)

Conc.H2S04

Al EtOH 383 K

A 2 EtOH

A3 EtOH

413 K

P(C) P(B)

(d) Both (b) and (c)

Column (B)

Bl ester

52 ether

53 ethylene 433 K (i) MeCOOH ^

A4 EtOH (ii) Pyrolysis 51

(a) Al -51 , A2-52 , A3 - S3 (b) A2 - 52, A3 - 53, A4 - 53 (c) Al - 51, A3 - 53, A4 - 53 (d) Al - 52, A2 - 53, A3 - 53

When 2-methylidene cyclobutanol is treated with dil. H2S04, (A) is obtained as major product. (A) is

(a)

(c)

(b)

(d)

CH2 —™ CH — CH — CH2

OH on treatment with Mn02 gives

OH OH

CH2CH(OH)Me

^^ CHr, — CH — COCH2 -CH2CH(OH)Me

( b ) CH2 = CHCH(OH) - CH2 J p y CH2COMe

CH, = CHCOCH, - CH2COMe

( d) CH2 - CH - C H ( O H ) C H 2 - ^ Q V

OH OH

CH,CHMe I OH

Me

PhCH2 - N+- CH2COCH3 Product.

Product is

CH2CMe3

Me

(a) PhCH2 - N - CHCOCH3

(b)

(c)

(d)

Me I

N - C H

CH2CMe3

CH2CMe3

^-CHjPh

^ C O C H 3

'3

Me 1 /CH,CMe 3

P h C H 2 - N - C H C \COCH3

Me I

PhCH - N - CH-jCOCH,

CH2CMe3

SOLUTIONS (b) (b) (a) (d) (d) 10 (c)

13 (a) (d) 15 (c) (c) (a) 20. (b)

• • Note : Chem Challenge published in January issue was contributed by

Nirmal Singh

(c) (d) (c) (a) (c) (b) (d) (d)

Forthcoming Competitive Exams CBSE P M T Preliminary Exam April 6 IIT-JEE April 13 West Bengal JEE April 20 MGIMS April 20 CBSE AIEEE April 27 AFMC May 4 BHU P M T Screening May 7 BITSAT May 9 to 12 June CBSE PMT Mains Exam May 11 DPMT May 18 DCE May 31 BHU P M T Mains June 15

B C H E M I S T R Y TODAY I FEBRUARY '08

CHEM

emus RULES

All students preparing for PMT/PET examinations can participate in Chem-Genius Contest. Answers marked only on the entry form of the magazine / photocopy of form will be accepted. More than one response to a question will be disqualified. Prizes • 1st Prize - LG Mobile phone • 2nd Prize - Adidas Bag • 3rd Prize - MTG Books (worth Rs. 500/-) The entries with maximum number of correct answers for three consecutive months (February '08 - April '08) will be awarded 1 st prize. 2nd and 3rd prize will be given to the next maximum scorers. In case of a tie, the winners will be decided through a lucky draw. The decision of the editor will be final and binding in all cases and will not be a matter for consideration of any court and no correspondence will be entertained. Name and photograph of the prize winners of this contest will be published in the June issue of 2008. MTG is not responsible for any postal delays, transit losses or mutilation of entries. Last Date The entries should reach on/betore 29lh February '08 to - Chem-Genius Contest-3, 406, Taj Apartment, Ring Road, Near Safdarjung Hospital, New Delhi-29. [Note : Enclosures include a passport size photograph and a photocopy of age proof.] The result of contest -2 (November '07 - January '08) will be published in March '08 issue.

Ustic acid is a natural compound found in lichens. Which statement correctly represents four of the functional groups present in ustic acid?

h 3 C x ^ c

o

H — C — O H C H 3 0 ^ J L / C 0 2 H

hoAAOH H

(a) Carboxylic acid, ketone, aldehyde and alcohol (b) Ether, ketone, alcohol and aldehyde (c) Carboxylic acid, ketone, phenol and alcohol (d) Ester, phenol, carboxylic acid and ketone

Which of the following constitutes irreversible colloidal system in water as dispersion medium?

(a) Clay (b) Platinum (c) Fe(OH)3 (d) All of these

The second electron gain enthalpies (in kJ mol-1) of oxygen and sulphur respectively are

(a) - 780, +590 (b) - 590, + 780 (c) + 590, + 780 (d) + 780, + 590

4.

(a) (d)

Which one of the following chlorides will not fume in air? BiCl, CCL PCI,

(a) (c)

(b) i5 (d) None of the above.

Partition coefficient of benzoic acid-ether-water in favour of ether is 2. A solution containing 8 g/litre benzoic acid in ether layer is shaken with 2 litre water. The concentration of acid in water layer 1 (b) 2 3 (d) 4

B C H E M I S T R Y TODAY I FEBRUARY '08

Of the five AH values required to calculate a lattice energy using the Born-Haber cycle, the one that is most difficult to measure is

(a) the electron affinity of the non-metal (b) the heat of formation of gaseous atoms of the non-

metal (c) the ionisation energy of the metal (d) the heat of sublimation of metal

7. Which of the following is a true peroxide? (a) N0 2 (b) Mn0 2

(c) BaOz (d) S0 2

8. If the distance between Na+ and Cl~ ions in sodium chloride crystal is X pm, the length of the edge of the unit cells is

(a) 4X pm (b) X/4 pm (c) XII pm (d) IX pm

The correct sequence in which bond order decrease in 0 2 , 0 J , 0 j and O§" is such that

(a) 0 2 > 0 2 > 0 2 > 0 2 -

(b) > 0 2 > 0 2~ > 0 2

(c) 0 2 > 0 2 ~ > 0 2 > 0 2

(d) 0 2 > 0 2 > 0 2 ~ > 0 2

10. Hydrazine reduces Fehling solution to (a) CuO (b) Cu 20 (c) Cu + 0 2 (d) Cu(OH)2

The final product of the following reaction is

(a)

(c)

(d)

Sodium nitrate decomposes above 800°C to give (a) N2 (b) 0 2

(c) N0 2 (d) Na 20

Which of the following is known as invert soap? (a) Pentaerythritol monostearate (b) Sodium stearyl sulphate (c) Trimethyl stearyl ammonium bromide (d) Ethoxylated nonyphenol

Potential energy of Li2+ electron is

(a)

^ 4K

3ez 3ez

4n e0 r

-2e2

4n e 0 r (b)

15. The enthalpy changes for two reactions are given by the equations

2Cr (s)+ ~ 02(g) —> Cr203(j )

c ( J ) +-o2 ( g )^co ( s )

AH = -1130 kJ

AH = -110 kJ

What is the enthalpy change, in kJ, for the reaction? 3C(S) + Cr203(s) —> 2Cr(s) + 3CO(g)?

(a) - 1 4 6 0 (b) - 8 0 0 (c) + 800 (d) + 1020

16. Arsine (AsH3) is a molecular hydride of a group V element. The most likely structure for the arsine molecule in the vapour phase is

H

(a) H—As—1-H £i 90°

As (C) I I < _ A ' " H

109° H

(b)

(d)

H K

,As ) 120°

As

h < 3 > H <109oH

17. Which one of the following notations shows the product incorrectly?

(a) 96Cm242 ( a , 2n) 97Bk243

(b) 5B10 ( a , n)7N1 3

(c) 7N14 (n, p) 6C14

(d) 14Si28 (d, n) 15P29

18. The concentration of a coloured substance is to be determined by measuring the absorbance of an aqueous solution of it and interpolating from a graph of absorbance versus concentration. Which procedural error will result in a concentration that is too high? Procedural Mistakes

I. rinsing the cuvette with water just before filling it with the unknown solution

II. measuring the absorbance of the unknown solution at a wavelength other than its maximum

III. using a cuvette for the unknown solution that has a slightly longer path length than you thought it had

(a) I only (b) III only (c) I and II only (d) II and III only

19. Pick up the wrong statement? (a) Anhydrous A1C13 exists as A12C16 (dimer).

7 0 CHEMISTRY TODAY I FEBRUARY '08

(b) Anhydrous A1C13 is a trigonal planar molecule. (c) Anhydrous A1C13 fumes in air. (d) Anhydrous A1C13 is ionic.

20. What will be the partial pressure of H2 in a flask containing 2 g of H2, 14 g of N2 and 16 g of 0 2 ?

(a) 1/2 of the total pressure (b) 1/3 of the total pressure (c) 1/4 of the total pressure (d) 1/6 of the total pressure

21. Parke's process is used (a) to extract Ag using NaCN (b) to extract Cu using CuFeS2

(c) to extract Ag from argentiferrous lead (d) to extract Ag by forming amalgam

22. In the following sequence of reactions, what is D1 CH, o [O]

>A SOC1, NaNO, Heat.

>D

(a) primary amine (b) an amide (c) phenyl isocyanate (d) a chain lengthened hydrocarbon

23. Heat liberated when 100 mL of 1 N NaOH is neutralised by 300 mL of 1 N HCl is

(a) 22.92 kJ (b) 17.19 kJ (c) 11.46 kJ (d) 5.73 kJ

24. Pick up the wrong statement? (a) Borax is used in the manufacture of optical glasses. (b) Borax is used as a flux. (c) Borax is used as a water softener. (d) Borax is not used for glazing paper.

25. The ratio of amounts of H,S needed to precipitate all the metal ions from 100 ml of 1 M AgN0 3 and 100 ml of 1 M CuS0 4 will be

(a) 1 : 1 (b) 1 : 2 (c) 2 : 1 (d) none of these

26. Which one of the following compounds is not a protonic acid?

(a) B(OH)3 (b) PO(OH)3

(c) SO(OH)2 (d) S0 2 (0H) 2

27. Calculate the percentage of pyridine, C5H5N, that forms pyridinium ion, C5H5NH+ , in a 0.10 M aqueous pyridine solution. (Kh for C5H5N = 1.7 x IO"9)

(a) 0.0060% (b) 0.013% (c) 0.77% (d) 1.6%

MCQ ' i Memory

C o n t e s t 1. Who can participate

If you have taken any of the exams given below and possess plenty of grey cells, photographic memory then you are the right candidate for th is contest. All you have to do is wr i te down as many questions (with all choices) you can remember, neatly on a paper w i th name of the exam, your name, address, age, your photograph and mail them to us.

2. The exams . PMT : AFMC, BHU, AIIMS, JIPMER, Delhi PMT, H.P., WB JEE, Punjab, Haryana CET, Safdarjung VMMC, CMC Vellore, Maharashtra CET, U.P. CPMT,OrissaJEE,M.P. PMT Engineer ing : DCE, UPSEE, Haryana CEE, MP PET, J & K CET, AMU, WB JEE, Bihar CECE, Jharkhand CECE, Orissa JEE, Maharashtra CET, PunjabPET,....

3. The Benefits Plenty! Each complete question wi th answer wi l l make you richer by Rs. 5* / - More the questions, the merrier it w i l l be. We wi l l make you famous by publishing your name (photo if possible). Also you can derive psychological satisfaction from the fact that your questions wi l l benef i t thousandsof readers.

4. and lastly the pitfal ls Don't send incomplete question. Our panel of experts wi l l cross-check your questions. You have to send it w i th in a month of giving the part icularexam. Mail to: The Editor, MTG,406,TajApt„ Ring Road,

Near Sa fda r j ung Hosp i ta l , New D e l h i - 2 9 . Tel. : ( 0 1 1 ) 2 6 1 9 4 3 1 7 , 2 6 1 9 1 6 0 1 .

* Conditions apply o Payment will be made after the MCQs are published. O Kindly note that each question should be complete. O Payment will be made only for complete questions. O Preference will be given to the reader sending the maximum

complete and correct questions. Other conditions apply. The decision of the Editor, MTG shall be final and binding.

B CHEMISTRY TODAY I FEBRUARY '08

Match list I with list II and select the correct answer using the code given below

List I List II Successive ionization energies

(kJ mol"1) Elements

IE] IE2 IE, 1. 2080 3963 6130 («) H

2. 520 7297 11810 (b) Li

3. 900 1758 14810 (c) Be

4. 800 2428 3660 id) B (e) Ne

(a) 1. (c), 2. (b), 3. {d), 4. (e) (b) 1. (a), 2. (c), 3. (b), 4. (d) (c) 1. (d), 2. (a), 3. (b), 4. (d) (d) 1. (e), 2. (b), 3. (c), 4. (d)

Which is the constitutional isomer of the given compound ?

(a)

(c)

(b)

(d) both (a) and (c)

Benzene diazonium chloride when reduced with sodium sulphite yields

(a) chlorobenzene (b) benzene (c) phenyl hydrazine chloride (d) phenol

Consider the following equilibrium

CH nCH -1 ' 2 CH3CH2CH2-

C H 2 C H 3

H CH2CH 2CH

Which of the following best describes the equilibrium constant K of this interconversion?

(a) K< 1 (b) K> 1 (c) K= 1 (d) K cannot be deduced from this information

Which of the following expression gives the effect of temperature on the rate constant?

(a) In k = In A - EJRT (b) In k = In A + EJRT

(a) (c) 34. (a) (b) (c) (d)

(c) In k = A - EJRT (d) k = In A + In EJRT

33 The solubility product for SrF2 is 7.9 x 10~n. A solution is 0.01 M in NaF. The minimum concentration of SrCl2 in the solution required to start the precipitation of SrF2 is equal to 0.89 x 10~5 M (b) 0.79 x 10"6 M 0.79 x 10"8 M (d) 0.89 x 10"7 M

Ellution in the chromatography is the process for crystallization of compound separation of compound extraction of compound distillation of compound

If phosphate is present as an interfering radical in a mixture but III, IV, V groups and Mg2+ are absent, then

(a) PO^"" must be removed before the analysis of III group and onwards

(b) PO^ - will not interfere

(c) PO4 - interferes in III group only

(d) Interference of PO^ - is must whether III group and other groups are present or not.

Magnetite, Fe304 can be converted into metallic iron by heating with carbon monoxide as represented by this equation. Fe304(l) + 4 COte) -» 3 Fe(j) + 4 C02(g)

How many kilograms of Fe304 must be processed in this way to obtain 5.00 kg of iron if the process is 85% efficient?

Substance Molar mass Fe304 232 g mol'1

(a) 6.92 kg (b) 8.15 kg (c) 20.8 kg (d) 24.4 kg

Which of the following is an unsaturated acid? (a) Linoleic acid (b) Stearic acid (c) Myristic acid (d) Laurie acid

Which of the following curve gives the variation

of A~ with 4 c for CH3COOH?

t Vc

(a)

t Vc

(b)

A„ A„

7 2 CHEMISTRY TODAY I FEBRUARY '08

t Vc

(c) (d) None of these

A,

Which of the following reaction is said to be entropy driven?

(a) Endothermic reaction with positive entropy change and high temperature

(b) Endothermic reaction with negative entropy change and low temperature

(c) Exothermic reaction with positive entropy change and high temperature

(d) Exothermic reaction with negative entropy change and low temperature

40, The size of the nucleus is approximately (a) 1/100th of an atom (b) l/1000th of an atom (c) 1/10000th of an atom (d) 1/100000th of an atom

41. Which of the following ion is coloured? (a) Cu+ (b) Cu2+

(c) Ti4+ (d) V5+

42 Which of the following does not show stereo isomerism

(a)

CH, /

H

CH, CH, H CO-NH

^ N H - C O ^ H

(c) CH3CH2C02H

(b)

(d)

CO-HN

CO ^ H N -H

CO,H I

H - C - O H I

H - C - O H

C02H

CH,

43. Argol, a brown crust formed during the fermentation of grape juice contains

(a) C0 2 (b) Fuse oil (c) Potassium hydrogen tartarate (d) Lye

44. A metal nitrate on reaction with KI gives black precipitate and with excess KI gives orange solution. The metal ion is

(a) Hg2+ (b) Pb2+

(c) Cu2+ (d) Bi3+

45. 23Na is the more stable isotope of Na. Find out the process by which Na can undergo radioactive

decay (a) (3-emission (b) cx-emission (c) [^-emission (d) K-electron capture 46. For anionic hydrolysis, pH is given by

(a) pH = i p K w - i p K f e ^ logC

(b) pH = ^ pK„

(c) P H = l p K H

2

,+ l p K a + l logC

(d) None of the above

47. What is the value for AG° when [Ag+] = [Cu2+] = 1.0 M?

(a) - 44.4 kJ (b) - 88.8 kJ (c) - 243 kJ (d) - 374 kJ

The hypothetical complex chlorodiaquatriam-minecobalt(III) chloride can be represented as

(a ) [ C O C 1 ( N H 3 ) 3 ( H 2 0 ) 2 ] C 1 2

(b) [Co(NH3)3(H20)Cl3]

(c ) [ C O ( N H 3 ) 3 ( H 2 0 ) 2 C 1 3 ]

(d) tCo(NH3)3(H20)3]Cl3

Which of the following does not represent a detergent molecule?

(a) CH 3 (CH 2 ) n CH 2 -^Q>—OCH 2 (CH 2 ) 1 2 CH 3

(b) CH3(CH2)nCH2 - o -

SOlNa

(c) C H 3 ( C H 2 ) 9 C H 2 - ^ Q > —

0(CH2)20(CH2)20(CH2)20H

(d) CH3(CH2)I2CH2C02- Na+

Mark the false statement. (a) A salt bridge is used to eliminate liquid junction

potential (b) The Gibbs free energy change, AG is related with

electromotive force E as AG = - n FE (c) Nernst equation for single electrode potential is

E = E RT

(d) The efficiency of a hydrogen-oxygen fuel cell is 23%.

B CHEMISTRY TODAY I FEBRUARY '08

The structure for neo-heptyl alcohol is CHoCHi i

(a) C H 3 C H 2 - C - O H

CH^CH^ CH 3

(b) C H 3 - C - C H 2 C H 2 C H 2 O H

C H 3

CH 3

(c) C H 3 - C - C H C H 2 C H 3 I I

H 3 C OH (d) n - C7H15OH

Which is the decreasing order of stability?

(i) C H 3 - C H - C H 3 (ii) C H 3 - C H - 0 - C H 3

(iii) CH 3 CHC0CH 3

(a) (i) < (ii) < (iii) (b) (i) > (ii) > (iii) (c) (iii) > (i) > (ii) (d) (ii) > (iii) > (i)

Which of the following pairs of compounds are enantiomers?

C H , CH 3

(a)

(b)

H-HO-

-OH -H

CH3

CH3

H-HO-

-OH -H

and

and

HO-HO-

-H -H

CH3

CH3

H -H -

OH - O H

CH3

CH,

CH3

CH, (c) HO-

H" •H -OH

and HO-HO-

-H -H

CH3

CH3

C H ,

(d) H-HO-

CH, -OH and HO-"H H-

CH,

-H -OH

13 CH3

Acrilan is a hard, horny and a high melting material. Its structure is

(a)

(c)

CH3 I

- C H 2 - C -I COOCH,

- C H 2 - C H -I

CI

(b)

(d)

- C H , - CH— I COC2H5

- C H 2 - C H -I CN

Cyclohexanol is dehydrated to cyclohexene on heating with conc. H2S04. If the yield of this reaction is 75% how much cyclohexene will be obtained from 100 g of cyclohexanol? (a) 61.5 g (b) 75.0 g (c) 20.0 g (d) 41.0 g

Number of n electrons present in naphthalene is (a) 4 (b) 6 (c) 10 (d) 14.

An ester (A) with molecular formula C9H10O2 was treated with excess of CH3MgBr and the compound so formed was treated with conc. H2S04 to form olefin (B). Ozonolysis of B gave ketone with formula C8HgO, which shows positive iodoform test. The structure of A is (a) CH3CH2COC6H5 (b) C6H5COOC2H5

(c) C6H5COOC6H5 (d) CH3COC6H4COCH3.

What is the concentration of nitrate ions if equal volumes of 0.1 M AgN03 and 0.1 M NaCl are mixed together? (a) 0.1 N (b) 0.2 N (c) 0.05 N (d) 0.25 N

100 mL of 1.0 M HC1 are mixed with 75 mL of 1.0 M Na2C03. The resulting solution will be (a) acidic (b) basic (c) neutral (d) amphoteric.

The density of H2S (mol wt. 34) at 27°C and 2 atm pressure is (R = 0.0821 atm/mol K) (a) 2.76 g/L (b) 27.6 g/L (c) 2.76 mg/L (d) none of these.

The standard reduction potentials for the two half cell reactions are given below

Cdw , £° = - 0.40 V Ctf\aq) + 2e-Ag\aq) + e-^Aj>(s),E> = 0.80 V

The standard free energy change for the reaction

B C H E M I S T R Y TODAY I FEBRUARY '08

2Ag+(a?) + Cd(j) • • 2Ag(j) + C d 2 +

W

is given by (a) 115.8 kJ (b) -115.8 kJ (c) -231.6 kJ (d) 231.6 kJ

A neutral fertilizer among the following is (a) CAN (b) ammonium sulphate (c) ammonium nitrate (d) Urea.

The correct order of basicities of the following compounds is

(1) CH3— c t

(3) (CH3)2NH

iNH

- N H 2

(2) C H 3 - C H 2 - N H 2

O II

(4) C H 3 - C - N H 2 .

(a) 2 > 1 > 3 > 4 (b) 1 > 3 > 2 > 4 (c) 3 > 1 > 2 > 4 (d) 1 > 2 > 3 > 4.

50 mL of 10 N HCl, 25 mL of 12 N HCl and 40 mL of 5 N HN0 3 were mixed together and volume of the mixture was made 1000 mL by adding water. The normality of the resulting solution will be (a) I N (b) 2 N (c) 3 N (d) 4 N.

The oxidation number of Co in [Co(NH3)3](N02)3

is (a) + 3 (b) 0 (c) - 3 (d) 6

The reaction

(CH3)3C - Br (CH3)3C - OH is (a) elimination reaction (b) substitution reaction (c) free radical reaction (d) addition reaction.

A compound formed by elements A and B has a cubic structure in which A atoms are at the corners of the cube and B atoms are at the face centres. The formula for the compound is (a) A2B3 (b) AB3 (C) A3B (d) A3B2.

Which of the following forms a toxic substance in blood by combining with haemoglobin? (a) carbon dioxide (b) carbon monoxide (c) oxygen (d) methane.

Which of the following rate law has an overall order of 0.5 for reaction involving substances x, y and z? (a) Rate = k(Cx)(Cy)(Cz) (b) Rate = k{Cx)°\Cy?\Cf * (c) Rate = k ( C x y \ C y r ( C z f (d) Rate = k(Cx)(Cy)0-5 / (Cz)2

In phosphorus pentoxide each P atoms is linked to (a) 4 oxygen atoms (b) 2 oxygen atoms (c) 3 oxygen atoms (d) 10 oxygen atoms.

^ J ^ + CHClBr2 •

Predominant product is

« ^

Me3COK^ p r 0 £ j u c t

(c) CI CI

(b)

(d)

Me Me

Which of the following transition have minimum wavelength? (a) n4 —» n1 (b) n2 —> n1

(c) «4 —» n2 (d) n3 —> «!

The Ksp of Mg(OH)2 is 1 x 10~12. 0.01 M Mg(OH)2

will precipitate at pH = (a) 3 (b) 9 (c) 5 (d) 8

Matte contains mainly (a) Cu2S (b) Fe (c) CuS and Fe2S3 (d) Cu2S and FeS

In P04~ ion, the formal charge on each oxygen atom and P-O bond order respectively are

(a) - 0 . 7 5 , 1.25 (b) - 0 . 7 5 , 1.0 (c) - 0 . 7 5 , 0 . 6 (d) - 3 , 1 . 2 5 .

The IUPAC name of CH3CH = CHCOOC2H5 is (a) ethylbut-l-enoate (b) ethylbut-2-enoate (c) ethylprop-2-enoate (d) none of the above.

If the observed and theoretical molecular mass of NaCl is found to be 31.80 and 58.50, then the degree of dissociation of NaCl is (a) 83.96% (b) 8.39% (c) 90% (d) 100%

For the redox reaction Mn04" + C204

2~ + H+ -4 Mn2+ + C0 2 + H 2 0 Correct stoichiometric coefficients of Mn04~, C204

2 - and H+ are (a) 2 , 5 , 1 6 (b) 1 6 , 5 , 2 (c) 5 , 1 6 , 2 (d) 2,16,5.

Which of the following is not a strong electrolyte? (a) NaCl (b) KN0 3 (c) NH4OH (d) FeS04 .

Which of the following is correct for lyophilic sol? (a) Irreversible sol (b) Formed from inorganic substances (c) Readily coagulated by addition of electrolyte (d) Self stabilized.

In the stoichiometry of natural /awja.vzYe-a zeolite with formula Nal[(A102)56 (Si02)136], 250H20 the value of x is (a) 56 (b) 136 (c) 250 (d) none of these.

B C H E M I S T R Y TODAY I FEBRUARY '08

STEM stands for (a) scanning transmission electron microscope (b) scanning tunneling electron microscope (c) both (a) and (b) (d) none of these

1 g atom of an a-emitting nucleide \ X (half life = 10 hours) was placed in a sealed container. 4.52 x 1023

He atoms will accumulate in the container in (a) 4.52 hrs (b) 9.40 hrs (c) 10.00 hrs (d) 20.00 hrs.

The half lives of two radioactive nuclides A and B are 1 and 2 min. respectively. Equal weights of A and B are taken separately and allowed to disintegrate for 4 min. What will be the ratio of weights of A and B disintegrated? (a) 1 : 1 (b) 5 : 4 (c) 1 : 2 (d) 1 : 3.

Which of the following is not the extensive property? (a) Gibbs free energy (b) Enthalpy (c) Entropy (d) Viscosity.

Peptization involves (a) disintegration and then condensation (b) condensation then disintegration (c) adsorption of common ions then disintegration (d) precipitation of charged colloids.

Which of the following does not contain P-O-P bond? (a) Isohypophosphoric acid (b) Diphosphorous acid (c) Diphosphoric acid (d) Hypophosphoric acid.

The magnetic moment of a transition metal ion is found to be 5.92 BM. The number of unpaired electrons present in it is (a) 2 (b) 3 (c) 4 (d) 5.

The flocculation value is expressed in terms of (a) millimole per litre (b) mol per litre (c) gram per litre (d) mol per millilitre.

C EE N

CH3MgBr ~> Q 3 > F OCH3

The product 'P' in the above reaction is

c h / o h C O - C H 3

^ CH 3

(a )

(c ) OCH 3

Which isomeric form of benzene hexachloride is used as insecticide? (a) a-form (b) (3-form (c) Y-f° r m (d) 8-form.

( a ) ( c )

(a ) (b) (c)

(d)

Acetamide is neutral acidic

(b) basic (d) amphoteric.

The complex ion [CU(NH3)4]2+ is tetrahedral and paramagnetic tetrahedral and diamagnetic square planar and paramagnetic square planar and diamagnetic.

A solid AS has NaCl structure. If the radius of cation A+ is 170 pm, calculate the maximum possible radius of the anion B~. (a) 210.3 pm (b) 397.4 pm (c) 410.6 pm (d) 347.9 pm.

The size of ionic species is correctly given in order (a) Cl7+ > Si4+ > Mg2+ > Na+

(b) Na+ > Mg2+ > Si4+ > Cl7+

(c) Na+ > Mg2+ > Cl7+ > Si4+

(d) Cl7+ > Na" > Mg2+ > Si4+.

The heat of combustion of carbon is - 94 kcal at 1 atm pressure, the intrinsic energy of C0 2 is (a) + 9 4 kcal (b) - 94 kcal (c) + 4 7 kcal (d) - 47 kcal.

In a catalytic conversion of N2 to NH3 by Haber's process, the rate of reaction expressed as change in the concentration of ammonia per time is 40 x 10~3 mol~V . If there are no side reaction, the rate of the reaction as expressed in terms of hydrogen is (a) 60 x 10"3 mol L"< s"1 (b) 20 x 10"3 mol L"1 s" (c) 1200 mol L"1 s-1 (d) 10.3 x 10~3 mol L"1 s"1.

The general order of reactivity of carbonyl compounds for nucleophilic addition reaction is (a) H2C = O > R2C = O > Ar2C = O > RCHO >

ArCHO (b) H2C = O > RCHO > R2C = O > ArCHO >

Ar2C = O (c) ArCHO > Ar2C = O > RCHO > R2C = O >

H2C = O (d) Ar,C = O > R2C = O > ArCHO > RCHO >

H 2 C = o Brown haematite is

(a) Fe203 (b) FeS2

(c) 2Fe203 .3H20 (d) none of these.

Alumina is insoluble in water because (a) it is a covalent compound (b) it has high lattice energy and low heat of hydration (c) it has low lattice energy and high heat of hydration (d) Al3+ and O2 ions are not excessively hydrated.

B CHEMISTRY TODAY I FEBRUARY '08

The prefix neo means a carbon atom having four alkyl groups, position of alcohol group should be at CJ.

Presence o f -7 effect in a carbocation decreases delocalisation of the positive charge and hence decreases stability of the carbocation and vice versa. Thus the carbocation (iii) having -COCH3 grouping is the least stable, follwed by (ii) having -OCH 3 grouping, followed by (i) having only alkyl group.

A pair of molecule which cannot be super-imposed on its mirror image (i.e., asymmetrical) related to each other as an object to its mirror image are known as enantiomers, So,

CH3 CH 3

H-HO-

- O H - H

and H O -H -

-H -OH

CH 3 CH3

are enantiomers. Acrilan is a cyanide compound. It has a structure

- C H 2 - C H -I

CN

OH H 2 S O 4 , + H 2 O

The weight of cyclohexene formed 82 x 100 75

= ~ 1 0 0 — l oo g

Naphthalene has a structure,

It has 571 bonds and thus the number of n electrons is 5 x 2 = 10.

Only C6H5COOC2H5 corresponds with the molecular formula of the ester A. Further it explains all the given reaction

CH 3 I Excess of

C 6 H 5 - COOC 2 H 5 > C 6 H 5 C - O H 6 (A) 2 5 CH3MgBr 6 5|

CH3 CH 2 0

conc.

H,SO,

O > C 6 H 5 - c — ~ > CGHS- c - CH; •3

(B) I v ; CH3 Iodoform > C6H5COONa + CHI3

AgN03(a9) + NaCl(ll9) -> NaN03(a9) + AgCl(J)

NOj ions remain in the solution. If V mL each of the two solutions be mixed, the volume of the resulting solution = 2V

M, - 0.05

Then 0.1xV=M2x2V 0 1

" 2 ' Na2C03 + 2HC1 2NaCl + H 2 0 + C0 2

Meq of HCl = 1.0 x 100 Meq of Na2C03 = 2 x 1.0 x 75 = 150 Thus, Na2C03 is in excess, and being the salt of weak acid (H2C03) and strong base (NaOH), undergoes hydrolysis to produce basic character. Na2C03 + 2H20 2Na+ + 20H" + H2C03

PV^JILRJ M

p Tn RT , v RT „ , PM

2 x 3 4 68 = 2.76 g/L 0.0821 x 300 24.63 E°cel l= 1.20 V

AG0 = -nFE° = - 2 x 96500 x 1.20 = -231.6 kJ CAN or calcium ammonium nitrate is a neutral

fertilizer. It gives non-acid hydrolysis products and it decreases acidity in the soil.

Protonated compound of 1 is stabilised by two equivalent resonating structures, hence, compound 1 should accept H+ most readily.

r H H 3 C - C - N H 2 H 3 C - C - NH 2<-

NH 2 N H ,

H 3 C - c - N H 2 < » H 3 C - C - NH 2

Further 2° amines are more basic than 1° amine while amides are least basic because electron pair on nitrogen is involved in delocalisation.

• o : o r II .. I +

C H 3 - C - N H 2 < > H 3 c - c = n h 2

So, the correct order of basicity is NH

C H 3 - C; / \ > (CH3)2NH NH, o

> CH 3 - CH 2 - NH 2 > CH 3 - c - NH 2

NV=N1V1+N2V2 + N3V3

N x 1000 = 50 x 10 + 25 x 12 + 40 x 5 N x 1000 = 500 + 300 + 200 N x 1000= 1000 ^ 1000 = i

1000 So, 1 N is the normality of resulting solution.

Oxidation state of NH3 = 0 and N0 2 = - 1 So, [Co(NH3)3(N02)3]

B C H E M I S T R Y TODAY I FEBRUARY '08

x + (0) + 3 x ( - 1) = 0; X - 3 = 0; JC = + 3 The replacement of an atom or group from a

molecule by a different atom or group is known as substitution reaction. (CH3)3C - Br - I I ^ ( C H 3 ) 3 C - O H

Substitution reaction

Number of corner atom (A) = 8 x ^ = 1

number of face centre atom (B) = 6 x ^ = 3 Hence formula for the compound is AB3.

CO dissolve more rapidly in blood than oxygen.

Rate = k (C,)1-5 (Cv)-1 (Cz)° Adding all exponential term = 1.5 + ( - 1) = 0.5

The structure of phosphorus pentoxide is :o:

So, each P atom is linked to 4 oxygen atoms, one by double bond and other three by single bond.

CHClBr2 + Me3COK —> CClBr2 :CClBr Chlorobromo carbene (: CClBr) undergoes addition.

r, he j-. 1 K ° r K

Thus, a decrease in wavelength represents an increase in energy for ra4 transition. Greater the energy difference will follow lower wavelength. Thus, for n4 - n, transition wavelength would be minimum.

Mg(OH)2 ± Mg2+ + 201T the solubility

product Ksp of Mg(OH)2 = [Mg]2+ [OH"]2

1 x 10-12 = [0.01][OH-]2

[ O H - ] 2 = i ^ = lxlO-10

[H+][OH_] = 10-14 or [H+] =

or [OH-] = 10-

= 1 0 ' 10":

pH = - log[H+] = - log 10"9 = 9 During metallurgy of Cu after smelting, mixture

of Cu2S and FeS is left, which is called matte.

Total number of electrons present in the valence shell of phosphate ion = 5 + 8 = 13 Total number of electrons involved in bond formation in P04

3" ion = 13 - 3 = 10 3 10

Number of bonds in P0 4 = — = 5

Average P—O bond order = ^ = 1.25 The average formal charge on each 'O' atom.

Total charge _ - 3 Total entity of 'O' atom ~ T

CH3CH = CHCOOC2H5 the IUPAC name is

' = - 0 . 7 5

ethylbut-2-enoate. . 58-50

a = -

31-80 i-1 1-8396

= 1-8396

1 = 0 • 8396 or 83 • 96%

31.

m - 1 2 - 1 The balanced equation is

2Mn04 + 5C204_ + 16H+ -> 2Mn2+ + 10C02 + 8H20. NH4OH is weak electrolyte. Lyophilic sols are self stabilised reversible,

organic macromolecules and are not coagulated by addition of electrolytes.

Here Si02 and H 20 do not carry any charge. The charge present in A102 unit is - 1 (+ 3 on Al and - 4 on two O-atoms). Thus negative charge present on 56 A102 units is - 56. Therefore 56 Na+ ions are present in one molecule of faujasite.

STEM stands both for scanning tunneling electron microscope and scanning transmission electron microscope.

1 g atom = 6.02 x 1023 atoms = N0\ tm = 10 hrs No. of atoms disintegrated = No. of a-particles

= No. of He atoms = 4.52 x 1023

No. of atoms still present = 6.02 x 1023 - 4.52 x 1023

= 1.50 x 1023

, 0.693 _ 0.693 . 10 '=0.0693 hr

t = log 6.02 xlO2 '•112

2.303 0.0693 ° 1.50x10

2.303x0.06021 0.0693

= 20 hours

For A ForB T - 4 min T = 4 min tm = 1 min h/2 ' ^ min N0 = x (say) N0 = x (say) n = 4/1 =4 n = 4/2 = 2

No _ x _ X 2" 24 16

w - _ x _x 2" 22 4

A disintegrated B disintegrated x 15x x 3x 16 16 ~X 4- 4

8 2 C H E M I S T R Y TODAY I FEBRUARY '08

Ratio of A : B disintegrated 15x 3x _ . ~ r r : —r- or 5 : 4 16 4

Viscosity is an intensive property. Peptization is the phenomenon of solution

preparation from freshly precipitated substance on adding an electrolyte having common ion to the precipitate. Common ions get adsorbed, impart charge then resultant repulsive interaction causes disintegration.

Hypophosphoric acid (H4P2Os) 0 0

/ \ / \ HO OH HO OH

Isophosphoric acid (H4P206) 0 0 II II

H - P - O - P - O H I I OH OH

Diphosphorous acid (Pyrophosphorous acid, H4P205) 0 0 II II

H —P — 0 - P - H I I OH OH

Diphosphoric acid (Pyrophosphoric acid, H4P207) 0 0 II II

H 0 ^ / ? V 0 / P ^ 0 H OH OH

Applying \Leff = Jn(n + 2), \leff = 5.92 BM is obtained from n = 5.

Minimum amount of the electrolyte in millimoles that must be added to one litre of the colloidal sol to bring about complete coagulation or flocculation, thus, the unit of flocculation value is millimole per litre.

Cyanide with Grignard reagentforms ketone.

C = N C, / C H 3

CH3MgBr _

O C H ,

NMgBr

+ 2H20

O C H ,

CO — C H ,

+ N H 3 + MG: . / B r \

O C H , OH

Benzene-hexachloride exists in eight isomeric forms namely a, P, y, 8, etc. Out of which y-isomer is most active and used as insecticide under the name of gammexane.

O

CH3— C — NH2 behaves as an acid as well as a base, the acidic character is due to resonance in the acetyl group.

O O -II .. I +

C H 3 - c - N H 2 < — > CH3— C = N H 2

[CU(NH3)4]2+ is square planar with one unpaired electron in 4d orbital.

For NaCl type structure

c* = 0.414-0.737

For maximum possible radius of A"

^ = 0.414; - - 1 7 0

0.414 = 410.6

These are isoelectronic species. The greater the nuclear charge, smaller is the size. So, the correct sequence of size is Na+ > Mg2+ > Si4+ > Cl7+

because nuclear charge of Na+ = 11, Mg2+ = 12, Si4* = 14, Cl7+ = 17

C + Oz C02 , AE = - 94 kcal

AE = AEf(C02) - AEy(C) - A£ /(0)2 = - 94

or A E / ( C 0 2 ) - 0 - 0 = - 9 4 , '

Ef(CO,) = - 94 kcal

N, + 3H, 2NH,

d\N2]. dt

1 d[ H2] 3 dt

1 ^[NH3] ' 2 dt

d[ H , ] 1 rf[NH3

dt 3 dt 2

= 60 x 10"3 mol L"1 s-1

48. (b ) :

An aryl group, due to - I effect is expected to speed up the reaction however, due to resonance it stabilises the reactant veiy much and thus causes net deactivation.

Haematite with three molecule of water is called brown haematite i.e., 2Fe203 • 3H20.

A1203 has high lattice energy and low heat of hydration. It is therefore, insoluble in water.

B CHEMISTRY TODAY I FEBRUARY '08

West Bengal JEE 2008 Exam on 20th April

2008 The pattern and the syllabus for WB JEE 2008 has been changed. Under the new pattern 80% questions will be multiple choice questions of 1 mark each. 20% questions (short answer type) will carry 2 marks each.

Time : 1 hour INSTRUCTIONS Max. Marks: 50 Question paper consists of two sections :

Section A contains multiple choice questions. Each question carries 1 mark. Section B contains short answer type questions. Each question carries 2 marks.

i t p i S E C T I O N A

In buckminster fullerene, each carbon atom is (a) s/?-hybridized (b) ,vp2-hybridized (c) sp3-hybridized (d) none of these.

CI, CH3CH2CH2CH3 hr> A+B

(monochlorination products)

The approximate ratio of percentage yields of A and B formed in the above reaction is (a) 50 : 50 (b) 72 : 28 (c) 45 : 55 (d) 60 : 40.

In alkaline medium, KMn04 reacts as follows: 2KMn04 + 2KOH 2K2Mn04 + H 2 0 + O. Therefore, the equivalent weight of KMn04 will be (a) 31.6 (b) 52.7 (c) 7.0 (d) 158.0 Which of the following cannot be made by using Williamson's Synthesis ? (a) Methoxybenzene (b) Benzyl-p-nitrophenyl ether (c) ferf-Butyl methyl ether (d) Di-terf-butyl ether

The product of the following reaction : CH, I (i) Hg(OAc)2, H2C>

CH, - C - C H - CH, 2 2 s

I C H ,

(ii) NaBH4

CH,

(a) CH, - C - CH - CH, I I

H3C OH CH, I 3

(b) CH3 - C - CH2 - CH2OH

CH3

OH I

(c) CH3 - C - CH - CH2OH

H3C CH,

CH3 I 3

( d ) H O C H 2 - C - C H 2 C H 3

C H 3

A radioisotope has a tm of 10 days. If today 125 g of it is left, what was its weight 40 days earlier? (a) 600 g (b) 1000 g (c) 1250 g (d) 2000 g.

Consider the following reaction :

RCOO~ Ag+ RBr + AgBr + C 0 2

In the statement, this reaction is known as X reaction and proceeds via a Y intermediate.'X ' and ' F stand respectively for (a) Hell-Volhard Zelinsky, carbanion (b) Sandmeyer, free radical (c) Wolff, carbene (d) Hunsdiecker, free radical.

The conjugate acid of HP042~ is

(a) H3P04 (b) P043"

(c) H2P04" (d) H 30+

Consider the isoelectronic series : K+, S2_, Cl~ and Ca2+, the radii of the ions decreases as (a) Ca2+ > K+ > CI' > S2" (b) CI" > S2" > K+ > Ca2+

(c) S2- > CI" > K+ > Ca2+

(d) K + > Ca 2 + > S2" > Cl-

Which of the following are arranged in the decreasing order of dipole moment ? (a) CH3C1, CH3Br, CH3F (b) CH3C1, CH3F, CH3Br (c) CH3Br, CH3C1, CH3F (d) CH3Br, CH3F, CH3C1

8 4 CHEMISTRY TODAY I FEBRUARY '08

If the dissociation constants of two weak acids HAj and HA2 are and K2, then the relative strengths of HA, and HA2 are given by

(a)

(c)

(b)

(d)

Friedel-Crafts reaction of benzene with ethylene in presence of HC1 and anhyd. A1C13 gives (a) styrene (b) stilbene (c) ethylbenzene (d) 1,1-diphenylethylene. 23Na is more stable isotope of Na. Find out the process by which Na can undergo radioactive decay (a) P'-emission (b) a-emission (c) (3+- emission (d) K-electron capture

The oxidation states of sulphur in the anions of SO32-, S 2 0 4

2 " and S 2 0 62 - follows the order:

(a) S 2 0 42 - < SO,2" < S 2 0 6

2 -(b) S0 3

2 - < S 20 42" < S 2 0 6

2 -(c) S 20 4

2" < S 2 0 62 " < SO32-

(d) S 20 62" < S 2 0 4

2 - < S032".

Number of g of oxygen in 32.2 g Na2S04-10H20 is (a) 20.8 (b) 22.4 (c) 2.24 (d)2.08

Temporary hardness of water is due to the presence of (a) MgS04 (b) Mg(HC03)2

(c) CaCl2 (d) CaC03

An aqueous solution of 6.3 g of oxalic acid dihydrate is made upto 250 ml. The volume of 0.1 N NaOH required to completely neutralise 10 ml of this solution is (a) 40 ml (b) 20 ml (c) 10 ml (d) 4 ml.

Benzene diazonium chloride can be converted into benzene on treatment with (a) H3PO3 (b) H 3 P0 4

(c) H 3 P0 2 (d) HPO3.

Which of the following is a redox reaction? (a) NaCl + KNO3 NaN0 3 + KC1 (b) CaC204 + 2HC1 CaCl2 + H 2C 20 4

(c) Mg(OH)2 + 2NH4C1 MgCl2 + 2NH4OH (d) Zn + 2AgCN 2Ag + Zn(CN)2.

Which of the following species is paramagnetic?

(a) 0 2 (b) CN- (c) CO (d) NO+.

Which one of the following reactions is an example for calcination process ? (a) 2Ag + 2HC1 + (O) -> 2AgCl + H 2 0 (b) 2Zn + 0 2 2ZnO (c) 2ZnS + 30 2 2ZnO + 2S02

(d) MgC03 H> MgO + C0 2

Sulphonation of phenol with concentrated H 2S0 4

at 288-293 K gives (a) o-phenolsulphonic acid (b) sulphanilic acid (c) p-phenolsulphonic acid (d) sulphone.

Sodium metal is prepared commercially by electrolysis of fused NaCl by (a) Down's process (b) Nelson cell (c) Solvay process (d) Castner and Kellner's cell.

The root mean square velocity of one mole of a monoatomic gas having molar volume M is «r m s. The relation between the average kinetic energy (E) of the gas and « r m s is

13 £ 12 E '' \2M (b) "r-ms- = \3M

[2E _ r r ' i u (d) Ur.m.s.-^3M

(a) "r.

(c) u.

Which of the following forms cationic micelles above certain concentration? (a) sodium dodecyl sulphate (b) sodium acetate (c) urea (d) cetyl trimethyl ammonium bromide.

In which of the following reactions, addition does not occur according to Markownikov's rule? (a) (b) (c) (d)

CH3CH = CH2 + HC1 CH 3 CH CH 2

CH 3 CH

CH2 + HBr CH2 + HI

ROORj

ROOR^

ROORs

CHCH, + HBr ROOR.,

van der Waal's equation of state is obeyed by real gases. For n moles of a real gas, the expression will be

(a)

(b)

P na n V2 j n-b = RT

„ a P + ~2 V2 (V-b) = nRT

B CHEMISTRY TODAY I FEBRUARY '08

( C )

(d)

na P + ~T (nV-b) = nRT

_ n2a | P + \(V-nb) = nRT

Tautomerism is not exhibited by

(a) Q ^ V o (b) (b) ^ C H = = N O H

O

(c) O

(d) O O

o o

When equal volumes of the following solutions are mixed, precipitation of AgCl (Ksp = l-8xlO"10) will occur only with (a) IO"4 M Ag+ and 10" 4 M CI (b) IO"5 M Ag+ and 10~5 M Cl" (c) 10"6 M Ag+ and IO"6 M Cl" (d) IO-10 M Ag+ and IO"10 M Cl~

At constant temperature, the equilibrium constant (Kp) for the decomposition reaction N 2 0 4 v

s 2N0 2 is expressed by Kp = (4x 2P)/( 1 - x2) where P = pressure, x = extent of decomposition. Which one of the following statements is true? (a) Kp increases with increase of P (b) Kp decreases with increase of x (c) Kp increases with decrease of x (d) Kp remains constant with change in P and x.

Which of the following will be most readily dehydrated in acidic conditions ?

O O H O H ( a ) ( b )

O o

(c) (d) O H O H

The electron affinities of halogens are : F = 332, CI = 349, Br = 324, I = 295 kJ mol"1. The higher value for CI as compared to that of F is due to (a) higher atomic radius of F (b) smaller electronegativity of F (c) weaker electron-electron repulsion in CI (d) more vacant p-subshell in CI.

In the following reaction sequence, CuCN/pyridine dU. H2SQ4

6 5 523 K *

The product Y is (a) benzonitrile (c) benzoic acid

(b) benzene (d) benzamide.

Which of the following compound is not known? (a) NC13 (b) NI3

(c) NC15 (d) SbCl3.

Among K0 2 , A102", Ba0 2 and N0 2+ , unpaired

electron is present in (a) N0 2

+ and Ba0 2 (b) KOz and A102~ (c) K0 2 only (d) Ba0 2 only.

A drag that is antipyretic as well as analgesic is (a) Chloropromazine hydrochloride (b) para - Acetamidophenol (c) Chloroquin (d) Penicillin

Which of the following salts is colourless? (a) CdCl2 (b) CuS04-5H20 (c) MnS04-7H20 (d) NiS04-7H20.

In DNA, the complementary bases are (a) Uracil and adenine : cytosine and guanine (b) Adenine and thymine : guanine and cytosine (c) Adenine and thymine: guanine and uracil (d) Adenine and guanine : thymine and cytosine.

The group reagent for third group is NH4OH in presence of (a) (NH4)2C03 (b) NaCl (c) (NH4)2S04 (d) NH4C1.

In nucleic acids, the sequence is (a) Phosphate-Base-Sugar (b) Sugar-Base-Phosphate (c) Base-Sugar-Phosphate (d) Base-Phosphate-Sugar

S E C T I O N - B

Pure aniline is colourless, but when kept exposed in air it becomes coloured. Why? How can you obtain colourless aniline from it? Glacial acetic acid is treated with acetylene gas in presence of HgS04 and the resultant product is distilled. Explain why ?

When a hydrogen bond is symbolised by X—H — Y, what do the solid and dotted lines represent? Which distance is shorter?

B C H E M I S T R Y TODAY I FEBRUARY '08

Write a plausible Lewis structure for the chlorite ion, C102~. Show the formal charges on each atom and the net charge. Identify redox reaction(s) and the reducing and oxidising agent(s) from the following reactions:

(i) 2Mn04" + 5S02 + 6H20 ->• 5S042- +

2Mn2+ + 4H30+

(ii) NH4+ + P04

3- NH3 + HP042"

(iii) HCIO + H2S H30+ + CI" + S

Write, with equation, what happens when Sodium sulphate is heated strongly with carbon and the residue is made to react with an aqueous solution of zinc sulphate. Sodium aluminate solution is heated with ammonium chloride. An aqueous solution of potassium iodide in which chloroform has been added is shaken gradually with chlorine water in excess. A mixed solution of sodium nitrate and caustic soda is heated with aluminium powder. Fluorine reacts with cold dilute solution of caustic soda.

How can you distinguish (i) Formaldehyde and acetaldehyde (ii) Methyl alcohol and ethyl alcohol Explain and exemplify (i) Haloform reaction (ii) Cannizzaro reaction.

Find the number of covalent bonds in C2H402. How can you explain the formation of (i) NH4

+

from NH3 and (ii) BH4" from BH3. (i) Which bond has greater ionic character: H - F or H - CI? Give reason. (ii) The boiling point of NH3 is greater than that of HC1. How can you account for this?

SOLUTIONS

In buckminster fullerene (Bucky ball, C60) each carbon atom is sp2 hybridised giving a spherical shape resembling to a soccer ball.

H-Butane has two types of H-atoms (six 1° and four 2°), therefore, it gives two monochloro-derivatives, i.e., 2-chlorobutane (A) and 1-chloro-butane (B). Their relative percentage yields are

Amount of A _ 4 3.8 _ 15.2 _ 72% Amount of B ~ 6 1.0 ~ 6.0 ~ 28%'

2KMn04 = O 2(158) 16 parts

8 parts of O = 158 parts of KMn04

Hence eq. wt. = 158.

Di-?er?-butyl ether cannot be made by Williamson's synthesis, since f-alkyl halides prefer to undergo elimination rather than substitution, i.e.,

CH, CH, I I

CH3 - C - Br + CH3 - C - 0~Na+ — > 3 | 3 | -NaBr

CH 3 CH 3 CH 3 CH 3

I I CH 3 - c = CH 2 + C H 3 C - O H

CH 3

Oxymercuration-reduction follows Markownikov's rule. Thus

CH3 I Hg(OAc)2, H2Q

C H 3 - c - C H C H 2 ( M a r k o w n j k o v ' s addition)"*

CH 3

CHo CHo I NaBH4 I

CH, - C - CH - CH, > CH, - C - CH - CH, I I I I I 3

H3C OH HgOAc H 3c OH

N = | G = 4, 125 G = ^ [ A O ]

or [A0] = 2000 g

Hunsdiecker reaction occurs by a free radical intermediate.

HP042" + H+ ; H2P04 .

Amongst isoelectronic ions, ionic, radii of anions is more than that of cations. Further size of the anion increases with increase in -ve charge and size of cation decreases with increase in +ve charge.

CH3C1, CH3F, CH3Br.

Strength of HA,/strength of HA2 = ^ K J K 2 .

CH 2 =CH 2 + HC1 + A1C13

CH 3 CH 2 +A1C1 4 "

CH 3 CH 2 + C 6 H 6 CH 3 CH 2 C 6 H 5

ethylbenzene

2f N a —£-» 23 Na

S 20 42 - (+3) < S03

2" (+4) < S2062" (+5)

1 mol Na2S04 • 10H20 = 14 g atoms of (O)

B CHEMISTRY TODAY I FEBRUARY '08

i.e. 322 g Na2S04 • 10H20 contain (O) = 224 g.

Ca and Mg-bicarbonates cause temporary hardness to water.

Normality of oxalic acid sol

6.3 1000 n A 1 > J = x = 0.4 N 250 63

10 x 0.4 = V x 0.1 or, V = 40 ml

Benzene diazonium salts on reduction with H 3P0 2 in presence of Cu+ ions gives benzene.

C6H5N2C1 + H 3P0 2 + H2O C u >

Zn Oxidation

C6H6 + N2 + HCl + H3PO3

-> Zn2+ + 2e- ;

Ag+ + e- R e d u c t i o n> Ag

0 2 contains one unpaired electron.

In calcination, carbonates are decomposed to C02 .

At 288-293 K, sulphonation is kinetically controlled and hence o-phenolsulphonic acid is formed.

Na metal is obtained by Down's process by electrolysis of fused NaCl.

R.M.S. of one mole of monoatomic gas is

= 3 RT M

Average kinetic energy is E = ^RT or, 2E = 3RT.

= [M "r.m.s ^ M •

Cetyl trimethyl ammonium cations aggregate to form cationic micelles.

In presence of peroxides, only the addition of HBr to unsymmetrical alkenes does not occur according to Markownikov's rule. Since alkene (b) is unsymmetrical while alkene (d) is symmetrical, therefore, the correct answer is alkene (b).

van der Waal's equation is 2'

' (V - nb) = n RT p I an

+ V2

Essential condition for tautomerism is the presence of an a-hydrogen on a saturated carbon attached to C = O, NO, N02 , etc. or should have a H-atom present on a heteroatom (O, N, S, etc.) attached to a double bond. Thus, option (a) does not have a a-hydrogen on a saturated carbon and hence

does not exhibit tautomerism.

After mixing, [Ag+] [Cl~] > Ksp only for (a). Kp is constant at constant temperature.

Aldols, i.e., (3-hydroxyaldehydes or (3-hydroxyketones readily undergo dehydration to form a , P-unsaturated aldehydes or ketones.

O OH + O OH, „+ u n O

P-Hydroxy ketone H H

C6H5Br C u C N W ^ QH.CN 523 K

[C6H5(OH): (unstable)

boil

3J C6H5COOH 2 benzoic acid

NC15, due to non availability of d-orbitals in case of nitrogen, is not known.

K + n q - o : ]

p-Acetamidophenol.

CdCl2 has Cd2+ ion (4d10) which is colourless. In DNA, adenine faces thymine and guanine

faces cytosine.

The group reagent for third group is NH4OH in presence of NH4C1.

Base—Sugar—Phosphate. Aniline is easily oxidised by oxygen of air in

presence of light giving first a light brown and then a deep brcwn colour. The presence of a lone pair of electron on the nitrogen atom makes it susceptible to oxidation by air. Colourless aniline can be obtained from this dark coloured one by distillation.

When glacial acetic acid is treated with acetylene gas in presence of mercuric sulphate, ethylene diacetate is formed when the mixture is distilled, it decomposes into acetic anhydride and acetaldehyde when the latter escapes first due to its much lower boiling point.

Solid line represents an ordinary chemical bond (o-bond, covalent bond) and the dotted line represents a hydrogen bond. The distance between X - H is shorter than the distance between H - Y.

Lewis dot structure of chlorite ion can be written as: :0: CI :0:

B C H E M I S T R Y TODAY I FEBRUARY '08

Formal charges are: O (-1), CI (+ 1) and O ( - 1) Net charge is ( - 1 + 1 - 1) i.e., - 1. (i) Redox reaction, Mn04

_ is oxidising agent and S0 2 is the reducing agent. (ii) Acid-base reaction. (iii) Redox reaction. HCIO is the oxidising agent and H2S is the reducing agent.

When sodium sulphate is heated strongly with carbon, it is reduced to sodium sulphide. Na2S04 + 4C Na2S + 4CO When the residue is heated with zinc sulphate solution, white zinc sulphide is precipitated Na2S + ZnS04 -H> Na2S04 + ZnS

When sodium aluminate solution is heated with ammonium chloride, white gelatinous aluminium hydroxide is precipitated and ammonia gas is given off. NaA102 + NH4 CI + H20->A1(0H)3 + NaCl + NH3

Chlorine liberates iodine from potassium iodide which goes into chloroform layer colouring it violet. 2KI + Cl2 -> 2KC1 + I2

Aluminium powder reacts with caustic soda solution liberating nascent hydrogen which reduces sodium nitrate solution giving off pungent smelling ammonia gas. NaOH + Al + H 2 0 H> NaA102 + 3 [H] NaN03 + 8 [H] NaOH + 2H20 + NH3

Over all reactions is: 5NaOH + 8A1 + 3NaN03+2H20 -> 8NaA102 + 3NH3

Fluorine reacts with cold dilute solution of caustic soda giving sodium fluoride, oxygen fluoride and water. 2F2 + 2NaOH 2NaF + 0F 2 + H 2 0

(i) When warmed with I2 and NaOH solution, acetaldehyde gives yellow precipitate of iodoform with characteristic smell. Formaldehyde does not give iodoform. (ii) When warmed with I2 and NaOH soln., ethyl alcohol gives yellow precipitate of iodoform with characteristic smell. Methyl alcohol does not respond to this test.

(i) Ethanol, CH3CH2OH, acetaldehyde, CH3CHO, ketones having the formula RCOCH3 and secondary alcohol of the type RCH (OH) CH3 reacts with halogen in presence of aqueous NaOH or KOH to give haloform. These reactions are classified as haloform reaction, e.g.

CH3COCH3 I2 + N a O H > CHI3 + CH3COONa CH3CH2OH c12 + k o h > CHC13 + HCOOK

(ii) Aldehydes having no a-hydrogen atom reacts with concentrated aqueous solution of caustic alkalies to give alcohol and salt of acid by self-oxidation reduction. 2HCHO + NaOH CH3OH + HCOONa

CHO CH2OH cOONa

2 ( Q ] + N a O H > ( Q ) + ( Q }

Number of covalent bonds in C2H402

H O I 11

H - C - C - 0 - H i s 8 . I

H (i) In the formation of NH4

+, the lone pair of electron on the nitrogen atom of ammonia molecule is donated to a proton, H+ which requires two electrons for its stability.

H H H - N = +H4" > H — N ->H+

I I H H

(i) In the formation of BH4~, electron pairs of H" ion is shared with boron atom in BH3 which has only six electrons and requires two more electrons for its stability.

H H - B - H + >R - B H"

l I H H

In both cases, co-ordinate covalent bonds are formed. (i) Fluorine is more electronegative than chlorine.

So the electronegativity difference between hydrogen and fluorine is more than that between hydrogen and chlorine. So HF has greater ionic character than H - CI. (ii) Though the electronegativity of chlorine is equal to that of nitrogen, hydrogen bonding in ammonia is of much higher degree than that in hydrogen chloride due to much larger size of chlorine atom. So association due to hydrogen bonding in ammonia is appreciable while this is negligible in hydrogen chloride and ammonia has greater boiling point than hydrogen chloride.

B C H E M I S T R Y TODAY I FEBRUARY '08

Objective Problems

I s I B n i l l a E M I U I *

l. CH, - CH—CH - O - C O

NaOH,

Col (aq) >(A) + (B)

CH, - C=C. / C H 3

\ CH,

NaOH, «

O - C

o

Me

Me

> ( Q + (5)

Both (A) and (Q can react with 2, 4-DNPH. (A) and ( Q can be distinguished by reagent R. R and (B) are:

(a) Fehling's solution, Me -^Q^)—COONa

(b) I2/KOH, Me - < Q > - C O O H

(c) AgN03 - NH4OH, Me C 0 0 N a

(d) PhNHNH2, Me ^ Q ) - C O O N a

pH = x, at which rate is

-NH 2 OH-maximum PCI,

(A) MB)

O Which of the following is/are correct?

O

(a) and (B) is N.

-OH O O

r - ^ N H (b) [ j and (B) is

and (B) is

H

•NH,

(c) N. \

N. OH \ CI

(d) (B) on hydrolysis gives dicarboxylic acid.

3. In which of the following pairs both reactions is/are spontaneous ? (a) (i) ROH + NaNH2—• (ii) PhOH + NaHC03

(b) (i) RCOOH + NaHC03 ->• (ii) PhOH + NaOH—> (c) (i) ROH + RLi -»(i i) PhOH + NaNH2 -> (d) (i) RCOOH + N a O H ^ (ii) ROH + NaHC03 - »

4. • ( A ) M g / e t h e r > ( B )

(i)[Z3(ii)H+

->(Q,

(a)

(c)

(Q is

(b)

(d)

5. Which of the following order is/are correct ? (a) H2C03 > H 2 0 > PhOH > ROH > CH = CH (acid

strength) (b) CF3COCF3 > CCl3CHO > CH3CHO > HCHO

(nucleophilic addition reactivity) CH2OH CH2OH CH(OH)CH3 CH(OH)CH3

<->(§}<(&<(§) < [O] NO2 CH3

(reactivity towards conc. HCl) (d) Me2C = CH2 < Me CH = CH2 < CH2 = CH2 <

CH = CH (reactivity towards hydrogenation)

6. Which of the following reaction(s) proceed through free radical mechanism?

B CHEMISTRY TODAY I FEBRUARY '08

(c) Me2C = CH2+ DC1, (PhC0)202

(d) CH2 = CH2 + HBr, Me3C - O - O - CMe3 -»• 7. 2-Bromobutane a lc ' K 0 H / h e a t > (A) as major alkene

2-Fluorobutane a l c-K O H / h e a t> (B) as major alkene Which of the following is/are correct? (a) (A) and (B) are geometrical isomers of each

other. (b) The heat of hydrogenation of (A) is numerically

smaller than that of (B) (c) (A) on treatment with Br2/CCl4 gives meso

product. (d) (5) on treatment with Baeyer's reagent gives

racemic mixture.

8. In column (A), a reaction is given and in column (B), the information is given regarding the product (or major product) formed in the reaction given in column (A). Match in appropriate manner. Column (A)

Co,(CO)g (i) CH2 = CH2 + CO + H2 V

H3PO4

(ii) Me2C = CH2 + CO + H 2 0 : >

(i)B2H6(ii)H20,

(i)Hg(0Ac)2,H20

(ii) NaBH4, NaOH

(iii) MeCH2CH = CH2

(iv)Me3C - C H = CH2

Column (B) (p) reduces Fehling's solution (q) produces C02 from NaHC03

(r) gives yellow ppt. with I2/OH (s) gives red colour in Victor Meyer test (t) gives blue colour in Victor Meyer test (u) liberates H2 by reacting with Na metal

9. In which of the following reactions the product shown is major of that reaction?

(a)

18 NNHCONH,

(b) 'H

CH. + CH3CH2OH

dry HC1 . OEt

Me

(c) CH3COCH2CH2CH2CH2CH2CHO Me OH

dil OH" CHO

(d) <Me

D

H

alc. KOH/heat

H CI Me 10. In the structure of glutamic acid

(HOOCCH2CH2CH^ ) at high pH there ^ C O O H

are three basic sites and their Kb values are x, y, z such that x < y < z. pi of glutamic acid is (a) 14 + log V xy (b) 14 + logVxz (c) 7 + log V yz (d) none of these

11. Which of the following is/are correct regarding the reaction CH3CH2CH = O + NaHS03 -> (a) due to reaction there is no change in O.N. of S (b) the product on treatment with NaOH gives

Na2S04 as an inorganic product (c) The number of sp3 hybridised atoms in the

product are 5 (don't consider those oxygens which are covalently bonded to just one atom.)

(d) The product on treatment with HC1 produces S02

gas.

Comprehension Sodium or potassium salt of carboxylic acid on electrolysis produces hydrocarbon and C0 2 at one electrode and H2 gas at other electrode. The reaction mechanism involves free radicals. (A) is a dicarboxylic acid. The aqueous solution of potassium salt of (A) on electrolysis gives (B)(g) and (C)fe) at one electrode and H2

at another electrode. (B) on treatment with HC1 gives (D) via intermediate (J). (D) on treatment with Mg - ether followed by reaction with (C) and then hydrolysis gives an acid (E). The calcium salt of (E) on heating gives an inorganic salt (F) along with organic compound (G). (G) is lowest ketone C„H2nO which cannot perform haloform reaction. (H) is higher homologue of (A). The calcium salt of (H) on heating gives (J) along with (F). Answer the following questions :

12. (7) is more stable than (a) CH2 = CH+ (b) (c) CH3CH2CH2

+ (d)

13. The aqueous solution of sodium salt of (E) on electrolysis gives.

CH3+

Ph+

6 CHEMISTRY TODAY JANUARY '08

(a) an alkane, (B) and C0 2 at anode (b) an alkane which on vapour phase nitration gives

a mixture of six nitro alkanes (c) an ester which on Bouveault - Blanc reduction

gives just one alcohol. (d) an alkane which on chlorination gives a mixture

of two isomeric dichlorides.

14. Which of the following is/are correct for (F)l (a) (F) is thermally more stable than MgC03

(b) (F) has greater Ksp than SrC03

(i) heat (ii) carbon ( c ) { F ) (iii) hydrolysis 3 m e t f a a p e

(d) (F) + dil HCl —» greenish yellow coloured gas.

Ph3P = CH, B2H6/THF H202/Na0H 1 J . (J) > > > conc. H,SO,/heat

— >(K)

(H) H 1 / r e d p> straight chain alkane (L). Which of

the following is/are correct for (K)l (a) (K) on oxidative ozonolysis gives (H) (b) (K) on reaction with CH2N2/^u gives a bicyclo

alkane of formula C6H10

(c) (K) on reaction with CH2/N2//zu gives a spiro alkane of formula C6H|0

(d) (K) on reaction with CH2N2/7zu gives a cyclo alkane of formular C6H12.

SOLUTION

• CH, - CH=:CH - O - cC ^ N a O H ,

Coj Me

C H , - C H = C H - O H

CH2 - CH2CHO

(A) Similarly:

.CH3 •H,

C H 2 - C H - C

( Q C H 3 0

2. (a)

+ N H O H Si'gMy acidic mediun^ (A)

o

PCL

N, OH

{ B ) hydro'y^'S NH2(CH2)4COOH

3. (b),(e) (A) ROH is stronger acid than NH3 therefore

ROH + NaNH2 RONa + NH3 is spontaneous. (B) PhOH(WA) + NaHC03 -> PhONa + H2C03(S.A)

non-spontaneous. ( Q RCOOH(SA)+NaHC03 -> RC00Na+H2C03(WA)

spontaneous. (D)PhOH(SA) + NaOH PhONa + HzO(WA)

spontaneous. (E) ROH(SA) + RLi ROLi + RH(WA) spontaneous. (F) PhOH(SA) + NaNH, -> PhONa + NH3 (WA)

spontaneous. (G)RCOOH(SA) + NaOH RCOONa + H20(WA)

spontaneous. (H)ROH(WA) + NaHC03 RONa + H2C03(SA)

non-spontaneous. 4. (c)

!CBr,

H Me3C - O-

-Br -Br

-Br -Br

Me3COH-

(A) M&/e ther > PhMgBr (B)

PhMgBr + CH2 - CH2

C H 2 - O -> Ph(CH2)3OMgBr

• Ph(CH,)3OH

B CHEMISTRY TODAY I FEBRUARY '08

UVJJ CH, ^ ^ HO 2 k^^J CH.

CHx r -H+ .

kJ 1 ^ C H 2

-CH, ( Q

5. (c), (d) Correct orders are : H2C03 > PhOH > H 2 0 > ROH > CH = CH

(acid strength) CF3COCF3 > CCl3CHO > HCHO > CH3CH0

(nucleophilic addition) CH2 CH2 CHCH3 CHCH3

: o> ro> ro> ro . NO, CH, J2 —

(stability of carbocation) 6. (a), (b), (d)

(i) Peroxide effect is not applicable in case of addition ofHClandHI

(ii) The reaction of alkene with NBS proceeds through attack of Br' at allylic C - H bond.

7. (b), (c), (d)

CH3CH2CHBrCH3 alc. KOH/heat (A) H Cri^

CH3CH2CHFCH3 alc- K O H / h e a t > CH3CH2CH— CH2 (B)

Me H Me J f Br2/CCl4(antiadditionj H - ^ - B r ( m e s o . p r o d u c t )

H ^ M E M E

C H 3 C H 2 C H = CH, KMNOK > CH3CH;CH(OH)CH2OH (racemic mixture)

8. (i) CH, = CH2 + CO + H2 Co2 (C0)8 *

CH3CH2CHO (p) only

H,PO 4 (ii) Me2C = CH2 + CO + H 2 0 — 2 — ^

Me,CH - COOH (q), (u)

(iii)MeCH2CH = CH2 W W i Q H A ,

MeCH2CH2CH2OH (s), (u)

(iv)Me3C - CH = CH2 v ' 3 z demercuration Me3C - CHOH - CH3 (r), (t), (u)

Ans. (i)-(p); (ii)-(q), (u); (iiiHs), (u); (ivHr),(t),(u)

9. (b),(d)

(a)

(b)

\ / n r r r \ / r OCH, OCH,

OCH,< > OCH,

/ \ pCH3 ii+ / \ / )Me

\ Et

(C)

A H CH2CH3

O CH H CH t 3 I

0 = C — C H ,

H

CH,

OH I

CH

• 0 = C CH CH, CH, C ^

CH,

CFL CEL / 2

CH,

(d)

D

H

anti elimination/-DCl

H CI

(3)

Me / /

10. (a) "OOC - CH2CH2 - CH„

(i) -NH,

-COO-Kb (1) = 2; K„ (3 )=y=* pK„ $)=>- log y Kb (2) = x => pKb (2) => - log x

pK(2) + pKa(3) Pl =

_ (14 + logx) + (14 + logy) = 1 4 + l o g V ^

11. (a), (c), (d) CH3CH2CH = O + NaHS03 - »

CH3CH2CH

CH3CH2CH;

CH3CH2CH;

-OH + NaOH

OH

O N a

O' 'O

~S03Na CH3CH2CH = O + Na2S03 + H 2 0

-OH + HC1 —¥

-S03Na CH3CH2CHO + NaCl + S02 + H 2 0

O. N of S in each case is +4.

B CHEMISTRY TODAY I FEBRUARY '08

Comprehension : 2RCOONa -> 2RCOO" + 2Na+

At cathode : 2H20 + 2e H 2 + 2GII

At anode : 2RCOO" 2RCOO' + 2e

2RCOO* 2R' + 2C02

2R-coupling

» R - R: 2R Disproportionation

> R - H +

alkene

The products are R-R, R-H , alkene and ester RCOOR.

(A) is CH 2 - CH 2 (B) is CH2 = CH , , (C) is C0 2

C O O H C O O H

(D) is CH3CH2C1, (£) is CH3CH2COOH,

(F) is CaC03, (G) is CH3CH2COCH,CH3

(B) is CH,

CH , - C O O H

(I) is CH3CH2

CH, C O O H / O

(J) is

12. (a), (b), (d); 13. (a), (b), (d) electrolysis

CH 3CH 2COONaw - - Na+at anode.

CH3CH2COO- -> CH3CH2COO + e

CH3CH2COO' CH3CH2 '+ C0 2

So the products at anode 3ie CH3CH2— CH2CH3

CH3CH2 - H, CH2 = CH2, CH3CH2COOCH2CH3

CH3CH,CH,CH3

vapour phase nitration

N 0 2 + CH3CH2*CH(N02)CH3 + CH3CH2CH2 - NO2 + CH3 - NO2 + CH3CH2 - NO2

Na/alc. , CH3CH2COOCH,CH3 chlorination

CH3CHJ

CH,CH,CH,OH+CH,CH,OH

» CH3CHC12 + C1CH2CH2C1

14. (a), (b) : The thermodynamic stability of alkaline earth

metal carbonates follows the order

BeC03 < MgC0 3 < CaC03 < SrC03 < BaC03

The Ksp order is

BeC03 > MgC0 3 > CaC03 > SrC03 > BaC03

CaC03 CaO + C02; CaO + 3C -> CaC2 + CO

CaC, + 2H20 C2H2 + Ca(OH)2

CaC03 + dil 2HC1 -> CaCl2 + C0 2 + H 2 0

15. (a), (b)

^ CH-, — COOH ttt / J n (H) CH, 2 H I / r e d P

? «-pentane (Z)

(K) | 11 + CH2N2

W U t / G -

A TEXT BOOK OF mm | H » m p K

I I ' . 1 I

Based on latest pattern of AIEEE/I IT and PMT's1 The book is represented in such a way that each student would be able to solve any problem of Engineering as well as Medical Entrance Exam Book covers complete syllabus in the form of concise text • Improve your problem solving skills with CHECK YOUR

CONCEPT boxes * Presentation makes Organic Chemistry interesting and easy * MCQ's of 3 different levels - Concept based, standard, numerical based with detailed solutions A number of objective and subjective problems to enhance problem solving approach.

Most Powerful Book for

AIEEE 11ST and all other Engg. Entrance Exams

C B S E - P M T / A I I M S / B H U / A F M C and all other Medical Entrance Exams

WI-FCG- M T G B O O K S 503, Taj Apt., Ring Road Near Safdarjung Hospital, New Delhi - 29

•««••» Tel.: 26194317, 26191601 e-mail: [email protected] website : www.mtg.in

B CHEMISTRY TODAY I FEBRUARY '08

2.

Nicotine is an alkaloidfound in the nightshade family of plants (Solanaceae), predominantly in tobacco, and in lower quantities in tomato, potato, eggplant (aubergine), and green pepper. Nicotine alkaloids are also found in the leaves of the coca plant. Nicotine has been found to constitute approximately 0.6 - 3% of dry weight of tobacco.

X are (a) (b) (c) (d)

CI, Benzotrichloride

C6H6 , C 6 H 5 CHO C 6H 5CH 3 , C 6 H 5 CHO C 6 H 5 ch 3 , C 6 H S COOH C6H6 , C 6 H 5 COOH

Hydrolysis > Y . X and Y

Cyclic hydrocarbon molecule, A has all the carbons and hydrogens in a single plane. All the carbon-carbon bonds are of same length and less than 1.54 A and more than 1.34 A . The C - C - C bond angle will be (approx). (a) 120° (b) 180° (c) 100° (d) 109°28'

The major product (70 - 80%) of the reaction between /w-dinitrobenzenc and (NH4)2SX is

NO, NH,

(a) 0 , N ^ N 0 2

NH,

(c) H2N NH,

(b)

(d)

NO,

NH,

HS NH,

Under ordinary conditions nucleophilic substitution is not possible in benzene ring of aryl halides, why? (a) C - X bond in aryl halides is stable due to

derealization of electrons by resonance. (b) C - X b o n d possess a double bond character. (c) C - if bond is stronger in aryl halides than C - X

bond in alkyl halides, due to resonance. (d) All the above.

Compound (A) C3H6C12 on reaction with alkali gives (.B) C3H60 or (C) C3H4. (B) on vigorous oxidation gave a compound C3H602 . C with dilute H2S04

containing Hg2+ ion gave (D) C3H60, which with

6.

bromine and alkali gave the sodium salt of C2H402 . A is (a) CH3CH2CHC12 (b) CH3CC12CH3

(c) CH2C1CH2CH2C1 (d) CH3CHC1CH,C1

An aromatic amine (A) was treated with alcoholic potash and another compound (Y), a foul smelling product was formed with formula C6H5NC. (J) was formed by reacting a compound (Z) with Cl2 in the presence of slaked lime. The compound (Z) is (a) C6H5NH2 (b) C2H5OH (c) CH3OCH3 (d) CHC13

Arrange the following compounds in order of ease of dehydrohalogenation by alcoholic KOH solution.

•-Br ^ ^ B r

M [5] (a) A < B < C < D (c) A < B < D < C

0 B r Q E [C] [D]

(b) A < D < B < C (d) C < D < A < B

Arrange the following compounds in order of ease of dehydrohalogenation by alcoholic KOH solution.

Br

(A) CH3CH2CCH3 (fi) CH3CH2CH2CH2CH2Br

CH3

( Q CH3CHCH2CH2CH3

Br (a) B< C<A (c )A<C<B

(b) A < B < C (d) None of these

Which of the following is the correct increasing order of strength of nucleophile?

(a) R-<CH30 < 0 H <CN <CH3COO < C 6 H 5 S0 2 0-

(b) CH 3 O <R <CH 3 COO <CN < O H < C 6 H 5 SO 2 O-

By : Pavan Kumar Kasu, Hyderabad. Mobile : 98853-19221, Email: [email protected]

B CHEMISTRY TODAY I FEBRUARY '08 M i

Problems in Organic Chemistry

(c) R-<CH,O->CH 3 COO-<CN-< C 6 H 5 SO 2 O < 0 H " (d) None of these.

Arrange the following in order of ease of solvolysis. CI CI Q

A

(a) A < C < B (c) C < B <A

B

(b) A < B < C (d) None of these

pKa values of CH3COOH, CCl3COOH, C6H5OH and C6H5S03H are 4.79, 0.9, 10.0 and -2.6. The leaving tendency of their conjugate bases increases in the order (a) C6H50- < CH3COO" < CCl3COO" < C6H ;S03

(b) C 6 H 5 O- < C 6 H 5 S0 3 " < CH 3 COO < CCL3COO (c) CCl3COO < C6H50 < C6H5S03 ' < CH3COO (d) None of these

Which of the following statement is incorrect for SN2 reaction? (a) This type of reaction occurs with inversion

product. (b) The order of reactivity of alkyl halides in this

reaction is in the order 1° > 2° > 3°. (c) Rearrangement reactions are uncommon in SN2. (d) None of these.

OH

(I) is more acidic than (II) due to

(a) More - 1 effect of-NO z in (I) than (II). (b) Mesomeric effect of -NO, is prevented in (I) due

to steric hindrance caused by /-butyl groups. (c) Mesomeric effect of - N 0 2 is prevented in (II)

due to steric hindrance caused by /-butyl groups. (d) /-butyl group shows more +1 effect in (I).

Methyl and ethyl halides can undergo nucleophilic substitution reactions to give different products via Sn2 mechanism with (a) AgN02 (b) RONa (c) NH3 (d) All of these

The presence of which of the following groups at o-and/or /^-position to halogen atom increases the reactivity of halogen atom towards nucleophilic substitution ?

(a) - N 0 2

(c) -CH3

(b) -OH (d) - OCOR

Which of the following statement is incorrect for the reaction between an alcohol and halogen acids to form alkyl halide ? (a) The reactivity of halogen acids is in the order

HI > HBr > HCl > HF. (b) The reactivity of alcohols is in the order

3 ° > 2 ° > 1° .

(c) The reaction is generally carried out in presence of anhy. ZnCl2 or conc. H2S04.

(d) None of these.

The order of reactivity of the following alcohols towards conc. HCl is

H 3 CV

C H 3

/ F < CH, CH

OH F V CI / X

Ph OH (I) (II) OH

(a) IV > III > II > I (b) (c) I > II > III > IV (d)

OH m (IV)

IV > III > I > II None of these

A compound with molecular formula C4Hi0O3

is converted by the action of acetyl chloride to a compound with molecular weight 190. The original compound has (a) one - OH group (b) two - OH groups (c) three - OH groups (d) no - OH group

Increasing order of acid strength among tertiary butanol, isopropanol and ethanol is (a) Ethanol < isopropanol < tertiary butanol (b) Tertiary butanol < isopropanol < ethanol (c) Isopropanol < tertiary butanol < ethanol (d) None of the above.

Identify (Z) in the series : CH3CH2CH2OH Conc.H^Oa 160-180°C > (A)

l.Alc.KOH>(Z) 2. NaNH,

C H 3 C H - C H 2 3I I (a) NH2 NH2

CH,

(b)

- CH - CH, I 1

OH OH

CH3 — C — CTL,

(C) OH (d) C H 3 - C = CH

SOLUTIONS

Light HjO/lT C6H5CH3 + Cl2

M > C6H5CC13(X) — — > C6H5COOH (Y)

B CHEMISTRY TODAY I FEBRUARY '08

Hence X and Y are C6H5CH3 and C6H5COOH.

The hydrocarbon is benzene.

Due to selective reduction by (NH4)2SV (yellow ammonium sulphide) the major product formed occurs at m-position.

(d)

C3H6C12 (A) + KOH -> C3H60 (B) C3H6C12 (A) + alc. KOH C3H4 (C). Thus

Hg^JT Br, H3CC = CH - CH3COCH3(D)

Alka l i CHBr3 + CH3COO-

Because B and D are different. B is CH3CH2CHO and A is CH3CH2CHC12.

C6H5NH2 (A) + CHC13 (Y) + 3KOH - » C6H5NC + 3KC1 + H 2 0

CH3CH2OH (Z) + Cl2 -»• CH,CHO + 2HC1 CH3CHO + 3C12 — CCI3CHO + 3HC1 2CCI3CHO + Ca(OH)2—>2CHC13( Y) + (HCOO)2Ca

Conjugated system is usually more stable than non-conjugated system due to resonance.

Dehydrohalogenation of alkyl halides follows the order 3° > 2° > 1° alkyl halides. Thus, order is B<C<A.

Poor base is usually a good leaving group, hence R"< CH,0 < OH"< CN~< CH3COO < C6H5S0,0-

The stability of carbonium ion formed as intermediate follows the order: Allyl > 3° > 2° > 1°.

C6H5OH < CH3COOH < CCljCOOH < C6H5SO3H 10 4.79 0.9 -2.6

Stronger the acid, weaker is the conjugate base and thus better is the leaving group. Hence leaving tendency of the conjugate base is C 6 H 5 O- < CH 3 COO- < C C I 3 C 0 0 - < C 6H 5SO 3 -

All the statements mentioned here are correct. Presence of /-butyl and -NO, in close vicinity in

compound (II) makes - N 0 2 loose its planarity with benzene ring and - M effect is not shown by that group. So, in (I) there is - I , - M effect of -NO, group whereas in (II) there is only - I effect.

N02" is ambident nucleophile, hence products are

Nitro alkane ( R - N { ) and

Alkane nitrite (R - O - N = 0). - N 0 2 is an electron withdrawing group, and

electron withdrawing groups increase reactivity of aryl halides towards nucleophilic substitution.

The reaction is reversible, hence the presence of a dehydrating catalyst such as conc. H,S04 or ZnCl2 retard the backward reaction by removing water. ROH + UX ^ RX+ H20.

C6H5CH,OH (IV) reacts with HC1 to give benzyl carbonium ion (C6H5

+CH2) which is resonance stabilized. Both CH3CHOHCH3 (III) and FCH2CH2CHOHCH3 (II) give secondary carbonium ions, but electron withdrawing F atom in (I) at (3 -carbon atom makes it unstable.

Mol.wt. of C4H10O3 = 48 + 10 + 48 = 106. On reaction with CH3COCl the H atom o f - OH group is replaced by COCH3 group that is, an increase in mol. wt. 12 + 16 + 12 + 3 - 1 = 42 units. If the mol.wt. becomes 190, it shows that the compound contains two - OH groups.

Reactivity involving H atom of alcohol or acidity of alcohols follows the order 1° > 2° > 3°.

CH3CH2CH,OH CH3CH = CH2(JQ CH3CHBrCH,Br(Y) -> CH3 - C = CH(Z). •

years (1994-2007)

Chapterwise AliMS Questions with Detailed Solutions

\ fa L G CHAPTERWISE £ 14 AliMS

Chapterwise Solutions H i f e M n v : r ravstcs 4 CWWISW;'! wdtBGV

RWLV ! FI>WFFC« -cateintn T>> £»PCRT"

• The' Essential Resoufc* for Aspiting Oocton .

.Success mPMT 2008 Mow Made Easy

PHYSICS | CHEMISTRY I BIOLOGY Available at leading

book-shops throughout India.

MTG BOOKS . O 5 0 3 . T a i AP*-> R i n 9 R o a d N e a r Safdarjung Hospital

New Delhi -110 029 Tel.: 26191601, 26194317

B C H E M I S T R Y T O D A Y I FEBRUARY '08

50 MCQs are given in this Practice Paper. Choose the most appropriate option. Answers are provided at the end of the paper.

AIIMS CBSE DPMT AFMC VMMC EAMCET

According D v a r l , l # > A 1 tonewpattem r i O O - I C C

WmSHKmSmm-' MP-PMT MG1MS PMDT Bihar PMT Haryana Kerala PMT j Raj. PMT TNPCEE

1. Styphnic acid is (a) 2,4,6-Trinitrophenol (b) 2,4,6-Trinitroresorcinol (c) 2,4-Dinitroresorcinol (d) Phloroglucinol

2. When 0.5 g of sulphur is burnt to S0 2 4.6 kJ of heat is liberated. What is the enthalpy of formation of sulphur dioxide ? (S = 32 and O = 16) (a) +147.2 kJ (b) -147.2 kJ (c) -294.4 kJ (d) +294.4 kJ

3. Assuming complete ionisation, the pH of 0.1 M HCl is 1. The molarity of H2S04 with the same pH is (a) 0.2 (b) 0.1 (c) 2.0 (d) 0.05

4. Conversion of CH4 to CH3C1 is an example of .... reaction (a) electrophilic substitution (b) nucleophilic substitution (c) free radical substitution (d) free radical addition

5. In a sample of pure compound Na = 0.0887 mole O = 0.132 mole C = 2.65 x 1022 atoms The empirical formula of the compound is (a) Na2C03

(b) Na302C5

(c) Na0 0887O0.i32C2.65 x io22

(d) NaCO

6. Which of the following compound has co-ordinate bond ? (a) NH4C1 (b) A1C13

(c) NaCl (d) Cl2

7. The tautomerism is exhibited by (a) R 3 C N 0 2 (b) (CH 3 ) 3 CNO (c) (CH 3 ) 2NH (d) R - C H 2 - N O 2

8. The mixture used in Holme's signals is (a) CaC, + CaCl2 (b) CaC2 + Ca3P2

(c) CaCl2 + Ca3P3 (d) CaC2 + Ca3N2

The reaction of NaCl and K2Cr207 mixture with concentrated H2S04 result in (a) Colourless gas (b) Violet coloured gas (c) Deep red gas (d) No gas formation

10. The IUPAC name of the compound is

(a) Bicyclo [2.1.0] pentane (b) 1, 2-Cyclopropyl cyclobutane (c) Cyclopentane [4.3] azulene (d) 1, 2-Methylene cyclobutane.

11. If n and I are respectively the principle and azimuthal quantum numbers, then the expression for calculating the total number of electrons in any energy level is

( a ) 1 2 ( 2 1 + 1) ( b ) '~i12(.2l +1)

(d) '2(27 + 1) v ' 1=0

(c)

1=0 l = n + l

I 2(21 + 1) ; = 0

12. The colour of the precipitate produced by adding NaOH solution to HgCl2 is (a) yellow (b) black (c) brown (d) white

13. What happens when KI is heated with conc. H2S04 ? (a) KI3 is formed (b) Only I2 is liberated (c) I2 and S02 are liberated (d) None of these

14. Addition of methanol to 2-methylpropene in the presence of conc.H2SG4 gives (a) tert-Butyl alcohol (b) tert-Butyl methyl ether (c) Di-fert-butyl ether (d) Dimethyl ether

15. In an electrolysis of metallic chloride 3.283 g of metal (molar mass = 197 g mol ') was deposited on the cathode by the passage of 4825 C of electric charge. The charge number of metal ion is (a) 0.5 (b) 1.0 (c) 2.0 (d) 3.0

6 2 CHEMISTRY TODAY I FEBRUARY '08

16. Acetone and acetophenone can be distinguished by (a) DNP test (b) Tollen's test (c) Saturated solution of NaHS03

(d) Schiff's reagent

17. In the chain reaction, 238 1 1

92U + Qn—^Nuclide A + Nuclide B + 3Qn + Energy E neutrons and energy produced at the (n - l)th step will be (a) 3n ,nE (b) 3n~\ 3n'2E (c) 3", 3 —lE (d) none of these

18. An aqueous solution containing 100 g of dissolved MgS0 4 is fed to a crystallizer where 80% of the dissolved salt crystallises out as MgS0 4 -6H 2 0 crystals. How many gram of the hexahydrate salt crystals are obtained from the crystallizer ?

(Given Mg = 24; S = 32; O = 16 and H = 1) (a) 152 (b) 80 (c) 100 (d) 120

. Me

Me H

Hydrogenation atom of the above compound in the presence of poisoned palladium catalyst gives (a) optically active compound (b) an optically inactive compound (c) a racemic mixture (d) a diastereomeric mixture

20. Which of the following is incorrect statement ? (a) Flourine does not form oxyacids (b) C1207 is an anhydride of perchloric acid (c) IC1 is a good conductor of electricity in fused

state (d) Melting and boiling points of HBr are less than

those of HC1 21. When primary amine is heated with CS2 in presence

of excess mercuric chloride, it gives iso-thiocyanate. This reaction is called (a) Hofmann-bromamide reaction (b) Hofmann mustard oil reaction (c) Carbylamine reaction (d) Perkin reaction

22. [Fe(CN6)]3~ and [Fe(H20)6]3+ show the hybridisation states respectively (a) sp3cf and (fsp3 (b) cPsp3 and sp3cP (c) both (a) and (b) (d) none of these

23. The geometry of IC12" ions is (a) trigonal with sp2 hybridisation

(b) tetrahedral with sp3 hybridisation (c) square planar with dsp2 hybridisation (d) trigonal bipyramidal with sp3d hybridisation

24. The most efficient agent for the absorption of S0 3

is (a) 98% H 2S0 4 (b) 80% H 2S0 4

(c) 20% oleum (d) 90% H 2S0 4

25. Which of the following has highest chlorine content ? (a) Pyrene (b) DDT (c) Chloral (d) Gammaxane

26. Which one of the following cation does not form an amine complex ? (a) Ag+ (b) Cu2+

(c) Cd2+ (d) Na+

27. The radius of hydrogen atom in the ground state is 0.53 A. The radius of Li+ ion (atomic number = 3) in a similar state is (a ) 1 . 0 6 A (b ) 0 . 2 6 5 A (c) 0 . 1 7 A (d) 0 . 5 3 A

28. KI on treatment with CuS04 solution gives (a) K2S04 + Cul2 (b) K2S04 + Cu4I4

(c) K2S04 + Cul2 + I2 (d) K2S04 + Cul + I2

29. The total number of valence electrons in 4.2 gm of N3" ion is (where, NA is the avogadro's number) (a) 2 . 1 A^ (b) 4 . 2 NA

(c) 1.8 NA (d) 3 . 2 NA

30. Ammonia form complexes with Ag+ according to the following reactions : (i) [Ag(H 20) 2r + NH3{fl9) = = = ^

[Ag(NH3)(H20(a9))]+ + H20(O (ii) [Ag(NH3)(H20 ( a? ))r + NH3(a?)

[ A g ( N H 3 ) 2 ( A 9 ) R + H 2 O I ( )

The equilibrium constants of equilibrium (i) and (ii) are 2.0 x 103 and 8.3 x 103 respectively. The equilibrium constant of the following reaction [ A g ( H 2 0 ) 2 ( f l ? 1 ] + + 2 N H 3 ( A 9 )

[Ag(NH3)2](a?)+ + 2H20 ( i )

(a) 4.15 (b) 2.0 x 103

(c) 8 . 3 x 103 (d) 16.6 x 106

31. A reaction mixture has been made by taken equal concentration of two reactants. It takes 40 minutes for the completion of 50% of the reaction. For the completion of next 50% of the reaction time taken is 80 min. What is the order of the reaction ? (a) 2 (b) 0 (c) 3 (d) 1

32. When m-cblorobcnzaldehyde is treated with 50% KOH solution, the product(s) obtained is(are)

B CHEMISTRY TODAY I FEBRUARY '08

33. A radioactive isotope has a half life of 2 days. What is the initial amount of the isotope will be left at the end of 8 days ?

(a) (b)

34.

J_ 4

'Azote' is (a) anhy. CaS0 4

(c) N2

1 <c> T6 (d)

1 56

(b) (d)

conc. HN0 3

o2

35. Two vessels A and B of equal capacity containing gases both at 1 atm are connected together. If the temperature remains constant the pressure of the mixture is (a) 0.5 atm (b) 1 atm (c) 1.5 atm (d) 2 atm

36. An example of a psychedilic agent is (a) DNA (b) LSD (c) DDT (d) TNT

37. The density of a certain mass of a dry gas at 27°C and 760 mm pressure is 2.5 gm/ltr. Its density at 7°C and 740 mm pressure will be (in gm/litre) (a) 0.9 (b) 1 (c) 1.2 (d) 2.3

38. The values of heat of formation of S0 2 and S0 3 are -298.2 kJ and -98.2 kJ. The heat of "formation of the reaction S0 2 + ( l /2)0 2 -> S0 3 will be (a) -200 kJ (b) -356.2 kJ (c) +200 kJ (d) -396.2 kJ

39. 'PYROPHORIC' is an (a) Alloy of rare earths with Fe, AI, C (b) Alloy of rare earths with S, C, Ca and AI (c) Alloy of transition elements with Fe, AI, C (d) None of these

40. Which of the following hormone increases the pulse rate and blood pressure ? (a) Estrogen (b) Adrenaline (c) Cortisone (d) Thyroxine

41. Solubility product of AgCl in water at 18°C is 1.5 x 10~5, then solubility of AgCl at this temperature will be (a) 17.34 x 10^ g/ltr (b) 17.58 x 10-4 g/ltr (c) 16.34 x 10^ g/ltr (d) 16.58 x IO*4 g/ltr

42.

43.

44.

The colour of p-amino azobenzene is (a) orange (b) yellow (c) bismark brown (d) indigo

Ka for ascorbic acid (HASC) is 5 x 10~5. Then the value of [H+] in an aqueous solution in which the concentration of ASC" ions is 0.02 (M) will be (a) 5 x 10~7 (b) 5 x IO"9

(c) 5 x 10"3 (d) 1 x IO"9

What mole of calcium hydroxide is dissolved in aqueous solution having 250 ml to give a solution of pH value 10.65? (Assuming full dissociation) (a) 0.47 x 10"4 (b) 0.48 x 10^ (c) 0.56 x 10^ (d) 0.58 x 10^

Wl-fcG-

. P ^ T MAINS

EXPLORER 4 Years

(2004-2007) Solved Papers

Available at leading book-shops throughout India.

Send D.D/M.O in favour of MTG Books.

Add Rs. 35 for postage.

wtfc©- M T G BOOKS 503, Taj Apt., Ring Road, Near Safdarjung Hospital New Delhi - 110 029 Tel.: 26191601, 26194317

Always insist on MTG Books -jfi-

6 4 CHEMISTRY TODAY I FEBRUARY '08

45. Aniline in cold reacts with nitrous acid (NaN02 + dil.HCl) to give (a) phenol (b) benzene diazonium chloride (c) nitrobenzene (d) chlorobenzene

46. Glycerine is a (a) secondary alcohol (b) trihydric alcohol (c) tertiary alcohol (d) ester

47. When excess of ethyl alcohol is heated with conc. H2S04 at 140°C, the product will be (a) C 2H 4 (b) C2HC

(c) C2H5 - O - C2H5 (d) C2H2

48. When alkyl halide reacts with dry AgzO powder, the compound formed is (a) alcohol (b) ether (c) aldehyde (d) ketone

49. A mixture of gases in a gas cylinder at 760 mm pressure contains 65% nitrogen, 15% oxygen and 20% carbon dioxide by volume. The partial pressure of nitrogen is (a) 400 mm (b) 490 mm (c) 494 mm (d) 500 mm

50. Urotropine is obtained when ammonia reacts with (a) HCHO (b) CH3CHO (c) CH3COCH3 (d) C6H5OH

SOLUTIONS

1. (b)

2. (c)

heat

2 . 4 , 6 - t r i n i t r o r e s o r c i n o l

S + 0 2 s- S02

0.5 gm sulphur when burnt to S02 gives - 4.6 kJ

32 gm sulphur when burnt to S02 gives

- 4.6 x 32 0.5 - 294.4 kJ

3.

4.

Hence, 294.4 kJ is the enthalpy of formation of S02. (d) : HC1 ionizes as HC1 * H+ + Cl~ and H2S04 as => H2S04 —> 2H+ + S04

2 . Since, pH of both is same, H+ concentration has to be equated Hence, 0.1MHC1 r= 0.05MH2S04

(c) : Conversion of CH4 to CH3C1 by free radical mechanism is as follows

• • • • l i g h t ( U . V ) • • :c i :c i : - J L - 1 — * : c i - + -ci:

Free radical

H H

Cl'+ H - C - H ' f ' H - C ' +HC1

H H H

H - C + Cl2—>CH3C1 + CI

H

....(i)

...(ii)

.(iii)

CI radical now repeats steps (ii) and (iii) and thus, a chain of reactions is set up.

5. (a) : No. of atoms of sodium in 0.0887 mole of Na _ 6.023 x l0 2 3 x 0.0887

1 = 5.32 x 1022

No. of atoms of oxygen in 0.132 mole of 0 2 = 6.023 x 1023 x 0.132 = 7.97 x 1022

No. of atoms of carbon = 2.65 x 1022

Hence ratios Na : O : C 2 : 3 : 1

Hence empirical formula = Na2CQ3

6. (a);

H I

H - N I

H

c r

C o - o r d i n a t e b o n d

7. (d) : Tautomers are as follows: H _ H

R - C - N

H

R - C —N OH

0

8. (b) 9. (c): Deep red coloured chromyl chloride (CrOzCl2)

gas is formed. 4NaCl + K2Cr207 + 3H2S04 >

2Cr02Cl2 + 2Na2S04 + K2S04 + 3H20

z \ 10. (a) :

B i c y c l o [ 2 . 1 . 0 ] p e n t a n e

11. (d): For a given value of n, I can have values from (n - 1) to zero. Further number of electrons in a subshell = 2(2/ + 1).

Total number of electrons in any energy level (n) I = n-1

= I 2(21 + 1)

C H E M I S T R Y TODAY I JANUARY '08

12. (a) : HgCl2 + 2NaOH •

13. (c) : KI + H2S04 — 2HI + H2S04 —

r \ 14. (b ) : CH3-*-C=CH :

4 CH,

CH, — C — CH,-3 i i 2

CH 3H

->Hg(OH)j + 2NaCl (Unstable)

I HgO + H20

Y e l l o w p p t .

KHS04 + HI 2H20 + S0 2 + I2

H +

•:o - C H ,

-H+ -»• CH,

0 - CH 1

• C - CH ! CH3

15. (d) : From Faraday's first law of electrolysis w = Zit or ZQ (Q - quantity of electricity in C, i = current)

Here w = 3.283 g E _ At. wt./Valency

96500" 96500 197/x

Z =

~ 96500 it = 4825 C

197x4825 3.283 =

1 xx96500

3.283x96500 x 197x4825

x = 3

16. (c) : Acetone with a saturated solution of sodium bisulphite gives crystalline sodium bisulphite addition compound but acetophenone does not.

CH

CH c = 0 + NaHS03

CH, OH

i3 CR/ \ o 3 - N a +

A c e t o n e s o d i u m b i s u l p h i t e

17. (b ) : Step I [3 neutrons, is(energy)] Step II [9 neutrons, 3£"(energy)] = [32«, 31/?] Step III [27 neutrons, 9£(energy)] = [33n, 32E] Step (n- 1) = [3""1 neutrons, 3" ~ 2 E (energy)]

18. (a) : 100 gm of MgS04 was initially fed to crystallizer. Out of 100 gm, 80 gm of MgS04 is crystallised as MgS04 .6H20

Mol. wt. of MgS04 = 120 Mol. wt. of MgS04 .6H20 = 228

120 gm of MgS04 gives 228 gm MgS04 .6H20

228x80 80 gm of MgS04 gives = — ^ —

= 152.00 gm 19. (b) : In the presence of Pd-BaS04 ci.v-addition of

H2 to triple bond takes place. This reduced product is optically inactive due to the presence of a plane of symmetry in its molecule.

M e u M e

H H

20. (d ) : Melting and boiling points of HBr are greater than those of HCl.

21. (b) : C,H5NH2 + CS2 + HgCl2

C2H5NCS + HgS + 2HC1 Ethyl i sothiocyanate

22. (b) : [Fe(CN)6]3_

Fe3+ [Z = 26] = \s2, 2s2p63s2p6d5

3d 4i 4p

?1 t i t !•• • • l -d2sp! h y b r i d i s a t i o n

[Fe(H20)6]3+

3 d

t t t t t

• sp d' h y b r i d i s a t i o n

3 s 3 p 3 d

»®'<S> ®@© o o o o o C h l o r i n e a t o m in g r o u n d s t a t e

® ®@© ® o o o o s p ' d h y b r i d i s e d s t a t e

CI

24. (a) : S0 3 on dissolving in 98% H2S04 gives oleum S03 + H,S04- •H 2S 2O 7

O l e u m

If dil. H2S04 is used, oleum will be immediately converted in H2S04.

H2S207 + H 2 0 > 2H2S04

25. (a) : CC14 (pyrene) has highest chlorine content due to greater number of chlorine atoms.

2 6 . ( d ) : A g + » [ A g ( N H 3 ) 2 I

Cu

Cd2

[Cu(NH3)4]

[Cd(NH,).]2

C o m p l e x i o n s a r e f o r m e d

6 6 C H E M I S T R Y TODAY I FEBRUARY '08

Na+ > No complex formation with ammonia solution

27. (c ) : Radius of any ion Radius of H atom in ground state

Z (of any ion)

= ° f A = 0.17A

28. (d) : [2KI + CuS0 4 > K2S04 + Cul2] x 2 2CuI2 » 2CuI + I2

4KI + 2CuS04 » 2K2S04 + 2CuI + I2

29. (c) : •.• 14 gm of nitride ion (N3~) has 6.023 x 1023N3^ion 4.2 gm of nitride ion (N3~)

= 6.023 x l0 2 3 x 4.2 14

Now, V 1 nitride ion (N3 ) has 6 valence electron

6.023 x l0 2 3 x 4.2 . / x t 3_, •'• J4 ion(N" )

6.023 x 1023 x 4.2 x 6 = p =1-8 NA

30. ( d ) : [Ag(H20)2]+ + NH3(Q9)

[Ag(NH3)(H20(a9))]+ + H2O(0 (1) Kx = 2.0 x 103

C[Ag(NH3)tH2Q( ))]+

= C 7c [Ag(H20)2]+ ^ ( a g )

[Ag(NH3)(H20(a?))]+ + NH3(a?) [Ag(NH3)2(a4))]+ + H20(Q (2) K2 = 8.3 x 103

K C[Ag(NH3)2(^)]+

( i i )

C[Ag(NH3)(H20(a9))] X CNH3(fl9)

[Ag(H20)2(a?)]+ + 2NH3(a?) [Ag(NH3)2(a?)]+ + 2H2O(0 (3)

c [Ag(NH3)2(ag)]+

c xr2 (»') L[Ag(H20)2(a?)] NH3

From equation (i), (ii) and (iii) it is evident K ~ K\K2

= 2 x 103 x 8.3 x 103 = 16.6 x 106

31. ( a ) : Initial concentration of both the reactant = a Half life (tU2) = 40 min After 50% completion of reaction = ^ Half life (tV2) = 80 min

(fl/2)i _ ( ^ Y " 1 40 = (_a_"" 1

OT' ^1/2)2 l " J ' 80 \2a

1 / I o r ' 2 = 2

n - 1 = 1 or, n = 2

32. (c) : m-Chlorobenzaldehyde has no a-hydrogen and hence gives Cannizzaro's reaction.

33. (c) : According to the question, half life (tU2) = 2 days

time (t) = 8 days So, n (no. of half life)

t n / 2

= t = 4

So, the fraction of isotope left \4 N 0 ) (2)

- (2) 16

34. (c) : Antoine Lavoisier proposed the name azote to N2 gas in 1789 because it did not support respiration and was therefore "lifeless".

35. (b) : Let the volume of each vessel be V litre. The volume of the mixture becomes 2V litre.

Since ~ PAVA+PbVB Volume of the mixture l x V + l x V

2V - = 1 atm

36. (b ) : LSD (Lysergic acid diethylamide) is an example of psychedilic agent, a hallucinogen drug which helps in controlling behavioral patterns, regulatory systems including mood, hunger, temperature, etc.

Pi P? 37. (d ) : As we know ~~pt~ ~ ~tTt~ rv\ 22

Here, v e are given Pi = 2.5 gm/litre, p2 = ? 7", = (273 + 27) K = 300 K T2 = (273 + 7)K = 280 K P1 = 760 mm, P2 - 740 mm

P2 = Pir2 . P2

'1 '1 2.5 x 280 x 740

760x300 = 2.3 gm/litre. 38. (c) : Values of heat formation of S02 = -298.2 kJ

and that of S0 3 = -98.2 kJ. Therefore, S w + O 2(g) ' •SO 2(g)

AHt = -298.2 kJ and S w + (3/2)02te)- •SO 3(g)

AH2 = -98.2 kJ

....(i)

...(ii)

39.

Subtracting equation, (i) from (ii), we get SOz + ( l /2)0 2 — ^ S03

Hence AH = -98.2 - (-298.2) = 200 kJ. (a) : 'PYROPHORIC' is an alloy of rare earth elements with Fe, Al and C and it is used to prepare ignition devices.

B C H E M I S T R Y TODAY I FEBRUARY '08

40. ( b ) : Helps in fight or flight reaction. 41. ( b ) : AgCl is a binary electrolyte.

AgCl Ag+ + Cl-

Therefore, .v2 = Ksp or s =

Solubility of AgCl = ^(1.5) x 10~10

= 1.225 x 10"5 gm-molecule/litre = 1.225 x 10"5 x 143.5 gm/litre = 17.58 x 10^ gm/litre

42. (b)

43. (b) As Kh = ^ 1x10' -14

5 x 10"J

= 2 x IO"10

Hydrolysis reaction ASC'l[lq)

At equilibrium (0.02 - x) = 0.02

Now, by applying the formula .2

HASC + OH" x x

Ku xx x

: a o 2 2

x 0.02

- 2 X 1 ° - , 0 = ( M => x2 = 2x IO"10 x 0.02 = 2 x IO"10 x 2 x 10"2

=> x2 = 4 x IO"12 = > ^ 2 x 1 0 ^ => [OH"] = 2 x IO"6

r I J + 1 1 xlQ- 1 4

••• = 5 > < 1 0 - 9

[Since, [H+] x [OH"] = 1 x IO"14] 44. ( c ) : pH + pOH = 14

10.65 + pOH = 14 pH = 14 - 10.65 = 3.35

[OH"] = 10"3'35 mole/litre .-. log [OH~] = -3-351ogl0

[OH-] = 4-47 x 10 4 mole/litre. Number of OH" moles in 250 ml

_ 4.47 x 10' 4

Soluble mole of Ca(OH)2

,-4 - = 1.12x10 ,-4

= - x 1.12 x l 0 ~ 4 = 0.56 x l O - 4

45. ( b ) : NaNOj + HCl-N H ,

»H N 0 2 + NaCl

O l + H N 0 , + H C l ^ - ^

Aniline

Benzene diazonium chloride

46. ( b ) : Glycerine is a trihydric alcohol.

Formula

CHjOH

CH0H I

CH20H 47. (c) : This reaction occurs in two steps.

o°c _

0 - S 0 2 - OH + HjO

140°C .

C2H5OH + H2S04 (conc.) CH3CH2

( E t h y l h y d r o g e n s u l p h a t e )

CHjCHJ-OH + H O S O j - 0 - CH 2 -CH 3

CH3CH2- 0 CH2CH3 + H2S04

Here H 2 S0 4 acts as a catalyst. Thus diethyl ether may be regarded as a dehydration product of ethyl alcohol.

A 4 8 . ( b ) : 2R - X + A g 2 0 ( d r y ) -»R - 0 - R + 2AgX

(Ether)

49. (c) : The partial pressure of nitrogen = total pressure x mole fraction of nitrogen

total pressure x volume of nitrogen total volume

760x65 100

- mm = 494 mm

50. ( a ) : HCHO reacts with NH3 to form hexamethylene tetramine

6HCHO + 4NH, ~~* (CH2)6N4 + 6H 2 0 Hexamethylene tetramine

(Urotropine)

Wt-fcG-

-PMT

14 Years (1994-2007)

Solved Papers

Available at leading book-shops

throughout India.

Send D.D/M.O in favour of MTG Books.

Add Rs. 35 for postage.

M T G B O O K S 503, Taj Apt., Ring Road, Near Safdarjung Hospital New Delhi - 110 029 Tel.: 26191601,26194317

1 R H H i h I 1

WWK3-f p f t ^ s . p ^

Qv m M •

E X P I O It £ R

81

B CHEMISTRY TODAY I FEBRUARY '08

Wl-fcG- CHEM

lEfflai All students preparing for PMT/PET examinations can participate in Chem-Genius Contest. Answers marked only on the entry form of the magazine / photocopy of form will be accepted. More than one response to a question will be disqualified. Prizes . 1st Prize - LG Mobile phone • 2nd Prize - Adidas Bag • 3rd Prize - MTG Books (worth Rs. 500/-) The entries with maximum number of correct answers for three consecutive months (Nov '07 - Jan '08) will be awarded 1st prize. 2nd and 3rd prize will be given to the next maximum scorers. In case of a tie, the winners will be decided through a lucky draw. The decision of the editor will be final and binding in all cases and will not be a matter for consideration of any court and no correspondence will be entertained. Name and photograph of the prize winners of this contest will be published in the March issue of 2008. MTG is not responsible for any postal delays, transit losses or mutilation of entries. Last Date The entries should reach on /be fo re 31s t January ' 08 to - Chem-Genius Contest -2 , 406, Taj Apartment, Ring Road, Near Safdarjung Hospital, New Delhi-29. [Note : Enclosures include a passport size photograph and a photocopy of age proof.]

1. The orbital angular momentum for an electron

revolving in an orbit is given by ^1(1 + 1)-^-. This momentum for a j-electron will be given by

1 h (a) +

2 27t

<•>5

(b) zero

(d) ^^2• 2n

2. F

(a)

(b)

. D M F / A

(i) N a N O , / H C l , 0-5°C

(ii) H j / N i >(B), B is

(c) o 2 N H ^ V N = r C H 3

(d) O 2 N ^ N H 2

3. A student plans to calculate the molar mass of an unknown gas from the mass of a measured volume of gas at a known temperature and pressure. To how many significant figures should the student report the molar mass if 175.0 mL of the gas weighs 1.0315 g at a temperature of 22°C and pressure of 742 mmHg? (a) Two (b) Three (c) Four (d) Five

4. Who is known as the "Father of chemistry"? (a) Faraday (b) Priestley (c) Rutherford (d) Lavoisier

B CHEMISTRY TODAY I FEBRUARY '08

Which one is the anhydride of HC104? ( a )C l 2 0 (b)C102 (c) Cl2Oe (d) C1207

There is no S — S bond in (a) S204

2" (b) S2052" (c) S203

2" (d) S2072-

The haemoglobin from the red corpuscles of most mammals contains app. 0.33% of iron by weight. The physical measurement indicates the molecular weight of haemoglobin 7000. The number of iron atoms in each molecule of haemoglobin is (At.wt.of Fe = 56) (a) 2 (b) 3 (c )4 (d) 5

A solution contains 20 mmol H3P04 and 10 mmol NaH2P04. How many mmol of NaOH must be added to convert all the phosphate to HP04? (a) 30 mmol (b) 50 mmol (c) 60 mmol (d) 80 mmol

9. "925 fine silver" means an alloy of (a) 7.5% Ag and 92.5% Cu (b) 92.5% Ag and 7.5% Cu (c) 80% Ag and 20% Cu (d) 90% Ag and 10% Cu

Identify X in the following reaction

11. C - CC1, 1

H The above structural formula refers to (a) BHC (b) DNA (c) DDT (d) RNA

K2[HgI4] detects the ion/group (a) NH2 (b) NO (c)NH4

+ (d) c i -

A solution of 1.25 g of non-electrolyte in 20 g of water freezes at 271.94 K. If Kf = 1.86 K kg mol~1, then the molecular weight of the solute will be (a) 179.79 g mol"1 (b) 207.8 g moH (c) 209.6 g mol"1 (d) 109.6 g mol"1

Pyrolysis of Me, Me'

0 "

Me + CD

would give (a) mixture of CH2 =: CH - CD3 and

CH3 - CH —CD2 (b) CH3-CH =CD 2

(c) Me2ft = C(CD3)CH3 (d) CH2 = CH - CD3

In the separation of Cu2+ and Cd2+ in 2nd group of qualitative analysis of cations, tetraammine copper (II) sulphate and tetraammine cadmium (II) sulphate react with KCN to form the corresponding cyano complexes. Which one of the following pairs of the complexes and their relative stability enables the separation of Cu2+ and Cd2+? (a) K3[CU(CN)4] : more stable and

less stable less stable and more stable more stable and less stable less stable and more stable

K2[Cd(CN)4] ( b ) K 2 [ C U ( C N ) 4 ]

K2[Cd(CN)4] ( c ) K 2 [ C U ( C N ) 4 ]

K2[Cd(CN)4] ( d ) K 3 [ C U ( C N ) 4 ]

K2[Cd(CN)4]

When an ideal gas undergoes unrestrained expansion, no cooling occurs because the molecules (a) are above the inversion temperature (b) exert no attractive force on each other (c) do work equal to loss in kinetic energy (d) collide without loss of energy

Me H

Me H Hydrogenation of the above compound in the presence of poisoned palladium catalyst gives (a) an optically active compound (b) an optically inactive compound (c) a racemic mixture (d) a diastereoisomeric mixture

The structure given below is known as

< g ^ C H 2 - C - N H ^ SY c

C H 3

(a) penicillin-F (c) penicillin-^-

C 6 O H

0 ' H

(b) penicillin-G (d) ampicillin

19 For a solid with the following structure, the coordination number of the point B is (a) 3 (b) 4 (c) 5 (d) 6

B CHEMISTRY TODAY I FEBRUARY '08

20.

21.

22.

[ C O ( N H 3 ) 5 N 0 2 ] C 1 2 a n d [ C O ( N H 3 ) 5 ( O N O ) ] C 1 2 a r e related to each other as (a) geometrical isomers (b) optical isomers (c) linkage isomers (d) coordination isomers

The strength as oxidising agents of the following species in acidic solution decreases in the order

(a) S 2 0 | - > Cr2C>7~ > MnO^

(b) Mn0 4 > Cr207_ > S 2 0 | "

(c) S 2 0 | - > MnC>4 > C^O,"

(d) Cr20^- > S 2 0 | - > Mn0 4

0 0 Compound^U^jJ^ on removal of proton gives a

carbanion. The most stable carbanion should be 0 0 II II

(b) / \ / \

0 0 II II

(a) / \ / \

0 0 II II

(c) / S / \

23.

24.

25.

26.

27.

(d) all of these.

When a given amount of water is heated from 2°C to 8°C, its volume varies with temperature according to the curve

(a)

(c)

f ( ° C )

/ A

(b)

(d)

vy T CC)

T(° C) T(°C)

Lattice defect per 1015 NaCl is 1. How many lattice defects are observed in 1 gram mole of NaCl? (a) 1014 (b) 6.02 x 1023

(c) 6.02 x 108 (d) none of these

When one mole of a substance melts reversibly at fusion point, 7} at constant pressure, which of the following relations is correct?

AH f (a) A S f = ~ f

T AHf <c> Tf = ~ASJ

Least mobile ion is (a) [Be(H20)„]2+

(c) [Mg(H20)„]2+

AS (b) A H f = Y

f

(d) both (a) and (c)

(b) [Na(H20)„]+

(d) [Li(H20)„]+

Which of the following reactions defines AWfr<.

CHEMISTRY TODAY I JANUARY '08

(a) ^(diamond) + ®2(g) C 0 2 ( ? )

(b) 1/2 H2(g) + 1/2 F % ) HFte)

(c) N2(g) + 3H2(g) 2NH3(

(d) CO(g) + 1/2 0 2 ( s ) C( [3(J)

'2(g)

28. The basic character of the transition metal monoxides follows the order (a) VO > CrO > TiO > FeO (b) CrO > VO > FeO > TiO (c) TiO > FeO > VO > CrO (d) TiO > VO > CrO > FeO

29. At CMC, the surface molecules (a) decompose (b) become completely soluble (c) associate (d) dissociate

30. The activity of a radioactive isotope is 3000 counts per minute at a certain time and 2736 counts per minute 48 hours later. What is its half-life? (a) 831 h (b) 521 h (c) 361 h (d) 1.44 h

31. Soap removes grease by (a) adsorption (b) emulsification (c) coagulation (d) none of these

32. For the concentrated solution of a weak electrolyte AxBy, the degree of dissociation is given by

(a) a = Keq/C(x + y) (b) a = ^KeqC/xy

K. xl Ix+y eq

^ a <xxyyCx+y~1

(d) a = ^Keq/xyC

33. The compressibility of a gas is less than one at STP. Therefore (a) Vm = 22.4 litre (b) Vm = 22.4 litre (c) Vm < 22.4 litre (d) Vm > 22.4 litre

34. Which of the following gases has highest root mean square velocity? (a) Carbon dioxide (b) Carbon monoxide (c) Sulphur dioxide (d) Oxygen

35. The number of ether metamers represented by molecular formula C4H10O is (a) 4 (b) 3 (c) 2 (d) 1

36. 114 cm3 of nitrogen are collected over water at 16°C and 753.5 mm pressure. The volume of dry gas at STP. (Aqueous tension at 16°C = 13.5 mm) (a) 104.85 cm3 (b) 10.48 cm3

(c) 48.01 cm3 (d) 480.0 cm3

37. Chargaff's rule states that in an organism

(a) amount of adenine (A) is equal to that of thymine (T) and the amount of guanine (G) is equal to that of cytosine (C).

(b) amount of adenine (A) is equal to that of guanine (G) and the amount of thymine (T) is equal to that of cytosine (C).

(c) amount of adenine (A) is equal to that of cytosine (C) and the amount of thymine (T) is equal to that of guanine (G).

(d) Amount of all bases are equal

38. Which of the following is considered to be an anticancer species?

(a) CI-

CI-:p t :

'C I

\ C 1

H 3 N \ (b) Pt

(c) H3N •

CI' pt:

'C I

"NH,

H3N

CK

/ -CI

\ C 1

CH,

(d) Pt CH, C l - ^ CI

39. The relative rate of diffusion of a gas (mol. wt. = 98) as compared to hydrogen will be (a) 1/5 (b) 1/4 (c) 1/7 (d) 1

40. In a first order reaction, the concentration of the reactant decreases from 800 mol/dm3 to 50 mol/dm3 in 2 x 104 sec. The rate constant of the reaction in sec-1 is (a) 2 x 104 (b) 3.45 x 10"5

(c) 1.386 x 10-4 (d) 2 x 10"4

41. In a hydrogen-oxygen fuel cell, combustion of hydrogen occurs to (a) produce high purity water (b) create potential difference between the two

electrodes (c) generate heat (d) remove adsorbed oxygen from electrode

surfaces.

42. Which of the following is expected to be a paramagnetic complex? (a) [Ni(H 2 0) 6 ] 2 + (b) [Ni(CO)4] (c ) [ Z n ( N H 3 ) 4 ] 2 + ( d ) [ C O ( N H 3 ) 6 ] 3 +

43. A mineral having the formula AB2 crystallizes in cubic close packed lattice with the atoms A occupying lattice points. The co-ordination number of A atoms, that of B atoms and the fraction of the tetrahedral sites occupied by B atoms are (a) 2, 6, 75% (b) 8, 4, 100% (c) 3, 1 ,25% (d)6, 6, 50%

44. The number of moles of KMn04 that will be needed to react completely with one mole of ferrous oxalate in acidic solution is (a) 3/5 (b) 2/5 (c) 4/5 (d) 1

45. Interparticle forces in nylon-66 are (a) dipole-dipole interactions (b) hydrogen bonding (c) van der Waals forces (d) ionic bonds

46. In the diagram given below, the value of x is

Cu E" = +0.15 ' Cu* = + 0 - 5 0 V > Cu

(a) 0.35 V (b) 0.65 V (c) 0.325 V (d) -0.65 V

47. In the following compounds

O O O g i H

I II

I H

III

H

IV

The order of basicity is (a) IV > I > III > II (b) III > I > IV > II (c) II > I > III > IV (d) I > III > II > IV

48. In the compound given below

the correct order of acidity of the position x, y and z is (a) z > x > y (b) x > y > z (c) JC > z > y (d) y > x > z

49. In a face centred cubic unit cell of close packed atoms, the radius of atom (r) is related to the edge length (a) of the unit cell by the expression

(a) r = V2

„ N A (b)r = -

(c) 2^2 (d) r = V3a

50. How many coulombs are required for the oxidation of 1 mole of H 2 0 to 0 2 ? (a) 1.93 x 105C (b) 9.65 x 104C (c) 3.86 x 105C (d) 4.825 x 104C

B CHEMISTRY TODAY I FEBRUARY '08

Very Similar MODEL TEST PAPER

for BHU (MAINS) 2006

Exam on 18th

J u n e 2006.

SECTION - A 1. KU for H 2 0 2 is of the order of (a) 10-'2 (b) IO'14 (c) IO"16 (d) 10-

2. The percentage by weight of hydrogen in H202 is (a) 5.88 (b) 6.25 (c) 25 (d) 50.

3. The equivalent mass of MnS0 4 is half of its molecular mass when it is converted to (a) Mn 2 0 , (b) Mn0 2 (c) Mn0 4 (d) Mn04

2".

4. Experiment shows that H 2 0 has dipole moment whereas C0 2 has not. Point out the structures which best illustrate these facts.

(a) o = c = o ; A (b) 0 = C = 0 ; H - 0 - H

, 0

(c)

d)

V ^ ; H - O - H

C = 0 ; H - 0 II I O H

5. The relationship between the dissociation energy of N2 and N2

+ is (a) dissociation energy of N2 = dissociation energy

of N2+

(b) dissociation energy of N2 can either be lower or higher than the dissociation energy of N2

+

(c) dissociation energy of N2 > dissociation energy of N2

+

(d) dissociation energy of N2+ > dissociation energy

of N2.

6. The half-life of a radioactive substance is 24 hours. Time required for 12.5% of the original radioactive substance W remains in (a) 1 day (b) 2 days (c) 3 days (d) 4 days.

7. Which of the fol lowing has more unpaired ^-electrons? (a) Zn (b) Fe2+ (c) Ni3+ (d) Cu+.

8. If r is the radius of first orbit, the radius of «th

orbit of the H atom will be (a) rn2 (b) rn (c) r/n (d) r2n2.

9. Titanium shows magnetic moment of 1.73 B.M. in its compound. What is the oxidation number of Ti in the compound? (a) +1 (b) +4 (c) +3 (d) +2.

10. AgCl precipitate dissolves in NH3 due to the formation of (a) [Ag(NH4)2]OH (b) [Ag(NH4)2]Cl (c) [Ag(NH3)2]OH (d) [Ag(NH3)2]Cl.

11. The ratio between the root mean square velocity of H2 at 50 K and that of 0 2 at 800 K is (a) 4 (b) 2 (c) 1 (d) 1/4.

12. A bottle of dry ammonia and a bottle of dry hydrogen chloride connected through a long tube are opened simultaneously at both ends, the white ammonium chloride ring first formed will be (a) at the centre of the tube (b) near the hydrogen chloride bottle (c) near the ammonia bottle (d) throughout the length of the tube.

13. Gas deviates f rom ideal gas nature because molecules (a) are colourless (b) attract each other (c) contain covalent bond (d) show Brownian movement.

14. A 0.5 molal solution of ethylene glycol in water is used as coolant in a car. If the freezing point depression constant of water is 1.86°C per mole, the mixture shall freeze at (a) 0.93°C (b) - 0.93°C (c) 1.86°C (d) -1.86°C.

15. Azeotropic mixture of HCl and water has (a) 84% HCl (b) 22.2% HCl (c) 63% HCl (d) 20.2% HCl.

16. The rate constant is given by the equation

CHEMISTRY TODAY JUNE 06 43

k = P-7.e~£tm. Which factor should register a decrease for the reaction to proceed more rapidly? (a) T (b) Z (c) E (d) P.

1/20 2 Or)

17. The reaction, N 2 0 5 ( i n C C 1 4 ) 2 N 0 2

is first order in N 2 0 5 with rate constant 6 . 2 x 10 4

What is the value of rate of reaction when [ N 2 O S ] = 1 . 2 5 m o l L " 1 ?

(a) 7.75 x 10"4 mol LH s~l

(b) 6.35 x l ( r 3 mol Lr1 s"1

(c) 5.15 x 10~5 mol L"1 s-' (d) 3.85 x 10^ mol L"1 s"1.

18. Consider the chemical reaction, N 2 W + 3H 2 t e ) ->2NH 3 0 ; ) .

The rate of this reaction can be expressed in terms of time derivative of concentration of N2(ff), H2 or NH3

Identify the correct relationship amongst the rate expressions.

d [ N 2 ] = l d [ H 2 ] = 1 4 N H 3 ] dt (a)

(b)

rate = -dt 3 dt 2

„ t e = = - 3 ^ ™ = 2d [ N R i ]

dt dt dt

r a t c ^ 4 N 2 ] = 1 4 H 2 ] = 1 4 N H 3 ] (C) dt 3 dt 2 dt

(d) rate = -4 N 2 ] _ 4 H 2 ] _ 4 N H 3 ]

dt dt dt

19. Solution of 0.1 NNH4OH and 0.1 NNH4C1 has pH 9.25. Then pKh of NH4OH is (a) 9.25 (b) 4.75 (c) 3.75 (d) 8.25

20. For the electrochemical cell, M\M+\\X~\ X, E°(M+/AI) = 0.44 V and

E°(x/x~) = 0-33 V, From this data, one can deduce that (a) M + X —> Ivt + X~ is the spontaneous reaction (b) M* + X~ —> M + X is the spontaneous reaction (c) ECM = 0.77 V (d) Ece]l = - 0.77 V. .

21. What is the decreasing order of strength of bases? OH", NH2-, H - C = C", CH3 - CH2-

(a) CH3 - CH2- > NH2^ > H - C = C- > OH" (b) H - C =s C" > CH3 - CH2~ > NH2" > OH~ (c) OH" > NH2- > H - C = C" > CH3 - CH2-(d) NH2" > H - C = C" > OH" > CH3 - CH2".

22. An SN2 reaction at an asymmetric carbon of a compound always gives (a) an enantiomer of the substrate (b) a product with opposite optical rotation

(c) a mixture of diastereomers (d) a single stereoisomer.

23. 3 moles of ethanol react with one mole of phosphorus tribromide to form 3 moles of bromoethane and one mole of X. Which of the following is X? ( a ) H 3 P 0 4 ( b ) H 3 P 0 2 (C) H P 0 3 ( d ) H 3 P 0 3 .

24. Deep blue colour formed by addition of copper(II) sulphate solution to ethylamine is due to formation of (a) free Cu2+ ions in solution ( b ) ( N H 4 ) 2 S 0 4 (C) [ C U ( C 2 H 5 N H 2 ) 4 ] 2 +

( d ) C U ( O H ) 2 .

2 5 . 1 — C O \ NH NaOH * I

Br,/KOH •II

C O -

In the above sequence, II is (a) P-alanine (b) a-alanine (c) ethylenediamine (d) y-aminobutyric acid.

SECTION - B Direction: In the following questions more than one of the answers given may be correct. Select the correct answers and mark it according to the code. Code: (a) 1, 2 and 3 are correct (b) 1 and 2 are correct (c) 2 and 4 are correct (d) 1 and 3 are correct

26. Which of the following have the same mass? (1) 0.1 mole of S0 2 gas (2) 6.02 x 1022 molecules of S0 2 gas (3) 1.204 x 1023 molecules of 0 2 gas (4) 0.1 mole of 0 2 gas.

27. At constant volume, for a fixed number of moles of a gas, the pressure of the gas increases with increase in temperature due to (1) increase in the average molecular speed (2) increased rate of collision amongst molecules (3) increase in molecular attraction (4) decrease in mean free path.

28. Ground state electronic configuration of nitrogen atom can be represented by

¥ 1 ( i )

(2)

(3)

(4)

t I t

t t t

t I

1 1

44 CHEMISTRY TODAY | JUNE 06

29. Dipole moment is possessed by (1) 1,4-dichlorobenzene (2) cz's-1,2-dichloroethene (3) /ra«5-l,2-dichloroethene (4) trans-1,2-dichloro-2-pentene.

30. Which of the following forms ideal solution? (1) ethyl bromide + ethyl iodide (2) ethyl alcohol + water (3) benzene + toluene (4) chloroform + benzene.

31. The enthalpy change for the process C (graphite) —» C (g) is called

(1) heat of vaporisation (2) heat of sublimation (3) heat of atomisation. (4) heat of allotropic change

32. Which of the following statement s) is(are) correct? (1) The conjugate base of H2P04~ is HP04~. (2) The pH of 1.0 x IO"8 M HCl solution is 8. (3) Auto protolysis constant of water increases with

temperature. (4) When a solution of a weak monoprotic acid is titrated

against a strong base at half neutralisation point, pH = l/2p Ka.

33. When Cl2 gas is passed through hot NaOH, oxidation number of CI changes from (1) - 1 to 0 (2) 0 to - 1 (3) 0 to +7 (4) 0 to +5.

34. Which of the following statement/s is/are not correct? (1) rust is Fe203

(2) saline water slows down rusting (3) pure metals undergo corrosion faster than impure

metals. (4) Zn-Cu cell is called Daniell cell

35. For a first order reaction (1) the degree of dissociation is equal to (1 - e~kl) (2) a plot of reciprocal concentration of the reactant

versus time gives a straight line (3) the pre-exponential factor in the Arrhenius equation

has the dimension of time-1. (4) the time taken for the completion of 75% of the

reaction is thrice the tm of the reaction

36. Decrease in atomic number is observed during (1) a-emission (2) positron emission (3) electron capture (4) P-emission.

37. Which of the following pairs of elements have almost similar atomic radii? (1) Zr, Hf (2) Mo, W (3) Co, Ni (4) Sn, Pb.

38. .^-Hybridisation is found in the structures of (1) 0 3 (2) NH3 (3) NOj- (4) H 2 0.

39. Highly pure dilute solution of sodium in liquid ammonia (1) shows blue colour (2) exhibits electrical conductivity (3) produces sodium amide (4) produces hydrogen gas.

40. Sodium sulphate is soluble in water but barium sulphate is sparingly soluble because (1) the hydration energy of Na2S04 is more than its

lattice energy (2) the lattice energy of BaS04 is more than its hydration

energy (3) the lattice energy has no role to play in solubility (4) the lattice energy of Na 2S0 4 is more than its

hydration energy.

41. The metals that cannot be obtained by electrolysis of the aqueous solution of their salts are (1) Ag (2) Mg (3) Cu (4) AI.

42. White phosphorus (P4) has (1) six P - P single bonds (2) four lone pairs of electrons (3) P - P - P angle of 60°. (4) four P - P single bonds

43. The anodic mud obtained during electrorefining of Cu contains (1) Ag (2) Fe (3) Au (4) Zn.

44. Tautomerism is exhibited by

(1) ( 0 ) - C H = C H - O H

( 4 ) 0 = Q = 0

45. An aromatic molecule will (1) have (4n + 2) 7t-electrons (2) be planar (3) be cyclic. (4) have 4n rc-electrons

45 CHEMISTRY TODAY | JUNE 06

46. The greater stability of ter/-butyl carbocation over methyl carbocation can be explained on the basis of (1) +I-effect of the methyl group (2) hyperconjugation effect of the methyl groups (3) electromeric effect of the methyl groups (4) -I-effect of the methyl groups.

47. When nitrobenzene is treated with Brz in presence of FeBr3 , the major product formed is m-bromonitrobenzene. The statements which are related to obtain the m-isomer are (1) the electron density on meta carbon is more that

at ortho and para-positions (2) the intermediate carbonium ion formed after initial

attack of Br+ at the /we?a-positions is least destabilised

(3) loss of aromaticity when Br+ attacks at the ortho and para positions and not at /wefa-position

(4) easier loss of H+ to regain aromaticity from the meta position than from ortho and para positions.

48. /7-Chloroaniline and anilinium hydrochloride can be distinguished by (1) Sandmeyer reaction (2) NaHC03

(3) AgN03 (4) carbylamine test.

49. Hydroxylamine reacts with (1) CH3COCl (2) CH3COCH3

(3) CH3COOC2H5 (4) CH3CONH2.

50. Under Wolff-Kishner reduction conditions, the conversions which may be brought about are (1) benzophenone into diphenylmethane (2) benzaldehyde into benzyl alcohol (3) cyclohexanone into cyclohexane (4) cyclohexanone into cyclohexanol.

SOLUTIONS

1. (a) : KA for H202 = 1.55 x 10"12. 2. (a) : Mol. mass of H 20 2 = 2 x 1 + 16 x 2 = 34

2 Percentage of hydrogen = — xlOO

100 17 = 5.88%.

+2 3. (b) : Mn2 +4

-*• Mn02 + 2e~

molecular mass _ M Eq. mass change in oxidation number 2

4. (a) : Water has a non-linear while C0 2 has a linear shape.

5. (c): N2 : KK a(2sf a*(2sf n(2Px)2 n(2Py)2 c(2pzf Bond order = 3 N2+ : KK o(2.?)2 cr*(2.v)2 n(2px)2 n(2P>)2 a(2pz)]

Bond order = 2. We know that higher the bond order, higher is the bond dissociation energy as well as greater is the stability.

Dissociation energy of N2 > dissociation energy of N2

+.

6. (c) : tm = 24 hrs = 1 day

X = ^ = 0.693 day'1

t = -

hn 2.303

I 2.303

. N0 2.303, 100 l o g — = log N 0.693 12,5

log8 = 3 days.

7.

8.

0.693 (b) : Zn : Is2 2s2 2p6 3s2 3p" 4s2 3d10

Fe2+ : Is2 2s2 2ph 3s2 3p6 3d6

Ni3+ : l.v2 2.S'2 2p" 3s2 3p6 3d1

Cu+ : Is2 2s2 2p6 3s2 3p6 3dw

(a) : If r is the radius of first orbit, then radius of orbit of the H-atom is rn2.

9. (c) : Magnetic moment = jn(n + 2) B.M Magnetic moment of Ti ion = 1.73 BM

V«(« + 2) =1.73 or, n(n + 2) = 3 or, n = 1. /. Ti ion has 1 unpaired electron i.e. 3d] configuration.

Complete configuration of Ti3+ ion is Ti3+ (Z = 22) : [Ar]18 3a".

10. (d) : AgCl + 2NH3 [Ag(NH3)2]Cl.

11. (c) : (i = 3 RT M

P L = 3RT\_ / 3R7i p2 V M\ / V M2

Hi I3RT, M2

o r ' ^ I MI 3 RT2

TXM2

MXT2

50x32 800x2

= 1.

12. (b) : Molecular mass of NH3 = 17 Molecular mass of HCl = 36.5

Rate of diffusion, r oc 1

molecular mass NH3 will diffuse faster than HCl.

So NH3 vapour will travel a longer distance than HCl vapour in same time.

Ring formed will be nearer to HCl bottle. 13. (b): For gases to show ideal behaviour there should be no attractive or repulsive forces among the molecules.

46 CHEMISTRY TODAY | JUNE 06

14. (b) : A7} = K,x m = 1.86 x 0.5 = 0.93°C A Tf= T°f - Tf

Tj = r f - A7>= 0 - 0.93 = - 0.93°C. 15. (d)

16. (c): When k increases, rate of reaction also increases. PZ

k = - E/RT For k to increase, P, Z, Tshould increase and E should decrease. 17. (a) : Rate = *[N205] = 6 . 2 x l O ^ x 1.25

= 7.75 x 10-4 mol L-1 s"1.

18. (a) : N 2 0 f ) + 3H 2 a f ) -»2NH 3 { l f )

Instantaneous rate = _ 4 N 2 ] I d [u 2 ] l 4 N H 3 ] dt 3 dt

[salt] dt

19. (b) : As pOH = pKh + log i

[base] but [NH4C1] = [NH4OH], pOH = pKh

:. pKh = pOH = 14 - pH = 14 - 9.25 = 4.75 20. (b) : M | M+ \\X~\X The two half cell reactions for the cell are

M M+ + e~ or, X + e~ X~

M+X^> M+ + X-£°oeii = E°X/X—E°M+/M = 0.33 V - 0.44 V

= - 0.11 V. As E° ceii is negative the reaction involved is non-spontaneous. The reverse reaction, however, will be spontaneous.

M + + X~ M + X 21. (a) : The strength of their conjugate acids are in the order:

H 2 0 > HC = CH > NH3 > CH3 - CH3

Stronger the acid weaker is its conjugate base. Therefore correct order of strength of their conjugate bases is

CH3 CH,~ > NH,- > HC = C" > OH-22. (d): In SN2 reaction, an optically active alkyl halide gives an optically active product. The product may not have opposite optical rotation to that of the alkyl halide as the two are not enantiomers. Therefore a single stereoisomer is the product.

23. (d) : 3CH3CH,OH + PBr3 3CH3CH2Br + H3P03

W 24. (c) : Cu2+ + 4C2H5NH2 [Cu(C2H5NH2)4]2+

tetraethylamine copper(ll) ion

(deep blue complex)

25. CHXONH,

CHjCOOH (I)

Br2/KOH ^ P C H 2 N H j

aCH2COOH p-alanine (II)

26. (a) : 0.1 mol of S02 = 6.4 g 6.02 x 1022 molecules of S0 2 = 0.1 mol S0 2 = 6.4 g 1.024 x 1023 molecules of 0 2 = 0.2 mol 0 2 = 6.4 g 0.1 mole of 0 2 = 3.2 g. 27. (b ) : With increase in temperature, average speed of the molecules as well as rate of collision increases. 28. (c) : In (2) and (4), the unpaired electrons have spin in the same direction.

CI H - C - CI

29. (c) : [ O J ( H - 0 ) , II (H#0) Y H - C - CI

CI

H - C - CI II (U = 0)

CI - C - H

CI CI C H , - C - C 1 I I II

C H 3 - C = C - C H , - C H 3 i.e., C1-C-CH,CH 3

(trans) (M. * 0)

30. (d) : (1) and (3) have pairs of similar liquids. Hence they form ideal solution. 31. (c) : C (() —> C ( j f ). This process is sublimation as well as conversion of crystalline allotropic form (graphite) to amorphous form (gas carbon). 32. (d)

-1 +5 33. (c) : 3C12 + 6NaOH—> 5NaCl + NaCIO, + 3H,0 .

34. (a) : Rust is Fe203-JcH20. Saline water speeds up rusting. Pure materials almost do not corrode. 35. (d) : Degree of dissociation = 1 - e~k'

k = Ae-W' As EJRT is dimensionless, therefore A has the dimension of k.

36. (a) : Electron capture is addition of . Hence atomic number decreases by one.

47 CHEMISTRY TODAY | JUNE 06

37. (a) : Due to lanthanide contraction, the atomic radii of Zr and Hf and Mo and W are almost similar. In case of Co and Ni, the contractive effect of increased nuclear charge is almost balanced by the increase in size due to increasing shielding effect. As a result, the atomic radii of Co and Ni are nearly the same. 38. (c) : NH3 and H 2 0 contain jp3-hybridized N and O atoms respectively. 39. (b) : Dilute solution of Na in liquid NH3 will not produce sodium amide and hydrogen gas instantaneously. These are metastable and when catalysed give hydrogen and amide. 40. (b) : The solubility increases as the hydration energy increases and the lattice energy decreases. 41. (c) : Mg and AI are highly electropositive metals and cannot be obtained by electrolysis of their aqueous salt solutions. 42. (a) : White phosphorus (P4) has tetrahedral structure in which each P atom lies at the corners of the regular tetrahedron. There are six single P - P bonds, four lone pairs of electrons and P - P - P bond angle of 60° in the P4 structure. 43. (d) : Noble metals like Ag and Au settle down as anode mud. 44. (a) : (1) shows tautomerism since aldehydes are more stable than vinyl alcohols.

C6H5 - CH = CH - OH •«—?

C6HS - CH2 - CH = O (2) shows tautomerism because enol form is stabilized by aromatic character.

r v o o

X //

OH

OH

(3) shows tautomerism because enol form is stabilized by H-bonding.

q ^ c h 0 O — H

(4) does not show tautomerism because it does not have hydrogens on a saturated carbon at a-positions. 45. (a) : Cyclic planar polyenes containing (4n + 2) Tt-electrons are aromatic in character. 46. (b) : +I-effect and hyperconjugation effect of the CH3 groups. 47. (b ) : Due to the electron-withdrawing effect of the NO : group, the electron density on m-carbon is more

than that at o- and /^-positions; and the intermediate carbonium ion resulting from meta attack of Br+ is least destabilised. Thus (1) and (2) are correct. 48. (a) : Sandmeyer reaction on p-chloroaniline gives p-dichlorobenzene which is a solid (m.p. 325 K) while that on anilinium chloride gives chlorobenzene which is a liquid (b.p. 405 K).

Sandmeyer CI NH,

reaction

m.p. 325 K.

NH,C1~ Sandmeyer^ < / Q ^ c 1

b.p. 405 K

reaction

Further, anilinium hydrochloride is an acid salt and hence liberates CO, from NaHC03 and gives white ppt. of AgCl with AgN03 solution.

C6H5NH3C1 + NaHCO,

C6H5NH3C1 + AgN03

• C02 +H 2 0 + NaCl

+ C6H5NH2

AgCl + C6H5NH3N07 /7-ChloroaniIine is neither acidic (actually it is basic) nor contains an ionic chlorine and hence does not give the above reactions. Thus (1), (2) and (3) are correct. However, both p-chloroani l ine and anilinium hydrochloride, being 1° amines, give carbylamine test.

O

49. (a) : CH, - C - CI -HCl

o II

CH3 - c - OC2H5 NH,OH

o II

C H 3 - C - N H O H hydroxamic acid

o C H 3 - C - N H O H

hydroxamic acid

O

C H 3 - C - C H 3

NOH NH;OH

-H ,0 CH3 - C - CH3

oxime

50. (d) : (C6H5)2C = O ™ : h " e r » (C6H5)2CH:

benzophenone diphenylmethane

O

Wolff-Kishner reaction

cyclohexanone •o cyclohexane

4 8 CHEMISTRY TODAY | JUNE 06

Practice Paper for

1. In a reaction 2A —> product, the concentration of A decreases from 0.5 mol/litre to 0.4 mol/litre in 10 minutes. The rate during this period will be (a) 0.010 (b) - 0 . 0 1 0 (c) 0.005 (d) - 0.005. 2. The value of AH for the reactions

C ( * ) + i 0 2 ( g ) - > C 0 ( g ) a n d

C0(g) + i 0 2 ( g ) - + C 0 2 ( g )

are 100 and 200 kJ respectively. The heat of reaction for C(s) + O, (g) —» C02 (g) will be (a) 50 kJ (b) 100 kJ (c) 150 kJ (d) 300 kJ. 3. The relative rates of diffusion of U235 and U238 are (a) 1.042 (b) 1.0062 (c) 1.612 (d) 1.0142. 4. What will be the mass of photon with a wavelength equal to 1.54 x IO"8 cm? (a) 1.42 x 10~32 kg (b) 2.24 * 10~30 kg (c) 1.60 x ]0~32 kg (d) 3.60 x 10"32kg.

5. If the half cell reaction A + e~ —» A~ has a large negative reduction potential, it follows that (a) A is readily oxidised (b) A is readily reduced (c) A~ is readily oxidised (d) A~ is readily reduced. 6. The vapour density of a gas is 11.2. The volume occuped by 11.2g of this gas at STP is (a) I litre (b) 11.2 litre (c) 22.4 litre (d) 2 litre. 7. What volume of 0.5 M solution contains 0.1 mole of the solute ? (a) 0.125 mL (b) 0.225 mL (c) 0.20 mL (d) 22.5 mL. 8. Heat of neutralisation for the reaction,

NaOH + HCl NaCl + H 2 0 is 57.2 kJ/mol. What will be the heat released when 0.25 mole of NaOH is titrated against 0.25 mole of HCl?

46

(a) 22.5 kJ/mol (b) 57.1 kJ/mol (c) 14.3 kJ/mol (d) 28.6 kJ/mol. 9. When IO-8 mol HCl is dissolved in one litre of water, the pH of the resulting solution will be ? (a) 8 (b) above 8 (c) 7 (d) above 7. 10. For which of the following reaction the equilibrium constant depend on the units of concentration? (a) _H, + I 2 ^ 2 H I

(b) N 0 ( g ) ^ N , ( g ) + l 0 2 ( g )

(c) CH3COOH(/) + C 2 H 5 O H ( / ) ^ CH3C00C2H5(/) + H20(/)

(d) C O C l 2 ( g ) ^ C O ( g ) + Cl2(g).

11. If Raoult's law is obeyed, the vapour pressure of the solvent in a solution is directly proportional to (a) mole fraction of solute (b) mole fraction of solvent (c) mole fraction of solute and solvent both (d) volume of the solution. 12. The standard oxidation potential o fZn and Ag in water at 25 °C are

Zn w Zn2+ + 2e~, E° = 0.76 V Ag w -> Ag+ + e~, E° = - 0.80 V

Which reaction actually takes place? (a) Zn2+(aq) + Ag+(aq) -> Zn(s) + Ag(j) (b) Zn(» + 2Ag(i) Zn2+(a<7) + Ag+(aq) (c) Zn(.?) + 2Ag+(aq) Zn2+(aq) + 2Ag(s) (d) Zn2+ + 2Ag+(s) 2Ag+(a<7) + Zn(j).

13. The pH of water at 25°C is 7. Which ofthe following should be true if water is heated to 50°C? (a) pH will increase (b) pH will decrease (c) pH will remain same (d) [H+] will increase but [OH ] will decrease. 14. The freezing point of a 0.05 molal solution of a non-electrolyte in water (Kf = 1.86) is (a) 1.86°C (b) -0.093°C (c) 0.093°C (d) 0.93C.

CHEMISTRY TODAY | JUNE 06

15. Which of the following will weight more at STP? (a) one litre H2 (b) one litre 0 2

(c) one litre N2 (d) one litre Cl2.

16. In the Haber process, metallic oxides catalyses, reaction between gaseous nitrogen and hydrogen to yield ammonia whose volume at STP relative to the total volume of reactants should be (a) the same (b) half (c) one fourth (d) three fourth.

17. Heat of reaction for C0(g) + 0 2 ( g ) - > C 0 2 ( g ) at

constant volume is -67.71 k. cal at 17°C. The heat of reaction at constant pressure at 17°C is (a) 68.0 k cal (b) - 68.0 k cal (c) - 67.42 k cal (d) 67.42 k cal.

18. 3-methyl-2-butanol on treatment with HC1 gives (a) 2-chloro-2-methyl butane (b) 2-chloro-3-methyl butane (c) 2, 2-dimethyl pentane (d) 2, 4-dimethyl pentane.

19. For a reaction 2A + B^C. + D, the partial pressures of A, B, C and D at equilibrium are 0.5, 0.8, 0.7 and 1.2 atmospheres respectively. The value of KP

for this reaction is (a) 0.24 atm 1 (b) 2.4 atirr1

(c) 0.42 a t m 1 (d) 4.2 atm-'.

20. HBr reacts fastest with (a) propan-l-ol (b) propan-2-ol (c) 2-methyl propan-l-ol (d) 2-methyl propan-2-ol.

21. CH3CHO and C6H5CHO can be distinguished by (a) Fehling's reagent (b) Tollen's reagent (c) Schiff's reagent (d) HCN.

22. Which one is a gem-dihalide? (a) CH3CHBrCH2CH2Br (b) CH3CHBrCH2Br (c) CH2BrCH2CH2CH2Br (d) CH3CHBr2.

23. in allene structure, three carbon atoms are joined by (a) three o-bonds (b) three cr-and three 71-bonds (c) two o-and two Ti-bonds (d) two cr-and one 7t-bond.

24. Which of the following aldehydes will not form an aldol when treated with dilute NaOH?

(a) C6H5CH2CHO (b) CH3CH2CHO (c) CH3CHO (d) (CH3)3CCHO.

25. Which of the following will be most highly ionised in water? (a) CH2ClCH2CH2COOH (b) CH3CHC1CH2C00H (c) CH3CH2CCl2COOH (d) CH3CH2CHClCOOH.

26. The amide contains (a) dative bond (b) sigma bonds and a n-bond (c) sigma bond only (d) 27i-bonds and aa-bond.

27. The reaction of a primary amine with an aldehyde gives a/an (a) acid (b) anils (c) nitrite (d) nitro compound. 28. Which of the fol lowing would not undergo Cannizzaro's reaction ?

CH, I

(a) C H - C - C H O (b) CH,CH,— CH— CHO

CH, CH,

(c) CH3CH2CH2CH2CH2CHO ,, C H - C H - O L C H O

(d) I " CH,

29. Which does not burn with sooty flame? (a) C6H6 (b) C6H5CH,OH (c) C6H5NH2 (d) C6H5OH. 30. The catalyst used for preparing toluene with a mixture of C6HS and CH3C1 is (a) anhydrous A1C13 (b) Pd (c) Pt (d) Ni.

31. The reaction of CH3COCH3 with HI in presence of red phosphorus yields (a) C2H5OH (b) CH3CH2CH3

(c) CH3CHICH3 (d) CH3CH2CHjOH.

32. Formation of disubstituted derivatives from monosubstituted benzene results the following number of isomers (a) one (b) two (c) three (d) four. 33. When propyne is treated with aqueous H2S04 in presence of HgS04 the major product is (a) propanal (b) acetone (c) propanol (d) propyl hydrogen sulphate.

47 CHEMISTRY TODAY | JUNE 06

34. Which does not decolourise bromine water? (a) CH 2 = CH-CH, (b) CH2=CH2

CHa / C H , (c) C H , C = C H (d) C H / c = c \ C H j -

35. A certain element forms a solid oxide which when dissolved in water forms an acidic solution. The element is (a) neon (b) sodium (c) phosphorus (d) sulphur. 36. The decomposition temperature is maximum for (a) BaCO., (b) CaC03

(c) MgCO, (d) SrC03 . 37. Ozone oxidises moist sulphur to (a) H2S04 (b) S0 3

(c) SO, (d) none of these. 38. Aqueous solution of CuS04-5H,0 changes blue litmus paper red due to (a) presence of Cu2+ ions (b) hydrolysis of Cu2+ ions (c) presence of S04

2" ions (d) reduction taking place. 39. C102 is an anhydride of (a) chlorous acid (HC102) (b) chlorine acid (HC103) (c) mixed anhydride of HC102 and HC103

(d) none of these. 40. On passing the vapours of Cr02Cl2 in NaOH and then adding CH3COOH and (CH3COO)2 Pb to it the ppt formed is (a) PbCl2 (b) PbCr04

(c) CrCl3 (d) none of these.

Instructions : In each of the following questions, a statement of Assertion (A) is given followed by a corresponding statement of Reason (R) just below it. Of the statements, mark the correct answer— (a) If both Assertion and Reason are true and the

Reason is a correct explanation of the Assertion. (b) If both Assertion and Reason are true but Reason

is not a correct explanation of the Assertion. (c) If Assertion is true but the Reason is false. (d) If both Assertion and Reason are false.

41. Assertion : Density is a derived physical quantity. Reason : A quantity, obtained with the help of two or more fundamental quantities, is a derived quantity.

42. Assertion : 2H,(j;) + 0, ( j ; ) —> 2H20(/) is the reaction of formation of water.

Reason : The oxygen is an excess reagent. 43. Assertion : Charles' law is applicable to liquids. Reason : Gas constant per mole is known as Hoffman's constant. 44. Assertion : Dissolving some salt in a liquid increases its viscosity. Reason : Viscosity is the resistance offered by a liquid to its flow. Reason : Osmosis is the process, in which solvent flows through semiperable membrane into the solution. 45. Assertion : In Balmer series of hydrogen spectrum, the values «, = 2 and n2 = 3, 4 and 5. Reason : The values of n for a line in Balmer series of hydrogen spectrum having the highest wavelength, is 4 and 6. 46. Assertion : The heat of neutralisation of

HNO, + NH4OH is - 51.66 kJ. Reason : The heat of neutralisation of

NH4OH + CH3COOH is - 55.78 kJ. 47. Assertion : Transport of oxygen by haemoglobin in the blood can be explained on the basis of Le Chatelier's principle. Reason : Removal of C 0 2 from tissues by the blood can also be explained by Le Chatelier's principle. 48. Assertion : Spectator ions are the species that are present in the solution but do not take part in the reaction. Reason : The phenomenon of formation of HzO, by the oxidation of HzO is known as autoxidation. 49. Assertion : The- order of electron affinity of halogens is I > Br > CI > F. Reason : Fluorine shows variable oxidation states. 50. Assertion : Explosion takes place when conc. H2S04 is added to KMn04 . Reason : In this reaction, an explosive acid is formed. 51. Assertion : Si02 is solid, but CO, is gas. Reason : C0 2 molecule is non-linear with net zero dipole moment.

52. Assertion : SiCl4 is resistant to hydrolysis, but CCI4 is easily hydrolysed. Reason : It is due to the presence of 3d orbitals in silicon atom.

53. Assertion : Alkali metals dissolve in liquid ammonia to give deep blue ammonical solution. Reason : Blue colour of the solution is due to the formation of ammonical electrons.

48 CHEMISTRY TODAY | JUNE 06

54. Assertion : [Fe(CN)6]3_ is weakly paramagnetic, while [Fe(CN)6]4~ is diamagnetic. Reason : In both cases, unpaired electrons are absent. 55. Assertion : Boiling points of alcohols are higher than their hydrocarbons. Reason : It is due to intermolecular hydrogen bonding in alcohols. 56. Assertion : In sublimation, organic substances change directly from solid to vapour without passing through liquid state and vice-versa. Reason : Distillation involves the process of heating liquid to convert it into vapour and condensing the vapour to get back the liquid. 57. Assertion : The molecule, in which one doubly-bonded carbon atom carries identical atoms or groups, exhibits geometrical isomerism. Reason : The geometrical isomer has identical physical properties. 58. Assertion : Ethers are insoluble in water. Reason : Ether molecules are not capable of forming H-bonds with water molecules. 59. Assertion : Fumes and smoke are gaseous air pollutants. Reason : Hydrocarbons and ozone are particulate pollutants. 60. Assert ion : The activity of enzymes is pH-dependent. Reason : A change in pH affects solubility of the enzyme in water.

ANSWERS

1. (d) : Rate = Rate of disappearance of A per mole

= _ i f M = _ 1 (0.5-0.4) = _ 0 , 0 0 5 2 dt 2 10

Negative sign indicates the fall in concentration of A.

2. (d) : C(.s-) + l o 2 ( g ) - > C O ( g ) + 100kJ

CO( g ) + | o 2 (g) -> C02 (g) + 200k J

On addition, we get C ( J ) + 0 2 (g) —> C 0 2 (g) + 300 kJ.

3. (b ) : According to the Graham's law of diffusion, i

rate of diffusion of a gas, r x ^molecular weight, M

= 1.0062.

h 4. (a) : According to de Broglie equation, X = —-

m = h/Xv Given that, X = 1.54 x IO"8 cm = 1.54 x IO"10 metre, h = 6.626 x 10~34 Js Velocity of photon = velocity of light = 3 x io8 m/s

6.626xlO-34 , „„„ , , „ „ , •• m = , r . II) , = 1-4285 x io-32 kg.

1.54x10 x 3 x l 0 5

5. (c) : Large negative reduction potential of A to A~ indicates that A~ has large oxidation potential, hence A~ is readiliy oxidised. 6. (b) : Vapour density of gas = 11.2

, . Mol.wt Vapour density=—

.-. mol. wt. = 2 x n . 2 = 22.4 Y 22.4 g of gas occupies = 22.4 litres at STP

11.2 g of the gas occupies

- 2 2 4 x 1 L2-litre at STP= n . 2 litre at STP.

7. (c) 22.4 Molarity

, , ys number of moles of solute(n) (M) = -

Volume(K) = -

8.

mole, i.e., • = 14.3 kJ/mol.

volume of solution in litres(K)

— = M = 0.20mL M 0.5

(c) : For neutralisation of 0.25 mole of NaOH by 0.25 mole of HCl, 0.25 mole of H 2 0 will be formed. Therefore, the amount of heat released would be one fourth of the amount of heat of neutralisation for one

57.2 4

9. (d ) : On dissolution of IO-8 mole HCl in one litre water, [H+] = IO"8 from HCl. But the [H+] from water can not be neglected. Hence the pH of the resulting solution will be above 7. 10. (d) depends upon where n = Number of moles of products -

Number of moles of reactants Kp = (atm)An and Kc = (mole/litre)^'

For (d), An = 2 - 1 = 1 Hence, unit of Kp and Kc will depend on concentration for (d). 11. (b): According to Raoult's law, the partial pressure of any volatile constituent of a solution at a constant temperature is equal to the vapour pressure of pure constituent multiplied by the mole fraction of that constituent in the solution. Vapour pressure of solvent °= mole fraction of solvent Vapour pressure of solute « mole fraction of solute

Units of equilibrium constant, K/t and Kc

49 CHEMISTRY TODAY | JUNE 06

12. (c) : According to given half cell reactions. Zn is oxidised whereas Ag is reduced. Therefore, reaction will be Zn(.v) + 2Ag+ (aq)^> Zn2*(aq) +2Ag(s).

13. (b) : The actual pH of the sample can change with temperature due to a change in the hydrogen ion activity in the solution, because ionization of compounds and hydrogen ion activity in the solution may be temperature dependent. For pure water, pH decreases with increasing temperature. But the degree of influence of temperature on pH is a function of the alkalinity of the water. [The molecules are moving faster, with more energy, so we may be sensing more of the ions in a time .frame]. 14. (b) : Depression in freezing point, ATR — KF XM = 1.86 x 0.05 = 0.093. Freezing point of the solution

= Freezing point of solvent -AT) = 0°C - 0.093°C = - 0.093°C.

15. (d) : At STP volume « number of moles weight

mol. weight and number of moles

weight .'. volumecc 2-;

mol. weight If volume remains constant then weight molecular weight.

One litre Cl2 weight more at STP. 16. (b) : In Haber process, N, + 3H2 ^ 2 N H 3

Four moles of reactants from two moles of NH,. Volume of NH, relative to the total volume of

reactants is half. 17. (b) : CO(g) + ^ 0 2 ( g ) —> C02 (g)

At constant volume A// =-67.71 kcal. Heat of reaction (at constant pressure)

AH =-67.71 + - i j x 2 x i 0 " 3 x290

= -67.71 -0.290 =-68.0 kcal. CH, I

18. (b) : CH — CI-I — C H - CH, + HC1—> I

OH 3-methyl-2-butanol

CH, ' I

CH - C H - C H - CH,+ H20

CI 2-chloro-3-methyl butane

19. (d) : 2A + B P P

-C + D _ 0.7x1.2

(PA) '„

= 4.2 atm" PH (0.5)2 xO.S

20. (d) : Tertiary alcohol, 2-methylpropan-2-ol CH, I

CH — C CH,^ w j ] | r e a c t fastest with HBr because its

OH -OH group will be replaced easily due to the +/ effect of three -CH 3 groups. 21. ( a ) : CH3CHO reduces Fehling solution to give a precipitate of Cu 20 while CsH5CHO does not have any reaction with Fehling solution. 22. (d ) : Gem-dihalides are those having two halogen atoms on the same carbon atom. 23. (b) : Allene is C H , = C = C H , Therefore, three C-atoms are joined by 3a- and 3n-bonds. 24. (d ) : Aldehydes which have atleast one H-atom at a-carbon atom show aldol condensation.

CI I

25. (c) : C H , - C H - C - C O O H is the strongest acid i CI

due to the - / effect of the two chlorine atoms on the alpha carbon. Therefore, it will be most highly ionised in water. 26. (b) : Amide R - C - N H ,

' II O

c-bonds and a rc-bonds 27. (b) : R—CH= O + H,NR'

(-11,0) R - C H = N - R '

Schiffs base (anils) 28. (a) : Compound (a) have no a-H-atom, which is necessary condition to show Cannizzaro's reaction. 29. (b) : Benzyl alcohol though aromatic does not burn with sooty flame (exceptional nature).

CH,

30. ( a ) : r V C H C 1 A n h y . A I C I , +HCI

(Friedal-Craft reaction) Anhydrous A1C13 acts as a halogen carrier. Mechanism : (i) CH3C1+ A1CI, H> CH,® + A1C14"

(ii) [ O J +eCH,—> ? H ,

50 CHEMISTRY TODAY | JUNE 06

A\ (iii) I •,©; lNCH,+ AICi;

C H ' \ RedP C H ' \

Propane +H,0 + 21,

32. (c): Three isomers ortho, meta and para are formed. 33. (b ) : When propyne is treated with aqueous H 2S0 4

in presence of HgS0 4 it adds one molecule of water. H,SOJ

CH,C— CH + H,0 ~ ,,„> - HgSO, OH O

C H , - C = C H , t a u t o m e " s a t i o "> CH,— C— CH, (Unstable) (Acetone)

34. (d)

35. (c) : Oxides of both phosphorus as well as sulphur are acidic in nature but 'P ' is a solid while 'S ' is a gas. Oxides of Na is a strong base. 36. ( a ) : Barium salts are quite stable because of great electropositive nature of Ba. Hence barium compounds possess high decomposition temperature. 37. (a) : Ozone oxidises moist sulphur to sulphuric acid.

S+ H , 0 + 30 3 -> H 2S0 4 + 30 2 t .

38. (b) : Hydrolysis of cupric ions produces H' ions according to the reaction

Cu2+ + 2 H , 0 Cu(0H)2 + 2H+

i.e., it acts as Lewis acid. Therefore turns blue litmus paper red (characteristics property of acids). 39. (c) : Hydrolysis of CIO;,

2C102 + HjO —> HCIO, + HC10, produces both HCIO, and HCIO,. Thus it is a mixed anhydride of both HC102 and HCI03 . 40. (b) : Chromyl-chloride test: Cr02Cl2 + 4NaOH Na2Cr04 + 2NaCl + 2HzO

yel low

(CH3COO)2Pb + Na 2 Cr0 4

PbCr04-i + 2CH 3 C00Na. yellow

41. (a) : A physical quantity, which is obtained with the help of two or more fundamental quantities, is a

derived quantity. Since density = 77-;—-— . Thus the J Volume

CHEMISTRY TODAY | MAY '06 5 1

j density is a physical quantity derived f rom two fundamental quantities mass and length.

42. (b) : The balanced equation for the formation of water is

2H2 f e ) + O 2 0 ; ) ^ 2 H 2 O (/)

In this reaction, 1 volume of hydrogen combines with 0.5 volume of oxygen to form 1 volume of water and 0.5 volume of oxygen remains unused. That is why. oxygen is an excess reagent.

43. (d) : According to Charles' law that at constant pressure, the volume of a given mass of a gas is directly proportional to its absolute temperature i.e.. T. And the Charles law is applicable only to gases. Gas constant {K) for 1 mole of a gas is termed as universal gas constant (R). • And for one mole (i.e.. n = 1) K = nR = 1 * R= R. 44. (a) : The viscosity is the resistance offered by a liquid to its flow. Dissolving some amount of sg.lt in a liquid increases its viscosity i.e., resistance to flow

• increases.

i 4,5. (c) : In the Balmer series of hydrogen spectrum, the highest wavelength or lowest energy is between n\ = 2 and n2 = 3 (where n = Principal quantum number). 46. (b) : Heat of neutralisation is the change in heat

i content of a system when one gram equivalent of an acid is neutralized by one gram equivalent of a base and vice versa. The heats of neutra l isa t ion of

10 Model Test Papers

12 Years Solved Papers (with detailed solutions)

Success Tips from AI1MS Toppers

FAQ's and Information on AIIMS

General Knowledge Questions

Assertion and Reason Questions

AIIMS EXPLORER

HNO, + NH 4OH and NH4OH + CH3COOH are -51.66 kJ and -55.78 kJ respectively.

47. (b) : From the Le Chatelier's principle Hb(s) + 0 ? u l ^ Hb02( s )

(Haemoglobin) (Oxygen) (Oxyhaemoglobin)

Thus in the lungs, partial pressure of oxygen (O) appreciably shows forward reaction. However, when the oxygen passes through the tissues, its partial pressure decreases to favour the backward reaction and releasing oxygen. Also from the Le Chatelier's principle that

CO, (f!) + H,0, / , ^ H+ („(/) + HC0 3- („v)

Thus in tissues, carbon dioxide (C02) dissolves in H : 0 due to high pressure, whereas in lungs C0 2 is released due to low pressure. 48. (b) : The species which are present in a solution but do not take part in the reaction and are also omitted while writing the net ionic reactions, are called spectator ions e.g., Zn + 2H4" + 2C1" Zn2+ + 2CL + H2. In this reaction, Cl~ ions could be omitted and are called spectator ions because these appears on both side of the reaction.Terpentine, phosphorus and metals like zinc and lead can absorb oxygen from air in the presence of water. The water is oxidised to hydrogen peroxide. The phenomenon of formation of H 20 2 by the oxidation of H 2 0 is known as auto-oxidation e.g., the chemical reaction PbO, + H 2 0 PbO + H202 .

49. (d) : The electron afinities of iodine (I), bromine (Br), chlorine (CI) and fluorine (F) are -295, -324, -348 and - 333 kJ moL' respectively. Thus the order of electron affinities of halogens is CI > F > Br > I. Fluorine has no vacant J-orbitals, therefore no excitation of valence electrons is possible. Thus it does not exhibit variable oxidation states.

50. ( c ) : 2KMn0 4 + H 2 S0 4 -> K2S04 + Mn207 + H , 0 In this reaction, explosion takes place due to the formation of manganese oxide (Mn207), which is explosive. 51. ( c ) : In silicon dioxide (Si02), the atom of silicon (Si) is tetrahedrally bonded to four oxygen atoms(O), and each oxygen atom is bonded to two silicon (Si) atoms. Thus the structure of S i0 2 extends in three dimensions to form a network solid. The molecule of carbon dioxide (C0 2 ) is linear with net zero dipole moment. That is why, Si02 is solid but C0 2 is gas.

52. (d) : CC14 + H 2 0 no action and (Carbon tetrachloride) (Water)

Si(CI)4 + H , 0 Si(OH)4 + 4HC1 ' (Silicicchloride) (Water) (Silicic hydroxide) (Hydrochloric acid)

52

Thus CCI4 is resistant to hydrolysis while SiCl4 is easily hydrolysed. Electronic configuration of silicon (Si) is Is2 ,2s2 ,2p", 3s2 ,3p2 . Thus it does not contain d-orbital in its atom. 53. (a) : All alkali metals dissolve in liquid ammonia to give deep blue ammonical solution. The dissolution of the metal is accompanied by its ionisation. Blue colour of the solution is due to the formation of ammonical electrons. 54. ( c ) : The electronic configuration of Fe-,+ ions in [Fe tCNy 3 - is \s2 2s2 2p" 3s2 3pb 3 a n d electronic configuration of Fe2+ ions in [Fe(CN)6]4" is Is2 2s2 2p" 3s2 3pb 3d6. Since [Fe(CN)6]3~ has unpaired electrons, that is why, it is weakly paramagnetic. And in [Fe(CN)6]4~ there are no unpaired electrons, that is why, it is diamagnetic.

55. (a) : The boiling points of alcohols are much higher as compared to the boiling points of their corresponding hydrocarbons due to the intermolecular hydrogen bonding in alcohols. 56. (b) : Certain organic substances change directly from solid to vapour, without passing through the liquid state and vice-versa. This is called sublimation. The substances, having vapour pressure equal to the atmospheric pressure but much below their respective melting points, can be sublimated and purified by this method. Distillation is the process of heating a liquid to convert it into the vapour, and then condensing the vapour to get back the liquid.

57. (d): In geometrical isomerism, two atoms or groups attached to each doubly-bonded carbon atom, should be different. If one of the two doubly-bonded carbon atoms carries two identical atoms or groups, then the molecule does not exhibit geometrical isomerism. 58. (a) : Ethers are insoluble in water, as their molecules are neither capable of forming H-bonds with water (H20) molecules nor capable of breaking H-bonds between water molecules. 59. (d) : Fumes and smoke are particulate pollutants. And hydrocarbon and ozone are gaseous air pollutants.

60. ( b ) : Enzymes are proteins, which act as catalysts in biochemical reactions. Since pH affects activity of enzymes as protonat ion-deprotonat ion causing denaturation of the protein structures, therefore the activity of enzyme is pH dependent. The change in pH value of an enzyme affects the solubility of the enzyme in water.

• CHEMISTRY TODAY | JUNE 06

Practice Paper for

UP-CPMT 2006

on 21s* May

2006

1. Ionic radii of the halogens is found to be greater than their corresponding atomic radii because addition of extra electrons cause (a) an increase in the number of protons (b) an increase in the number of neutrons (c) a greater repulsive force among the electrons (d) none of these.

2. Which of the following statement is correct? (a) H 2 0 and OF, both are linear in shape (b) H20 is linear whereas OF2 is bent molecule (c) both H 2 0 and OF2 have a bent structure •(d) H , 0 is bent whereas OF, has a linear structure.

3. Arrange the following compounds in order of increasing dipole moment. 1. toluene II. /n-dichlorobenzene III. o-dichlorobenzene IV. p-dichlorobenzene. (a) I < IV < II < III (b) IV < I < II < III (c) IV < I < III < II (d) IV < II < I < III.

4. The volume of a gas (a) depends on the temperature only (b) depends on pressure only (c) depends on the number of moles of the gas only (d) depends on the temperature, pressure and the number

of moles of the gas.

5. If 40.00 ml of 1.60 M HCl and 60.00 ml of 2.00 M NaOH are mixed, then the molar concentration of OH~ in the resulting solution will be (a) 0.056 M (b) 0.28 M (c) 0.56 M (d) 0.053 M.

6. For a solution in which both the components are volatile, Raoult's law states that (a) the relative lowering of vapour pressure is equal

to the mole fraction of the solute (b) the relative lowering of vapour pressure is

proportional to the amount of solute in solution (c) the vapour pressure of the solution is equal to the

mole fractions of the solvent

By: Er. Arvind Tripathi (6. Tech, IT-BHU) Momentum, Gorakhpur

(d) the saturated vapour pressure of each component in the mixture is equal to the product of the mole fraction of that component and the saturated vapour pressure of the component when it is pure.

7. The vapour pressure of a dilute aqueous solution of cane sugar (C l 2H2 2On) at 373 K is 740 mm Hg. Calculate the mole fraction of cane sugar, (a) 0.974 (b) 0.36 (c) 0.029 (d) 0.026

8. The addition of more nitrogen on the equilibrium • . • N J W + 3 H I W ^ 2 N H J W

in a vessel at constant volume is that it (a) shifts the equilibrium to the right (b) shifts the equilibrium to the left (c) increases the rate of formation of ammonia by 10

times (d) does not affect the equilibrium.

9. Which of the following salt will hydrolyze in water to form basic solutions? I. KCI II. CuS04 III. K,P0 4

(a) I, II and III (b) I and II only (c) I only (d) III only.

10. For the reaction, A + B -> C, the following results were obtained for kinetic runs at the same temperature.

Ml. mol L"1

|0 |o mol L_l

initial rate molLr's-1

0.20 0.10 0.20 0.40 0.10 0.80 0.40 0.20 0.80

What is the overall order of the reaction? (a) 0 (b) 1 (c) 2 (d) 3.

11. For an elementary process (a) the order and molecularity are identical (b) the order is twice of molecularity (c) the order is three times the molecularity (d) there is no relation between the order

molecularity. and

53 CHEMISTRY TODAY | JUNE 06

12. Given £°A|3+/AI = -1.66 V, E°Cu2+/Cu = +0.34 V. What will happen if we store aluminium nitrate in coper tumbler? (a) the tumbler gets coated with aluminium (b) the solution becomes blue (c) an alloy of copper and aluminium is formed (d). there is no reaction.

13. The length of the side of the unit cell for lithium is 352 pm, calculate its atomic radius. Lithium forms body-centred cubic crystal. (a) 304 pm (b) 76 pm (c) 608 pm (d) 152 pm.

14. If the radii of A+ and B~ are 95 pm and 181 pm respetively, then the coordination number of A+ will be (a) 12 (b) 8 (c) 6 (d) 4.

15. The nuclear fission reaction involves the (a) combination of two nuclei to form single nuclide (b) splitting of single nuclide into two nuclei (c) combination of alpha and beta particles

(d) combination of [H and ^H .

16. If 360 grams of glucose (C6H,206) is dissolved in 2000 grams of water (essentially 2 litres), what will be the freezing point of this solution? (a) - 6.58°C (b) - 3 . 7 2 ° C (c) -1 .86°C (d) - 0.51°C.

17. Sodium metal is obtained by (a) the electrolysis of concentrated aqueous NaCl (b) heating N a 2 0 with H2

(c) heating fused sodium chloride (d) electrolysis of fused sodium chloride.

18. Which of the following tetrahalide of lead is not known? (a) Pbl4 (b) PbBr4 (c) PbCl4 (d) PbF4.

19. In the reaction : NaOH + S -> X + H 2 0 + Na2S, X is (a) Na ,S0 3 (b) Na 2S0 4

(c) N a , S , 0 , (d) Na2S2.

20. The oxidation state of iron in its brown ring complex (test for nitrate ion) [Fe(H 2 0) 5 N0]S0 4 is (a) +1 (b) +2 (c) +3 (d) 0.

21. Among the fo l lowing , the lowest degree of paramagnetism per mole of the compound at 298 K will be shown by (a) MnS0 4 -4H 2 0 (b) FeS0 4 -6H,0 (c) CuS0 4 -5H 2 0 (d) NiS0 4 -6H,0.

22. In P4O10, each phosphorus atom is linked with .. oxygen atoms. (a) 2 (b) 3 (c) 4 (d) 5.

23. The correct IUPAC name of the compound

O CH3 II I

CH3 - CH - C - CH, - CH is

CH,

CH,

CH,

(a) 2,5-dimethylheptan-4-one (b) 3,6-dimethylheptan-4-one (c) 2-ethyl-5-methylhexan-3-one (d) 1 ,l-dimethyl-4-ethylpentan-3-one.

24. Which of the following Newman projection formula for 1,2-dichloroethane represents the staggered form?

25. The product X in the following reaction is

CH3CH CHCH, H , 0 ,

-»- X, HIO4 > X

(a) C H , C H - C H - C H , (b) CH, - COOH

OH OH

(c) CH,CHO

OH O I II

(d) CH,CH - C - CH,

26. The compound X in the following reaction is

(OO, HC CH

(ii) HT X

(a) glyoxal only (b) mixture of glyoxal and formic acid (c) butanone (d) ethanal only.

27. Chloretone is (a) CCljNO, (b) (CH3),C(OH)CCl3

54 CHEMISTRY TODAY | JUNE 06

(c) CH3C0CH2CHC12 (d) (CH3)2C = CC12

28. The compound X in the following reaction H,/Ni

CHj - CH = CH - CHO » X is

(a) CH3CH2CH2CHO (b) CH3CH — CH - CH2OH (c) CH3CH2CH2C'H2OH (d) CH, - CH — CH - CH3.

29. A—j-'l ,, —> a-hydroxy propionic acid. (//) dilute H 2S0 4

The structure of A is (a) CH3COOH (b) CH2 = CH - COOH (c) CH3CHO (d) CH3COCH3. 30. Identify the compound B in the following reaction.

(a )

(c )

( Q y so 3 H

< O ^ 0 H

NaN02/H2S04

0°C -+A steam

distilled

(b) H O 3 S - © - N H 2

(d)

31. When 10 ml of ethyl alcohol (density 0.7893 g/ml) is mixed with 20 ml of water (density 0.9971 g/ml) at 25°C, the final solution has a density of 0.9571 g/ml. The percentage change in total volume on mixing is (a) 3.1% (b) 2.4% (c) 1% (d) none of these.

32. Which is incorrect statements? (a) a solute will dissolve in water if hydration energy

is greater than lattice energy (b) if the anion is large compared to the cation, the

lattice energy will remain almost constant (c) solubility of 1IA hydroxide is in order

Be(OH)2 < Mg(OH)2 < Ca(OH)2 < Sr(OH), (d) all are correct.

33. Which of the following pairs of isomers and types of isomerism are correctly matched? 1. [Co(NH3)5(N02)]Cl2 and [Co(NH3)5(ONO)]Cl2

.... linkage 2. [Cu(NH3)4][PtCU] and [Pt(NH,)4][CuCl4]

.... coordination 3. [Pt(NH3)4CI,]Br2 and [Pt(NH3)4Br2]Cl2

.... ionisation Select the correct answer using the codes given below: (a) 2 and 3 (b) 1 ,2 and 3 (c) 1 and 3 (d) none of these.

34. 0.0012 mol of CrCl3-6H20 was passed through a cation exchange resin and acid coming out of it required 28.5 mL of 0.125 M NaOH. Hence complex is (a) [Cr(H20)5Cl]Cl2 • H 20 (b) [Cr(H20)4Cl2] • 2H 2 0 (c) [Cr(H20)6]Cl3 (d) [Cr(H20)3Cl3] • 3H20.

35. Which of the following statement about Millikan's oil drop experiment is true? (a) when the electrifc field is turned on, all the oil

drops move towards the positively charged plate (b) the charge on each oil drop is the electronic charge (c) in the absence of the electric field, the speed with

which the drop falls depends only upon the acceleration due to gravity

(d) some oil drops become positively charged and some become negatively charged after colliding with gaseous ion.

36. Total vapour pressure of mixture 1 mol A (P°4 = 150 torr) and 2 mol B ( = 240 torr) is 200 mm. In this case (a) there is positive deviation from Raoult's law (b) there is negative deviation from Raoult's law (c) there is no deviation from Raoult's law (d) molecular masses of A and B are also required.

37. If AG = AH- TAS and AG = AH + 7 Jcl(AG) dT

then variation of EMF of a cell E, with temperature T is given by

AH AG AS -AS (a )

AG nF (b) ~~nF

AS (c) — (d)

nF nF

38. Silver salt of a dibasic acid has 54% silver. Hence molecular weight of the organic compound is (a) 200 (b) 400 (c) 93 (d) 186.

39. ZnS does not crystallise in the NaCl structure. It is due to (a) the r+/r~ ratio is 0.402, too low to avoid anion-

anion contact in the NaCl structure (b) the r + / r ratio is 0.402, too high to avoid cation-

cation contact in the NaCl structure (c) both are correct (d) none is correct.

H,

C u - Z n

CO, H „ [CoH(CO),l 40. CH,CH = CH, —

J - 125° pressure

product This represents oxo method of alcohol synthesis. Product can be

55 CHEMISTRY TODAY | JUNE 06

(a) CH3CH,CH2CH2OH (b) CH3-CHCH2OH

CH,

(c) both are true (d) none is true.

41. End product of the following sequence of reactions is

C H = C H CH3MgBr> CO^H^O* > HgSOj/HiSOj >

o I!

(a) CH3-C-COOH (b) CHj(COOH)2

O II

o

(c) CH j - C - C H O " (d) H - C - C H 2 C O O H

42.

A and B are (ii) H , 0 +

(a) H O - ^ - T M O , ( C H 3 ) 2 N H

(b) H 0 - ^ O ) - O H ' ( C H 3 ) 2 N H

(c) HO - < 0 > - N O , CH,CH2NH2

(d) none of these.

4 3 ' ( 3 ) ~ M g B r + C H2 - C H 2 —

(a) ( Q ) - C H 2 O H ( b ) < g ) - C H 2 C H 2 O H

(c) ( 0 / ~ R*71" C H 2 n o n e o f t h e s e -V

44. Match the reactions taking place in a blast furnance with temperature-range of operations.

Reactions Temperature range A. Fe20 + CO -> C0 2 + FeO I. 1800 °C B. CO C + CO, II. 800°C C. FeO + CO Fe + C0 2 111. 500-600°C D. CaO + Si02 CaSi03 IV. 400°C

Choose the correct alternate. (a) A-1V, B-11I, C-II, D-I (b) A-IV, B-II, C-III, D-I (c) A-l, B-ll, C-1V, D-II1 (d) A-I, B-II, C-III, D-IV.

45. When KI (excess) is added to I. CuS04 II. HgCI, III. Pb(N03)2

(a) a white ppt. of Cul in I, an orange ppt. of Hgl2 in II and a yellow ppt. of Pbl, in III

(b) a white ppt. of Cul in I, an orange ppt. dissolving to Hgl4

2~ in II, and a yellow ppt. of Pbl, in III (c) a white ppt. of Cul, Hgl, and Pbl2 in each case (d) none is correct.

46. A B (oxide) + CO,, B + H 2 0 -> C, C + C 0 2 -> A (milky), C + NH4C1 — ^ D (gas) D + H 2 0 + C02 E ; E + NaCl F F - A * Na2C03 + C0 2 + H , 0

Name of the process is (a) Solvay (b) ammonia-soda (c) both are correct (d) none is correct.

47. CO, gas along with solid (K) is obtained when sodium salt (X) is heated. (AO is again obtained when CO, gas is passed into aqueous solution of (Y). X and Y are (a) Na2C03, Na 2 0 (b) Na,C03 , NaOH (c) NaHCOj, Na,C0 3 (d) Na,C03 , NaHCO,.

48. The statement is not true for the long form of the periodic table. (a) it reflects the sequence of filling the electrons in

the order of the sub-energy shells .s, p, d and / (b) it helps to predict the stable valency states of the

elements (c) it reflects trends in physical and chemical properties

of the elements (d) it helps to predict the relative ionicity of the bond

between any two elements.

49. CH3CH2CH2CH,CI

I

CH3CHCH2CI

CH,CH,CHCH, 3 V 3

ci 11

CH,

C H 3 - C - C ]

CH3 C H J

III IV Increasing tendency for SN1 and SN2 reaction is A. SN1 : I < III < II < IV B. S n2 : IV < II < III < I (a) A and B both are correct

56 CHEMISTRY TODAY | JUNE 06

(b) only A is correct (d) both incorrect.

(c) only B is correct

50. An organic compound of molecular formula C4H6

(A), forms precipitates with ammoniacal silver nitrate and ammoniacal cuprous chloride. A has an isomer B one mole of which reacts with one mol of Br2 to form l,4-dibromo-2-butene. A and B are

CH, - CH2 - C = CH and CH, = CH - CH = CH2

CH3 - C = C - CH3 and CH3 - CH = C = CH,

CH2 — CH C=CH 2 and '

CH

(a)

(b)

(O H2CV

(d)

" CH,

CH 3 - c == c - CH 3

H2C

/ C 1 K (d) C H 2 ^ | > H 2

^ C H ^

ANSWERS

1. (c) 2. (c) 3. (b) 4. (d) 5. (c) 6. (d) 7. (d) 8- (a) 9. (d) 10. (c) 11. (a) 12. (d) 13. (d) 14. (C) 15. (b) 16. (c) 17. (d) 18. (a) 19. (c) 20. (a) 21. (c) 22. (c) 23. (a) 24. (d) 25. (c) 26. (b) 27. (b) 28. (c) 29. (c) 30. (c) 31. (a) 32. (d) 33. (b) 34. (c) 35. (d) 36. (b) 37. (c) 38. (d) 39. (a) 40. (c) 41. (b) 42. (a) 43. (b) 44. (a) 45. (b) 46. (c) 47. (c) 48. (d) 49. (a) 50. (a)

UP-CPMT EXPLORER

10 Model Test

Papers +

Wt&2006

UTTAR PRADESH

E X P L O R E R 12 years

Solved Papers (1994-2005)

(with detailed solutions)

. .. 12 years Solved Papers

(1994-2005) (with detailed solutions)

the Examination* I SyHabus of U.R CPMY » :;

12 years Solved Papers

(1994-2005) (with detailed solutions)

J/m! Tip*from Tww -* - /- -- - ' 10 Model Test Papers -

12 years Solved Papers

(1994-2005) (with detailed solutions) '"WaM j Pr ice : Rs. 200 k

MTG-MCQ's Memory Contest

a I. The exams: National Chemistry 1L Olympiad / International Chemistry '>\ Olympiads/OrissaJEE/AIIMS/AFMC/

\ EAMCET / DCE/ Karnataka CET/

(UP SEE / Bihar CECE/Jharkhand CECE/Haryana CET/ Punjab PMT & PET / NDA /UGC - JRF/JIPMER/ Maharashtra CET/Raj. PMT & PET / MP PMT & PET/AMU. . . . 2. The requirements: Plenty of grey cells, photographic memory, neat handwriting, eagerness to do something different and of course, taking exams.

3. The task: Take the exam, come home and write down as many questions (with

all choices and their answers) you can remember, neatly on a paper with your name,

| your address, age, which exam you sat for, your photo and mail them to us.

4. The benefits: Plenty 1 Each complete question with answer will make you richer by Rs. 5*/-. More the questions, the merrier it will be. We will make you famous by publishing your name (photo if possible). Also you can derive

; psychological satisfaction from the fact | that your questions will benefit I thousands of readers.

5. and lastly the pit falls: Don't send incomplete questions or incorrect answers. Our panel of experts will crosscheck your questions. You have to send it within a month of giving the

particular exam. Mail to: The Editor, MTG 406, Taj Apartment, Ring Road,

New Delhi-29. Ph: 26194317 * Conditions apply

Payment wilt he made after the MCO V are published. Kindly note that each question should he complete with the answers. Payment will he made only for complete questions. Preference will he given to the reader sending the maximum complete and correct questions. Other conditions apply. The decision of the editor, MTCi shall he final and binding.

57 CHEMISTRY TODAY | JUNE 06

SOLVED PAPER

CBSE-PMT (Prelims) - 2006 1. The orientation of an atomic orbital is governed by (a) principal quantum number (b) azimuthal quantum number (c) spin quantum number (d) magnetic quantum number.

2. The number of unpaired electrons in a paramagnetic diatomic molecule of an element with atomic number 16 is (a) 1 (b) 2 (c) 3 (d) 4.

3. Which of the following is not a correct statement? (a) multiple bonds are always shorter than

corresponding single bonds (b) the electron-deficient molecules can act as Lewis

acids (c) the canonical structures have no real existence (d) every ABS molecule does in fact have square pyramid

structure.

4. Which of the following species has a linear shape? (a) 0 3 (b) N0 2" (C) S0 2 (d) N02

+ .

5. Which of the following is not isostructural with SiCl4? (a) NH4

+ (b) SC14 (c) S042" (d) P04

3".

6. Which of the following is the most basic oxide? (a) SeO, (b) Al203 (C) Sb203 (d) Bi203.

7. The correct order regarding the electronegativity of hybrid orbitals of carbon is (a) sp < sp2 < sp3 (b) sp > sp2 < sp3

(c) sp > sp2 > sp3 (d) sp < sp2 > sp3.

8. The correct order of the mobility of the alkali metal ions in aqueous solution is (a) Rb+ > K+ > Na+ > Li+

(b) Li+ > Na+ > K+ > Rb+

(c) Na+ > K+ > Rb+ > Li+

(d) K + > Rb+ > Na+ > Li+.

9. The electronegativity difference between N and F is greater than that between N and H yet the dipole

moment of NH3 (1.5 D) is larger than that of NF3

(0.2 D). This is because (a) in NH3 the atomic dipole and bond dipole are in

the opposite directions whereas in NF3 these are in the same direction

(b) in NH3 as well as in NF3 the atomic dipole and bond dipole are in the same direction

(c) in NH3 the atomic dipole and bond dipole are in the same direction whereas in NF3 these are in opposite directions

(d) in NH3 as well as in NF3 the atomic dipole and bond dipole are in opposite directions.

10. Which one of the following orders is not in accordance with the property stated against it? (a) F2 > Cl2 > Br2 > I2 : bond dissociation energy (b) F2 > Cl2 > Br2 > I2 : oxidising power (c) HI > HBr > HC1 > HF : acidic property in water (d) F2 > Cl2 > Br2 > I2 : electronegativity.

11. In which of the following molecules are all the bonds not equal? (a) NF3 (b) C1F3

(c) BF3 (d) A1F3.

12. The IUPAC name of CI

0 (a) 2,3-dimethylpentanoyl chloride (b) 3,4-dimethylpentanoyl chloride (c) 1-chloro-l-oxo-2,3-dimethylpentane (d) 2-ethyl-3-methylbutanoyl chloride.

13. The human body does not produce (a) enzymes (b) DNA (c) vitamins (d) hormones.

14. During the process of digestion, the proteins present in food materials are hydrolysed to amino acids. The two enzymes involved in the process

proteins e" z y m e ( ' 4 )> polypeptides enzyme (B) > amino acids,

are respectively

58 CHEMISTRY TODAY | JUNE 06

(a) invertase and zymase (b) amylase and maltase (c) diastase and lipase (d) pepsin and trypsin.

15. The general molecular formula, which represents the homologous series of alkanols is (a) C„H2„ + 2 0 (b) C„H2„02

(c) C„H2„0 (d) C„H 2 n + I 0.

16. The appearance of colour in solid alkali metal halides is generally due to (a) interstitial positions (b) F-centres (c) Schottky defect (d) Frenkel defect.

17. CsBr crystallises in a body centred cubic lattice. The unit cell length is 436.6 pm. Given that the atomic mass of Cs = 133 and that of Br = 80 amu and Avogadro number being 6.02 x 1023 mol-1, the density of CsBr is (a) 4.25 g/cm3

(c) 0.425 g/cm3 (b) 42.5 g/cm3

(d) 8.25 g/cm3.

18. Given : The mass of electron is 9.11 x 10-31 kg, Planck constant is 6.626 x 10"34 Js, the uncertainty involved in the measurement of velocity within a distance of 0.1 A is (a) 5.79 x 105 ms-' (b) 5.79 x 106 ms"1

(c) 5.79 x io7 ms"1 (d) 5.79 x 108 ms"1.

19. A plot of log(x/m) versus log p for the adsorption of a gas on a solid gives a straight line with slope equal to (a) log/: (b) -logK (c) n (d) 1 In.

20. A solution containing 10 g per dm3 of urea (molecular mass = 60 g mol"1) is isotonic with a 5% solution of a nonvolatile solute is (a) 200 g mol-1 (b) 250 g mol"1

(c) 300 g mol-' (d) 350 g mol-1.

21. 1.00 g of a non-electrolyte solute (molar mass 250 g mol-') was dissolved in 51.2 g of benzene. If the freezing point depression constant, Kf of benzene is 5.12 K kg mor 1 , the freezing point of benzene will be lowered by (a) 0.2 K (b) 0.4 K (c) 0.3 K (d) 0.5 K.

22. Identify the correct statement for change of Gibbs energy for a system (AGsystem) at constant temperature and pressure. (a) If AGsystem < 0, the process is not spontaneous.

(b) If AGsystem > 0, the process is spontaneous. (c) If AGsystem = 0, the system has attained equilibrium. (d) If AGsystem = 0, the system is still moving in a

particular direction.

23. Assume each reaction is carried out in an open container. For which reaction will AH = AE? (a) 2CO (g) + 0 2 —> 2C02

(b) H2 f e ) + Br 2 f e ) ->2HBr ( K )

(c) C w + 2H20 - » 2H2 + C 0 2

(d) P C I S W - * P C 1 3 W + C12W .

24. For the reaction: CH4 + 20 2 (g) C 0 2 + 2 H 2 0 m ,

AHr = -170 .8 kJ mol"1. Which of the following statements is not true? (a) The reaction is exothermic. (b) At equilibrium, the concentrations of C0 2 ^ and

H 2 0 (/) are not equal. (c) The equilibrium constant for the reaction is given

[co2] [CH 4 ] [0 2 ]"

(d) Addition of CH cause a shift to the right.

by

4 t e ) or 0 2 at equilibrium will

25. For the reaction, 2A + B 3C + D, which of the following does not express the reaction rate?

(a)

(c)

d[A] 2 dt

d[B] dt

(b) -

(d)

d[C] 3 dt

d[D] dt

26. Consider the reaction: N2 (jr) + 3H2 (x) —> 2NH, 4 N H 3 ]

The equality relationship between 7 — and

d[ H2] dt

(a)

(c) +

(d) +

4NH 3 J _ 4 H 2 ] dt dt

4 N H 3 ] _ 2C/[H2] dt 3 dt

4 N H 3 ] _ 3 J [ H 2 ]

(b)

dt

4 N H 3 ] 1 4 H 2 ] dt 3 dt

dt 2 dt

2 7 . £°F e2+/Fe = - 0 . 4 4 1 V a n d £°F e3+ / F e2+ = 0 . 7 7 1 V, t h e

standard EMF of the reaction Fe + 2Fe3+-> 3Fe2+ will be (a) 0.111 V (b) 0.330 V (c) 1.653 V (d) 1.212 V.

59 CHEMISTRY TODAY | JUNE 06

28. A hypothetical electrochemical cell is shown below. ©

A | A+ (.xM) || B+ (yM) | B The emf measured is + 0.20 V. The cell reaction is (a) A + B+ A+ + B (b) A+ + B A + B+

(c) A+ + e~ A; B+ + e~ B (d) the cell reaction cannot be predicted.

29. A solution of acetone in ethanol (a) obeys Raoult's law (b) shows a negative deviation from Raoult's law (c) shows a positive deviation from Raoult's law (d) behaves like a near ideal solution.

30. During osmosis, f low of water through a semipermeable membrane is (a) from solution having lower concentration only (b) from solution having higher concentration only (c) from both sides of semipermeable membrane with

equal flow rates (d) from both sides of semipermeable membrane with

unequal flow rates.

31. Which of the following pairs constitutes a buffer? (a) HC1 and KC1 (b) HNO, and NaNOz

(c) NaOH and NaCl (d) HN0 3 and NH4N03 .

32. The hydrogen ion concentration of a 10"8 M HC1 aqueous solution at 298 K (KW = 10~14) is (a) 1.0 x 10"8 M (b) 1.0 x 10"6 M (c) 1.0525 x lO"7 M (d) 9.525 x 10"8 M.

33. The enthalpy and entropy change for the reaction: Br2(J) + C l 2 t e ) - » 2 B r C I w

are 30 kJ mol"1 and 105 J K 1 mol"1 respectively. The temperature at which the reaction will be in equilibrium is (a) 300 K (b) 285.7 K (c) 273 K (d) 450 K.

34. A1203 can be converted to anhydrous A1C13 by heating (a) A1,03 with Cl2 gas (b) A1203 with HC1 gas (c) A1203 with NaCl in solid state (d) a mixture of A1203 and carbon in dry Cl2 gas.

35. In which of the following pairs are both the ions coloured in aqueous solution? (a) Ni2+, Cu+ (b) Ni2+, Ti3+

(c) Sc 3 \ Ti3+ (d) Sc3+, Co2+. (At. no. : Sc = 21, Ti = 22, Ni = 28, Cu = 29, Co = 27)

36. Copper sulphate dissolves in excess of KCN to give

(a) Cu(CN)2

( c ) [ C U ( C N ) 4 ] 3 -

(b) CuCN ( d ) [ C U ( C N ) 4 ] 2 ~

37. More number of oxidation states are exhibited by the actinoids than by the lanthanoids. The main reason for this is (a) more active nature of the actinoids (b) more energy difference between 5/and 6d orbitals

than that between 4 / and 5d orbitals (c) lesser energy difference between 5/and 6d orbitals

than that between 4/and 5d orbitals (d) greater metallic character of the lanthanoids than

that of the corresponding actinoids.

38. [Cr(H20)6]Cl3 (at. no. or Cr = 24) has a magnetic moment of 3.83 B.M. The correct distribution of 3d electrons in the Chromium of the complex is

(a) 3dxy}, 3dyz\ 3 d j

(b) 3 d u 2 _ y 2 ) \ 3 d / , 3dj

(c) 3dxy\

(d) 3 d x y \ 3dyz\ 3dx;

3 9 . [ C O ( N H 3 ) 4 ( N 0 2 ) 2 ] C 1 exhibits (a) linkage isomerism, geometrical isomerism and

optical isomerism (b) linkage isomerism, ionization isomerism and optical

isomerism (c) linkage isomerism, ionization isomerism and

geometrical isomerism (d) ionization isomerism, geometrical isomerism and

optical isomerism.

40. In a set of reactions propionic acid yielded a compound D.

CH3CH2COOH SOC1, ^ B-

N H , ^ c KOH

Br, > D

The structure of D would be (a) CH3CH,NH2 (b) CH3CH,CH,NH, (c) CH3CH,CONH, (d) CH3CH,NHCH3.

41. Which of the following is not chiral? (a) 2-hydroxypropanoic acid (b) 2-butanol (c) 2,3-dibromopentane (d) 3-bromopentane.

42. Ethylene oxide when treated with Grignard reagent yields (a) primary alcohol (b) secondary alcohol (c) tertiary alcohol (d) cyclopropyl alcohol.

6 0 CHEMISTRY TODAY JANUARY '06

43. Which of the following is more basic than aniline? (a) benzylamine (b) diphenylamine (c) triphenylamine (d) p-nitroaniline.

44. The enthalpy of hydrogenation of cyclohexene is -119.5 kJ mol"1. If resonance energy of benzene is -150.4 kJ mol"1, its enthalpy of hydrogenation would be (a) - 358.5 kJ mol"1 (b) - 508.9 kJ mol"1

(c) - 208.1 kJ mol-' (d) - 269.9 kJ mol"1.

45. Nucleophilic addition reaction will be most favoured in (a) CH,CHO

O

(b) CH, - CH2 - CH2C - CH, (c) (CH3)2C = O (d) CH3CH2CHO.

46. The major organic product in the reaction, CH3 - O - CH(CH3)2 + HI products is

(a) CH3I + (CH3)2CHOH (b) CH3OH + (CH3)2CHI (c) ICH2OCH(CH3)2 (d) CH3OC(CH3),

I

47. Self condensation of two moles of ethyl acetate in presence of sodium ethoxide yields (a) ethyl propionate (b) ethyl butyrate (c) acetoacetic ester (d) methyl acetoacetate.

48. A carbonyl compound reacts with hydrogen cyanide to form cyanohydrin which on hydrolysis forms a racemic mixture of a-hydroxy acid. The carbonyl compound is (a) formaldehyde (b) acetaldehyde (c) acetone (d) diethyl ketone.

49. >AvfNH(CH2)NHCO(CH2)4COt^ is a (a) homopolymer (b) copolymer (c) addition polymer (d) thermosetting polymer.

50. Which one of the following is a peptide hormone? (a) adrenaline (b) glucagon (c) testosterone (d) thyroxin.

SOLUTIONS

1. (d): Principal quantum number represents the name, size and energy of the shell to which the electron belongs. Azimuthal quantum number describes the spatial distribution of electron cloud and angular momentum.

Magnetic quantum number describes the orientation or distribution of electron cloud. Spin quantum number represents the direction of electron spin around its own axis.

2. (b) : Molecular orbital configuration of a diatomic molecule of an element with atomic number 16 is KK(c2.j)2 (cr*2.s)2 (02px)2 (n2p})2 (nip,)1 (n*2p,,)' (n*2p.)1

Number of unpaired electrons = 2.

3. (d) : For AB5 molecules, there are three possible geometries i.e. planar pentagonal, square pyramidal and trigonal bipyramidal.

N

o 115° O

planar pentagonal square pyramidal trigonal bipyramidal

Out of these three geometries, it is only trigonal pyramidal shape in which bond pair-bond pair repulsions are minimum and hence this geometry is the most probable geometry of AB5 molecule.

4. (d ) : N 0 2 " : Due to sp2

hybridisation of N-atom and the presence of one lone pair on it, N02" has angular shape.

0 3 : ^ - X * V-shaped O 116.8° o

S0 2 : Due to the presence of one lone pair of electros in one of the three sp2-hybrid orbitals and sp2

hybridisation of S or S+ atom, S0 2

molecule has angular (V-shaped) structure. N0 2

+ : Due to sp hybridisation of N+, N0 2+ ion has

linear shape.

: 0 = N r = 0 :

5. (b) : SiCl4, NH4+, S04

2" and P043" ions are the

examples of molecules/ions which are of ABT type and have tetrahedral structure. SC14 is ,-lB4(lone pair) types species. Although the arrangement of five sp3d hybrid orbitals in space is trigonal bipyramidal, due to the presence of one lone pair of electron in the basal hybrid orbital, the shape of AB4 (lone pair) species gets distorted and becomes distorted tetrahedral or see-saw.

o 119.5° 0

61 CHEMISTRY TODAY | DECEMBER '05 47

6. (d) : Se02 —> acidic oxide A1203 —> amphoteric Sb203 —> amphoteric Bi203 —» basic oxide.

7. (c): Electronegativity of carbon atom is not fixed. It varies with the state of hybridisation. Electronegativity of carbon increases as the j-character of the hybrid orbital increases.

C (sp) > C (sp2) > C (sp1) 8. (a) : The alkali metal ions exist as hydrated ions M+(H20)J. in the aqueous solution. The degree of hydration, however, decreases with the ionic size as we move from Li+ to Cs+. In other words, Li+ ion is most highly hydrated. e.g. [Li(H20)6]+. Since the mobility of ion is inversely proportional to the size of their hydrated ions, therefore, amongst the alkali metal ions, lithium has the lowest ionic mobility.

Rb+ (aq) > K+ (aq) > Na+ (aq) > Li+ (aq) 9. (c) : The dipole moment of NF3 is 0.24 D and of NH3 is 1.48 D. The difference is due to fact that while the dipole moment due to N - F bonds in NF3 are in opposite direction to the direction of the dipole moment of the lone pair on N atom which partly cancel out, the dipole moment of N - H bonds in NH3 are in the same direction of the dipole moment of the lone pair on N atom which adds up as shown below.

t ; x;

NF, (moments subtract)

H H

NH3 (moments add)

10. (a) : X-X bond F - F CI - CI Br - Br I - I Bond dissociation 38 57 45.5 35.6 energy (Kcal/mol) The lower value of bond dissociation energy of fluorine is due to the high inter-electronic repulsions between non-bonding electrons in the 2p-orbitals of fluorine. As a result F - F bond is weaker in comparison to CI - CI and Br - Br bonds.

11. (b) : The basal CI - F (CI - FA) bond length is equal to L 7° " 1.60 A while each of the two axial CI • CI - F (CI - Fa) bond lengths is 1.70 A | equal to 1.70 A . F

1.60 A,

12. (a) :

4 [2

o

.CI

2,3-dimethylpentanoyl chloride

13. (c) : Certain organic substances required for regulating some of the body processes and preventing certain diseases are called vitamins, which cannot be synthesised by the organism.

pepsin

14. (d) : Proteins p r o t e a s e s* polypeptides trypsin

cliemotrypsin* 3 1 1 1 1 1 1 0 a C ' d s

15. (a) : All alcohols follow the general formula C„H2„ + 2 0 .

CH3OH [CH2 + 2 0] ; C2H5OH [C2H(2 x 2) + 2 0] n = 1, n — 2

16. (b): F-centres are the sites where anions are missing and instead electrons are present. They are responsible for colours.

Z x M 17. (a) : Density of CsBr

1x213 in i — r r - = 4.25 g/cmJ

(436.6 x 10 ) x 6.023 x 10 18. (b) : Ax-mAv = h/4n

a i m-io A 1 1 . 3 1 \ 6.626x10 0.1x10 x9.11x10 x Av — -34

4x3.143

Av 6.626x10 -34

0.1 xlO -10 x9.1 lx 10~31 x 4x3.143 = 5.79 x 106 ms-1.

19. (d ) log

slope = — n

log K (intercept)

O log P--X

This is according to Freundlich adsorption isotherm. 20. (c) : For isotonic solution, osmotic pressure of urea = osmotic pressure of non-volatile solute

10 _ 5 6tbdO(W= wxToO => - = 300 gmol- ' .

,„ 1 0 0 0 x / C / x w n c A 1000x5.12x1 21. (b): m = — — — — — or, 250 = -

W x A7" 51,2x A r 50

-10 CHEMISTRY TODAY | FEBRUARY '06 127

1000x5.12x1 • AT= — — = 0.4 K.

51.2x250 22. (c) : The criteria for spontaneity of a process in terms of AG is as follows: (a) If AG is negative, the process is spontaneous. (b) If AG is positive, the process does not occur in the forward direction. It may occur in the backward diection. (c) If AG is zero, the system is in equilibrium. The process does not occur. 23. (b) : AH = AE + AnRT For H. 2 Or) + Br2 2HBr An = 2 - (1 + 1) = 0. 24. (c) : CH4 + 20 :

Kn=-

4 Or)

Pco7

y2 fe)

Or) AH = AE.

± C0 2 to + 2H20

PCH4 • Po2

25. (b) : 2A + B 3C + D

ratc _ ~d[A\ _ d[B]_d[C]_d[D] 2 dt dt 3dt dt

Negative sign shows the decrease in concentration. 26. (c) : N2 + 3H2 - » 2NH3 w

- 4 N , ] _ 4 H 2 ] _ 4 N H 3 ] Rate = -

Hence

dt 4 N H 3 ] _

dt

3dt 2 d[H2] 3 dt

2 dt

27. (d) : Fe2+ + 2<r Fe ; E° = -0.441 V ... (i) Fe3+ + cr Fe2+ ; E° = 0.771 V ... (ii) Fe + 2Fe3+ 3Fe2+ ; E° = ?

To get the above equation, (ii) * 2 - ( i ) 2Fe3+ + 2e~ -> 2Fe2+ ; E° = 0.771 V —Fe2+ ± 2e~ - Fe ; E° = -0.441 V 2Fe3+ + Fe 2Fe2+ ; E° = 1.212 V

28. (a) : From the given expression: At anode : A —» A+ + e At cathode : B* + e -» B Overall reaction is : A + B+ —> A+ + B. 29. (c) : Both the components escape easily showing higher vapour pressure than the expected value. 30. (d) : Osmosis is the phenomenon of flow of pure solvent from the solvent to the solution or from a less concentrated solution to a more concentrated solution through a semipermeable membrane. Common semipermeable membranes are permeable to certain solute particles also. Infact, there is no perfect semipermeable membrane. Therefore we can say that flow of water through a semipermeable membrane takes

place both sides with unequal rates. 31. (b) HN02 and NaNO, are examples of acidic buffer. 32. (c) : IO"8 M HCl = IO"8 M H+. Also from water, [H+] = IO"7. Total [H+] = IO"7 + 0.10 x IO"7 = 1.1 x IO'7. 33. (b) : Br2 (/) + Cl2 2BrCl (s)

AH = 30 kJ mol-1, AS = 105 JK"1 mol-'

AS = — i.e. 105 = —xlOOO

r = 3 0 x l M = 2 8 5 . 7 K . 105

34. (d) : A1203 + 3C + 3CI2 1000°C > 2A1C13 + 3CO

vapours

cool

solid anhydrous aluminium chloride 35. (b) : Sc [Ar] 3d 4s2, Sc3+ [Ar] Ti -> [Ar] 3d2 4s2, Ti3+ [Ar] 3d Ni [Ar] 3d 4s2, Ni2+ [Ar] 3d Cu [Ar] 3d'0 4s1, Cu+ [Ar] 3 d w

Co [Ar] 3d 4s2, Co2+ -> [Ar] 3d1

Ti3+, Ni2+ and Co2+ are coloured due to presence of unpaired electrons. 36. (c) : First cupric cyanide is formed which decomposes to give cuprous cyanide and cyanogen gas. Cuprous cyanide dissolves in excess of potassium cyanide to form a complex, potassium cyanide [K3CU(CN)4].

[CUS04 + 2KCN CU(CN)2 + K2S04] x 2 2Cu(CN)2 CU2(CN)2 + (CN), CU2(CN)2 + 6KCN -» 2K3CU(CN)4

2CUS04 + 10KCN 2K3CU(CN)4 + 2K2S04 + (CN), 37. (c) : The 5/"-orbitals extend into space beyond the 6s and 6p-orbitals and participate in bonding. This is in direct contrast to the lanthanides where the 4/-orbitals are buried deep inside in the atom, totally shielded by outer orbitals and thus unable to take part in bonding.

38. (d) : Magnetic moment = + 2)

3.83 = + or, (3.83)2 = n(n + 2) or, 14.6689 = n2 + 2n On solving the equation, n = 3. Cr3+ -> [Ar] 3d

d„. d d 2

50 -10 CHEMISTRY TODAY | FEBRUARY '06 128

39. (c) : Ionization isomerism arises when the coordination compounds give different ions in solution.

[ C O ( N H 3 ) 4 ( N 0 2 ) ] C 1 ^ [ C o ( N H 3 ) 4 ( N 0 2 ) 2 } + + CL~

Linkage isomerism occurs in complex compounds which contain ambidentate ligands like N02", SCN~, CN", S 2 0 3

2 ' a n d C O .

[ C O ( N H 3 ) 4 ( N 0 2 ) 2 ] C 1 a n d [ C O ( N H 3 ) 4 ( O N O ) 2 ] C 1

are linkage isomers as N0 2" is linked through N or through O. Octahedral complexes of the type Ma462 exhibit, geometrical isomerism.

NH3

NH,

NH,

cis-

40. (a) : CH3CH2COOH NH3

SOCI,

> CH3CH2CONH2-

Br

trans-

->• CH3CH2C0C1 KOH Br,

•> CH3CH2NH2

41. (d) : H3C - CH2 - CH - CH, - CH,

Due to absence of asymmetric carbon atom. O •

/ \ 42. (a) : H2C—CH2 + CH3MgBr *•

CH3CH2CH2OMgBr H CH3CH2CH2OH primary alcohol

43. (a) : Any group which when present on benzene ring has electron withdrawing ( - N02 , - CN, - SO,H, - COOH, - CI, - C6H5 etc.) decreases basicity of aniline, e.g. aniline is more basic than nitroaniline and diphenyl amine. While a group which has electron repelling effect ( - NH2, - OR, - R etc.) increases basicity of aniline. Thus, benzylamine is more basic than aniline.

44. (c) : + H,

AH = -119.5 kJ moL

+ 3H,

Enthalpy of hydrogenation of benzene

= 3AH - resonance energy = 3 x (-119.5) - (-150.4) = -358.5 + 150.4 = -208.1 kJ moL1.

45. ( a ) : The reactivity of the carbonyl group towards the addition reactions depends upon the magnitude of the positive charge on the carbonyl carbon atom. Hence aryl substituent that increases the positive charge on the carbonyl carbon must increase its reactivity towards addition reactions. The introduction of negative group (-1 effect) increases the reactivity while introduction of alkyl group (+1 effect) decreases the reactivity.

H CH, CH,

/ ; c = o >

H

\ C — O >

H CH, / c = o

+I-effect and steric hindrance increases

46. (a) : With cold HI, a mixture of alkyl iodide and alcohol are formed. In the case of mixed ethers, the halogen atom attaches to a smaller and less complex alkyl group.

CH3OCH(CH3)2 + HI —> CH3I + (CH3)2CHOH

47. (c): Ethyl acetate undergoes Claisen condensation in presence of sodium ethoxide involving a-hydrogen atom in which two molecules of ethyl acetate combine together to form acetoacetic ester.

CH3COOC,H5 + CH3COOC,H5 C : H ' ° N a »

CH3COCH,COOC,H5 + C,H5OH acetoacetic ester

48. (b) : CH3CHO + HCN II I

-»- CH,-C—OH I CN

II 11,0

CH3-C—OH I COOH

lactic acid 49. (b ) : ^NH(CH,)NHCO(CH,)4COiwv is formed by the condensation of adipic acid and hexamethylene diamine. It is a copolymer (a polymer made from more than one type of monomer molecules is referred to as copolymer).

50. (b) : Glucagon is a single chain peptide of 29 amino acids, synthesised by the A cells in the Islets of Langerhans of the pancreas.

65 CHEMISTRY TODAY | DECEMBER '05 47

U .

(For Q.lWo. 1 to 60) Only one option is correct and there will be negative marking in these questions.

PASSAGE 1 : Read the following passage and answer the questions numbered 1 to 5. When aluminium salts are added to water, Al3+ ions are immediately attracted to the negative end of polar water molecules. They form hexaaquaaluminium (III) ions, [AI(H20)6]3+. This is often written simply as Al3+ (aq). However the electric field associated with small, highly charged Al3+ ion is so intense that it draws electrons in the O - H bonds of water towards itself. This enables the water molecules to become donors. In aqueous solution, free water molecules act as bases and the following equilibrium is established. [A1(H20)6]3++ H 20 [A1(0H)(H20)5]2+ + H3O+

Thus, solutions of Al3+ salts are acidic, in fact as acidic as vinegar. When a base stronger than H20, e.g. S2~ is added to aqueous aluminium salts further H+ ions are removed from [A1(H20)6]3+ and insoluble aluminium hydroxide precipitates. 2[A1(H20)6]3+ + 3S2~ (at)

2[A1(0H)3(H20)3] w + 3H2S More stronger base can remove 4H+ ions as follows: [A1(H20)6]3+ + 40H- — [AI(OH)4(H2O)]-w + 4H2O(0

1. Which of [A1(H20)6]3+, H,S or H,0 is the strongest acid? (a) [A1(H20)6]3+ (b) H2S (c) H 20 (d) all are same.

2. A base which will behave just like S2~ (a) C03

2- (b) CH3OH (c) NH4 (d) NH 2 -

3. Another ion that would behave similar to Al3+

in forming an acidic solution is (a) Kf (aq) (b) Ba2+ (aq) (c) Be2+ (aq) (d) Tl+ (aq).

(HI)

4. [A1(0H)3(H20)3] Al(OH)3 (S) is

(.v) more simply written as

(a) acidic (c) neutral

(b) basic (d) amphoteric.

5. Which gas would get absorbed when passed into a solution of Al3+ (aq)? (a) (c)

NH 3

CO (b) NO (d) 0 2 .

PASSAGE 2 : Read the following passage and answer the questions numbered 1 to 5. Structural isomers have different covalent linkage of atoms. Stereoisomers are compounds that have same sequence of covalent bonds but differ in the relative dispositions of their atoms in space. Geometrical and optical isomers are the two important types of configurational isomers. The compound with double bonds or ring structure have restricted rotation, so exist in two geometrical forms. The double bonds in larger rings (ring size 10 carbon or large) can also cause geometrical isomerism. The optical isomers rotate the plane of plane-polarised light. A sp3-hybridised carbon atom bearing four different types of substituents is called an asymmetric centre or chirai centre. A chirai object or molecule cannot be superimposed on its mirror image. Stereoisomers that are mirror images of each other are called enantiomers. The stereoisomers that are not mirror images of each other are called diastereomers. Diastereomers have different physical properties. A racemic mixture is optically inactive and contains equal amounts of both the enantiomers. Resolution refers to method of separating a racemic mixture into two pure enantiomers. A meso compound is an optically inactive stereoisomer, which is achiral due to the presence of an internal plane of symmetry or centre of symmetry within the molecule.

6. The pair showing identical species is

Contributed by Deptt. of Chemistry, Resonance, Kota (Rajasthan)

-10 CHEMISTRY TODAY | FEBRUARY '06 06

(a) and -""V'

Ci r

" ' / / H

CI Me Br

(b) •OH D -

and -Br H"

•Et -OH

Et

' V (c)

(d)

HOOC

COOH

OH

and

Br

f CI

OH

CH,

Me

HOOC H

Br

COOH OH

HOH

and

II3C-CI

7. Observe the following reaction. CH,

"l H

NH,

HOOC .CH, \ / 3

— ( R ' R ) + (R'S)

CI H (R')-l-plienylcthylamine (R + S)-2-cliloropropanoic acid

separation by recrystallisation

u 7 "

-> (R'R) hydro'ySS (R') + (R)

(R'S)

(3)

hydrolysis > (RO + (S)

Which statement is not correct about the above observation. (a) the product mixture of step-1 is optically active (b) the products R'R and R'S have identical structural

formula (c) R'R is nonsuperimposable on R'S (d) R'R and R'S have same solubility in water.

8. Which statement is correct about the following pair of compounds?

(I) (II) (a) both give same product with Br,/CCl4 solution (b) I and II cannot be separated by fractional distillation (c) I and II have different linkage of atoms (d) I and II after catalytic hydrogenation, followed by

photochemical bromination and then treatment with alcoholic KOH followed by reaction with HBr/R,0, give same major product.

9. The number of chiral centres present in the following compound is

CH,OH

H A O H

HO OH H

H HO

O D (+) sucrose

OH H (a) 7 (c) 9

(b) 8 (d) 10.

10. The following two compounds are

I II (a) identical (b) conformational isomers (c) geometrical isomers (d) structural isomers.

11. CoCI42" („v) is blue in colour while [Co(H20)c,]2+

(U(/)

is pink. The colour of reaction mixture Co(H20)6

2+(at/) + 4Cl-(a9) — CoCl4

2- (uq) + 6H,0 (/)

is blue at room temperature while it is pink when cooled hence (a) reaction is exothermic (b) reaction is endothermic (c) equilibrium will shift in forward direction on adding

water to reaction mixture (d) equilibrium will not shift on adding water to

equilibrium mixture.

12. Which one of the following best explains why the lattice enthalpy of magnesium chloride is much larger than that of lithium chloride? (a) magnesium has greater electronegativity than lithium (b) magnesium ions have greater polarising power than

lithium ions

-10 CHEMISTRY TODAY | FEBRUARY '06 7

(c) magnesium ions have greater radius than lithium ions

(d) magnesium ions have greater charge than lithium ions.

13. The percentage of copper in a copper (II) salt can be determined by using a thiosulphate titration. 0.305 g of a copper (II) salt was dissolved in water and added to an excess of potassium iodide solution liberating iodine according to the following equation.

2Cu2%„;) + 41- (aq) — 2CuI (s) + I2(a<;)

The iodine liberated required 24.5 cm3 of a 0.100 mole dm"3 solution of sodium thiosulphate.

2S203~ („(/) + 12 —> 21 (I(I/) • S4Os2 (aq)

The percentage of copper, by mass in the copper (II) salt is [Atomic mass of copper = 63.5] (a) 64.2 (b) 51.0 (c) 48.4 (d) 25.5

14. Consider the reactions shown below.

Cr04=- (aq) H 2 SQ 4 (ag)

AgN0 3 (aq) V X

silver chromate (VI)

> C r 2 0 72 - (aq)

FeS0 4 (aq)IH+ (aq)

C r 3 + (aq)

Zn (J)/H+ (aq) V C r 2 + (aq)

Which of the following statements is false? [Atomic mass of zinc = 65.4] (a) silver chromate (VI) has the formula Ag2Cr04

(b) the minimum mass of zinc required to reduce 0:100 mole of Cr3+ to Cr2+ is 6.54 gm

(c) the conversion of Cr042- into Cr207

2~ is not a redox reaction

(d) the equation, Cr2Ov2- + I4H+ + 6Fe2+

6Fe3+ + 2Cr3+ + 7H 2 0 correctly describes the reduction of Cr207

2 - by acidified FeS04.

15. A complex cation is formed by Pt (in same oxidation state) with ligands (in proper number so that coordination number of Pt becomes six). Which of the following can be its correct IUPAC name? (a) d iamminee thy lened iamined i th iocyana to -S-

piatinum (II)* ion (b) d iamminee thy lened iamined i th iocyana to -S-

platinate (IV) ion (c) d iamminee thy lened iamined i th iocyana to -S-

platinum (IV) ion

(d) diamminebis (ethylenediamine) dithiocyanate-S-platinum (IV) ion.

16. Zinc oxide is white powder at room temperature but becomes yellow in colour on heating, it is due to (a) Schottky defects produced in zinc oxide crystals

due to heating (b) it loses oxygen at high temperatures and some

electrons are trapped in anionic vacancies causing yellow colour to crystals due to excitations

(c) it just glows at high temperature just like other substances which burn with yellow flame

(d) due to heating zinc ions and oxide ions gets displaced from their locations and due to their movement yellow colour to crystals is obtained.

17. The standard reduction potentials of Cu2+/Cu and Cu2+/Cu+ are 0.337 V and 0.153 V respectively. The standard electrode potential of Cu+/Cu half cell is (a) 0.184 V (b) 0.827 V (c) 0.521 V (d) 0.490 V.

18. 100 mL of 0.02 M benzoic acid (pKa = 4.2) is titrated using 0.02 M NaOH. pH after 50 mL and 100 mL of NaOH have been added are (a) 3.50, 7 (b) 4.2, 7 (c) 4.2, 8.1 (d) 4.2, 8.25

19. 0.1 mole of which of the following when added to 1 L water, the aqueous solution obtained will have the lowest freezing point? (a) C6H5NH3C1 (b) K3[Fe(CN)6] (c) K4[Fe(CN)6] (d) AI(N03)3.

20. At 27°C the ratio of rms velocities of ozone to oxygen is (a) 7575 (b) V4/3 (C) V575 (d) 0.25

21. A gaseous mixture of three gases A, B and C has pressure of 10 atm. The total number of moles of all the gases is 10. If the partial pressures of A and B are 3.0 and 1.0 atm respectively and C has molecular weight of 2.0. What is the weight of C. in gms present in the mixture? (a) 6 (b) 8 (c) 12 (d) 3.

22. If 1023 gas molecules each having a mass of 10-25 kg, placed in a 1 L container, moving with rms speed of 105 cm/sec then the total kinetic energy of

-10 CHEMISTRY TODAY | FEBRUARY '06 132

gaseous molecules and pressure exerted by molecules, respectively are (a) 10 kJ, 3.33 x 10° Pa (b) 5 kJ, 3.33 x 10b Pa (c) 10 kJ, 3.37 x 107 Pa (d) 5 kJ, 3.33 x 107 Pa.

23. In hexagonal close packing of sphere in three dimensions (a) in one unit cell there are 12 octahedral voids and

all are completely inside the unit cell (b) in one unit cell there are six octahedral voids and

all are completely inside the unit cell (c) in one unit cell there are six octahedral void and of

which three are completely inside the unit cell and other three are from contributions of octahedral voids which are partially inside the unit cell

(d) in one unit cell there are 12 tetrahedral voids, all are completely inside the unit cell.

24. Which of the following is wrong? (a) bond order of 0 2 is greater than bond order of 02" (b) bond order of 0 2 is less than bond order of 02~ (c) bond order of N2 ' is less than that of N2

(d) 022- is diamagnetic while 02~ is paramagnetic.

25. How many of di, tri, tetra, penta and hexa chloroderivatives of benzene will have dipole moment identical to that of chlorobenzene? (a) 4 (b) 5 (c) 6 (d) 7.

26. Identify incorrect order or bond angles. (a) C120 > F20 and F 2 0 < H 2 0

(b) Asl-j > AsBr, > ASC13

(c) NO,+ > NO, (d) H A B H , , < H , B H , where H , is the terminal hydrogen

of B 2 H ( , and H ; , is the bridging hydrogen of B 2 H 6 .

27. A white crystalline solid is treated with hot concentrated H2S04 whereupon a gaseous mixture is released. The mixture is cooled during which liquid drops from which are capable of turning anhydrous CuS04

blue. The remaining dry gas exhibits a 50% volume contraction on passing through I205 (s). How many moles of acidified KMn04 would be required to oxidise, 1 mole of the white solid in aqueous solution? (a) 0.4 mol (b) 0.2 mol (c) 0.66 mol (d) 2 mol.

28. Two types of carbon-carbon covalent bond lengths are present in

(a) diamond (c) C60

(b) graphite (d) benzene.

29. Consider the structures of the following two molecules.

X : F2C = C = CF, Y : F2B - C = C - BF2

In which of these two it is impossible for all the four F atoms to lie in the same plane? (a) * (b) )' (c) both (d) none of these.

30. The HCH bond angle in H2C = O will be (a) 120° (b) more than 120° (c) less than 120° (d) nothing can be said.

31. Assuming C - C bond distance equal to that found in benzene ( 1 . 3 7 A) , the length of the following aromatic molecule will be nearly (a) 9.69 A (b) 11.2 A (c) 12.32 A (d) 15 A. 32. A diatomic molecule has a dipole moment of 1.2 D. If its bond distance is equal to 1.0 A then the fraction of an electronic charge on each atom is (a) 25% (b) 37% (c) 52% (d) 42%.

33. At 300 K. and 1.0 atm pressure, the density of gaseous HF is found to 3.1 g/L (atomic mass H = 1, F = 19). This means (a) HF gas consists of HF, H2 and F2 molecules (b) HF gas consists of clusters of HF molecules such

that each cluster consists of 4 HF molecules bonded through H-bonds

(c) a cluster consists of 6 HF molecules bonded with H-bonds

(d) a cluster consists of 2 HF molecules which are covalently bonded with each other.

34. Consider the following sets of H-bonds. I

P : - O - H N -I I

Q : - O - H O -

I I R : - N - H N -

I I I

S : - N - H O -

The correct order of H-bond strengths is

- 1 0 CHEMISTRY TODAY | FEBRUARY '06 10

(a) Q > P > S > R (c) R > S > P > Q

(b) R > Q > S > P (d) P > Q > R > S.

35. Which of the fol lowing carbides will give CH3 - C = C - H on reaction with water? (a) Be2C (b) SrC2

(c) Mg2C3 (d) none of these.

36. Which of the following gases exhibits green house effect? (a) Cl2 (b) C102

(c) NH, (d) C0 2 .

37. Which is correct about the cyclic silicate [Si60,g]"~? (a) the value of n is 12 (b) each Si atom is bonded with three oxygen atoms (c) each oxygen atom is bonded with two Si atoms (d) all of these.

38. In which of the following compounds B atoms are sp2 and sp3 hybridisation states? (a) borax (b) diborane (c) borazole (d) all of these.

39. Which is correct about D 2 0? (a) its boiling point is higher than that of H 2 0 (/)

(b) O - D O bond is stronger than O - H O bond

(c) D 2 0 (s) sinks in H 2 0 (f)

(d) all of these.

40. Which of the following is ionic solid? (a) XeF6 (.v) (b) PBr5(,, (c) CaC2 w (d) all of these.

41. Which of the following is V-shaped? (a) S3

2" (b) I3" (c) N3~ (d) none of these.

42. Tailing of mercury is a test for (a) H 2 0 2 (b) 0 3

(c) Na2S203-5H20 (d) H2S.

43.- Which of the following reactions is likely to be most exothermic? (a) C w + 2F2 0 0 - » C F 4 t e )

(b) C (v) + 2C12 q,) —> CC14 ((r)

(c) C M + 2 B r 2 W - > C B r 4 w

(d) C w + 2 I 2 0 ; ) ^ C I 4 f e ) .

44. According to molecular orbital theory which of the following is correct?

(a) LUMO level for C2 molecules is a a2p orbital (b) in C2 molecule both the bonds are n bonds (c) in C2

2- ion there is one a and two n bonds (d) all of these.

45. Which of the following names is impossible? (a) potassium tetrafluorooxochromate (VI) (b) barium tetrafluorobromate (III) (c) dichlorobis (urea) copper (II) (d) all are impossible.

46. The rate of esterification of CH3COOH is fastest with

OH OH

(a)

OH Me

( c Q -

47. The major product (ester) of the following reaction is

O H2so4 M e V * N * A Me

C - C - O H + H O - C C ^ I I ' Et HN

D H (a) a single stereoisomer (optically active) (b) a mixture of diastereomers (both optically active) (c) a racemic mixture (optically inactive) (d) a mixture of four stereoisomers (two racemic

mixture). The following questions consist of two statements one labelled assertion (A) and the another labelled reason (R). Select the correct answers to these questions from the codes given below: (a) both A and R are true andR is the correct explanation

of A (b) both A and R are true but R is not the correct

explanation of A (c) A is true but R is false (d) A is false but R is true.

48. Assertion : 3-chlorocyclopropene is solvolysed in methanol at much higher rate than 5-chlorocyclopenta-1,3-diene. Reason : The intermediate carbocation of 5-chloro-cyclopenta-1,3-diene is more stable than the carbocation formed by 3-chlorocyclopropene.

-10 CHEMISTRY TODAY | FEBRUARY '06 1 1

49. The correct stability order of following species is

(w) (x) / V / V

(y) (a x > y > w > z (c) x > w > z > y

(z) (b) y > x > w > z (d) z > x > y > w.

50. The major monochlorination product of the following compound is

Me

o , n - < Q > - N - C - < Q > - N H 2

o Me

(a) NH,

CI O Me

(b) 0 2 N H f \ - N - C - f \ N H ,

CI O Me CI

(c) V N - C - / / V I w

NH,

O Me I

(d) 0 ,N-< / y—N - C —U \ NH,

O CI

51. The major product of the following reaction sequence will be

( T \ _ Q H PliCOCI/Pyridine > CH3CQC1/A1C13 ^ \ = / ~ (1) (2)

Zn-Hg/HCl/A ^ B r 2 / F e >

(3) (4)

(a) C H 3 C H 2 H Q - 0 - C ^ >

O Br

(b) H 3 C - C H 2 - O - 0 - C - ^

Br o

(c) Br - ( / \ y - 0 - C - V V

O

C H , - C H ,

(d) C H 3 - C H .

Br

52. The dehydration of ethyl alcohol with sulphuric acid at 140°C produces diethyl ether, but ethylene is produced at higher temperature. Assume that ethylene is produced from the ether. Which statement is incorrect regarding mechanism of this reaction? (a) it will be a (3-elimination reaction (b) ethyl alcohol is the leaving group (c) the protonated ether undergoes 1,2-eIimination

reaction (d) the energy of activation of ether formation

(SN reaction) is higher than energy of activation of alkene formation (elimination reaction).

53. The major product of the following reaction is Br

CN C2HSCT C,H,CR H 2 C <

(a)

CN (1)

COOH

(2) (3) H3O+, A

(C) ^ N ^ C O O H

(b)

(d)

(4)

c o X C 0 o

(5)'

o

\

54. The reagents of following reaction sequence are respectively

P h - = M (1)

JL (2)

> Ph - = - v - © O M

(4) '

NH 2

P h ^ ^ y

(s)

,C — N

(5)

(a) NaNH, ; CH,CHO ; TsCl ; NaCN ; LiAlH4

(b) Na/Iiq. NH3 ; ; TsCl ; KCN ; LiAlH4

(c) Li/NH3 ; \ Q / ; TsCl ; NaCN ; H,/Pd

(d) KNH, ; CH, = C = O ; TsCl ; KCN ; H,/Ni.

-10 CHEMISTRY TODAY | FEBRUARY '06 12

55. Which of the following is/are the proper sequence of reagents for the conversion?

,OH > O:

Ph

OH OH

O II

(a)

( b )

(c)

PBr3 M g / E t , 0 P h - C - H H , S 0 4 0 s 0 4

^ ^ > ~A > „ n > A H , 0

PCI3 ^ P h M g B r ^ Br2 ^ a l c , KOH PhCQ3H >

H 30+ H 3O

+

o CH 3 MgBr P h - C - H H 2 S 0 4 0 s 0 4 alkaline

h3O+

( d ) PhMgBr ^ H 2 SQ 4 ^ cold alkaline ^

H 3 O T A K M n 0 4 solution

56. In the following reaction the major product is

MeOH, H®v

M e O N / \

(a) T J (b)

o O' MeO O

OMe

(c) (d) C. " OMe

0 / / X H

57. Arrange the alcohols in order of reactivity towards gaseous HBr.

F P F F ^ / r p

OH OH I II

(a) II > III > IV > I (c) I > II > III > IV

OH OH III IV

(b) I > IV > II > III (d) III > II > IV > I.

58. Compounds A'and T(C4H803) on heating produce W and S (C4H602) respectively. Only S decolourise bromine solution. Both A'and 1'turn blue litmus solution to red and both give white turbidity with Lucas reagent. W is -sweet smelling liquid. X and )' are

OH O 0

(a) OH OH OH

(b) HO

(c)

(d)

O OH O

cA = A, A OH

H O . „ , 0 Y » ;

o O P OH O

HON

OH OH

59. The structure of the starting material X is PBr, Mg/ether

> z -X N a 2 C r 2 0 7 , H 2 S 0 4 I V-

(a)

(C)

3,4-dimethyl-3-hexanol OH

OH

HjO"1

(b)

-OH (d) / \ ' I OH

60. Which laboratory test can not be given by the end product of reaction sequence?

CH3

Br alc. K O H ^ cold. KMnQ4 HIO.

(a) NaHC03

(c) NH40H/AgN03

soln

(b) 2,4-DNP (d) L/OH-.

ANSWERS

1. (a) 2. (a) 3. '(c) 4. (d) 5. (a) 6. (a) 7. (d) 8. (d) 9. (c) 10. (c) 11. (b) 12. (d) 13. (b) 14. (b) 15. (c) 16. (b) 17. (c) 18. (c) 19. (c) 20. (c) 21. (c) 22. (b) 23. (b) 24. (b) 25. (a) 26. (d) 27. (a) 28. (c) 29. (a) 30. (c) 31. (a) 32. (a) 33. (b) 34. <d) 35. (c) 36. (d) 37. (a) 38. (a) 39. (d) 40. (d) 41. (a) 42. (b) 43. (a) 44. (d) 45. (a) 46. (b) 47. (a) 48. (c) 49. (c) 50. (d) 51. (b) 52. (d) 53. (c) 54. (b) 55. (d) 56. (b) 57. (b) 58. (d) 59. (b) 60. (a)

Note : For detailed solutions please log on to our website www.resonance.ac.in

-10 CHEMISTRY TODAY | FEBRUARY '06 13

Time : 2 hrs.

Mock Test Paper for

IIT-JEE 2006 Exam on 9th April

2006

Max. Marks : 150

U s e f u l D a t a : At. wt. : H = 1, 0 = 16, C = 12, N = 14, Na = 23, K = 39, Mn = 55, Ag = 108, CI = 35.5, Ba = 137, F = 19, Co = 59, Cu = 63.5, Fe = 56, S = 32, P = 31, Au = 197, Mg = 24, Pb = 207, Sn = 118.7, Ca - 40, V = 51, Br = 80, B = 10.8, Li = 7, I = 127, Ti = 48, Cs = 133, U = 238, Zn = 65.5.

S E C T I O N A

1. The orbital diagram in which the aufbau principle is violated is

(a)

(c)

n n t | w 111 i n t t

t i t t t (d) | n | | n n t

2. Which of the following contains maximum number of lone pairs on the central atom? (a) CIO," (b) XeF4 (c) SF4 (d) 13"

3. Ethylene dibromide (C,H4Br2) and 1,2-dibromo propane form a series of ideal solutions over the whole range of composition. At 85°C, the vapour pressure of these two liquids are 173 and 127 torr respectively. What would be the mole fraction of ethylene dibromide in a solution at 85°C equilibrated with 1 : 1 molar mixture in the vapour? (a) 0.423 (b) 0.846 (c) 0.648 (d) none.

4. Calculate the enthalpy of formation of ammonia. Given that the N - H, H - H and N = N bond energies are 389, 435 and 945.4 kJ mol-1 respectively. (a) -81.4 kJ mol"1 (b) - 41.8 kJ mol-' (c) -65.8 kJ mol-' (d) -21.2 kJ mol"1

5. Estimate the minimum potential difference needed to reduce A 1,0., at 500°C. The free energy change for

2 4 the decomposition reaction, — AI,0, —> — AI + O, is 3 J

z

AC = + 960 kJ (F = 96500 C mol"1) (a) 2.487 V (b) 8.247 V (c) 7.428 V

Comprehension (Q.6

(d) 4.287 V

Q.10) : A lead storage battery

consists of a lead anode and a grid of lead packed with lead dioxide as the cathode. The electrolyte taken is 39% H,S04 by mass having a density of 1.294 g mL"1. The battery holds 3.5 L of the acid. During the discharge of the battery, the density of H,S04 falls from 1.294 g m/L to 1.139 g mL"1 which is 20% H2S04 by mass. 6. The reaction occuring at the cathode during charging is (a) (c) (d)

Pb2+ + 2e~ Pb Pb -> Pb2+ + 2e~ 2PbS04 + 2H20 2Pb02

(b) Pb2+ + S042" PbS04

4H + S042" 2e~

Moles of sulphuric acid lost during discharge is 9.88 (b) 8.88 (c) 7.88 (d) 6.88

Molarity of the solution after the discharge is 8.136 (b) 4.068 (c) 2.32 (d) 1.16

7. (a) 8. (a) 9. The amount of charge in coluombs used up by the battery is nearly (a) 954180 (b) 477090 (c) 95418 (d) 47709

10. The number of ampere-hour for which the battery must have been used is (a) 2650.5 (b) 265.05 (c) 26.505 (d) 2.6505

11. The potential energy diagram for reaction, R —> P is given in figure. AH° of the reaction corresponds to the energy (a) a (b) b (c) c (d) a + b

12. The disperse phase in colloidal iron (III) hydroxide and colloidal gold is positively and negatively charged

Reaction coordinate

Contributed by Deptt. of Chemistry, KLiitjee, Kota (Rajasthan)

1 4 CHEMISTRY TODAY I APRIL '06

respectively. Which of the following statements is not correct? (a) Magnesium chloride solution coagulates the gold

sol more readily than the iron (III) hydroxide sol. (b) Sodium sulphate solution causes coagulation in both

sols. (c) Mixing the sols has no effect. (d) Coagulation of both sols can be brought about by

electrophoresis.

13. In an adsorption experiment, a graph between \og(x/m) versus log p was found to be linear with a slope of 45°. The intercept on the log (x/rn) axis was found to be 0.3010. Calculate the amount of the gas adsorbed per gram of charcoal under a pressure of 0.5 atmosphere, (a) 1.0 (b) 2 (c) 3 (d) 1.5

14. Which of the following reaction is incorrect? (a) (b) (c) (d) BC1, + 2NMe3 CI3B(NMe3)2

15. (a) (c)

BF3 + 3C,HsMgl B(C,H5)3 + 3MgIF BC13 + 3EtOH —> B(OEt)3 + 3HC1 BBr + F,BNMe3 BF3

BCI, + 2NMe3

When XeF, is treated with water, it forms 4

Br3BNMe3

(b) both Xe and Xe03

both XeO, and XeOF, 3 4

only Xe only Xe03 (d)

16. Which of the following is the correct order of second ionization energy? (a) V > Cr > Mn (b) V < Cr < Mn (c) V < Cr > Mn (d) V > Cr < Mn

17. Which of the statement about the following structures is false?

CI en en

(1)

(a) (b) (c) (d)

on 'Cr' en

q >-" en I and either of the II and III are trans and c/s-isomers. I and II are enantiomers. II and III are optical isomers. I, II and 111 are isomers.

Comprehension (Q.18-Q.21) : A green solid (A) is water-soluble and its solution gives a white precipitate of AgCl soluble in NH,. When NH4OH is added to green solution of (A), a green-precipitate (5) is formed. When (5) is boiled with NazO, and water, a colour change to yellow results and solution (C) is obtained. Acidification of (C) causes the colour of the solution to change from yellow to orange solution (D). Addition of dilute aqueous Pb(NO,)0 to the orange solution (D) produces a bright yellow precipitate (£) in alkaline medium.

18. Compound A is (a) NiCI4 (b) NiCl, (c) CrCI, (d) FeCI,

19. Compound C is (a) Na,Cr04 (b) NaCr04

(c) NaJNi(02)3] (d) Na,Cr,04

20. Compound D is (a) Na,Cr707 (b) (NH4)?Cro07

(c) [Ni(NH3)4]S04 (d) [Ni(NH,)6]Cl2

21. Compound E is (a) PbCr04 (b) PbCr,07

(c) NiCrO„ (d) Ni,Cr207

22. Which of the following will produce 2,4-dichloro phenoxy acetic acid, i.e., (2,4-D), a weedicide?

OH PI (I) NaOH v

(a) | C1 + ClCH2COOH V( 2 ) h c [ >

(b)

-(c)

(d)

0 H + C1CH.COOH (I) NaOH (2) HC'I

CI ( I ) NaOH . + C CH,COOH — 1 >

2 (2 ) HC1

O H (1) NaOH v U H + CICH 2 COOH > (2) HC1

23. When C F 3 - C - 0 - 0 H is reacted with mesitylene O a

(1,3,5 trimethyl benzene) in the presence of BF,, the product formed is

Me Me

(a) (b) Vic M e

BF.,

Me

Ol

X C H - C F ,

M e H (c)

Me'' 011

(d) Me' Me

C - C F , II O

CHEMISTRY TODAY | MARCH '08 1 5

reacts with Br,(«g). Ozonolysis of B produces a compound ure

ClLCIiCl C\ C.HsO,. Which of the following is structure of .4?

(a)

(c)

O" (b)

-C3H3

T I

H,C CI

(d) -CH,CH,

Assertion (A) and Reason (R) Type questions

in the following questions (Q. 35-37), a statement of Assertion (A) is given followed by a corresponding s ta tement of Reason (R) jus t below it. Of the statements, mark the correct answer-(a) If both A and R are time and R is correct explanation

of A. (b) If both A and R are true and R is not the correct

explanation of A. (c) If A is true but R is false. (d) If A is false but R is true.

35. Assertion : CH = CH reacts with HCl in the presence of HgCI, while CH, = CH, does not. Reason : There is more unsaturation in CH = CH than in CH, = CH,.

36. Assertion : Unsubstituted alcohols give dehydration reaction only in acidic medium. Reason : If a strong base is used, then alcohols undergoes elimination reaction.

37. Assertion : Treatment of 1,3-dichloro propane on reaction with alcoholic KOH gives CH, = C = CH, \ Reason : It is nucleophilic elimination reaction.

Comprehension (Q. 38-Q.41) : Characteristic reactions of some of the important groups are given below (I) Phenols and carboxylic acids are soluble in dil.

NaOH, however only carboxylic acids are soluble in very dilute (5%) aqueous NaHCO^ solution with j the evolution of CO, gas.

(II) Lower alcohols containing five or less carbon atoms i react with Lucas reagent, while the higher alcohols ;

do not react with the reagent. (III) Aldehydes and ketones reacts with 2 ,4 -

dinitrophenylhydrazine to give orange coloured crystals, however only aldehydes reduce Fehling solution.

(IV) Acetaldehyde gives yellow precipitate with alkaline iodine.

O I!

CH, -C II i /OH" -> CHIjxl + HCOO"

The following flow-sheet diagram was prepared on the basis of the above reactions.

Water soluble Unknown

I 5% NaHCO,

No reaction Reaction r 2,4-DNP

Reaction No reaction r

Reaction

Fehling's Lucas test • test "

No immediate N o reaction] Reac t ion reaction |

KMnOi | No immediate

Reaction 1 reaction

VII I ' IV

Iodoform test

No reaction Reaction

VI V

38. Compound II should have (a) - COOH group (b) an alcoholic group (c) both (a) and (b) (d) a phenolic - OH group

39. Compound 111 is (a) C6H5OH (c) C H OH

(b) C H„OH (d) any of the three

40. Which of the following compound can be an aldehyde (a) V (b) VI (c) VII (d) IV

41. Compound V can be (a) CH3CHO (b) CH,COCH3

(c) CH,ICOC,H. (d) any1 of the three

S E C T I O N B

42. Which of the following statements are correct? (a) the electronic configuration of Cr is [Ar] 3d54s'

(At. no. of Cr = 24) (b) the magnetic quantum number may have a negative

value (c) in silver atom, 23 electrons have a spin of one type

and 24 of the opposite type (At. no. of Ag = 47) (d) the oxidation state of nitrogen in HN3 is -3 .

43. Which of the following are true? (a) In NaCl crystals, Na+ ions are present in all the

octahedral voids. (b) In ZnS (zinc blende), Zn2+ ions are present in alternate

tetrahedral voids. (c) In CaF„ F" ions occupy all the tetrahedral voids. (dj In Na,0, 0 : ions occupy half the octahedral voids.

1 8 CHEMISTRY TODAY I APRIL '06

44. A compound A reacts with KI to form yellow ppt. B. It reacts with BaCI2 to form white ppt. C which is soluble in excess of NH, solution and forms D. A also reacts with NaOH to form brown ppt. E. Compound A and E are (a) AgN03 , AgOH (b) Pb(NO,)„ Pb(OH), (c) CuS04 , Cu(OH), (d) Ca(N03)2, Ca(OH)~

Comprehension (Q. 45-Q. 47) : There is negligible intermolecular force of attraction between gas molecules. Molecules are constantly moving in zig-zag motion colliding with each other as well as with walls of the container and exert pressure. The kinetic energy is directly proportional to absolute temperature. Different molecules move with different velocities at a particular temperature. When temperature is increased, the average velocity of molecules increases but distribution of velocities remain the same.

45. The velocity possessed by maximum number of molecules is called (a) average velocity (b) root mean square velocity (c) most probable velocity (d) none of these

46. The kinetic energy of 8 gm of O, at 300 K. is (a) 0.9353 kJ (b) 1.870 kJ (c) 3.74 kJ (d) 0.0935 kJ

47. The root mean square velocity is equal to 100 m/s. What would be average velocity? (a) 92.13 ms-1 (b) 101 ms"1

(c) 81.6 ms-' (d) 102 ms-'

Comprehension (Q. 48- Q.50) : A spinel is a class of oxides in which two types of cations are present, bivalent cations and trivalent cations. Oxide ions are arranged in CCP layers Bivalent cation occupy l/8 ,h of the tetrahedral voids. Trivalent cation occupy 1/2 of the total number of octahedral voids.

48. If.4 is bivalent, B is trivalent cation, what is formula of oxide having spinel structure? (a) AB,0, (b) A0BO, (c) ABO, ( d ) A , B O

49. One trivalent ion will occupy the void formed by (a) six O2- ion (b) four 02~ ion (c) three O2- ion (d) eight oxide ions

50. I f — lies between 0.414 to 0.732, the cation will r

occupy

(a) trigonal void (c) octahedral void

(b) tetrahedral void (d) cubic void

51. Which of the following are optically active, optically inactive and which are coloured due to charge transfer spectra and which due to d —> d transition and which are colourless (1) [Fe(en),]3+ (2) [Pt(NH3)(H,0)CI, Br]2+

(3) M(a — a') trans form (4) [Ti(H,0)J4 +

(5) [Ti(H,0)J3 + (6) MnCV (7) Agl (a) (l)-optically active, (2)-opticalIy active (b) (l)-optically active, (2),(3)-optically inactive (c) [T i (H 2 0)J 4 + colourless, optically inactive,

[Ti(H,0)6]3+ coloured due to d —> dtransition and optically inactive

(d) (6), (7) coloured due to charge transfer

52. For the following conversions

CI 01

Conversion A

Conversion B :

NO,

NO,

o NO,

NO,

(a) for conversion A, the steps in Z (i) aq. OH /A (ii) H+ /H,0

(b) for conversion B, the steps in P (i) NH, + CuO/A (ii) C1-CH,CH,CH,-CH,-CH,-C1

(c) for conversion A, the steps in Z (i) aq. NaOH at -1000°C , (ii) H+ /H,0

(d) for conversion B, the steps in P (i) f J

Y H

53. H 2 /catalyst

1 atm 25"C •> A K.MnQ4

IN DENE

H , /Catalyst

H 2 0 / A C

N H ,

100 atm 100°C B

V D

In the following reaction sequence given

(a) compound A is

CHEMISTRY TODAY | MARCH '08 1 9

(b) compound B " C O (c) compound C is [ |

COOH

(d) compound D is

COOH

O

H

54. X*

O CH2-CH3

alk. KMnOj

H+

r

A + c o 2

H+/IL,0 KMnOj + CH,-C-0-C-CII, J

' II II •' o o p H+/H-°> O

B

In the reaction sequence given for toluene, identify A, B, X, )', P, O .

COOH CH2COOK

(a) A =

(b) = Q r "

(c) X =

, B

COOK - ^ . C O O I I

JQ^ / C I

C H , - C H O

(D) P = I I C H :

o II

, 0 - C - C H ,

^O-C-CH, II O

Q = C H O

Comprehension (Q.55-Q.59) : Amino acids contain an -NH^(basic) as well as a - COOH(acidic) group. They exist as zwitter ions.

R R

H , N - C H - C O O H H J N - C H - C O O -

which explain their several characteristic properties, like decomposition on heating, solubility in water, large dipole moment. Thus in solution, amino acids may exist as dipolar ion (neutral pH), cationic (in strongly acidic solution), or anionic (in strongly basic solution). Amino acids undergo usual reactions of the - COOH group as well as - NH, group.

55. At intracellular pH (-6—7), amino acids can be divided into four types positively charged, negatively

charged, hydrophobic and hydrophilic. Which is the correct classification of the following four amino acids?

+ NH, OH + NH 3 I I I "

(I) C 6H 5CH 2CHCOO" (II) CH:, C H - C HCOO~ +NH,

I (III) H 2 N - ( C H 2 ) 3 - C H - C O O "

+ NH 3

(IV) HOOCCH2 C H - COO" (a) (I)-hydrophobic, (11)-+ vely charged, (III)- - vely

charged, (IV)-hydrophilic (b) (I)-hydrophobic, (Il)-hydrophilic, (III)- + vely charged

(IV)- - vely charged (c) (I)-hydorphilic, (Il)-hydrophobic, (111)- + vely charged

(IV)- - vely charged (d) (I)- + vely charged, (II)- - vely charged,

(111) hydrophobic, (IV)-hydrophilic

56. Amino acids are (a) as basic as a typical amine and as acidic as a

carboxylic acid. (b) less basic than a typical amine and less acidic than

a - COOH. (c) more basic than a typical amine and more acidic

than a - COOH. (d) nothing is certain.

57. Base treatment of an amino acid usually results in the conversion of the acid to a derivative via the amino carboxylate salt. The above procedure (a) decreases the rate of electrophilic reaction of the

free amino group. (b) decreases the rate of nucleophilic reaction of the

free amino group. (c) enhances the rate of nucleophilic reaction of the

free amino group. (d) enhance the rate of electrophilic reaction of the

free amino group.

58. Benzoylation of an amino acid can best be done by treating the amino acid with benzoyl chloride (a) in presence of dil. NaOH (b) in presence of conc. NaOH (c) in absence of NaOH (d) in presence of HCl

+ N H , I

59. R - C H - C O O " pH » 12

NH,

^ R - C H - C O O "

-> Z. Compound Z is

2 0 CHEMISTRY TODAY I APRIL '06

NH,

(a) R - C H - C O O -

+ N H , i

(c) R - C H - C O O H

+ NH 3

(b) R - C H - C O O '

N H , I

(d) R - C H - C O O H

60. Histidine, a heterocyclic amino acid has following structure at pH < 1.82,

UN TL VCH,CHCOOH

N II

at pH > 1.82 it should have which structure?

(a)

(b)

. (c)

( d )

UN //

NH, I '

CH,CHCOO"

HN-T\ O - C H ;

NH, I 3

CHCOOl I

I1N-//

r \ H,

NH, I -

-CH,CI 1COOH

NH, 1

-CH,CHCOOH

ANSWERS

1. <b) 2. (d) 3. (a) 4. (b) 5. (a) 6. (d) 7. (a) 8. (c) 9. (a) 10. (b) II. (c) 12. (c) 13. (a) 14. (d) 15. (b) 16. (c) 17. (b) 18. (c) 19. (a) 20. (a) 21. (b) 92 (a) 23. (c) 24. (c) 25. (a) 26. (b) 27. (b) 28. (c) 29. (a) 30. (a) 31. (b) 32. (a) 33. (c) 34. (b) 35. (b) 36. (c) 37. (a) 38. (c) 39. (c) 40. (c) 41. (d) 42. (a, b, c) 43. (a, b, c) 44. (a) 45. (c) 46. (a) 47. (a) 48. (a) 49. (a) 50. (c) 51. (b, c, d) 52. (a, d) 53. (a, b. c, d) 54. fb, c, d) 55. (b) 56. (b) 57. (c) 58. (a) 59. (b) 60. (a)

ONLINE TRAINING

FOR BITSAT'06 5000 Students will get I unfair advantage... I

Yes! we are now offering the

Onl ine Training Module for the

Preparation of BITSAT. The

p r e c i s e c o u r s e m a t e r i a l

designed by the professionals

w i l l offer you the picture perfect

direct ion to BITSAT, so that you

can easi ly understand and

master it and check out your

path to Victory over BITSAT.

For more informat ion, log on to

w w w . m t g . i n

2 2 CHEMISTRY TODAY I APRIL '06

Very Similar MODEL TEST PAPER

for A I E E E - 2 0 0 6

Exam on' I 30th Apri l

2006

1. Which of the following statements is incorrect? (a) the third ionization potential of Mg is greater than

third ionization potential of Al (b) the first ionization potential o fNa is less than first

ionization potential of Mg (c) the second ionization potential of Mg is greater than

the second ionization potential of Na (d) the first ionization potential of Al is less than the

first ionization potential of Mg.

2. In the following compounds, the order of basicity will be

n o o o N N ^ N I I I H H H ( I ) ( I I ) ( H I ) ( I V )

(a) I > II > III > IV (b) IV > III > II > I (c) 1 > III > II > IV (d) IV > I > II > III.

3. Which of the following species has unpaired electron? (a) Ba02 (b) K0 2

(c) AI02- (d) N02+ .

4. Compound [Cr(NH3)5NCS][ZnCl4] will be (a) colourless (b) diamagnetic (c) green in colour (d) none of these.

5. Which of the following compounds is both paramagnetic and coloured? (a) K3[Cu(CN)4] (b) (NH4)2[TiCl6] (c) K2Cr04 (d) VOCl2.

6. The number of enantiomeric pairs that can be obtained by monochlorination of 2-methyl butane is (a) 4 (b) 2 (c) 1 (d) 3.

7. The molar heat capacity of water in equilibrium with ice at constant pressure is (a) zero (b) 40.45 kJKr1 mol 1

(c) infinity (d) none of these.

3 4

8. The property of alkaline earth metals that increases with their atomic number is (a) solubility of their sulphates (b) electronegativity (c) ionization potential (d) solubility of their hydroxides.

9. Nitrobenzene is prepared from benzene by using a nitrating mixture (HN03 + H2S04). In the nitrating mixture nitric acid acts as a (a) acid (b) base (c) reducing agent (d) catalyst.

10. The first use of quantum theory to explain the structure of atom was made by (a) Heisenberg (b) Einstein (c) Bohr (d) Planck.

11. Which of the following groups will have the best leaving ability? (a) CH3COO- (b) CH30-(c) MeS03" (d) -0S0 2 CF 3 .

12. The standard molar enthalpy of formation of C0 2

is equal to (a) the sum of standard molar enthalpies of CO and 0 2

(b) zero (c) the standard molar enthalpy of combustion of gaseous

carbon (d) the standard molar enthalpy of combustion of

graphite.

13. Which of the following statements is incorrect? (a) the rate of nitration of toluene is greater than that

of benzene (b) the rate of nitration of benzene is almost same as

that of hexadeuterobenzene (c) nitration is a nucleophilic substitution reaction (d) during nitration of benzene nitric acid acts as a base.

14. The intermediate during the addition of HC1 to propene in presence of peroxide is

CHEMISTRY TODAY I APRIL '06

(a) C H 3 - C H 2 - C H 2

(C) CTI3-CH-CH3

(b) C H 3 - C H 2 - C H 2

(d) CH 3 -CH-CH 2 C1

15. The correct statement about compounds (A), (B) and (C) is

COOCII,

H-H-

-OH -OH

H-H-

COOH

— OH H-— OH HO-

COOH

COOH

— OH — H

COOCH3

( Q

COOCH, W (.B)

(a) (A) and (B) are diastereomers (b) (A) and (C) are enantiomers (c) (A) and (B) are enantiomers (d) (A) and (8) are identical.

16. The oxidation states of the most electronegative element in the products of a reaction between Ba02

and H2S04 are (a) 0 and 1 (b) - 1 and - 2 (c) - 2 and - 2 (d) - 2 and 0.

17. The molar heat of a gas at constant pressure is 37.2 J. Then its atomicity will be (R = 8.3 J) (a) 1.4 (b) 1.3 (c) 1.6 (d) none of these.

18. In van der Waal's equation of state for a non-ideal gas the term that accounts for intermolecular forces is (a) (V-b) (b) RT

(c) P + - (d) (RT)~

19. The vapour pressure of pure benzene at a certain temperature is 200 mm Hg. At the same temperature the vapour pressure of a solution containing 2.0 g of non-volatile/non-electrolytic solid in 78 g benzene is 195 mm Hg. The molecular weight of the solid will be (a) 40 (b) 80 (c) 20 (d) 10.

20. Ten grams of a substance were dissolved in 250 ml of water and the osmotic pressure of the solution was found to be 600 mm of mercury at 15°C. Then, the molecular weight of the substance will be (a) 1198 (b) 1298 (c) 119.8 (d) 1.198

21. 20% N 2 0 4 molecules are dissociated in a sample

of gas at 27°C and 760 torr. Then the density of equilibrium mixture will be (a) 3.12 gm/lit (b) 31.2 gm/lit (c) 312 gm/lit (d) 3120 gm/lit.

22. Consider two gaseous equilibria involving S02 and corresponding equilibrium constants at 298 K.

SO 2 te) 2SO

+ l / 2 O 2 0 ! ) ^ S0 3 te); K 2SO, /,,) + 0 2 ; K2 3 (.?)

The values of equilibrium constants X, and A'-, are related by (a) K2 = Kt (b) K2 = (Ki)2

(c) = V ( A , ) 2

(d) K 2 = ~ 7 -

23. Equilibrium constant for the reaction 2NO f e ) + Cl2 ( , ) 2NOC1 U'>

is correctly given by the expression

(a) K =

(c) K =

[2NOC1] [2N0][C12]

[NOC1]2

[NOj2[Cl2]2

(b) K =

(d) K =

[NOC1]2

[NO]2[CI2] [NOCI]'/2

[NO]2 + [Cl2]

3 (x)

24. For the reaction, 2S0 2 t e ) + 0 2 t e ) ^ 2SO

the unit of K(; will be (a) lit mol"1 (b) mol lit"1

(c) (mol lit"1)2 (d) (lit mol"1)2.

25. In a chemical reaction, A + B C + D, the concentrations of A, B, C and D (in moles/lit) are 0.5, 0.8,0.4 and 1 respectively. Then the value of equilibrium constant will be (a) 0.1 (b) 1 (c) 10 (d) 00

26. The reagent with which both acetaldehyde and acetone react easily is (a) Fehling solution (b) Grignard reagent (c) Schiff's reagent (d) Tollen's reagent.

27. The compound which reacts fastest with Lucas reagent at room temperature is (a) butan-l-ol (b) butan-2-ol (c) 2-methylpropan-l-ol (d) 2-methylpropan-2-ol.

28. In the following compounds, the order of their acidic strength is

35 CHEMISTRY TODAY | DECEMBER '05 47

OH O H

OH CHj

(I) (II) (III) (IV) (a) III > IV > I > II (b) I > IV > III > II (c) II > I > III > IV (d) IV > III > I > II.

29. The formation of cyanohydrin from a ketone is an example of (a) electrophilic addition (b) nucleophilic addition (c) nucleophilic substitution (d) electrophilic substitution.

30. When acetaldehyde is heated with Fehiing's solution, it will give a precipitate of (a) Cu (b) CuO (c) Cu 20 (d) (Cu + CuO + CuzO).

31. What is the resultant oxidation number of chlorine in a molecule of bleaching powder? (a) 0 (b) +1 (c) - 1 (d) +2.

32. The type of hybridisation present in chlorine atom of C102~ ion is (a) sp (b) sp2

(c) sp3 (d) dsp2.

33. In which of the following compounds iodine atom has oxidation number less than +7? ( a ) H 5 I 0 6 ( b ) H 3 I O S

(c) H4I209 (d) none of these.

34. Which of the following oxyacids does not exist? (a) H3Sb03 (b) H3As04

(c) HBi03 (d) H3Bi04.

35. Which of the following is an anhydride of H4N204? (a) NO (b) N 2 0 (c) NO, (d) N203 .

36. Beta-hydroxy propionic acid on heating gives (a) ethylene (b) acrylic acid (c) acetone (d) propanal.

37. Which of the following compounds is formed when ammonium oxalate is heated with phosphorus pentoxide? (a) oxalic acid (b) C0 2 + H 2 0 (c) cyanogen gas (d) hydrocyanic acid.

38. Which of the following compounds will show the phenomenon of tautomerism? (a) diethyl ether (b) ethyl alcohol (c) cyanic acid (d) methanal.

39. Methanal and phenol react in the presence of a base to give (a) dacron (b) nylon-66 (c) polyethylene (d) bakelite.

40. Which of the following ligands forms chelate compounds? (a) acetate (b) cyanide (c) oxalate (d") ammonia.

41. Which ofthe following solutions will have highest boiling point? (a) 0.1 M glucose (b) 0.1 M NaCl (c) 0.1 M B?C12 (d) 0.1 M sucrose.

42. The molality of a solution having 18 g glucose (mol. wt. = 180) dissolved in 500 g of water will be (a) 0 m (b) 1 m (c) 0.2 m (d) 2 m.

43. Which of the following species behaves as Bronsted acid and Bronsted base? (a) OH- (b) HC03-(c) NH4

+ (d) H2S.

44. For which of the following properties of halogens the sequence F > CI > Br > I holds good? (a) boiling point (b) atomic size (c) electronegativity (d) electron affinity.

45. When oxalic acid is heated with conc. H2S04 at 90°C, the products formed will be (a) H 2 0 and C0 2 only (b) H 2 0 a"d CO only (c) CO and C0 2 only (d) H20, TO and C02 .

46. Which of the following cations will require lowest concentration of OH" ions for its precipitation as hydroxide? (a) Cr2+ (b) Cr3+

(c) Fe2"1" (d) Fe3+.

47. Which of the following is the correct sequence of the thermal stability? (a) KC1 > CsCl > RbCI > NaCl (b) RbCI < KC1 > CsCl < NaCl (c) RbCI > CsCl > KC1 > NaCl (d) NaCl < KC1 < RbCI < CsCl.

145 145 CHEMISTRY TODAY I APRIL '06

48. An aqueous solution freezes at -0 .150°C. (Kf= 1.50 and Kh = 0.41). What will be the elevation in boiling point? (a) 0.150 (b) 0.410

0.41 (c) — (d) 0.041

49. When /z-butyl benzene is oxidised by hot KMn04

solution, which of the following is formed? (a) benzoic acid (b) phenyl propionic acid (c) phenyl butyric acid (d) phenyl butyraldehyde.

50. Which of the following classes of organic compounds forms cyanohydrin, when reacts with HCN? (a) ethers (b) carboxylic acids (c) alcohols (d) carbonyls.

51. Ketones when react with hydroxy! amine, the products formed are (a) hydrazones (b) carbazones (c) oxirnes (d) cyanohydrins.

52. For a homogeneous gaseous reaction, 2SO, + 0 2 — 2S03

the dimension of K c will be (a) conc.+l (b) conc.-1

(c) cannot be predicted (d) it has no dimension.

53. Which one of the following scientists gave an idea of elliptic?.! orbits? (a) Moseley (b) Dalton (c) Sommerfeld (d) Pauling.

54. Benzoin condensation takes place, when (a) benzene is refluxed with sodium (b) benzaldehyde is refluxed with NaCN (c) acetophenone is heated with KCN (d) benzoyl chloride is hydrolysed.

55. Which one of the following acids behaves as a salt in its aqueous solution? (a) propionic acid (b) methanoic acid (c) ethanoic acid (d) glycine.

56. Which one of the following pollutants given by motor car, may cause cancer? (a) oxides of nitrogen (b) carbon dioxide (c) polynuclear hydrocarbons (d) none of these.

57. Which of the following statements is correct regarding the aldol condensation?

(a) all aldehydes give this reaction (b) this reaction proceeds in presence of dilute acid (c) ketones do not give this reaction (d) ketones, in which re-hydrogen atom is present, give

this reaction.

58. The energy of the first Bohr orbit of hydrogen is -13.6 eV. What will be the energy of third Bohr orbit of hydrogen atom? (a) - 40.8 eV (M -1.5 eV (c) - 4.53 eV (d) - 3.0 eV.

59. For a reaction, 2A + 3B 2C + 5D, when the concentration of A is doubled and that of B is halved, the reaction rate becomes (a) half of initial rate (b) double of initial rate (c) three times of initial rate (d) 1/4th of initial rate.

60. If K is ionization constant of water and Ku. is ionic product of water, which of the following represents the correct relationship between them? (a) K = (b) K = Kv. x 55.4

„ K (c) K = j f j (d) K = "1JB x*,,,.

f SOLUTIONS j

1. (c) : Na = I.?2, 2s2 2pb, 3^' Na + = \s2,2s22pb

Mg = Is2, 2s2 2/A 3s2

Mg+ = Is2, 2s2 2pb, 3.S1

Mg2+ = 1 j2, 2s2 2pb

Al = l i2 , 2s2 2p6, 3s2 3p]

Al+ = Is2, 2s2 2pb, 3s2

Al2+ = \s2, 2s2 2pb, 351

In case of Mg, the electron has to be removed from a • stable fully filled 3s orbital. Hence, I, of Mg is greater than I, o f N a and I, of Al. I3 of Mg is greater than I3 of Al, because in Mg electron is removed from coniDletely filled 2p-orbital. L of Na will be greater than I, of Mg because in Na second electron will be removed from fully filled 2p orbital while in case of Mg second electron will be removed from 35 incomplete orbital.

2. (b) : The lone pair of electrons on nitrogen atom (N) in structure (I) is involved in resonance, therefore, it is almost neutral. (IV) is a strongest base because nitrogen in this is in sp3 hybridised state like NH3 or aliphatic amines.

35 CHEMISTRY TODAY | DECEMBER '05 47

48. An aqueous solution freezes at -0 .150°C. (KF = 1.50 and KH = 0.41). What will be the elevation in boiling point? (a) 0.150 (b) 0.410

0.41 (c) — (d) 0.041

49. When /7-butyl benzene is oxidised by hot KMn04

solution, which of the following is formed? (a) benzoic acid (b) phenyl propionic acid (c) phenyl butyric acid (d) phenyl butyraldehyde.

50. Which of the following classes of organic compounds forms cyanohydrin, when reacts with HCN? (a) ethers (b) carboxylic acids (c) alcohols (d) carbonvls.

51. Ketones when react with hydroxyl amine, the products formed are (a) hydrazones (b) carbazones (c) oximes (d) cyanohydrins.

52. For a homogeneous gaseous reaction, 2SO, + 0 2 ^ 2S03

the dimension of Kc will be (a) conc.+l (b) conc.-1

(c) cannot be predicted (d) it has no dimension.

53. Which one of the following scientists gave an idea of elliptical orbits? (a) Moseley (b) Dalton (c) Sommerfeld (d) Pauling.

54. Benzoin condensation takes place, when (a) benzene is refluxed with sodium (b) benzaldehyde is refluxed with NaCN (c) acetophenone is heated with KCN (d) benzoyl chloride is hydrolysed.

55. Which one of the following acids behaves as a salt in its aqueous solution? (a) propionic acid (b) methanoic acid (c) ethanoic acid (d) glycine.

56. Which one of the following pollutants given by motor car, may cause cancer? (a) oxides of nitrogen (b) carbon dioxide (c) polynuclear hydrocarbons (d) none of these.

57. Which of the following statements is correct regarding the aldol condensation?

35 CHEMISTRY TODAY | DECEMBER '05

(a) all aldehydes give this reaction (b) this reaction proceeds in presence of dilute acid (c) ketones do not give this reaction (d) ketones, in which a-hydrogen atom is present, give

this reaction.

58. The energy of the first Bohr orbit of hydrogen is -13.6 eV. What will be the energy of third Bohr orbit of hydrogen atom? (a) — 40.8 eV (H -1.5 eV (c) — 4.53 eV (d) 3.0 eV.

59. For a reaction, 2/1 + 3 B 2C. + 5D, when the concentration of A is doubled and that of B is halved, the reaction rate becomes (a) half of initial rate (b) double of initial rate (c) three times of initial rate (d) 1/4th of initial rate.

60. If K is ionization constant of water and K,, is innic product of water, which of the following represents the

j correct relationship between them? (a) K = KW (b) K = KW X 55.4

K (c) (d) K= 1J8 x K„.

j SOLUTIONS

1. (c) : Na = Is2, 2s2 2pb, 3s1

Na+ = Is2, 2s2 2pb

Mg = Is2, 2s2 2pb, 3s2

Mg+ = Is2, 2s2 2p \ 3s1

Mg2+ = Is2, 2s2 2p6

AI = Is2, 2s2 2p\ 3s2 3p l

Al+ = Is2, 2s2 2p", 3s2

Al2+ = Is2, 2s2 2p \ 3s1

In case of Mg, the electron has to be removed from a -stable fully filled 3s orbital. Hence, I, of Mg is greater than 11 of Na and 1, of AI. I3 of Mg is greater than 13 of AI, because in Mg electron is removed from comDletely filled 2p-orbitaI. L of Na will be greater than l2 of Mg because in Na second electron will be removed from fully filled 2p orbital while in case of Mg second electron will be removed from 3s incomplete orbital.

2. (b) : The lone pair of electrons on nitrogen atom (N) in structure (I) is involved in resonance, therefore, it is almost neutral. (IV) is a strongest base because nitrogen in this is in sp3 hybridised state like NH3 or aliphatic amines.

47

AlCV +

3. (b) : (a) Ba2+ | j O — O 2

(b) K+ o ] u n P a i r e d e l e c t r o n

(c) Al3+[0,]4 '

(d)

4. (c) : Complex compound [Cr(NH3)sNCS][ZnCl4] can be represented as [Cr(NH3)5NCS]2+[ZnCl4]2~. It contains Cr3+ and Zn2+ ions with the configuration in d orbit as Cr3+ = 3d3, Zn2+ = 3d [ a . Due to 3 unpaired electrons in 3d, the compound will be of green colour as other trivalent chromium compounds. 5 . ( d ) : K 3 [ C U ( C N ) 4 ] ; C U + = 3d10

(No unpaired electron hence, neither coloured nor paramagnetic). K2Cr04 , Crs+ = 3d w

(No unpaired electron hence not paramagnetic but coloured). (NH4)2[TiCl6], Ti4+ = 3d° (No unpaired electron) VOCl2, V4+ = 3d1

(One unpaired electron hence, paramagnetic and blue coloured).

CH,

6. (b) : CH, - CH, - CH - CH, d l l o r" i a t i o n>

(i) CH2C1-CH2

CI CH,

CH, I '

• CH - CH3

CH,

(ii) CH, - CH - CH - CH3 (iii) C H 3 - C H 2 - C - C H ,

CI (iv) CH, - CH2 - CH - CH3

CH,C1 Compounds (i) and (iii) have no chirai carbon atom and hence, they are optically inactive. Compounds (ii) and (iv) both have chirai carbon atoms and hence, they are optically active. Thus, the two pairs of enantiomers will be as

H I

CH, - C - CHMe2 I CI

enantiomers

H

M e 2 C H - C - C H 3

CI

H

C2H5 - C - CH3

CH2C1

H

H 3 C - C - C 2 H 5

CH,CI enantiomers

7. (c) : As we know that C-r = dqldt. Since dl = 0, hence C,, = infinity.

8. (d)

9. (b) : In nitrating mixture HN0 3 acts as a base as

- © ©

HO - NO, + 2H2S04 11,0+ + 2HS04" + NO, 10. (c)

11. (d) : Highly electronegative fluorine atoms pull electrons strongly from sulphur and thus, lower its nucleophilic power and hence, it will have highest leaving ability.

0 F 1 4

O - S ^ C - ^ F I * o F

12. (d)

13. (c) : Nitration involves the formation of an + electrophile i.e. NO, ion.

14. (c) : The peroxide effect does not apply to the addition of HCI. It proceeds with formation of more stable

CH3 -CH -CH3 carbocation.

15. (c)

16. (b) : BaO, + H,S0 4 BaS04 + H 2 0 , In the products electronegative element is oxygen. Hence, the oxidation state of O atom in BaS0 4 = - 2 and in H 20 2 = - 1 .

17. (b) : As, CP = 37.2 J, R = 8.3 J Now by applying the formula, CP - Cv = R => 37.2 — Cy= 8.3 .-. C|/ = 28.9.

Since, y = £e- = — = 1.280 = 1.3. ' Cv 28.9

18. (c) : As the correction in ideal gas equation due to

intermolecular forces of attraction is y%

19.(b) : Given, P° = 200 mm, P = 195 mm w = 2.0 g, M= 78, W = 78, m = ?

3 8 CHEMISTRY TODAY APRIL 06

Applying P°-P wM

P° Wtn 200-195 2x78

200 m = 80.

7 8 x / «

5 _ 2 200 m 40 fl? '

20. (a) : As, according to vant Hoff 's equation, VI'

nV = nRT => nV = — RT m

wRT

Now, by substituting the values, we get 10x0.082x288 „ „ „

m = = 1 1 9 8 . (600/760) x 0.25

21. (a) : N 2 0 4 2NO, Initially 1 0 Finally ( 1 - 0 . 2 ) 2 * 0.2

= 0.8 = 0.4

Now by substituting the values in V = nRT

1.2x0.082x300 1

= 29.52 lit.

Now, density Mol. wt. 92

22. (c) : As, K, =

K-, -

Volume 29.52 (mol. wt. of N 20 4 = 92)

= 3.12 gm/lit.

[S03] 1/2 [S0 2 ] [0 2 ]

[SO2]2[Q2] [SO,] 2 (ii)

It is clear from (i) and (ii) that K2 = (K i)"

23. (b) : As the expression for Kc, for the reaction, 2 N 0 W + C12 W ^ 2NOC1 (g)

[NOC1]2 K(.=

[N0]2[C12]'

24. (a) : As, for the reaction 2S0 2 ( k ) + 0 2 ( s ) ^ 2S0 3 t e )

(mol/lit)2

Kr=- • = lit mol (mol/lit)2 x (mol/lit)

25. (b) : As, K(: for the equation A + B ^ C + D ^ K _ . [n [P ]^ [0 .4 ] [1 .0 ]__0 .4^ 1 Q

r [A][B] [0.5][0.8] 0.4 .-. Kr = 1.0

26. (b) : CH3CHO acetaldehyde

C.2H5

C 2 H 5 M g I Grignard reagent

CFI3 - C - OMgl — C 2 H 5 C H O H C H , + Mg(OH)I HCl

H secondary alcohol

o OMgl

C H 3 - C - C H , + CH3MgI acetone Grignard

reagent

HCl

—> CH3 - C - CH3

CH3

CH3

> CH3 - C - OH + Mg(OH)I

CH, tertiary alcohol

27. (d) : 2-methyIpropane-2-ol is OH

CH, - C - CH,

CH3

It is a tertiary alcohol, tertiary alcohols react very fast with Lucas reagent (conc. HCl + ZnCl2). 28. (d) : Phenol is acidic in nature due to extra stability of phenoxide ion (C6H50~) due to resonance. Acidic strength of phenol increases if a group or atom having -ve inductive effect is attached to the ring. e.g. - CH3, - C2H5 etc. Since - NO, group is stronger electron attracting as compared to - OH group. So nitrophenol will be strongest.

CH3 group has +ve inductive effect so C H 3 ~ O H

will be weakest. Therefore, the correct sequence is

0,N •OH

OH

> CH

29. (b) : > C ^ = 0 :

r ® - OH

\ C—O,: + H + C N " nucleophile

— > > 9 - OH

CN cyanohydrin

So, it is a nucleophilic addition.

30. (c) : RCHO + 2CuO > RCOOH + Cu,Oi (Fehling . (red ppt.) solution)

35 CHEMISTRY TODAY | DECEMBER '05 47

31. (a) : The structure of bleaching powder molecule is

Ca / CI

\ Oui

The CI attached to Ca has oxidation number - 1 and that which is attached to O atom has +1 oxidation number as electronegativity of O is greater than CI. Thus, the resultant oxidation number of CI

+ 1 + H ) • = o.

32. (c) : C102~ ion involves sp1

hybridisation in chlorine atom and has given structure. Thus the structure of CIOY is angular.

33. (d) : In all three compounds, the oxidation number of iodine is +7. 34. (d) : Arsenic forms two oxyacida H3AsQ3 and H3AS04. Antimony forms only one cxvaeid H3SbO,. Bismuth forms only one oxvacid i.e. metabismuthic acid HBi03 (Scarlet red solid). 35. (a) : H 4 N 2 0 4 is known as nii ioxylic acid or hydronitrous acid. It is supposed to be hydrate of nitric oxide, 2NO 2H 2 0. Ktn„c, it» anhydride is NO.

A 36. (b) : HO-CH 2 -CH,COOH [3-hydroxypropionic acid

R> CH: = CH.COOH acrylic acid

37. (c) COONH4 I COON1 !4

ammonium oxalate

P..O, CONH, -2H 20

P,O5 CN

CONH2 " 2 H : ° CN oxamide cyanogen

38. (c) : Cyanic acid exhibits tautomerism as H - 0 - C = N ; = ± H - N = C = 0

cyanic acid isocyanic acid

39. (d)

40. (c) : Oxalate ion forms chelate compounds with metals. The chelates are actually complex compounds in which closed rings are formed.

o = c - o . M-C — 0 n 2 _

I V > ^ ' > V I 2 X

O — C — O 0 - C = 0

Five membered rings are formed.

40

41. (c) : Boiling point is a colligative property which depends upon the number of particles.

42. (c) : When 180 g of glucose is dissolved in 1000 g of water, the solution is said to 1 molal. v 500 g contains 18 g glucose

18x1000 _ 1000 g contains — — — = 36 g

500 180 g in 1000 g means m solution

mx 36 mxI 36 g in 1000 g means = 0.2 m.

180 5 43 . ' (b ) : HC03" behaves both as Bronsted acid and Bronsted base as

HC03" H+ + C032"

bronsted acid

44. (c)

45. (d) :

bronsted base

COOH c o n c . H s o

90°C H,0 + CO + CO,

COOH oxalic acid

46. (d): The solubility products of different hydroxides in question are given as : Cr(OH)2 = 1 x 10"17 Cr(OH)3 = 1 x 10"33

Fe(OH)2 = 8 x 10~16 Fe(OH)3 = 4 x 10"40

As the solubility product of Fe(OH)3 is lowest and therefore, Fe3+ will require lowest concentration of OH" ions for the precipitation as Fe(OH)3.

47. (a) : The standard heats of formation of chlorides of alkali metals are as: NaCl = - 400 kJ mol 1 , KC1 = - 428 kJ mol-' RbCI = - 423 kJ mol"1, CsCl = - 424 kJ mol"' Hence, sequence of thermal stability will be as

KC1 > CsCl > RbCI > NaCl

48. ( d ) : Given, A7>= - 0.150°C, KF = 1.50, KH = 0.41 A7}= Kj x molality (m)

Molality (m) AT, 0.150

.50 = 0.1 m

ATH = KH x molality (M) = 0.41 x 0.1 = 0.041°C.

49. (a): C H , - C H 2 - C H , - C H 3 -hot K M n O

n-butyl benzene ( O T C O O H+ c o ,

benzoic acid

50. (d) : Carbonyl compounds (aldehydes or ketones) form cyanohydrin on reacting with HCN. 150 CHEMISTRY TODAY I APRIL '06

It is a nucleophilic addition reaction which proceeds by attack of nucleophile. (: CN" ion).

\ / C I.

- C - CN I o-

H+ > - C - C N

: CN

51. (c) : Oximes are formed as :

H

OH cyanoliydrin

\ / c o

ketone

+ : NH2OH -hydroxy! amine

- C - N H O H I OH

/ C = NOH + H,0 oxt me

52. (b) : For the reaction, 2S0 2 + O, 2S03

[S03]2

Kc = [ s o 2 f t o 2 ]

[mol/lit]2 1 - = conc.

[mol/Iit]2[mol/lit] [mol/lit] 53. (c)

54. (d) : When benzaldehyde is refluxed in presence of NaCN or KCN, benzoin is formed.

H O H O I II K C N I II

benzaldehyde benzaldehyde OH

benzoin

55. (d) : The amino acids which have one carboxylic acid group and one amino group behave as a neutral compound. This is due to the fact that amino and carboxylic groups, being of opposite character neutralize one another intramolecularly and form an internal salt. Thus, in aqueous solution a-amino acids behave as salt.

56. (c)

57. (d)

58. (b) : Energy of first Bohr orbit of hydrogen is

Z2 1 E, =-13.6x——eV = - 1 3 . 6 x - = -13.6 eV

' n2 1 and energy of third Bohr orbit is \

E3 =-13.6X4T eV = -1.5 eV. 3

59. (a) : Reaction \2A + 3B 2C + 5D r oc [A]2 [5]3

roc [2A]2

2

~ 4[A]2 -\B? 8L J

- 4 x - W [ f i f

1 [Af[3f

Hence, rate of reaction will become half of initial rate. 60. (c) : H 2 0 H+ + OH" According to law of mass action,

[H+][OH~] ionization constant of water, K = -

[H 20]

As water is very less ionized the value of H 2 0 remains almost constant and hence

£ [H 2 0] = [H+][OH"] K[U20] = Kw = [H+][OH-]

K = o r , K = [H20] 55.4

[H20] = 55.4 mole/litre]

Based on Latest Pattern • 10 Very Similar

Practice Test Papers

• 2002-2005 Expert's Solved Papers

• 2000+ MCQ's with detailed Solutions

• Frequently Asked Questions

oj^M ^ f Rs.200 % ^,pages.1ioo$

CHEMISTRY TODAY | APRIL 06 41

( A I I M S ; C B S E ] ( D P M T ) ( A F M C ] ( V M M C ) (EAMCET) ( WB-JEE) ( B H U ) ( C M C )

2006 Medical Entrance Exam ( U P C P M T )

(CET Karnataka Practice Test Paper (Manipa l PMTj

( J I P M E R )

MP PMT) M G I M S PMDT B i h a r ) fPMT HaryanlTj ( K e r a l a P M T ) ( R a j . P M T ] f T N P C E E )

1. Sulphuryl chloride S02C12 reacts with water to give a mixture of H2S04 and HC1. Mole of NaOH required to neutralise the solution formed by adding 1 mole of S02C12 to excess water is (a) T (b) 2 (c) 3 (d) 4.

2. 5.6 g of a metal forms 12.7 g of metal chloride. Hence, equivalent weight of metal is (a) 127 (b) 254 (c) 56 (d) 28.

3. Mole fraction of ethanol in ethanol-water mixture is 0.25. Hence, percentage concentration of ethanol by weight of mixture is (a) 25% (b) 75% (c) 46% (d) 54%.

4. When N2 changes to N2+, the N - N bond distance

and when 0 2 changes to 02+ , O - O bond

distance (a) increases, decreases (b) decreases, increases (c) increases in both cases (d) decreases in both cases.

5. In which there is change in the type of hybridisation when (a) NH, combines with H+

(b) A1H, combines with H (c) in both cases (d) none of these.

6. Hypervalent compound is (a) S03

2- (b) P043-

(c) S042~ (d) all are correct.

7. The correct order of increasing C - O bond length of CO, CO,2", COz is (a) C03

2- < C0 2 < CO (b) C0 2 < C032~ < CO

(c) CO < C032" < C0 2 (d) CO < C0 2 < C03

2-.

8. Amongst Ni(CO)4, [Ni(CN)4]2~ and NiCl42"

(a) Ni(CO)4 and NiCl42" are diamagnetic and [Ni(CN).,]2

is paramagnetic (b) NiCl4

2" and [Ni(CN)4]2~ are diamagnetic and Ni(CO)4

is paramagnetic

(c) Ni(CO)4 and [Ni(CN)4]2~ are diamagnetic and NiCl42~

is paramagnetic (d) Ni(CO)4 is diamagnetic and NiCl4

2-and [Ni(CN)4]2" are paramagnetic.

9. COC13-4H,0 is an anhydrous binary solute. Hence, its Werner's representation is

CI CI | OH, | OH,

(a) C l - C o - O H , (b) CI — Co — OH, H2O q OH2" H20 £[ OH2

CI OH2 \ l (c) H,0 — CoI — OH, (d) none of these.

C 1 O H >

10. KMn04 oxidises X n + into X03~, itself changing to Mn2+ in acid solution. 2.68 x 10~3 mol of X"* require 1.61 x l o - 3 mol of Mn04". Hence value of n is (a) 2 (b) 3 (c) 4 (d) 0.

I 11. In Aufbau rule is not used, 19th electron in Sc (Z= 21) will have (a) n = 3, / = 0 (b) n = 3, / = 1 (c) n = 3, / = 2 (d) n = 4, 1= 0.

12. The compressibility factor of gas is less than unity at STP, therefore (a) V,„ (molar volume) > 22.4 lit (b) Vm< 22.4 lit (c) Vm = 22.4 lit (d) Vm = 44.8 lit.

13. Which gas shows real behaviour? (a) 8 g O, at STP occupies 5.6 litre (b) 1 g H, in 0.5 L flask exerts a pressure of 24.63 atm

at 300 K (c) 1 mol NH3 at 300 K and 1 atm occupies volume

22.4 litre (d) 5.6 litre of C02 at STP is equal to 11 g.

14. For the non-zero value of froce of attraction between gas molecules, gas equation will be

50 CHEMISTRY TODAY JANUARY '06

(b) PV= nRT + nbP

(d ) P = n R T V-b

:A

K,[Fe(CN 0.01 M FeCI,=

(a) PV = nRT —-V

(c) PV - nRT

15. FeCIj on reaction with K4[Fe(CN)s] inaq. solution gives blue colour. These are separated by a s e m i p e r m e a b l e membrane A B as shown in figure. Due to osmosis there is (a) blue colour formation in side X (b) blue colour formation in side Y (c) blue colour formation in both of sides X and Y (d) no blue colour formation.

16. See the table.

side A' ; ; ; ; ; side Y I EE B

A+ B-^ C

[A] IB] [rate]

1 1.0 1.0 0.25 2 2.0 1.0 0.50 3 1.0 2.0 0.25

Rate is expressed in mol lit-1 min'1. In the above reaction, the order is (a) zero in A and one in B (b) one in A and zero in B (c) one in both A and B (d) zero in both A and B.

17. Rate constant of a reaction is 0.0693 min"1. Starting with 10 mol, rate of the reaction after 10 min is (a) 0.693 mol min'1 (b) 0.0693 mol min'1

(c) 0.0693 x 5 mol min'1

(d) 0.0693 x (5)2 mol m i n - i .

18. If Ag+ + 2NH3 = Ag(NH3)2+, K, = 1.8 x

Ag+ + CI' = AgCl, K2 = 5.6 x io9

then for AgCl + 2NH, = Ag(NH3)2+ + CI',

equilibrium constant K will be (a) 0.32 x 10'2 (b) 0.31 x 103

(c) 1.00 x 1017 (d) 1.00 x 10'17.

107

19. Using only the following data, I. II. TheA//° value, in kilojoules, for the reaction

Fe,03 (,) + 3CO fe ) = 2Fe„., + 3CO20r), AH° =-26.8 kJ Fe (5) + CO, = FeO (.s) + CO te), AH° = 16.5 kJ.

III. Fe ,0 2W3 (.v) CO fe) 2FeO co2 2 fe) is calculated to be

(a) - 4 3 . 3 (b) - 1 0 . 3 (c) + 6 . 2 (d) + 1 0 . 3

20. The dissociation energies of CH4 and C,H6 to convert them into gaseous atoms are 360 and 620 kcal mol"1

respectively. The bond energy of C - C bond is (a) 260 kcal mol"1 (b) 180 kcal mol"1

(c) 130 kcal mol"1 (d) 80 kcal mol'1.

21. In the reaction : [A1(H,0)6]3+ + H , 0 = [A1(H20)5]2+ + H 3 0 +

(a) [A1(H,0)6]3+ is a base (b) [A1(H,0)6]3+ is an acid (c) both are correct (d) none is correct.

22. pH of Ca(OH)2 is 12. Milliequivalents of Ca(OH)2

present in 100 mL solution will be (a) 1 (b) 0.5 (c) 0.05 (d) 5.

23. For the first order reaction A —> product, the half-life time is 100 seconds. The rate constant of the reaction is (a) 6.93 x I0"2 s'1 (b) 6.93 x 10"4 s'1

(c) 6.93 x 10'3 s"1 (d) 6.93 s'1.

24. Maximum enolisation takes place of (a) CH3COCH3 (b) CH3COCH2CHO

(c) CH3COCH2COCH3 (d) O o

25. Type of isomerism shown by the product of the reaction between benzaldehyde and hydroxy! amine is (a) syn and anti geometrical (b) cis and trans geometrical (c) E and Z geometrical (d) none of these.

26. Most stable free radical is

(a) CH, — CH (b) CH, = CHCH,

(c) CH, (d) CH3

27. Major product of the following SN1 reaction is

CH, - CH - CH - CH, + OC,H, > 3 I I 3 2 5

Br CH, CH3 I

(a) CH3CHCHCH3 (b) CH,CHCH,CH,OC2H5

OC,H, CH, CH,

(c) CH3CH,CCH3 (d) none of these.

OC2H5

35 CHEMISTRY TODAY | DECEMBER '05 47

46. EN o f the e l e m e n t s is £ , and ionisation potential is E2. Hence energy o f activation will be (a) 2 £ , - E2

(c) E, - 2E2

(b) £ , - E2

(d) (£, - E2)/2.

47. Stability order o f +3 and +1 states o f II1A (boron family) element is (a) Ga3 + < In3+ < Tl3 + (b) Ga+ > In+ > Tl+

(c) Ga+ < ln+ < Tl+ (d) Ga3+ < Ga+.

48. In P4 (tetrahedral) (a) each P is jo ined to four P (b) each P is jo ined to three P (c) each P is jo ined to two P (d) P4 does not exist.

49. A g , S + NaCN A + Zn - » B is a metal. Hence, A and B are (a) Na , [Zn(CN) 4 ] , Zn (c) Na[Ag(CN) 4 ] , Ag

(b) Na[Ag(CN) 2 ] , A g (d) Na 3 [Ag(CN) 4 ] , Ag.

50. B<:

I.

Lindlar H,

Li/NH, -> A .A and B are

c = c-

(a) I in both cases (c) A is 1, B is 11

II.

(b) II in both cases (d) A is II, B is I.

S O L U T I O N S

1. (d) : SO,CI, + 2 H , 0 H , S 0 4 + 2HC1 1 mol H 2 S 0 4 = 2 mol NaOH 2 mol HC1 = 2 mol NaOH.

2. (d) : CI combined with metal = 12.7 - 5.6 = 7.1 g 7.1 g CI = 5.6 g metal 35.5 g CI = 28 g metal..

3 . ( C ) :

So,

/^(ethanol) 1 _ _ _ % «, (ethanol) + /ij (water) 4

/7,(water) HJO

So,

A;, (ethanol) + w, (water) 4 n(ethanol) _ 1 _ 1 mol ethanol (46 g)

(water) 3 3 m o l H 2 0 ( 5 4 g ) Total weight o f solution = 100 g % ethanol by weight of solution = 46%.

4. (a) : Species Bond order N 2 3

N , + 2.5 O, 2 0 , + 2 .5

f Bond length oc ! V bond order Hence, N 2 —> N , + , bond length increases. O , —» 0 2

+ , bond length decreases.

5. (b) : N H , sp

•NH 4

sp'

A1H3 > AIH4

sp2 sp3

6 . ( d ) : Hypervalent compound/ion is that in which the Lewis structure demands the presence o f more than an octet (8) o f electrons around at least one atom.

7. ( d ) : Bond length <x ? no. o f bonds

CO : triple bond, C 0 2 : double bond COj2": between single and double bond due to resonance. Hence, C - O bond length will be in order

CO < CO, < C032".

8. (c) : CO, CN" are strong l igands and unpaired electrons in bonding metal/metal ion get paired. CI" is a weak ligand.

9. (c) : COC1 3 -4H 20 is a binary solute, thus, can be written as [CO(H ,0 ) 4 CI 2 ]C1. Thus, two CI" satisfy primary as wel l as secondary valencies and are shown by rrrr: . Four H 2 0 molecules satisfy only secondary valencies. Hence shown by — . One CI" satisfies only primary valency and is shown by 10. (a) : M n O " + .Y"

I AO, + Mn2+

oxidation (5 - n ) e lost 1 reduction Se gained

Thus 0 1 0 1 6 o f M n 0 4 5 - n 1.61x10 2.68x10" mole oiX"+

which gives n = 2.

11. (c) : If Aufbau rule is not fo l lowed , electronic configuration o f SC is l.v2 2S2 2p6 3s2 3 p b 3tP. Hence, for 19 ,h electron, n = 3, / = 2.

12 . ( b ) : If compressibility f a c t o r s < 1 then

PV ——— = z < 1 RT

P and T remains constant is less than 22 .4 litres. 13. (c) : If gas is real, then PV * nRT.

Vm (molar vo lume)

14. (a) : P + - [V - nb] = nRT

154 CHEMISTRY TODAY I APRIL '06 35

If force of attraction exists then nb = 0.

P + -V2 V = nRT . P V + l l = n R T ,

V

15. (d) : Osmosis of solvent takes place hence, no interaction between solute particles.

dx 16. ( b ) : y - } = k[AY[BX

If concentration of the reactant is made m times, rate becomes m" times for «,h order reaction. From (i) and (ii), if concentration of A is doubled rate is also doubled. Hence 2" = 2 a = 1. From (i) and (iii), if concentration of B is doubled, rate remains unchanged. Hence, 2b = 1 => b = 0.

17. (c ) : k = 0.0693 min-1 (first order since unit of k is time"1).

0.693 T =- = 10 min.

If initial concentration is 10 mol, after 10 minute = 5 mol (50% reacted)

= k(A) = k(a-x) = 0.0693 x 5 mol min"1.

18. (a) : K = — = 0.32x10"2. k2

19. (c) : I + 2(11) = III.

20. (d) : [CH4 —> C (g) + 4H ^ 1 4 ( B E ) C _ H = 360 kcal

[C2H6 —» 2C + 6H Q,) ] ( B E ) C ^ C + 6 ( B E ) C _ H = 6 2 0 k c a l

Hence, (BE)C_C = 80 kcal.

21. (b) : [A1(H20)6]3+ changes to [A1(H,0)50H]2+ by loss of H+ hence, it is an acid.

22. (a) : pH = 12 . pOH = 2 .

[OH"] = IO"2 N = [Ca(OH)2] Hence, in 100 ml solution equivalent of

10~2 xlOO Ca(OH)2 = - = 1 0 " J

1000 Milli equivalent = 10"3 x 1000 = 1.

23. (c) : Rate of half-time =

0 . 6 9 3

0 . 6 9 3

k = 100 = 0 . 6 9 3 X L 0 " Z = 6 . 9 3 X IO"3 s"

OH is most stable due to resonance. 24. (d) :

25. (a) : CHO + NH2OH >

CH = N - OH

Benzaldoxime shows syn and anti geometrical isomerism.

26. (d) : Stability of free radicals - 1° < 2° < 3°.

27. (c) : CH,CHCHCH, ! 1 3

Br CH,

CH,CHCHCH, • © |

CH, 2° carbocation

1,2 H~ shift OC2H5 . OC2H5

> CH,CH,CCH, CH.CHXCH

CH, 3 21 3

CH3 3° carbocation

28. (b) : By 1,2-methyl shift, 2° carbocation changes to 3° carbocation. [Substituted alkenes are more stable],

29. (c) : Aldehydes/ketones with atleast one a - H give aldol condensation reaction.

30. (a) : Product due to ozonolysis is malonic acid CH2(COOH)2 which is decarboxylated to CH3COOH on heating. Decarboxylation takes place in : * If two - COOH groups on same C.

f 0 ^

If there is - C - CH,COOH

vp " / keto group at

3-position * If there is ( - C = CCH2COOH), double bond at P,

v p y position.

31. (c): Fehling test is used to detect the reducing and non-reducing sugars. It reduces the glucose but not the sucrose.

32. (b) : CH3CH,CN -> CH3CH2NH2

(one C less)

33. (d) : All the given ions are isoelectronic (each having ten electrons). For such cases, (RH °= 1/Z). Hence, Mg2+ < Na+ < F~ < O2". Z = ( 1 2 ) ( 1 1 ) ( 9 ) ( 8 )

34. (b) : KNO, + CH3COOH HNO,

50 CHEMISTRY TODAY JANUARY '06

HNO, is complexing agent as well as oxidising agent for Co2+.

Co2+ N O , - , N O , -Co3+ ^ [CO(NO,)6]3"

35. (d) : Fe3+ Fe2+

deep yellow light green SnCl2, Zn and H,S all reduce Fe3+ to Fe2+. 36. (b) : CCI,CHO is hydrated since hydrate formed is stable due to intramolecular H-bonding.

CI H \

\ C l - C C — H

/ N o CI H ^ (chloral hydrate)

37. (a) : CI is best leaving group. 38. (a) :

Species

(a) M(OH),

(b) M(OH),

K,,

1 X 10~33 = 27s4

1 x io-32 = As2

1 x IO"30 = s2

Solubility (s)

1x10" 27

1x10" 4

VlxlQ-3" VlxlO"2*

(c) MOH

(d) MOH Most soluble is thus (a).

39. (d) : For dilute solution, relative decrease in V.P.

P°~P = x >h .. >h P n, + n2 n2

n| (moles of solute), /?, (moles of solvent) ii' g solute (mol. wt. = m) is dissolved in solvents (V.P. of pure A = p°A)

PA - P ii'/ M w M . w,/M. M w.

Similarly, P, -P ii' M„ w M H

m n'

B„t =

40. (b) : + Eu +

which gives MA = 2 MH

H Eu

( a complex)

41. (b) : CaH, + 2H,0 -> Ca(OH)2 + 2H,. 42. (b) : CaSiO, + 6HF CaSiF6 + 3H,0.

glass

O O

43. (d) : CH3C CH, > CCH,

f 0 1 V C H 3 C J IS electron withdrawing.

44. (b) : If there is at least one a - H, KMn04 oxidation of alkyl benzene gives benzoic acid. If no a - H, then under vigorous oxidation by KMn04 , ring is cleaved, a - H present :

( O ^ C . L , < Q > - CH2CH,

Mn04 | CH(CH3), ^ I O

a - H absent :

< Q > - C(CH3)3 > (CH3)3C • COOH

45. (a) : /, < I2 « < /3 (very high) After two electrons have been removed, removal of third electron requires very big ionisation potential (1500 eV). Hence M2+ has stable inert gas configuration.

46. (a ) : (EN)={'F) + {EA) (Mullikan scale) 2

47. (c) : By inert pair effect, stability of higher states decreases and that of lower state increases while going down the group in p-block elements (starting from fourth period).

P

48. (b) : P

49. (b) : Ag2S + 4NaCN - 4 2Na[Ag(CN),] + Na,S A

2Na[Ag(CN),] + Zn Na,[Zn(CN)4] + 2Ag B

Rv

H"

/ R , Lindlar 50. ( d ) : / C = C ^ H « H R - C = C - R

Li/NH3 ^ H-R /

C = C trans-

-H

is cis and II is trans.

59 CHEMISTRY TODAY | DECEMBER '05 47

(For Q.No. 1 to 40) Only one option is correct

1. The hybridisation of iodine in iodosobenzene is (a) sp (b) sp2

(c) sp3 (d) sp3d.

2. Which of the following pairs is not isomorphous? (a) BaS04, KMn04 (b) KC104, KBF4

(c) FeS04-7H,0, MgS04-7H,0 (d) NaCIOj, NaNO,.

3. Which of the following is correct order for solubility in water? (a) NaCI04 < KCI04 (b) NaBF4 > NaF (c) MgS04 < SrS04 (d) all of these.

4. Which of the following can be best explained on the basis of hydration of ions? (a) Mg(C104)2 is a powerful water absorbent (b) LiC104 forms a stable hydrate but other alkali metals

perch lorates do not (c) electrical conductivity of Li+ in aqueous solution

is much less than expected (d) all of these.

5. In which of the following N atom is not sp2

hybridised?

(a) HN04 (b) FNOj

(c) NH2 (d) B3N3H6.

6. In which of the following Xe atom is sp3cf hybridised? (a) [XeOh]4~ (b) [XeF3]+

(c) Xe04 (d) XeF2.

7. Which of the following is correct statement? (a) diamond is a good conductor of heat (b) ozonide ion is diamagnetic (c) hydroazoic acid is a linear molecule (d) none of the above.

will be negative marking in these questions)

8. Which of the following orbital overlappings is not possible according to VBT?

(o O C E X D (d) all of these.

9. Which is correct regarding the cyclic trimer of S03? (a) it contains three S - S, o bonds (b) it contains three O - O, o bonds (c) it contains six O - O, ji bonds (d) the total number of a and n bonds in it are 12 and

16 respectively.

10. Which of the following species does not exist? (a) Cl4 (b) [Si04]4-(c) [CO,]4" (d) C2

11. Which of the following is expected to have resultant dipole moment equal to zero? (a) H202 (b) 0 3

(c) S0 2 (d) XeF4.

12. The correct order of sulphur-oxygen bond length is (a) SOF2 > SOCl2 > SOBr, (b) SOCl2 > SOBr2 > SOF2

(c) SOF2 = SOCl2 = SOBr2

(d) SOBr2 > SOCI2 > SOF,.

13. Which is correct statement? (a) the BrPBr bond angle must be nearly 120° in the

molecule PBr2CI3

(b) bond angle in PF3 is more than in NF3

(c) bond angle in PH3 is more than in NH3

(d) none of these.

S8PLE S W . E R f o r

| IIT-JEE 2006

10 Contributed by Deptt. of Chemistry, Resonance, Kota (Rajasthan)

CHEMISTRY TODAY | JANUARY '06

14. Among the fol lowing compounds the one that is polar and has central atom with sp3 hybridisation is (a) H2CO, (b) SiF4

(c) BF, (d) HC102 .

15. Which of the fol lowing is ionic? (a) KHF, (.,) (b) CaC2(.5)

(c) PCI5(.t) (d) all o f these.

16. Dipole moment is higher for (a) CH,F (b) CD3F (c) equal for both (d) nothing can be said.

17. According to structure of H2SO s ,

O ^ 1 1 ^ (H)- O - O - S - O -(H) a II R

O which H will be released first as H+? (a) a (b) 3 (c) both together (d) above acid is not possible.

18. According to VSEPR theory in [I02F2]" ion the F1F bond angle will be nearly (a) 120° (b) 90° (c) 109° 28' (d) 180°.

19. The least stable carbocation intermediate of the fol lowing acid catalysed isomerisation reaction would be

X ' OH

CH = C H - C H ,

H isomerisation

O

CH, - CH,

XZ (a) u ^ " C H - C H 2 - C H 3

(b) [XcH„. ' CH2 - CH - CH3

(c) p j > - CH = CH - CH3

(d) OH

C H , - C H 3

2 0 . The product III of the fol lowing reaction sequence is

[ J c = C H 0) (I) (+2HCOOH)

- a COOH

+ (II)

(ii) (H) P h M g B r ) (Hi)

. ^ ^ C H , - COOH ^ . C O O H

(«) (Q) a » (Qj

(c) CqJ (d) [ Q J

21. The end product of the fol lowing reaction would be

C , H , 0 H / H g S 0 4 / H ® CH3 - C = C - H — — — >

(a) H , C - C H - C H 3

V (b) CH 3 - C H , - O - CH 2 - CH = O

CH3

(c) C H 3 - C H 2 - O - C = CH,

(d) CH 3 - C H = CH - O - CH 2 - CH 3 .

22. Which route is appropriate for the f o l l o w i n g conversion reaction?

0. C H O ^ X H O

N H C 6 H , C 0 3 H NaOH 0 s 0 4 H , 0

(a) 5( 1 )

3 > — ^ r ^ (b) — ^ (2) (I) (2)

, alkaline K M n 0 4 H , 0 ( c ) (i>

Br , NaOH * : (d) ,,, >

(1) (2)

23. Which statement is correct about the fol lowing reaction? C H , - C H = C H 2 NOCKni tmsy l c h l o r i d e ^ ^

tautomerises ^ ^

(a) X and Y do not have stereoisomers (b) X i s a racemic mixture, Y is optically inactive achiral

compound (c) A'is mixture o f two stereoisomers and Kis a mixture

of four stereoisomers

10 CHEMISTRY TODAY | JANUARY '06

I

(d) the electrophile is CI® ion in the above addition reaction.

24. The correct statement about the following reaction is Br

C H , - C = C - C H 3

(1)

Br2/CCl4(leq)^ = 1 _

Br (in Cl Br

C ' 2 / C C S C H 3 - C - C - C H 3 ( [ V )

Br CI dihalide

(III) (excess) (a) but-2-yne is more reactive than but-2-ene towards

bromine solution (b) rate of addition of Cl, to (11) is faster as compared

to but-2-ene (c) the product (III) is a mixture of two enantiomers

Cl I

(d) IV is C H , - C = C - C H 3 + ZnBr,.

Cl

25. Four reagents (I, II. Ill and IV) are added to two compounds R and S. R responded positively to all reagents while S responded negatively to all reagents. The reagents are I. AgN03 + NH4OH (II) Br,/CCI4

III. CH,MgBr (IV) alkaline KMn04. R and S are respectively be (a) CH3 - CH2 - CH, - C = CH and

CH, - CH2 - CH, - CH = CH, (b) CH, = CH - CH : - OH and

CH, - CH, - CH, - CH : - CH3

(c) i ^ l j and ^

(d) CH3 - CH, - CH, - C = CH and CH3 - CH, - O - CH, - CH,.

26. Dehydrohalogenation by a strong base is slowest in

Cl Cl .Cl CII

(a)

(c)

' , Cl Cl Cl (b)

Cl. CI

Cl CI

CI (d) Cl

27. In which case ar alkene is formed on reaction with a strong anionic base?

H

(a)

(c)

(b)

(d)

Cl

28. The product X of the following sequence mainly exists in enolic form. The enol is

C H , = C— CH, NaNH2/NH3 (/) CH3COCl

HgS04/H,S04/H,0 > A'

(a) CH2— C - C H , - C - C H 3

OH ' 0

(b) C H 3 - C = C H - C - C H 3 I II 3

OH 0

(c) CH = CH - CH, - C - CH3

OH O (d) C H 2 = C - C H , - C = C H ,

OH OH

The following questions consist of two statements one labelled Assertion (A) and the other labelled Reason (R). Select the correct answers to these questions from the codes given below. (a) Both A and R are true and R is the correct explanation

of A. (b) Both A and R are true but R is not the correct

explanation of A. (c) A is true but R is false. (d) A is false but R is true.

29. Assertion : Both MeOH/H® and MeOH/Br,/H® give electrophilic addition reaction to R - CH = CH, type of alkene according to Markownikof's rule. The products are an ether and a (3-bromoether respectively. Reason : In both the cases, the electrophile H® ion adds to that side of double bond which has more number of hydrogen atoms.

30. Assertion : Rate of catalytic hydrogenation is faster

CHEMISTRY TODAY JANUARY

but rate of electrophilic addition is slower in ethyne as compared to ethene.

Reason : Due to linear shape of ethyne its rate of adsorption at the surface of metal catalyst is faster than ethene for catalytic hydrogenation but the intermediate vinylic carbocation is less stable than ethyl carbocation in electrophilic addition.

PASSAGE 1 : Read the following passage and answer the questions numbered 31 to 35. They have only one correct option. All the boron trihalides except Bl3 may be prepared by direct reaction between the elements. Boron trihalides consist of trigonal-planar BX3 molecules. Unlike the halides of the other elements in the group they are monomeric in the gas, liquid and solid states, BF3 and BC1, are gases, BBr3 is a volatile liquid and BI3 is a solid. Boron trihalides are Lewis acids because they form Lewis complexes with suitable bases, as in the reaction.

B F 3 W + : N H 3 0 r t - > F 3 B - N H 3 W

However, boron chlorides, bromides and iodides are susceptible (sensitive) to protolysis by mild proton sources such as water, alcohols and even amines for example BCI, undergoes rapid hydrolysis.

BC13 (x) + 3H20 (/) B(OH)3 Uut) + 3HCI <„,„ It is supposed that the first step in the above reaction is the formation of the complex C13B OH, which then eleminates HC1 and reacts further with water.

31. Which of the following is the best order of Lewis acid strength of BF„ BC13 and BBr3? (a) BF3 > BC13 > BBr3 (b) BF3 = BCI3 = BBr3

(c) BF, < BCI3 < BBr3 (d) BBr3 > BF, > BCI,.

32. Which of the following is the correct prediction about observed B - X bond length, in BX3 molecule? (a) B - F bond length in BF3 is found to be less than

theoretical value because the electronegativity values of B (2.04) and F (4.0) suggest the bond to be ionic and hence the attraction between oppositely charged ions must decrease the bond length

(b) BF, and [BF4] have equal B - F bond length (c) the decrease in the B - F bond length in BF3 is due

to delocalised pn - p

n bonding between vacant 2p

orbital of B and filled 2p orbital of F (d) the correct B - X bond length order is

B - F > B - CI > B - Br > B - I.

33. Which is correct about the hydrolysis of BX,? (a) all BX3 undergo hydrolysis to produce B(OH)3 im/)

and HX (H(/)

(b) BF3 does not undergo complete hydrolysis due to formation of HBF4

(c) BBr3 does not undergo hydrolysis at all because it cannot form H-bonds with water

(d) all the above are correct.

34. Which of the following reactions is incorrect? (a) BF3 (x) +. F~ (n</) [BF4]~ (aq)

(b) BC13 + 3EtOH {/) B(OEt), (/) + 3HC1 ((f)

(c) BBr, w + F3BN(CH3)3 m B F 3 + Br,BN(CH3)3 „, (d) BC13 w + 2C5H5N m C13B(C5H5N)2

e x c e s s

35. Which of the following is correct? (a) B(OCH3)3 is much weaker Lewis acid than BBr3

(b) B(OH)3 {aq) behaves as a triprotic acid (c) [H,B03]~ („(/) is a conjugate base of H3B03 (u</)

(d) all of the above.

PASSAGE 2 : Read the following passage and answer the questions numbered 36 to 40. They have only one correct option. In presence of metal catalyst (Ni, Pt, Pd) hydrogen adds to a "^C = C < and - C = C - bonds. The metal catalysts are used in finely divided state in solvent like ethanol. These are classified as heterogenous catalysts. Both H, and alkene get absorbed at the surface of metal. The less crowded alkene adsorbs with faster rate. Both breaking of 7t-bond of alkene and a bond of H, take place at metal surface. Hydrogenation reactions are exothermic in nature. If palladium is precipitated at the surface of calcium carbonate and it is treated with quinoline it becomes deactivated for adsorption of alkenes. But it can effectively catalyse hydrogenation of - C = C - . The alkyne can adsorb at the surface of poisoned metal catalyst and syn addition of hydrogen atoms take place. The relative rate of hydrogenation follows the order:

- C = C - > / C = C < > / C = 0 > C6H(, If sodium or lithium metals dissolved in liquid ammonia are used with alkynes then the metal transfers its electrons to7i-frameworkof-C = C-bond and an anion radical is formed, Liquid ammonia provides H atom and a vinylic radical is formed. The process is repeated and an anti addition of H atoms occurs at - C = C - bond. A

CHEMISTRY TODAY | FEBRUARY '06 160

vinyl carbanion / C = C ^ is more stable than

^ C — C — .So only alkynes (nonterminal) are reduced

by this method and not alkenes (nonterminal).

36. Which of the following involves homogeneous reduction?

HJNi. (a) CH, = CH, C H 3 - CLI,

(b) C H 3 - C E H C - C H 3

CH, - CH— CH - CH, + NaNH,

H 2 /Pd/CaC0 3 /qu inol ine > CHjCHO ( c ) C H 3 C O C I

H,/Pd/BaSO,/quinoline (d) CH,C = CH — 2 > C H 3 C H = C H 2

37. The relative rate of catalytic hydrogenation of following alkenes is

C X N_ W (I) (II) (III) (IV)

(a) I > IV > 111 > II (b) II > III > IV > I (c) HI > IV > I > II (d) II > IV > 1 > III.

38. The product of following reaction is O

J]

OH

Pd-CaC0 3 -quinol ine

OH

(a) h (b)

( 0 ( V ^ W

0

JJ I 39. The product of following reaction is

/ (CH 2) , \ / C H , o Na, NH, „

n - C4H, (CH, ) / (a) a mixture of four stereoisomers (b) a mixture of cis, cis and trans, trans isomers (two

stereoisomers) (c) a mixture of cis, trans and trans, cis isomers (two

stereoisomers) (d) a single stereoisomer (cis, trans).

40. In which case the reaction is most exothermic with H2/Ni?

(a)

(c)

(b)

(d)

One or more than one options may be correct (for Q. 41 to 55)

41. In which of the following pairs hybridisation of the central atom is same? (a) C1F3, C1F30 (b) C1F30, CIF302

(c) [C1F20]+, [ClF 4 0r (d) [C1F40]-, [XeOF4],

42. There are three elements P, O and R which belong to p-block of the periodic table they all form trifluorides with F2, such that PF-, is Lewis acid but 0F3 is a weaker Lewis base (dipole moment = 0.23 D). These two compounds react with each other in presence of F2 to produce C?F4

+ PF4_. The compound ^F3 is a T-shape in

interhalogen molecule. Which of the following is/are correct statements with reference to above informations? (a) all the R- F bond lengths are equal in RF3 molecule

F

(b) in the structure the angle 0*180°

(c) the O and P both are sp3 hybridised in the salt OF4

+PF4" (d) the elements P, O and R can be B, N and Cl

respectively. 43. On the basis of structure of graphite, which of the following is/are true for it? (a) it is a diamagnetic substance (b) it behaves like metallic as well as semiconductor (c) it is less dense than diamond (d) it reacts with F2 to form product which is more

conducting than graphite.

44. Which of the following is correct about B2H6? (a) each B atom is sp3 hybridised (b) it consists of two - "3 centre 2 electron bonds" (c) the two bridging H-atoms are in the plane of the

molecules (d) all of the above.

1 0 CHEMISTRY TODAY | JANUARY '06

45. Given that interionic distance in Na+F~ crystal is 2.31 A , which of the following predictions will be wrong? (a) rNa+/rF- = 0.7 (b) coordination number ofNa+ = coordination number of F " = 6 (c) Na+F must have rock salt type crystal structure (d) effective nuclear charge for Na+ and F~ are equal.

46. Which of the following is/are examples of banana bond? (a) A12CI6 (b) A12(CH3)6

(c) B2H6 (d) I2C16.

47. In which of the following species bond angle is expected to be more than 120°? (a) PC14

+ (b) NOf (c) N0 2

+ (d) XeF2.

48. Which of the following has/have linear shape? (a) HCN ' (b) (CN)2

(c) C 30 2 (d) CO,.

49. Which is/are wrong about P4O l0 molecule? (a) each P atom can be considered to be sp3 hybridised (b) there are four P - P bonds in the molecule (c) there are two types of P - O bond lengths (d) P O P angle is 180°.

50. Alkenes X and Y on hydration with dilute H2S04

give same alcohol Z. Reductive ozonolysis of X gives P and Q, where only Q gives silver mirror test positively. Ozonolysis of Y under similar conditions gives only R, which is a dicarbonyl compound and gives positive Tollen's test as well as iodoform test. The product Wof catalytic hydrogenation of X and )' is same which can also be obtained from reduction of all of the following compounds (1 - 5) by Zn-Cu/C,H5OH.

Br

Br xy B,xy & o cr* (V) (2) (3) (4) (5)

Which ofthe following is/are correct informations about the above reactions?

O (a) R is

O (b) X and )' are geometrical isomers

(c) X Br

(d) Z - a OH.

51. Which of the following would be the significant product/s of the given reaction?

H —C==C-CH-CH ==CH2 H S ( l r a c e S ) >

I OH

(a) C H 2 = C H - C - C H = CH,

O (b) CH2 = CH - CH = CH - CHO

(c) H - C = C - C - C H , - C H 3

O (d) CH, - CH = CH - C = CH

OH

52. But-l-ene is formed in reaction/s 0

heat (500°C).

(1) CH,CO,H N 1 — | (2) A (heat)

(C)

(d)

OH

OH

53. About oleic acid, the correct statement/s is/are COOH

H H (oleic acid)

(a) with Br,/CCl4 oleic acid produces a mixture of two optically active stereoisomers

(b) on hydration an optically inactive alcohol derivative is formed (meso isomer)

(c) oleic acid — 1 — " 3 >

V -O

(iodolactone)

10 CHEMISTRY TODAY | JANUARY '06

(d) one of its reductive ozonolysis product liberates C0 2 with NaHC03 .

54. In which of the following reaction/s alcohol will be formed as the final product?

(a) (CH3CH2CH, -)3B

(b) CHo — CH — CH,

\ / Hg .

+2

(CH3COO")2 H , 0 N a B H , - >

O,, A H,0 (c) CH3 - CH = CHMgBr -

V 7 THF (d) CHj - CH2-CH2MgBr ——^ >

55. When neopentyl alcohol is treated with H2S04 a mixture of two alkenes (85 : 15) is formed. Which statement/s is/are correct about these alkenes? (a) both give same majour product with HBr (b) both give different products (major) with HBr/R202/

light (c) the alkene which is formed in 85% concentration

has higher heat of hydrogenation than the other

one obtained in 15% concentration (d) both give same product on ozonolysis.

ANSWERS

1. (c) 2. (d) 3. (b) 4. (d) 5. (c) 6. (a) 7. (a) 8. (d) 9. (d) 10. (c) 11. (d) 12. (a) 13. (a) 14. (d) 15. (d) 16. (b) 17. (b) 18. (d) 19. (c) 20. (b) 21. (c) 22. (b) 23. (c) 24. (d) 25. (d) 26. (c) 27, (b) 28. (b) 29. (c) 30. (a) 31. (c) 32. (c) 33. (b) 34. (d) 35. (a) 36. (b) 37. (b) 38. (c) 39. (d) 40. (b) 41. (a, b, d) 42. (b, c, d) 43. (a, b, c) 44. (a, b) 45. (d) 46. (b, c) 47. (c, d) 48. (a, b, c, d) 49. (b, d) 50. (a, c, d) 51. (b, d) 52. (a, b, c, d) 53. (a, c, d) 54. (a, b, d) 55. (a, b)

Note : For detailed solutions please log on to our website www.resonance.ac.in

Can We Drink Too Much Water? Q. Can w e Real ly Drink Too Much Water? In a word, yes. Drinking too much water can lead to a condition known as water intoxication and to a related problem resulting from the dilution of sodium in the body, hyponatremia. Water intoxication is most commonly seen in infants under six months of age and sometimes in athletes. A baby can get water intoxication as a result of drinking several bottles of water a day or from drinking infant formula that has been diluted too much. Water intoxication and hyponatremia result when a dehydrated person drinks too much water without the accompanying electrolytes. Q. What H a p p e n s Dur ing Water Intoxication? When too much water enters the body's cells, the tissues swell with the excess fluid. Our cells maintain a specific concentration gradient, so excess water outside the cells (the serum) draws sodium from within the cells out into the serum in an attempt to re-establish the necessary concentration. As more water accumulates, the serum sodium concentration drops — a-condition known as hyponatremia. The other way cells try to regain the electrolyte balance is for water outside the cells to rush into the cells via osmosis. Although electrolytes are more concentrated inside the cells than outside, the water outside the cells is 'more concentrated' or 'less dilute' since it contains fewer electrolytes. Both electrolytes and water move across the cell membrane in an effort to balance concentration. From the cell's point of view, water intoxication produces the same effects as would result from drowning in fresh water. Electrolyte imbalance and tissue swelling can cause an irregular heartbeat, allow fluid to enter the lungs, and may cause fluttering eyelids. Swelling puts pressure on the brain and nerves, which can cause behaviours resembling alcohol intoxication. Swelling of brain tissues can cause seizures, coma and ultimately death unless water intake is restricted and a hypertonic saline (salt) solution is administered. Q. It's N o t H o w Much w e Drink, It's H o w Fast w e Drink It! The kidneys of a healthy adult can process fifteen liters of water a day. We are unlikely to suffer from water intoxication, even if we drink a lot of water, as long as we drink over time as opposed to intaking an enormous volume at one time. As a general guideline, most adults need about three quarts of fluid each day. Much of that water comes from food, so 8-12 eight ounce glasses a day is a common recommended intake. We may need more water if the weather is very warm or very dry, if we are exercising, or if we are taking certain medications.

-10 CHEMISTRY TODAY | FEBRUARY '06 163

Practice Paper for ¥ E x a m o n

West Bengal J EE 2006 > 2 2 n d a n d 23 r d

- L A p r i l 2 0 0 6 ,

1. The compound which does not react with CH3COCI is (a) RNH2 (b) R2NH (c) R3N (d) all of these.

2. Which of the following is the strongest acid? (a) CHjCOOH (b) HCOOH (c) ClCH2COOH (d) CI2CHCOOH.

3. Carboxylic acids do not give the characteristic reactions of C = O group because of (a) polar nature (b) resonance (c) symmetrical structure (d) attached alkyl group.

4. A Idol contains (a) - CHOH and COOH groups (b) - CHO and - CH2OH groups (c) - CHO and - CHO groups (d) - CHOH and C = O groups.

5. A ketone that does not form a cyanohydrin is (a) C2H5COCH3 (b) CH3COCH3 (c) C6H5COC6H5 (d) CH3COC6H5.

6. Which is true about ionic bonds? (a) directional nature (b) isomerism (c) high melting and boiling points (d) non crystalline nature.

7. Which is not true in case of an ionic bond? (a) it is a linear bond (b) no bond is 100% ionic (c) it is formed between two atoms with large

electronegativity difference (d) its formation is favoured by low charge cation and

anion.

8. The element with atomic number 20 will most likely combine chemically with the element whose atomic number is (a) 14 (b) 10 (c) 11 (d) 16.

9. pi bond (a) increases bond length (b) decreases bond length (c) distorts the geometry of molecule (d) makes homoatomic molecules more reactive.

10. The bond formed in fluorine molecule is due to (a) s-s overlapping (b) s-p overlapping (c) p-p overlapping (d) hybridisation of orbitals.

11. The mixture of ethanol and sulphuric acid is heated in a closed flask at 1 4 0 ° C . The flask would contain (a) H 2 S 0 4 and C 2 H 5 O C 2 H 5 only (b) H 2 0 , H 2 S 0 4 and C 2 H 5 H S 0 4 only (c) H 2 0 , C 2 H S H S 0 4 , C 2 H 5 O C 2 H 5 and H 2 S 0 4 only ( d ) H 2 0 , C 2 H S O C 2 H 5 , C 2 H 5 H S 0 4 a n d C 2 H 5 O H .

12. (CH3)2CHC1 + Nal acetone (CH3)2CHI + NaCl The above reaction is known as (a) Perkin's reaction (b) Finkelstein reaction (c) Stephan's reaction (d) Sabatier and Senderson's reaction.

13. The reaction : CH3Br + OH"" CH3OH + B r is (a) SN1 (b) S n 2 (c) S E 1 (d) Se2.

14. An alkyl halide reacts with metallic sodium in dry ether to form an alkane. The reaction is known as (a) Frankland's reaction (b) Sandmeyer's reaction (c) Wurtz reaction (d) Wurtz-Fitting reaction.

15. SN1 displacement is rapid for (a) 1-bromopentane (b) 2-bromopentane (c) 2-bromo-2-methylbutane (d) all have equal rates.

16. On passing steam over silicon, we get (a) (c)

SiOz + H2

SiO, + N , (b) (d)

Si0 2 + H 2 0 SiO.

17. Fluorine can be freed from HF by passing the mixture through

164 CHEMISTRY TODAY JANUARY '06

(a) an alkaline solution (b) conc. H2S04

(c) H 2 0 (d) NaF.

18. Fluorine does not show variable oxidation state because of (a) its small atomic size (b) its high electronegativity (c) non-availability of (/-orbital (d) low dissociation energy of F - F bond.

19. When calcium fluoride is heated in a glass tube, the vapours come out from glass tube are (a) H2SiF6 (b) Si02

(c) HF (d) Na,SiF6.

20. Fluorine is a stronger oxidising agent than chlorine 'In aqueous solution. This is attributed to many factors except (a) heat of dissociation (b) electron affinity (c) ionisation potential . (d) heat of hydration.

21. When heat is evolved in a reaction, it is called (a) an endothermic reaction (b) an exothermic reaction (c) a redox reaction (d) none of these.

22. Heat of a reaction does not depend on (a) temperature of the reaction (b) the path by which final product is obtained (c) physical states of the reactants and products (d) whether the reaction is carried out at constant pressure

or at constant volume.

23. The relationship between enthalpy change (AH) and internal energy change (AE) for a system is given by (a) AH = AE + PAV (b) AH = AE - PAV (c) AH = AE + PV (d) AH = AE + RT.

24. The heat of neutralisation of a weak acid is than that of a strong acid. (a) less than (b) more than (c) equal to (d) none of these.

25. The heat change in a chemical reaction at constant pressure is (a) AH (b) AE (c) AT (d) E.

26. A nuclear reaction must be balanced in terms of (a) mass (b) energy (c) number of electrons (d) mass and energy.

27. If the quantity of a radioactive element is doubled, the rate of disintegration (a) is halved (b) is doubled (c) slightly increases (d) remains unaffected.

28. In the third period from left to right (a) reducing property decreases (b) reducing property increases (c) oxidising property increases (d) oxidising property remains same.

29. The first ionisation potential of Si, P and S are in the order (a) Si < P < S (b) Si > P > S (c) Si < P > S (d) Si > P < S.

30. Which of the following is crystal lattice of aluminium? (a) body centered cubic (b) face centered cubic (c) hexagonal closely packed (d) none of these.

31. Electrode potential of standard hydrogen electrode is (a) zero (b) 2 V (c) 0 . 3 V (d) 1.6 V.

32. The time required in hours for a current of 3 ampere to decompose electrolytically 18 g of HzO is (a) 9 (b) 18 (c) 36 (d) 54.

33. IO-2 g atom of Ag can be oxidised to Ag+ in the electrolysis of AgN03 solution using silver electrode by (a) 965 coulomb (b) 96500 coulombs (c) 9650 coulombs (d) 96.5 coulombs.

34. In a Galvanic cell, energy changes are (a) chemical energy —» electrical energy (b) electrical energy —> chemical energy (c) chemical energy —» internal energy (d) internal energy —> electrical energy.

35. Silver from silver nitrate is deposited by copper because ( a ) £ ° ( C u 2 + . C u , < £ 0 ( A g + , A B )

(b) £°(ab+, Ag) < (Cu2+ Cu)

( c ) £ ° ( C u 2 + , Cu) = (Ag+, Ag)

(d) none of these.

-10 CHEMISTRY TODAY | FEBRUARY '06 13

36. If 92U235 decay only by emitting two a - and one (3-particles, the possible product of decay is (a) 89Ac224 (b) 89Ac225

(c) 89AC226 (d) 89 AC"27.

37. The elements of atomic number larger than 92 show (a) artificial radioactivity (b) natural radioactivity (c) artificial as well as natural radioactivity (d) no radioactivity.

38. In the colloidal state, particle size ranges from (a) 1 to 10 A (b) 10 - 20 A (c) 10 - 1000 A (d) 100 - 280 A. 39. Which of the following is correct? (a) Langmuir adsorption is highly specific (b) van der Waal's adsorption is reversible (c) both (a) and (b) are exothermic (d) all are correct.

40. Which one of the following is an example of adsorption? (a) anhydrous calcium chloride with water vapours (b) silica gel in contact with water vapours (c) cotton clothes dipped in dye solution (d) ammonia gas in contact with water.

41. Which one of the following represents standard hydrogen electrode correctly? (a) Pt, H2 (1 atm) | H+ (1 M), 298 K (b) Pt, H2 (1 atm) | H+ (1 M), 273 K (c) Pt, H 2 0 (0.1 atm) | H+ (1 M), 273 K (d) Pt, H2(0.1 atm) | H+ (0.1 M), 273 K.

42. Choose the incorrect relation. pH + pOH

(a) 14 : 1

(c) pOH = 14 - pH

(b) pH + pOH = 14

(d) pH = 14 + pOH.

43. An aqueous solution of HC1 is IO"9 M HC1. The pH of the solution should be (a) 9 (b) between 6 and 7 (c) 7 (d) unpredictable.

44. An acid solution of pH six is diluted hundred times. The pH of the resulting solution becomes (a) 6.95 (b) 6 (c) 4 (d) 9.

45. A1C1, + CI- AlCLf. In this reaction, A1C13 can be classed as

(a) acid (c) salt

(b) base (d) none of these.

46. The solubility product of Ag2S is given by the expression (a) K,„ = [Ag']2 [S2 ]2 (b) KS„ = [Ag ] [S2"] (c) KSP = [Ag :]2 [S2 ]1 (d) KS„ = [Ag+] |S2 ]2.

47. Which of the following combinations will constitute buffer solution? (a) FeS04/H2S04

(b) CH3COOH/CH3COONH4

(c) CH3COOH/CH3COONa (d) KCI/KOH.

48. When 1.0 ml of dilute HCI is added to 100 ml solution of pH 4, the pH of the solution (a) becomes 7.0 (b) does not change (c) becomes 2.0 (d) becomes 10.0

49. To which of the following, Ostwald's dilution law is applicable? (a) CH 3 COOH (b) HCI (c) H N 0 3 (d) H 2SO 4 .

50. Which one of the following is a Bronsted acid but not a Bronsted base? (a) H2S (b) H , 0

HC03" (d) NH3. (c)

51. In the reaction,

C , H 5 N H , NaNO, /HCl

.4 HCI K.CN. > C the final product is (a) propane nitrile (c) propyl amine

(b) ethane nitrile (d) formo nitrile.

52. Propanoic acid on warming with Cl2 in presence of red P gives (a) CH3CH2COC! (b) CH3CH2CH,C1 (c) CH3CHClCOOH (d) CH2ClCH2COOH.

53. A metamer of a 2-pentanone is (a) 3-pentanone (b) 2-pentanol (c) 1-pentanal (d) 2,2-dimethyl propanal.

54. The difference between aldol condensation and Cannizzaro reaction is that (a) the former takes place in the presence of ct-H atom (b) the former takes place in the absence of a-H atom (c) the latter takes place in the absence of a-H atom (d) none of these.

10 CHEMISTRY TODAY | JANUARY '06

55. Base catalysed aldol condensation occurs with (a) propionaldehyde (b) 2-methyl propionaldehyde (c) both of the^bove » (d) none of these.

56. Element X is strongly electropositive and element )' is strongly electronegative. Both are univalent. The compound formed would be (a) )'" (b) X V+

(c) X-Y (d) X—> Y.

57. Of the following solvents, the one most likely to dissolve ionic compounds is (a) carbon tetrachloride (b) methanol (c) water (d) butyl ether.

58. The maximum covalency is generally equal to (a) the number of unpaired s-electrons (b) the number of paired p-electrons (c) the number of unpaired s- and ^-electrons (d) the actual number o f s and /^-electrons in the valence

shell.

59. pi bond is formed by (a) the overlapping of atomic orbitals on the axis of

nuclei (b) by mutual sharing of p electrons (c) by sidewise overlapping of parallel oriented half-

filled p-orbitals (d) overlapping of s-orbital with p-orbitals.

60. Noble gases do not react with other elements because they (a) are monoatomic (b) are not found in abundance (c) have very small size of their atoms (d) have completely paired up and stable electron shells.

61. When diethyl ether is treated with chlorine in dark, it forms (a) per chlorodiethyl ether (b) ethyl chloride (c) acetyl chloride (d) 1,1-dichloroether.

62. Hansdiecker reaction is the (a) reaction of alcohol with phosphorus tri-iodide (b) reaction of alkyl halide with sodium metal in the

presence of dry ether (c) formation of alkyl bromide containing one carbon

atom less from silver salt of carboxylic acids (d) reaction of alcohol with thionyl chloride.

63. Among isomeric halides, the boiling point is more for (a) CH3(CH2)3C1 (b) (CH3)2CHCH2C1 (c) C(CH3)3C1 (d) none of these.

64. Formation of an alkane by the action of Zn on alkyl halide is called (a) Wurtz reaction (b) Kolbe's reaction (c) Ullman's reaction (d) Frankland's reaction.

65. Substitution of chlorine takes place at high temperature for (a) CH3 - CH = CH2 (b) H2C = CH, (c) HC = CH (d) none of these.

66. Heat of neutralisation of a strong base with a strong acid is (a) - 1 3 . 7 kcals (b) +13.7 kcals (c) -12.7 kcals (d) none of these.

67. H2 + Cl2 -> 2HC1; AH = -44 kcal mol"1. In this reaction heat of formation of HCl in kcal is (a) - 44.0 kcal mol"1 (b) +44.0 kcal mol'1

(c) -22.0 kcal mol'1 (d) +22.0 kcal mol-1.

68. By convention the enthalpy of an element in its standard state is assumed to be (a) 1 kcal mol"1 (b) 100 kcal mol-' (c) 0 kcal mol-1 (d) 250 kcal m o K

69. Enthalpy of a reaction is given as (a) H = E + PV (b) H = E - PV (c) AH=AE + PAV (d) AH = AE - PAV.

70. In an endothermic reaction (a) heat is converted into electricity (b) heat is absorbed (c) heat is evolved (d) heat is converted into mechanical work.

71. Quantity of electricity is measured in (a) ampere sec (b) ampere (c) ampere sec"1 (d) ampere-1 sec.

72. Time taken to deposit 100 g of aluminium from an electrolytic cell containing A1203 at a current of 125 ampere is (a) 1.2 hr (b) 1.8 hr (c) 2.4 hr (d) 3.2 hr.

73. Passing a current through molten aluminium chloride for some time produced 11.2 litres of Cl2 at NTP at anode. The quantity of aluminium deposited at cathode is

-10 CHEMISTRY TODAY | FEBRUARY '06 13

(a) 29 g (c) 9 g

(b) 18 g (d) 4.5 g.

74. The current strength in amperes required to liberate 10 g of bromine from KBr solution in half an hour is (a) 6.7 (b) 7.6 (c) 5.7 (d) 4.7

75. In a galvanic cell, a salt bridge is used to (a) complete the circuit (b) reduce electric resistance in the cell (c) attain equilibrium (d) carry salts for reaction occurring in cell.

76. Moderate electrical conductivity is shown by (a) silicon (b) graphite (c) diamond (d) carbon.

77. The purpose of addition of KF to HF in the preparation of F2 by electrolysis is (a) to decrease the conductivity of HF (b) to increase the conductivity of HF (c) to increase the concentration of F2

(d) none of these.

78. Fluorine reacts with strong NaOH to give (a) 0 2 (b) OF2

(c) H2 (d) 0 3 .

79. Which is the strongest reducing agent? (a) HF (b) HCI (c) HBr (d) HI.

80. Which of the following is a false statement? (a) Halogens are strong oxidising agents. (b) Halogens show only - 1 oxidation state. (c) HF molecules form inter halogen molecular H-bonds. (d) Fluorine is highly reactive.

81. Which of the following represents acid-base pair of the equilibrium given below?

HCI + 2H20 H,0 + + Cl" (a) HCI, H ,0 +

(c) C1-, H 2 0 (b) HCI, Cl-(d) HCI, H20.

82. The pH of a solution is 5.0. If its hydrogen ion concentration is decreased 100 times, the solution will be (a) more acidic (b) more basic (c) neutral (d) of the same acidic strength.

83. A pH of 7 signifies

54

(a) pure water (c) basic solution

(b) neutral water (d) acidic solution.

84. pH of human blood is (a) 7.9 (b) 6.8 (c) 7.1 (d) none of these.

85. The standard electrode potentials of Ag, Cu and Zn are 0.8 V, 0.34 V and -0.76 V respectively. Which one of the following can be stored? (a) CuS04 in zinc vessel (b) AgN03 in copper vessel (c) Cu(N03)2 in silver vessel (d) AgN03 in zinc vessel.

86. The activity of radioisotopes changes with (a) temperature (b) pressure (c) chemical environment (d) none of these.

87. A wooden article and freshly cut down tree are 7.6 and 15.2 min"1 g~' of carbon ( t m = 5760 years) respectively. The age of the article is

(a) 5760 years (b) 5760 x f — U 5 / 2 years

( 15 2 (c) 5760x1 - j - ^ | years (d) none of these.

88. As we move from Na to CI along third period (a) metallic character increases (b) non-metallic character decreases (c) metallic character decreases (d) metalloid character remains same.

89. Ionisation potential of P is higher than S because (a) P is more electronegative than S (b) the 3p orbital is more than that of S (c) the 3p orbital is half filled in P while it is partially

filled in S (d) none of these.

90. The structure of magnesium metal is (a) body centred cubic (b) face centred cubic (c) hexagonal (d) tetrahedral.

91. Which of the following particles is emitted in the nuclear reaction?

13A127 + 2He" ,5P30 + (a) o ( b ) (c) ,H> (d) |H3.

-10 CHEMISTRY TODAY | FEBRUARY '06 168

92. Lighter elements such as Li, B, C, O etc. show (a) artificial radioactivity only (b) natural radioactivity (c) artificial as well as natural radioactivity (d) no radioactivity.

93. Tyndall effect in colloidal system is due to (a) scattering of light (b) reflection of light (c) absorption of light (d) presence of electrically charged particles.

94. In adsorption to Freundlich adsorption isotherm, the adsorption at low pressure is proportional to (a) P (b) P-]

(c) Pm (d) P" (where n = 0 to 1)

95. Which of the following is true about physical adsorption? (a) it is reversible (b) decreases with temperature (c) it is exothermic (d) all of these.

96. The solubility product of As2S3 is given by the expression (a) Ksp = [As3+] [S2 ] (b) Kxp = [As3+]2 [S213

(c) Ksp = [As3+]' [S2]1 (d) Ksp = [As3+]3 [S2-]2.

97. Let solubilities of AgCl in H20, 0.01 M CaCl2, 0.01 M NaCl and 0.05 M AgN03 be Su S2, S3 and SA

respectively. What is the correct relationship between these quantities? (a) Si > S2 > S3 > S4 (b) > S2 = S3 > S4

(c) S, > S3 > S2 > S4 (d) S4 > S2 > S3 > Sj.

98. A solution which resists the change in its pH value on addition of some amount of an acid or a base is called

(a) isochoric solution (b) amphoteric solution (c) buffer solution (d) neutral solution.

99. The value of Ksp can be (a) negative (b) positive (c) negative or positive (d) is always a whole number.

100. Which of the following is a Lewis acid? (a) H 2 0 (b) SnCl4

(c) C2H5OH (d) Cl".

ANSWERS

1. (c) 2. (d) 3. (b) 4. (d) 5. (c) 6. (c) 7. (a) 8. (d) 9. (b) 10. (c) 11. (c) 12. (b) 13. (b) 14. (c) 15. (c) 16. (a) 17. (d) 18. (c) 19. (a) 20. (C) 21. (b) 22. (b) . 23. (a) 24. (a) 25. (a) 26. (d) 27. (b) 28. (b) 29. (c) 30. (b) 31. (a) 32. (b) 33. (a) 34. (a) 35. (a) 36. (d) 37. (c) 38. (c) 39. (d) 40. (b) 41. (a) 42. (d) 43. (b) 44. (a) 45. (a) 46. (c) 47. (c) 48. (b) 49. (a) 50. (a) 51. (a) 52. (c) 53. (a) 54. (a) 55. (c) 56. (a) 57. (c) 58. (d) 59. (c) 60. (d) 61. (d) 62. (c) 63. (a) 64. (d) 65. (a) 66. (a) 67. (c) 68. (c) 69. (a) 70. (b) 71. (a) 72. (c) 73. (c) 74. (a) 75. (a) 76. (b) 77. (b) 78. (a) 79. (d) 80. (b) 81. (b) 82. (b) 83. (b) 84. (d) 85. (c) 86. (d) 87. (a) 88. (c) 89. (c) 90. (c) 91. (a) 92. (a) 93. (a) 94. (a) 95. (d) 96. (b) 97. (c) 98. (c) 99. (b) 100. (b)

E S S Complete Book for

WB JEE'06

CHEMISTRY TODAY | JANUARY '06 13

The nanoscience of MMHNMMI RSW BMMI

KAJAL Ask the millions of women who routinely ring

their eyes with kajal about nanotechnology, and chances are you would receive a vacant

stare in return. For eons now, Indian women have adorned their eyes with the sooty cosmetic little knowing that one day it would excite the world of molecular science with its simplicity. Just the other day, the Indian Institute of Technology, Kanpur, announced that kajal constituted carbon nanotubes (CNT). So? Now consider this : till now the generation of CNT was an expensive affair in which an organic compound like benzene was vapourised and CNT extracted from the deposition of the smoke. The equipment used in the process alone costs over Rs. 10 lakh. And how much does it costs to burn oil in an ordinary lamp and catch the soot on a plate - the way kajal is made at homes all over India? Nano derives from the Greek nanos, or a dwarf. In actual measurement, a nanometre is a billionth of a metre, nanotechnology, therefore, dealing with particles smaller than in microtechnology. Nanotech, in its basic sense, is using molecular building blocks in an array of applications, from medicine and manufacturing to biotech and optics. Unique among elements, carbon can bond to form sheets that can be rolled into tubular fibres, known to be among the strongest fibres known, 10 to 100 times stronger than steel by weight. Sabyasachi Sarkar, a chemistry professor at IIT-K, was working on the production of fullerenes, another ball-shaped, sub-micro structure, in 1992, when he detected that waste insoluble carbon soot seemed to have a substantial amount of CNT. Based on this finding, Sarkar tried to produce CNT from the traditional oil-burning lamp in which pure carbon, or kajal is produced. After his initial studies on generation of CNT, Sarkar was awarded a Rs. 32 lakh project by the Indian Government's Department of Science and Technology (DST) in September 1994. He said his focus, as of scientists elsewhere, was on developing a cost-effective method to generate CNT and finding a way to make CNT water soluble for medical use.

He chuckles about how blind we have been over the years. Even the Ramayana and Mahabharata mentioned the kajal. "Traditionally, the grandmother applies fresh kajal to the six-day-old baby to prevent eye-ailments", he added. No one knew this pure carbon could be a source of much-needed carbon nanotubes. As for the other eyeliner, the surma, Sarkar says it is not pure carbon like kajal but emanates from lead sulphide and is of Arabic origin. Sarkar was able to extract up to 40 percent of the kajal as CNT. More important, Sarkar not only produced cheap CNT through a traditional method but also managed to make it water soluble. He did this by treating CNT with nitric acid, which incorporated some hydrophilic groups and thus made it soluble. CNT has multifarious uses. "The dry-state uses include manufacturing of picture tube for display devices, which would be much cheaper and less power consuming", says Sarkar. He also envisages CNT replacing optic fibres used in telecommunication since it minimises distortion of sound and disturbances in the line. Other uses of CNT are in manufacturing of reusable packaging material and in electronics. Wet-use CNT can be applied in drug delivery. Sarkar said, for instance, that if drug for cancer in a specific body part was to be delivered without affecting the other parts, the magnetic property of CNT was extremely helpful. "The drug is placed in a CNT and injected into the blood. It is then dragged to the affected part with magnetic material and then blasted using electro-magnetic radiation," he explained. Despite the obvious fallout of his discovery, Sarkar has not patented the process. "I do science, not business," he said. He suggests the production of kajal should be promoted as a cottage industry that would generate employment for thousands of educated, unemployed people.

Courtesy : I n d i a n E x p r e s s

10 CHEMISTRY TODAY | JANUARY '06

A I I M S ( C B S E ) ( D P M T ) ( A F M C ~ ] ( V M M C ) (EAMCET) (WB-JEE) ( B H U ) ( C M C )

2006 Medical Entrance Exam ( U P C P M T J

(CET Karnataka] Practice Test Paper (Manipa l PMT)

( J I P M E R

(MP PMT] ( M G I M S ) (PMDT B i h a r ) (PMT Haryana ) ( K e r a l a P M T ) ( R a j . P M T ) T N P C E E )

1. Which of the following is slag? (a) Si02 (b) CaSi03

(c) CaO (d) Ca(0H)2.

2. Which one of the following is an important ore of copper? (a) malachite (b) calamine (c) galena (d) dolomite.

3. Which one of the following metals corrodes readily in moist air? (a) gold (b) silver (c) nickel (d) iron.

4. In the extraction of Fe from haematite ore, CaC03

acts as (a) flux (b) slag (c) lining material (d) none of these.

5. The froth floatation process is used for the concentration of (a) oxide ores (b) sulphide ores (c) minerals (d) slag.

6. An oxide of nitrogen has a molecular weight 30. The total number of electrons in one molecule of the compound (at. no. N = 7, O = 8) is (a) 14 (b) 15 (c) 7 (d) 16.

7. If n = 3, / = 0, m (for outermost shell) = 0 then the probable atomic number may be (a) 10, 11 (b) 11, 12 (c) 12, 13 (d) 13, 14.

8. Pick out the isoelectronic species from the following. I. CH3

+ II. H3O+

III. NH, (a) I and II (c) I and III

IV. CH 3 \ (b) III and IV (d) II, III and IV.

9. In which of the following one electron system is the radius of the first Bohr orbit minimum? (a) hydrogen-atom (H) (b) deuterium ion (D+) (c) singly ionized helium (He+) (d) doubly ionized lithium (Li2+).

10. The energy of the electron of the H-atom orbiting in a stationary orbit of radius rn is proportional to (a) r„ (b) r2 (c) 1 !rn (d) 1 !rn\

11. Cyanide process is used to obtain (a) Cu (b) Ag (c) Zn (d) Ni.

12. Which of the following is ferrous alloy? (a) invar (b) solder (c) magnalium (d) type-metal.

13. Correct formula of potassium ferricyanide is (a) K4[Fe(CN)]6 (b) K3[Fe(CN)6] (c) KFe(CN)s (d) K2Fe(CN)4.

14. Sea-weed is important source of (a) iron (b) chlorine (c) iodine (d) bromine.

15. The acid employed for etching glass is (a) H2S04 (b) HC103

(c) HF (d) aqua regia.

16. The size of the species given below increases in the order of (a) Mg 2 +< N a + < F" < Al3+

(b) F" < AI3+ < Na+ < Mg2+

(c) Al3+ < Mg2+ < Na+ < F (d) Na+ < Al3+ < F" < Mg2+.

17. Which of the following has largest radius? (a) Mg2+ (b) Na+ (c) 02~ (d) F".

18. Which of the following represents the electronic configuration of J-block elements? (a) (« - 1 )s2 d> ' 10 (b) (k - 1 )d]-10 ns2

(c) (n - l)dl- 10 ns2 np4 (d) (n - ])p4 ns2.

19. If the speed of the electron in the first Bohr orbit of the hydrogen atom be x, then the speed of the electron in the third Bohr orbit is (a) x/9 (b) x/3 (c) 3x (d) 9x.

CHEMISTRY TODAY | JANUARY '06 13

20. The ratio of energy of a photon with 2000 A wavelength to that of a photon with 4000 A wavelength is (a) 1 : 2 (b) 1 : 4 (c) 2 : 1 (d) 4 : 1.

21. Cl2 gas is dried over (a) CaO (b) NaOH (c) K.OH (d) conc. H,S04 .

22. Iodine deficiency in diet causes (a) beri-beri (b) goitre (c) rickets (d) night blindness.

23. There is no S - S bond in (a) S203

2 (b) S2042"

(c) S2Os2" (d) S207

2".

24. Oxalic acid when heated with conc. H2S04 gives out (a) H 2 0 and C0 2 (b) CO and C0 2

(c) oxalic sulphate (d) C0 2 and HgS.

25. A solution of S 0 2 in water reacts with H2S precipitating sulphur. Here S02 acts as (a) an oxidising agent (b) a reducing agent (c) an acid (d) a catalyst.

26. The first ionization energies are in the order of (a) Na < Mg > AI < Si (b) Na > Mg > AI > Si (c) Na < Mg < AI < Si (d) Na > Mg > AI < Si.

27. When the first ionization energies are plotted against atomic numbers, the peaks are occupied by (a) alkali metals (b) halogens (c) noble gases (d) transition elements.

28. Which of the following has the biggest ionic radius? (a) O2" (b) Na+ (c) K+ (d) Cl".

29. An element A (Z = 13) and another element B (Z = 8) combine together to form a compound. The formula of the compound is (a) AB (b) A2B (C) AB2 (d) A2B3.

30. The highest covalent character is found in (a) CaF2 (b) CaCl2 (c) CaBr2 (d) Cal2.

31. Which one of the folllowing molecules contains a lone pair of electrons on the central atom? (a) Cl2 (b) CH4 (C) CHCI3 (d) NH3.

32. Which of the following oxides of nitrogen is the anhydride of HN0 3?

(a) NO (b) N 20 3 (C) N204 (d) N2Os.

58

33. The starting material in the Birkland Eyde process for manufacture of nitric acid is (a) ammonia (b) N0 2 gas

| (cj air (a) cmli saltpetre.

| 34. The molecular formula of phosphorus is j (a) P (b) P2 (c) P5 (d) P4.

35. The basicity of H3P04 is (a) 2 (b) 3 (c) 4 (d) 5.

36. Which of the following contains both ionic and covalent bonds? (a) CC14 (b) CaCl2 (c) NH4C1 (d) H 20.

37. The shortest length of H-bond is present in (a) HF (b) H 2 0 (c) NH3 (d) C2H5OH.

38. Which of the following does not exhibit resonance? ( a ) C 6 H S O H ( b ) C 2 H 5 O H

(c) N 2 0 (d) S02 .

39. The number of mole of atoms present in 10 g of calcium is (a) 0.1 mole (b) 0.2 mole (c) 0.25 mole (d) 0.4 mole.

40. The number of molecules of oxygen present in 280 ml of oxygen at STP is (a) 7.53 x 1021 (b) 7.50 x io21

(c) 6.02 x 1023 (d) 7.53 x 1022.

41. The number of mole of C0 2 left after removing 1021 molecules from 200 mg of C0 2 will be (a) 2 x 10 3 (b) 3 x IO"4

(c) 3 x IO"3 (d) IO3 .

42. The number of milli moles contained in 1.6 g of NaOH is (a) 0.04 (b) 0.4 (c) 4 (d) 40.

43. If 1.12 litres of methane contains 3 x 1022 molecules at STP, the number of molecules in 5.56 litres of oxygen at STP, will be (a) 6 x 1022 (b) 1.5 x 1022

(c) 0.15 x 1022 (d) 3 x 1022.

44. A gaseous mixture contains oxygen and nitrogen in the ratio of 1 : 4 by weight. The ratio of number of molecules is (a) 1 : 4 (b) 2 : 7 (c) 7 : 32 (d) 3 : 16.

45. The number of atoms in 0.004 g magnesium will be close to

-10 CHEMISTRY TODAY | FEBRUARY '06 172

(a) 24 (b) 96 (c) 1020 (d) 6.02 x 1020.

46. Which of the following is not a characteristic of entropy? (a) it is a state function (b) it is a macroscopic property (c) it is extensive property (d) it is intensive property.

47. Which of the following is insoluble in dilute HCl? (a) aniline (b) triphenylamine (c) ethylamine (d) dimethylamine.

48. Aniline is basic because of the presence of (a) - N H 2 (b) - N H -(c) - CN (d) - C O N H 2 .

49. The compound formed when aniline is gently heated with chloroform and potassium hydroxide solution is (a) benzonitrile (b) phenyl isocyanide (c) phenol (d) ammonia.

50. Which of the following may be prepared by Gabriel-phthalimide synthesis? (a) aliphatic amines (b) aromatic amines (c) aliphatic amides (d) aromatic amides.

51. Schiff's bases are formed when aniline is condensed with (a) phenols (b) aryl chlorides (c) aromatic aldehydes (d) aliphatic alcohols.

52. Nitrobenzene on reduction with Zn-dust and aqueous NH4C1 produces (a) aniline (b) phenyl hydroxylamine (c) nitrosobenzene (d) none of these.

53. When nitrobenzene is heated with fuming HN03

and concentrated H2S04 for several hours, it gives (a) dinitrobenzene (b) trinitrobenzene (c) DDT (d) none of these.

54. At 60°C a mixture of concentrated HN0 3 and concentrated H2S04 converts benzene into (a) nitrobenzene (b) /w-dinitrobenzene (c) benzene sulphonic acid (d) 1,3,5-trinitrobenzene.

55. When nitrobenzene is reduced with Sn and concentrated HCl, the product is (a) C6H5NH2 (b) C6H5NHC1 (c) C6H5OH (d) C6H5COOH.

56. The weight of iron which will be converted into its oxide by the action of 18 g of steam on it will be (Fe = 56, O = 16) (a) 20 g (b) 40 g (c) 45 g (d) 42 g.

57. The weight of 50% solution of HCl required to react with 100 g of CaC03 would be (a) 73 g (b) 100 g (c) 146 g (d) 200 g.

58. What will be the weight of oxygen that is required for the complete combustion of 2.8 g of ethylene? (a) 2.8 kg (b) 6.4 kg (c) 96 kg (d) 9.6 kg.

59. The weight of carbon required to reduce 165 g of C0 2 to CO will be (a) 40 g (b) 43 g (c) 45 g (d) 48 g.

60. The minimum amount of hydrogen required to reduce completely 7.95 g of CuO (mol. wt. 79.5) will be (a) 0.5 g (b) 1 g (c) 2240 ml at STP (d) none of these.

61. The volume of S0 2 at STP formed when 1260 g of sodium sulphite react with sufficient sulphuric acid is (a) 112 litres (b) 224 litres (c) 22.4 litres (d) 11.2 litres.

62. The weight of ammonium chloride required to prepare 22.4 litres of ammonia at STP is (a) 53.5 g (b) 107 g (c) 26.75 g (d) 40 g.

63. 18 g of water is electrolysed. Volume of oxygen obtained at STP is (a) 22.4 litres (b) 11.2 litres (c) 5.6 litres (d) 2.8 litres.

64. 15 litres of hydrogen combine with excess of nitrogen to produce ammonia. The volume of ammonia in litres formed is (a) 5 litres (b) 8 litres (c) 10 litres (d) 12 litres.

65. 15 ml of a gaseous hydrocarbon required 45 ml of oxygen for complete combustion and 30 ml of C0 2 were formed. The formula of the hydrocarbon is (a) C3H6 (b) C2H6 (C) C2H2 (d) C2H4.

66. Among the following, the most basic compound is (a) benzylamine (b) aniline (c) acetanilide (d) /?-nitroaniline.

67. Toluene reacts with Cl2 in the presence of sun light to give

CHEMISTRY TODAY | JANUARY '06 13

(a) benzyl chloride (b) benzoyl chloride (c) p-chlorotoluene (d) o-chlorotoluene.

68. Which of the following does not undergo Friedel-Craft's reaction? (a) aniline (b) phenol (c) nitrobenzene (d) all of these.

69. Which of the following is not used as a reagent in Friedel-Craft's reaction? (a) alcohol (b) alkene (c) chlorobenzene (d) alkyl-halide.

70. Toluene on oxidation with alkaline KMn04 gives (a) benzaldehyde (b) potassium benzoate (c) benzoic acid (d) benzo quinone.

71. The reaction of toluene with chlorine in the presence of ferric chloride gives predominantly (a) benzoyl chloride (b) m-chloro toluene (c) benzyl chloride (d) o- and p-chlorotoluene.

72. Toluene is oxidised to benzaldehyde with chromyl chloride. The reaction is known as (a) Oppenauer reaction (b) Etard reaction (c) Wurtz-Fittig reaction (d) Rosenmund reaction.

73. Which xylene is most readily nitrated? (a) o-xylene (b) p-xylene (c) m-xylene (d) all at the same rate.

74. Which of the following shows maximum molar conductivity? ( a ) [ C O ( N H , ) 6 ] C 1 3 ( b ) [ C O ( N H 3 ) 5 C 1 ] C 1 2

( c ) [ C O ( N H 3 ) 4 C I 2 ] C I ( d ) [ C O ( N H 3 ) 3 C 1 3 ] ,

75. Which of the following deactivates benzene substitution? (a) - NHR (b) - OH (c) - COOR (d) - OR.

76. 20 ml of gas was collected at STR The pressure was then doubled and the temperature gradually changed until the volume of the gas became 20 ml again. The temperature at which this happened was (a) 173 K (b) 273 K (c) 373 K (d) 546 K.

77. A certain mass of H2 at 273°C and 3 atm pressure occupies 2 litres. What will be the volume of the gas at STP? (a) 2/3 litres (b) 1 litre (c) 2 litres (d) 3 litres.

78. The density of a gas (mol. wt. 44.8) in gm/litre at

273 °C and 1 atm pressure is (a) 8.9 (b) 11.2 (c) 1 (d) 2.

79. The density of a certain mass of a dry gas at 27°C and 760 mm pressure is 2.5 gm/litre. Its density at 7°C and 740 mm pressure will be (in gm/litre) (a) 0.9 (b) 1 (c) 1.2 (d) 2.3

80. A vessel contains 1 mole of 0 2 at 27°C and 1 atm. A certain amount of the gas was withdrawn and the vessel was heated to 327°C to maintain the pressure at 1 atm. The amount of gas removed was (a) 8 g (b) 0.2 mole (c) 0.25 mole (d) 0.5 mole.

81. Which produces ammonia on reaction with caustic soda? (a) ethyl amine (b) dimethyl amine (c) acetamide (d) aniline.

82. The isomer of an oxime is (a) RN0 2 (b) R - O - C = N (c) R - O - N = O (d) R - CONH2.

83. Which of the following is most reactive towards nucleophilic substitution reaction? (a) RC1 (b) (RC0) 2 0 (c) RCOC1 (d) RCONH2.

84. The most suitable reagent for laboratory preparation of acetyl chloride from acetic acid is (a) PC15 (b) HCI (c) SOC1, (d) PC13.

85. Which ofthe following has the highest melting point? (a) (CH3C0)20 (b) CH3CN (c) CH3CONH2 (d) CH3COCl.

86. Which of the following is not a /w-directing group? (a) - S03H (b) - N 0 2 (c) - CN (d) - NH2.

87. Among the following the compound that can be most readily sulphonated is (a) benzene (b) nitrobenzene (c) toluene (d) chlorobenzene.

88. Gammexane is (a) DDT (b) benzenhexachloride (BHC) (c) chloral (d) hexachlorobenzene.

89. Benzene reacts with CC14 in the presence of anhydrous A1CI3 to give (a) tetraphenylmethane (b) chlorobenzene

10 CHEMISTRY TODAY | JANUARY '06

(c) tetrachlorobenzene (d) triphenyl methylchloride.

90. Anhydrous A1C13 in Friedel-Craft's reacton is used to (a) generate an electrophile (b) generate a nucleophile (c) generate a free radical (d) absorb water.

91. When a solution of a primary aliphatic amine is treated with NaN0 2 and HCl the effervesescence occurs due to formation of (a) CO, (b) N 0 2

(c) N2 (d) H,.

92. The carbylamine reaction is associated with the name of (a) Wurtz (b) Kolbe (c) Hinsberg (d) Hoffmann.

93. Hinsberg reagent is (a) benzene sulphonic acid (b) benzene sulphonyl chloride (c) /7-toluene sulphonyl chloride (d) none of these.

94. When an organic compound was treated with sodium nitrite and HCl in the ice-cold, nitrogen gas was evolved. The compound is (a) a nitro compound (b) a primary amine (c) a secondary amine (d) a tertiary amine.

95. An aromatic amine (A) was treated with alcoholic potash and another compound (F) when a foul-smelling gas was formed with formula C6H5NC, (7) was formed by reacting a compound (Z) with Cl, in presence of slaked lime. The compound (Z) is (a) C6H5NH2 (b) CH3OH (c) CH3COCH3 (d) CHCI3.

96. Benzene is converted into toluene by (a) Wurtz reaction (b) Etard's reaction (c) Fittig reaction (d) Friedel-Craft's reaction.

97. In Friedel-Craft's reaction, the catalyst used is (a) Raney Ni (b) Mg and ether (c) anhydrous A1C13 (d) Na and liquid NH3.

98. Benzene undergoes predominant reactions by (a) electrophilic addition (b) nucleophilic addition (c) electrophilic substitution (d) nucleophilic substitution.

99. In the presence of bright sunlight benzene reacts with chlorine to form (a) chlorobenzene (b) benzyl chloride (c) hexachloro-benzene (d) benzene hexachloride.

100. The main source of aromatic compound is (a) wood (b) coal tar (c) coal gas (d) petroleum.

ANSWERS

1. (b) : The undesired impurities present in ores are called gangue or matrix. To remove gangue, certain substances are mixed with concentrated ore which combine with gangue to form fusible material which is not soluble in molten metal. The substances used are called fluxes and the fusible material formed during reduction process is called slag.

Si02 + CaO - » CaSiOj flux gangue slag

2. ( a ) : Malachite - CuC03 • Cu(OH)2

Calamine - ZnC03

Galena - PbS Dolomite - CaCO, • MgC03

3. (d) : Corrosion is def ined as the gradual transformation of a metal into its combined state because of the reaction with the environment. When iron is exposed to moist air, it is found covered with a reddish brown coating which can easily be detached. The reddish brown coating is called rust. 4. (a) : CaCO, decomposes into CaO and CO,. CaO acts as a flux as it combines with silica present as an impurity to form a fusible slag.

CaC03 CaO + CO, CaO + Si02 -> CaSi03

5. (b) 6. (b) : Molecular formula weight of the compound

= 3 0 The oxide would thus be NO = (14 + 16) = 30 At. number of N and O being 7 and 8, the total number of electrons present = (7 + 8) = 15.

7. (b) : Electronic configuration of the element may be Is2 2s2 2pb 3s1 or Is2 2s22pb 3s2. Hence, its atomic number will be 11, 12.

8. (d) : The number of electrons in I are (6 + 3 - 1) = 8 II are (3 + 8 - 1) = 10 III are (7 + 3) = 10 IV are (6 + 3 + 1) = 10.

-10 CHEMISTRY TODAY | FEBRUARY '06 175

9. (d) : As we know that r„ «2/Z For the same value of principal quantum number the radius decreases as the atomic number Z increases. For, H, Z = 1; D+, Z = I; He+, Z = 2 ; Li2+, Z= 3 The radius of first Bohr orbit is the smallest one for Li2+.

1 2 10. (c) : Kinetic energy of the electron = — wv

and potential energy of the electron =

Total energy (E) of the electron = K.E.

1 2 —mv 2

Ze1

4 7 t £ 0 r

+ P.E.

Ze1

4 ne0r Centripetal force = (coulombic atractive force

between electron and proton)

mv r

E =

Ze2

47te0/-' mv2 =

Ze 8 i t E 0 r

Ze Zez

47tE 0r Ze2

8ne0r [8ne0r That is E <*= Mr.

11. ( b ) : 4Ag + 8NaCN + 2H20 + O, 4NaAg(CN)2 + 4NaOH

sodium argentocyanide

NaAg(CN)2 + Zn -> Na2Ag(CN)2 + 4NaOH soluble complex

12. ( a ) : Invar is an iron alloy containing 36% Ni.

13. (b) 14. (c) : Laminaria species contain 0.5% of iodine in their ashes (known as kelp) in the form of iodides. 15. (c) : Glass being a mixture of sodium and calcium silicates reacts with hydrofluoric acid forming sodium and calcium fluorosilicates respectively.

Na2Si03 + 3H2F2 Na2SiF6 + 3H20 CaSi03 + 3H2F2 -> CaSiF6 + 3H20

16. (c) Species Z e Z/e Mg2+ 12 10 12/10 = 1.2 Na+ 11 10 11/10 = 1.1 F- 9 10 9/10 = 0.9

Al3+ 13 10 13/10 = 1.3 We know the bigger the value Z/e, the smaller is the ionic radius. Hence increasing order of radius

AI3+ < Mg2+ < Na+ < F". 17. (c) : All these species are isoelectronic. Again the larger value of proton/electron the smaller is

the radius. The oxide ion has only 8 protons, the ratio is the smallest one. Hence 02~ has largest radius. 18. ( b ) : In cf-block elements the electrons occupy the c/-subshell and the last two principal shells are incomplete. Again, after filling the ns orbital the electron enters the {n-\)d orbital.

T 19. (b) : We know that, velocity = -

no. of Bohr orbit Therefore, the velocity of the electron in the third Bohr orbit = x/3.

, he 20. (c) : As we know, £ = «o = —

K 1 £, _ X2 _ 4000 _ 2

E x — . Therefore, c - , ~ , X ' E2 2000 1

21. (d) : As others (1), (2) and (3) reacts with Cl2. 22. (b) 23. (d) : The structure of S207

2" is

/ s — o — s ^ . .

•• :o: :o: Therefore, it has no S - S bond. Rest of them have S - S bond in their structures.

24. (b) : H,C204 cone. H , S O . . . 2 4 COT + CO,T

dehydration of H 2 0

25. (a ) : S02 + 2H2S 2H20 + 3S Here oxidation number of S reduces from +4 to 0. But it oxidises H2S (-2 to 0) So, S02 in this reaction acts as oxidising agent. 26. (a) : All these elements belong to period 3 in the periodic table. The first ionization energy increases in a period, with the exception that ionisation energy of group IIIA is less than that of group IIA element. Hence, the order of increasing ionisation energy is

Na < Mg > Al < Ai. 27. (c) : As we know that the noble gases have the highest value of first ionisation energy in their period, hence noble gases occupy the highest points in the curve. 28. (d) : Isoelectronic cations have smaller radius than anions and the radius increases in a group. 29. (d) : Electronic configuration of A

(Z = 13), 2, 8, 3 Electronic configuration of B (Z = 8), 2, 6

50 CHEMISTRY TODAY | JANUARY '06

A by losing 3 electrons and B by gaining two electrons achieve the noble gas configuration. Hence, valency of A is +3 and that of B is -2 . Therefore, the formula of the compound is A2B3. 30. (d) : The cation in all the compounds is the same Ca2+. The anion is halide ion, The size of the iodide ion is greatest of all other halide ions. By Fajan's rule, the maximum covalent character of these exists in Cal2. 31. (d) 32. (d) : 2 H N 0 3 ~ H 2 ° > N 2 0 5

33. (c): For the manufacture ofHN03 , reactions taking place in air are as follows:

N2 + 0 2 ^ 2NO - 43,200 calories 2NO + 0 2 -> 2NO, 2N02 + H20 HNO3 + HN02

3HN02 HN03 + 2NO + H20

34. ( d ) : As the molecular formula of phosphorus is P4. 35. (b): As in orthophosphoric acid (H3P04), number of ionisable hydrogen atoms (basicity) is three. 36. (c) : The Lewis structures of these compounds are

: C l : . . I

: C I - C -•• I

:ci: oo H I

H - N - H

ci: ,[ca2+] [:ci:]_

(b)

H M A L J • H H M W)

(a) and (d) have only covalent bonds (b) contains an ionic bond (c) contains both covalent and ionic bonds. 37. (a) : Considering the size of the F, N, O the atom of the smallest radius of the three is fluorine and fluorine is the most electronegative atom. The H-bond in HF is, therefore, shortest in length.

38. (b) : The species exhibiting resonance requires the presence of a rc-bond or the lone pair suitably placed in the structure of molecule or ion. The structures are

CH3: CH2: O : H

: N = N ^ O 0 = S—O

-10 CHEMISTRY TODAY | FEBRUARY '06

UjpJ » C H 3 - C H , - 0 - H ,

® ..'— .. © : N = N = 0 : , : 0 = S — O:

The condition is not present in ethanol (C2H5OH). 39. (c) : This atomic weight of calcium = 40 .". 40 g of Ca make 1 mole

10 g of Ca make — x 10 = 0.25 mole. 40

40. (a) : Since, one mole of oxygen = 32 g = 6.02 x io23 molecules of 0 2

= 22400 ml 0 2 at STP Number of molecules of oxygen in 280 ml of 0 2 at

280 STP = x 6.02 x 1023 molecules 22400

= 7.53 x io21 molecules. 41. (c) : v 6.02 x io23 molecules of C02

= 1 mole of C02

1021 molecules of CO, = 10 ,21 10 ,-2

6.02 xlO23 6.02 mole

200 1 Now, 200 mg of C02 = — : * — mole

10 _9 = —xlO mole

22 10

Number of mole of CO, left = 1

xlO"

42. (a) : Millimoles of NaOH Mg of NaOH

22 6.02, = 3 x 10"3 mole.

1600 formula wt. of NaOH in mg 40x 1000

_ *600 _ 0 m j | j j m o l e

40,000

43. (b) : As, vol. of methane at STP _ No. of molecules of CH4

vol. of oxygen at STP No. of molecules of O, 3x10 22 1.12

0.56 no. of molecules of 0 2

3xl()22 x0.56 => No. of molecules of O, =

44. (c) : As,

1.12 1.5 x io22.

No. of molecules of 0 2

No. of molecules of N2

_ moles of 0 2 ^ Avogadro's number moles of N2 Avogadro's number

13

wt. of O, mol. wt. of O, 1/32 1 28 7

wt. of N , 4/28 32 4 32 mol. wt. o f N 2

45. (c) : Since, 24 g of magnesium contains 6.02 x 10" atoms

6.02 x l0 2 3 x 0.004 0.004 g of Mg contains = ;

24 = IO20 (nearly).

46. (d) 47. (b) 48. (a)

49. (b) : [ O J + CHC1, + 3K.OH >

N = C + 3KC1 + 3H20

This reaction is called isocyanide reaction and is used as a qualitative test for primary amines.

50. (a)

51. (c) : Aniline reacts with benzaldehyde in 1 : 1 molar ratio to produce benzalaniline. (also called Schiff's base or anil)

A C6H5NH2 + O = CH - C6H5 Benzaldehyde

NO,

- H , 0 » C6H,N = CH

C6H5

52. (b) : Zn/NH.CI (aq)

NO,

C6H5NHOH phenyl hydroxyl

amine

NO,

53. (b) : [ O J fuming H N O ,

high temp. NO: 0 NO, 2 v

1,3,5-trinitrobenzene

54. (a)

conc. H,SO / 1 + conc. HNO, 2— 50-55°C

NO,

benzene

55. (a) :

nitrobenzene

NO, (i) Sn/conc. HCI

(ii) NaOH nitrobenzene

NH,

aniline

56. (d) : 3Fe + 4H,0 Fe304 + 4H, (3 x 56)g (4 x 18) g

Since, 72 g of steam reacts with 168 g of Fe

168x18 „„ 18 g of steam will react with — — — = 42 g.

57. (c) : CaCO, + 2HCI CaCI, + CO, + H , 0 100 g 73 g

Amount of HCI required for 100 g of CaC03 = 73 g v 50 g of HCI are contained in 100 g of 50% acid

100x73 1/1/f .-. The wt. of 50% acid = — — — = 146 g .

58. (d) : C2H4 + 3 0 , 2C02 + 2H,0 28 kg 96 kg

Y 28 kg of ethylene are completely burnt by 96 kg of 0 2

2.8 kg of ethylene are completely burnt by

^ = 9.6 kg. 28

59. (c) : CO, + C 2CO 44 g 12 g

44 g of CO, are reduced by 12 g of carbon 12

.-. 165 g of CO, are reduced by — x 165 g of carbon 44

= 45 g. 60. (c) : CuO + H2 Cu + H 2 0

79.5 g 2 g

79.5 CuO is reduced by 2 g of hydrogen .-. 7.95 g CuO is reduced by 0.2 g of H2

Again, 2 g of hydrogen is equivalent to 22400 ml of H, at STP.

0.2 g of H2 is equivalent to 2240 ml H, at STP. 61. (b) : Na2S03 + H,S04 Na,S04 + H , 0 + S0 2

(46 + 32 + 48) 22 .4 litres at STP

= 126 g

. . given wt. vol. of unknown Again, - — : — = _ (per mole)

mol. wt. 1260 x 126 22.4 lit.

22.4 litres 1260x22.4

126 = 224 litres.

62. (a) : NH4C1 NH, + HCI (14 + 4 + 3 5 . 5 ) g 22.4 lit at STP

given volume wt. of unknown

lit.

Again, 22.4 litres

22.4 _ A-

22.4 ~ 53.5

mol. wt.

=> x = 53.5 g.

electrolysed 63. (b) : H 2 0-

18 g

64. (c) : 15 * 3H2 + N2 2NH3

3 2

-O,

(11.2 litres of O , at STP)

6 4 CHEMISTRY TODA\ JANUARY '06

given volume _ unknown volume no. of molecules no. of molecules 15 x j 2

65. (d) : Combustion reaction of hydrocarbon CrHv

, y i

A- = 10 litres.

C , H V + [ X + ^ I O 2 = * C O 2 + | H 2 O

v o ' ' x v 0 ' 1 vol

15 ml 15| -T + - J ml I5x ml

Now 15A: = 30 (given) (As volume of C0 2 is 30 ml) . x = 2.

Again, 15lx + ^-J = 45 (volume of 0 2 reacted)

^- = 3 - j c = 3 - 2 = 1 => jc + —= 3 4 4

=f y = 4. ;. Formula of hydrocarbon = CVH;. = C2H4. 66. (a)

benzyl chloride

68. (d) : The reaction of an alkyl halide or acyl halide with benzene in the presence of a Lewis acid, generally A1C13, is known as Friedel Craft's reaction. 69. (c) 70. (b) : - CH3 group is oxidised to - COOH group.

C H 3 C H 3 C H 3

71. ( d ) : [ O j

72. (b) :

Cl, FeCl3

C H ,

o-chlorotoluene CI p-chlorotoluene

CHO CS 2 (25-45°C)

(toluene) (benzaldehyde)

So, toluene is conveniently oxidised with Etard reagent (chromyl chloride CrO,Cl2)to benzaldehyde. This reaction is known as Etard reaction. 73. (c) 74. (a) : Because it contains maximum number of ions. 75. (c)

76. (d) : Here K, = 20 ml, K, = 20 ml r, = 273 K, T2 = ? Pt = 1 atm, P2 = 2 atm

20x20x273 i 2 -W _P2V2

75 T-> P\V\ 1x20 = 546 K.

77. (d) : Pf = 3 atm, P2 = 1 atm, V2 = 2 litres, V2 = ? = (273 + 273) K = 546 K, T2 = 273 K

. , IWi As we know from the gas equation. 5H r. P~>

lr P.V.T, 3x2x273 V-, = 1 1 2 = = 3 litres.

TXP2 546x1 PM RT

= 1 gm/litre.

78. (c) : As we know that, density, P =

Here, we are given P = 1 atm, R = 0.082 litre atm K"1 moL1

M = 44.8 gm/mole, T=(273 + 273) K = 546 K 1 44.8

.'. P = x 0.082 546

79. (d) : As we know, = P{l] P2T2

Here we are given p, = 2.5 g/litre, p, = ? 7, = (273 + 27) K = 300 K T2 = (273 + 7) K = 280 K Px = 760 mm, P2 = 740 mm

p,r, P2 _ 2.5x280x740 P2 = „ = 2.3 g/litre.

P\ 7] 760x300 80. (d) : Suppose the gas removed = x mole

The amount remaining in the vessel = (1 - x ) mole Volume is constant and pressure is kept the same under this condition.

nlTl = n2T2

Here, nx = 1 mole, n2 = (1 — x) mol r , = 300 K, T2 = 600 K Therefore, 1 x 300 = (1 - x) 600

1 = 2(1 - x ) = 2 - 2x .'. x = 1/2 mole = 0.5 mole. 81. (c) : CH3CONH2 + NaOH (aq)

CHjCOONa + NH3

This is also test for amide - (CONH,) group. 82. (d) 83. (c): Reactivity towards nucleophilic substitution lies in the following sequence.

RCOCI > (RC0)20 > RCOOR' > RCONH, Nucleophilic substitution takes place readily at an acyl carbon than at saturated carbon. Thus for nucleophilic substitution acid chlorides are more reactive than alkyl chlorides; amides are more reactive than amines and esters

CHEMISTRY TODAY | JANUARY '06 13

are more reactive than ethers. RCOCI > RC1, RCONH2 > RNH2, RCOOR' > ROR' 84. (c) : Thionyl chloride is the better reagent because the byproducts of the reaction are the gases S02 and HCI, which are easily separated. 85. (c)

86. (d) : - NH2 is an o-, p-directing group, as it has at least one pair of non-bonding electrons on the atom adjacent to the benzene ring. 87. (c)

88. (b) : Asgammexane is nothing but C6H6C16 (B.H.C). A1C1,

89. (d) : C6H6 (excess) + CCI4

(C6H5)3C - CI (C6H5)3C - CI is called triphenyl methyl chloride. 90. (a) : Reaction —> alkylation

Reagent —> R - CI Catalyst AICI-,

Electrophile —> R

Product CfiH, - R (CH3 )

91. (c) : C2H5NH2 or R - N H 2 (NaNQ2+HCl) or HN02 (0°C) » C ,H 5 OH+N,

92. (d)

93. (b) : Benzene sulphonyl chloride (C6H5S02C1) is known as Hinsberg's reagent, and used for the separation of primary, secondary and tertiary amines. 94. (b): This reaction is used to make distinction between primary, secondary and tertiary amines. 95. (c) : C6H5NH2 + CHC13 + 3KOH (a fc) ->

A Y C6H5NC + 3KC1 + 3H20

CH3COCH3 + 3C12 CCl3COCH3 + 3HC1 Z

OH CCljCOCHj 2CHCI CHjCOO-^ C a < +

^ O H C C 1 3 C 0 C H 3 C H , C O O ' ; ca

96. (d) - C H j C l A1C1, C H 3

(benzene) (toluene)

This reaction is called Friedel-Craft's reaction. 98. (c)

bright sunlight (U.V.), > C6H6C16

97. (c)

99. (d) : C6H6 + 3C12

(B.H.C) 100. (d) : Coal and petroleum are the two important sources of aromatic hydrocarbons.

Wl-fcG- Most Powerful Books for CBSE-PMT/AIIMS/AFMC & Other PMT Exams a ^ « r ^ 2i5 sJ i J li ffii

iiTdw'®! 53FI1 i j l g m l iiTdw'®! 53FI1

3332EII mmrm

f-m PfltUMlSAXY EXAM io Y

H j ;

Assertion A s s e r t i o n & Reason For Competitive Exams

-iSBotany ^ ^ •"-^. 'Zoology _

Add Rs.35 for postage/handl ing charge if ordering a single book.

P o s t a g e H 3 H f ° r order value above Rs. 500

How to order Send a demand Draft in favour of

'MTG Books' payable at Delhi.

MTG BOOKS 503. Taj Apt., Ring Road, Near Safdariung Hospital. New Demi-29. Tel.: 26194317,26191601

10 CHEMISTRY TODAY | JANUARY '06

Q h M S ) C B S E ) f D P M T ) A F M C ) f v M M C ) (EAMCET) (WB-JEE) ( B H U ) ( C M C )

2006 Medical Entrance Exam { U P C P M T )

fCET Karnataka] Practice Test Paper (Manipa l PMT)

( J I P M E R ) (MP PMT) ( M G I M S ) ( P M P T B i h a r ) (PMT Haryana ) ( K e r a l a PMT ) ( R a j . P M T j f T N P C E E )

1. Iodised table salt contains 7.6 x IO-5 g of KI per gram of NaCl. The concentration in ppm of KI is (a) 76 (b) 152 (c) 500 (d) 760.

2. The heat of combustion of ethane, hydrogen and graphite are -33.0, - 68.5, - 94 K cal/mole, the heat of formation of ethane would be (a) -188 K cal/mole (b) -383.5 K cal/mole (c) -350.5 K cal/mole (d) +53.5 K cal/mole.

3. A compound A'reacts in the following ways. BaC12 (aq)^

X(aq)

yellow precipitate

NaOH (aq)

brown precipitate

white ppt.

I NH3 (aq) \ colourless solution

The compound X is likely to be (a) Pb(N03)2 (b) CaCr04

(c) Ag2S04 (d) AgN03 .

4. Holmes signal makes use of a mixture of (a) calcium carbide and calcium chloride (b) calcium carbide and sodium phosphide (c) calcium carbide and calcium phosphide (d) calcium chloride and sodium phosphide.

5. The oxyacid of phosphorus that has some reducing property is (a) H3P04 (b) HP03

(c) H3P03 (d) H2P207.

6. Portland cement contains the following amount of CaO. (a) 5-15% (b) 20-35% (c) 50-60% (d) 70-80%.

7. The ether that undergoes electrophilic substitution reaction is

(a) CH3OC2H5

(c) CH3OCH3

(b) C6H5OCH3

(d) C2H5OC2H5.

8. Predict the final product B in the sequence of reaction. 3 0 % H , S 0 4

CH = CH „ 4> A > B H g S 0 4 NaOH

(b) CH3COOH (d) C H 3 - C H - C H 2 - C H O

OH

(a) CH3COONa (c) CH3CHO

9. An organic compound with the formula C6H ! 206

forms 20% a yellow crystalline solid when treated with phenylhydrazine and gives a mixture of sorbitol and mannitol when reduced with sodium. Which among the following could be the compound? (a) fructose (b) glucose (c) mannose (d) sucrose.

10. Caprolactum polymerises to give (a) terylene (b) teflon (c) glyptal (d) nylon-6.

11. The SN1 mechanism for the hydrolysis of an alkyl halide to an alcohol involves the formation of (a) carbanion (b) carbocation (c) free radical (d) pentavalent carbon in the transition state.

12. The relationship between standard reduction potential of cell and equilibrium constant is shown by

(a) E°, cell ' " log Kc (b) / ^ — - l o g / Q . 0.059

(c) £°cen = 0.059 n log/Q;

,,, r-o _ 'og ( d ) £ cell n

13. Colourless solutions of the following four salts are placed separately in four different test tubes and a strip of copper is dipped in each one of these. Which solution will turn blue? (a) KN03 (b) AgN03

CHEMISTRY TODAY | DECEMBER '05 47

(c) Zn(N03)2 (d) ZnS04.

14. Concentrated H2S04 has a density of 1.98 gm/ml and is 98% H2S04 by weight. Its normality is (a) 2 N (b) 19.8 N (c) 39.6 N (d) 98 N.

15. Which of the following reactions occur at the cathode during the charging of lead storage battery? (a) Pb2+ + 2e~ Pb (b) Pb2+ + S04

2" PbS04

(c) Pb H> Pb2+ + 2e~ (d) PbS04 + 2H20 -> Pb02 + 4H+ + S04

2" + 2e~

16. The number of dative bonds in sulphuric acid molecule is (a) 0 (b) 1 (c) 2 (d) 4.

17. The correct sequence of decrease in the bond angle of the following hydrides is (a) NH3 > PH3 > AsH3 > SbH3

( b ) N H 3 > A S H 3 > P H 3 > S b H 3

(c) SbH3 > A S H 3 > P H 3 > N H 3

( d ) P H 3 > N H 3 > A S H 3 > S b H 3 .

18. Which of the following reactions is not exothermic? (a) C ( „ + 0 2 t e ) - > C 0 2 t e )

(b) C W + 2 S W . - » C S 2 W

(c) CH4 w + 20 2 -> C0 2 w + 2H20 m

( d ) C O W + 1 / 2 0 2 W - 4 C 0 2 ( S )

19. The favourable conditions for Haber's process are (a) low temperature and low pressure (b) low temperature and high pressure (c) high temperature and low pressure (d) high temperature and high pressure.

20. A graph plotted between logKvs 1/7"for calculating activation energv is shown by

t log K

(a)

1/7" • 1 IT •

t log K

(C)

t log K

(d)

1/7" • 1/7" •

21. The oxidation number of Fe in compound

46

[ F e ( H 2 0 ) 5 N 0 ] S 0 4 j s

(a) +2 (b) - 2 (c) 0 (d) +1.

22. The equivalent weight of sodium thiosulphate (Na2S203-5H20) in the reaction

2Na2S203 + I2 -> 2NaI + Na2S406 is (a) 248 (b) 124 (c) 596 (d) 62.

23. 'Mischmetal' is an (a) alloy of rare earth elements with Fe and traces of

S, C, Ca and Al (b) alloy of rare earth elements with Cu and traces of

S, C, Ca and Al (c) alloy of rare earth elements with Zn and traces of

S, C, Ca and Al (d) alloy of transition elements with Ca and traces of

S, C, Ca and Al.

24. In the reaction,

[A] Br, /P - H B r . * [ S ] [C] -co.

(C 5 H 1 0 O 2 ) contains an (No geo.

[D]

(alkene)

(E) and (F) are

asymmetric isomerism) carbon

ozonolysis > ^ ^ [yr]

(positive (negative Sch i f f s test) Sch i f f s test)

O II

(a) HCHO and CH3 - C - C2H5

O II

(b) HCHO and CH3 - C - CH3

C H

(C) C H CH • COOH and HCHO

(d) none of these. 25. The amount of NH3 and NH4C1 required to prepare a buffer solution of pH = 9, when

(pKh for NH3 = 4.7, log2 = 0.30) (a) 0.2 mol/lit and 0.4 mol/lit (b) 0.4 mol/lit and 0.2 mol/lit (c) 0.2 mol/lit and 0.2 mol/lit (d) 0.2 mol/lit and 0.02 mol/lit. 26. Ka for ascorbic acid (HASC) is 5 * 10~5. Then the value of [H+] in an aqueous solution in which the concentration of ASC~ ions is 0.02 (M) will be (a) 5 x 10~7 (b) 5 x IO"9

CHEMISTRY TODAY | DECEMBER '05 47

(c) 5 X 10- (d) 1 x 10"

27. The increasing order of thermal stability for oxyhalide anion/the salts of CIO", C102", C103", C104" is as follows. (a) CIO < ClOY < C103- < C104-(b) c i o - > c i o 2 - > d o , - > c i o 4 -(c) CIO" > C104- < C102 < CIO3-(d) C104" > C102- > CIO- > CIO3-

28. Which of the following will displace Br, from an aqueous solution containing bromide ions? (a) Cl2 (b) I3-(c) I2 (d) Cl".

29. Ionic solids with Schottky defects contain in their structure (a) equal number of cationic and anionic vacancies (b) anionic vacancies and interstitial ions (c) cationic vacancies (d) cationic vacancies and interstitial cations.

30. Which one of the following pairs of atoms or ions will have identical ground state electronic configuration? (a) Li+ and He" (b) Cl" and Ar (c) Na and K (d) F+ and Ne.

31. A compound is treated with NaNH2 to give sodium salt. Identify the compound. (a) C2H, (b) C6H6

(c) C2H6 (d) C2H4.

32. Chlorobenzene reacts with Mg in dry ether to give a compound (/I) which further reacts with ethanol to yield (a) phenol (b) benzene (c) ethyl benzene (d) phenyl ether.

33. Benzene reacts with n-propyl chloride in the presence of anhydrous A1C13 to give (a) 3-propyl-l-chlorobenzene (b) /7-propyl benzene (c) no reaction (d) isopropylbenzene.

34. At 90°C, pure water has [H+] = IO"6. What will be the value of Kw at 90°C? (a) IO"12 (b) IO"11

(c) 10~8 (d) 10-6.

35. A solution of potassium bromide is treated with each of the following. Which one will liberate bromine? (a) hydrogen iodide (b) sulphur dioxide (c) chlorine (d) iodine.

36. If a 0.1 molar solution of glucose (mol. wt. = 180) and a 0.1 molar solution of urea (mol. wt. 60) are placed on the two sides of a semipermeable membrane to equal heights, then it will be correct to say that (a) there will be no net movement of solutions across

the membrane (b) glucose will flow across the membrane into urea

solution (c) urea will flow across the membrane into glucose

solution (d) water will flow from urea solution into glucose

solution.

37. The reaction of toluene with chlorine in presence of ferric chloride gives mainly (a) «-chlorotoluene (b) benzyl chloride (c) o- and p- chlorotoluene (d) benzoyl chloride.

38. A solution containing ammonium chloride and sodium nitrite on boiling produces (a) ammonia (b) nitric oxide (c) nitrogen (d) nitrous oxide.

39. In the manufacture of bromine from sea water, the mother liquor containing bromides is treated with (a) carbon dioxide (b) chlorine (c) iodine (d) sulphur dioxide.

40. When chlorine is passed over dry slaked lime at room temperature, the main reaction product is (a) Ca(C102)2 (b) CaCl2

(c) CaOCl2 (d) Ca(OCl)2

41. In a molecule of carbon tetrachloride the four C - Cl bonds are directed towards the corners of a (a) square (b) tetrahedron (c) prism (d) cube.

42. Cu2+ and Ag+ are both present in the same solution. To precipitate one of the two ions and leave the other in solution, we shall add (a) HCl (fl?) (b) NH 4 N0 3 (aq)

(c) H2S {aq) (d) HN0 3 (aqy

43. The root mean square speeds at S.T.P. of molecules H2, N2, 0 2 and HBr are in the order of (a) N2 > 0 2 > HBr > H2

(b) 0 2 > N2 > H2 > HBr (c) HBr > 0 2 > N2 > H2

(d) H2 > N2 > 0 2 > HBr.

CHEMISTRY TODAY | DECEMBER '05 47

44. Which one of the following carboxylic acids will have lowest acidic strength? (a) BrCH,COOH (b) CH3COOH (c) CHjCHCICOOH (d) ClCH2COOH.

45. The hydrolysis of PC13 produces (a) (H3P03 + HCl) (b) (PH3 + HCIO) (c) (H3P03 + HCIO) (d) (H3P04 + HCl).

46. Calcium carbonate decomposes on heating according to the equation : CaC03 = CaO + C02 . At S.T.P. the volume of C0 2 obtained by thermal decomposition of 50 g of CaC03 will be (a) 22.4 litres (b) 44 litres (c) 11.2 litres (d) 1 litre.

47. 12.5 ml of a gaseous hydrocarbon were mixed with 100 ml of oxygen and exploded. After cooling the residual gas occupied 68.75 ml. The volume was further reduced by 50 ml by passing the gases in KOH solution and the remaining gas was oxygen. The molecular formula of the hydrocarbon is (a) C2H6 (b) C4H8

(c) C4H,0 (d) C5H12.

48. Two containers A and B contain the same gas. If the pressure, volume and absolute temperature of gas in A are two times as compared to those in B and if density of gas in A is d, then density of gas in B is (a) d/2 (b) d (c) 2d (d) 4d.

49. XC + THN03 C0 2 + N0 2 + H20. The value of X and Y to balance this equation is (a) X=4, Y= 3 (b) X= 3, 7 = 4 (c) X=4, 7 = 1 (d) X = 1, Y = 4.

50. The compound which gives the most stable carbonium ion on dehydration is (a) 2-methyl-l-propanol (b) 2-methyl-2-propanol (c) 1-butanol (d) 2-butanol.

SOLUTIONS

1. (a) : 7.6 x 10'5 g in one gram = 7.6 x 10~5 x 106 = 76 ppm.

7 2 ( g ) '

AH = - 3 3 . 0 Kcal/mol

2. (c) : C2H6 w + - 0 2 (g) -> 2C02 + 3HzO

... (i) I

H2(g) + 2 02(g)- H 2 0 ;

AH = -68.5 Kcal/mol c (.«) + °2 (g) c o 2 fe)

AH = -94 Kcal/mol Heat of formation of ethane is given by

... (ii)

... (iii)

2C (.V) 3H20 (,,) —» C2H6 (g), AH, = ? The above equation can be obtained by multiplying equation (ii) by 3 and equation (iii) by 2, and then subtracting (i) from the resultant equation (iv).

1 [H2 fe) - 0 2 to H 2 0 ; AH = -68.5] x 3

[C w + 0 2 to -» CP 2 to ; = -94] x 2 1

2C (,, + 3H2 + 3 - 0 2 (s) 3H20 (x) + 2CO: ' (s) T J n 2 (g) 1 J 2 w2 <g) AH = -383.5 Kcal/mol

2 (g) ... (iv)

3H20 + 2COz —> C 2 H g + — 02; AH= +33 Kcal/mol

2C w + 3H2 w -> C2H6^y, AW" = -350.5 Kcal/mol.

3. (d) : Reaction with KI and NaOH indicate that cation is Ag+. i.e. Ag+ + KI —> Agl + K+

yellow ppt. Ag+ + NaOH AgOH + Na+

brown

Reaction with BaCl2 indicate that anion is either S042^

or N03" as white ppt. may be formed due to BaS04 or AgCl. However, AgCl is soluble in NH3 and not BaS04, so X is AgN03.

4. (c) 5. (c) : Oxyacid in which P is in low valent oxidation state can act as reducing agent. Oxidation state of P in H3P04 = +5 Oxidation state of P in HP03 = +5 Oxidation state of P in H3P03 = +3 Oxidation state of P in H4P207 = +5 As oxidation state of P in H3P03 is +3, hence it can be oxidised to +5, thus shows some reducing behaviour. 6. (c) : Composition of Portland cement is CaO - 60%, Si02 - 22.5%, A1203 - 7.5%, MgO - 2.5%, Fe203 - 2.0%, K 2 0 - 1.5%, S 0 3 - 1.0%, Na 20 - 1.5%.

% SiO-, Essentially, % AI2O3

% CaO

• = 2.5 to 4.0

% SiO, + %A1,0, + %Fe,0, - = 1.9 to 2.1

4 8

7. (c) : Aromatic ring undergoes electrophilic substitution reaction due to rich Jt-electron cloud in the ring.

CHEMISTRY TODAY | DECEMBER '05 47

8. (D) : C H = C H 3 0 % H * S ( S CH,CHO

HgSQ 4

9. ( a ) : CH,OH I C = o I

(CHOH)3

CH2OH fructose

C H , - C H - C H , C H O I O H B

(aldol)

C H = N N H C S H 5

C = N N H Q H , I

( C H O H ) 3

CH2OH

fructose osazone (yellow)

C H 2 O H I > H - C - O H + H O - C - H

CH2OH

[H]

( C H O H ) 3

CH2OH sorbitol

10. (d)

11. (b) : SN1 mechanism.

RX

(CHOH)3

CH2OH mannitol

> R+ + X~ > RQH+X" (carbocation)

12. (b) : AG = -nFE Cell reaction at equilibrium has AG = 0. This means, nFE = 0 or, E = 0. Consider a reaction,

Zn + Cu2+ ^ Zn2+ + Cu, .-. K = [Zn2+] [Cu2+]

As only ions are present in solution, then

E-E° + 3 L ] o g I C O ce" nF g c [Zn2+]

eeU nF [Zn2+] nF [Cu2+]

RT, ,„ 2.303/?7\ £0ce.i = — log. K = — log10 K

nF nF

£ ° c e „ = ^ l o g , o ^ ( a t 2 9 8 K ) n

Also, nFE° = RT logeK or, -AG° = RT logeK. 13. (b) : Cu + 2AgN03 Cu(N03)2 + 2Ag

colourless green As, Ag is below the Cu in electrochemical series, hence

it is reduced to Ag. Cu is oxidised to Cu :" ion.

98 g H,S04 14. (c) : 98% H2S04 by weight means = 1 0 0 g w a t e r

Volume of solution = 100 100

N,

d 1.98 g eq. of H2SQ4 98

"2S°4 volume of solution 49x100 1.98x1000

JVH2so4 = 19.8 x 2 = 39.6 15. (d)

O H

16. (a) : 0 = S = 0 I OH

17. (a) : This is due to decreased bond-bond repulsions as electronegativity of X decreases.

18. (b) : All combustion reactions are exothermic.

19. (b) : Formation of NH3 proceeds with decreasing of moles of products, hence pressure decreases. It is an exothermic reaction. According to Le-Chatelier principle, to counter the physical changes during reactions, high pressure and low temperature is required. However, at very low temperature rates of reaction decreases considerably, hence reaction is carried out at optimum temperature at the expense of yield.

20. (a) : In K = A + j or, 2.303 log K = A + j

1 log K vs — is linear with a positive slope.

21. (a) : Oxidation number of Fe in compound [Fe(H20)5N0]S04 will be as follows -

x + (0 x 5) + (0 x 1) + 1 x (-2) = 0 => x = +2. Hence, oxidation state of Fe in compound [Fe(H20)5N0]S04 will be +2.

22. (a) : The molecular weight of hydrated sodium thiosulphate (Na2S203-5H20) is 248. From reaction, 2Na2S203 + I2 2NaI + Na2S406

it is clear, that change in oxidation number = 5 - 4 = 1 mol. wt.

Now, E (Na2S203) = c h a n g e i n 0 . N

248 248 5 - 4 1

= 248.

CHEMISTRY TODAY | DECEMBER '05 47

23. (a) : 'Mischmetal' is an alloy of rare earth elements with iron and traces of S, C, Ca and AI. It is used to improve the workability of steel.

24. (b) : The reaction is

C I S CO + HCHO <°ZON°'YSIS C H 3 > C = CH2 X CH, 2 CH3

(F) (E)

- c o 2

(D)

CH,

CH C = CH.COOH 4

HBr

( Q Br

C H 3 V I > C H . C H C O O H

CH, J (S)

CH

CH. ^>CH.CH2COOH

(A) O

Hence (E) and (F) are HCHO and CH3 - C - CH3.

25. (a) : Let the concentration of NH4CI (salt) = x mol/lit

.'. Concentration of NH3 (base) = 0.6 - x [salt] [base]

x

For alkaline buffer, pOH = pKh + log]0

5 = 4.7 + log|0 * => 0.3 = log]0 (0.6- x)

=> logl0 2 = log,0 —— => 2 = — ^ 0.6-x 0 .6 -x

=> x = 0.4 .-. Concentration of [NH3] = 0.2 mol/lit. and concentration of [NH4C1] = 0.4 mol/lit.

-14

0 . 6 - x

A' 1x10" 26. (b) : Kh =

Ka 5x10" Hydrolysis reaction, ASC~ At equi l ibr ium (0.02 - x)

= 0.02

Now by applying the formula,

= 2x10"

HA + OH-x

K,=- 2x10" =-0.02 0.02 0.02

=> x2 = 2 x IO"10 x 0.02 = 2 x IO"10 x 2 x io-2

=i» x2 = 4 x IO"12 => x = 2 x 10"6

=> [OH-] = 2 x 10-6.

... [ H+ ] = l i l i ^ - = 5xlO-9.

2x10" [Since, [H+] x [OH"] = 1 x IO"14].

27. (a) : As, the thermal stability of both the acids and their salts increases with the increasing order of oxidation

5 0

state of halogen or with the increase of the number ol oxygen atoms. So, the correct order of thermal stability for oxyhalide anions CIO , C102- C103", C104~ is as

cio- < cio2- < cio3- < C104 . 28. (a) : Cl2 + 2 B r 2CL + Br2

29. (a) : Such a defect which involves a cation and anion valency in the crystal lattice is called Schottky defect. 30. (b) : Both having the same number of electrons. i.e., 18 electrons.

31. (a) : Because, acidic hydrogens are present in acetylene.

32. (b) : C6H5C1 + Mg d r y e t h e r > C6H5MgCl (A)

C2H5OH + C6H5MgCl C6H6 + Mg

33. (d)

(benzene)

A1C1, + CH3CH,CH2C1 ^

OC2HS

Cl

a CH3 I 3

C H - C H , + HCl

34. (a) : As [H+][OH"] = Kw

But, [H+] = [OH-] = 10"6

[10 6][10-6] = IO12 =

35. (c) : Because, chlorine is more electronegative than bromine. Therefore, it can replace bromine in KBr.

2KBr + Cl2 2KC1 + Br2

36. (a) : Such solutions are said to be isotonic.

38. (c) : NH4C1 + NaN02 N2 + NaCl + 2HzO

39. (b) : 2KBr + Cl2 2KCI + Br2

40. (c) : Ca(OH)2 + CI2 CaOCI2 + H 2 0

41. (b) : In carbon tetrachloride covalent bonding involves sp3 hybridisation. So C - Cl bonds are directed towards four corners of a tetrahedron.

CHEMISTRY TODAY | DECEMBER '05 47

42. (a) : Silver will be precipitated in 1 st group as silver chloride on addition of HCI.

43. (d): Because, the root mean square speed is inversely proportional to the square root of molecular weight.

44. (b): Presence of halogen in the alkyl chain enhances the acidic strength by increasing the ionisation of carboxylic group due to inductive effect.

45. (a) : Hydrolysis o f P C l 3 -PC1, + 3HOH H3P03 + 3HC1.

46. (c) : CaC03 = CaO + CO, (40 + 12 + 48) g 22.4 litres at STP

v 100 g CaC03 produces 22.4 litres C0 2 at STP .'. 50 g CaC03 will produce 11.2 litres C0 2 at S.T.P. 47. (c) : Volume of hydrocarbon taken = 12.5 ml Volume of oxygen taken = 100 ml Volume of C0 2 produced = 50 ml Volume of oxygen left = (68.75 - 50.00) ml = 18.75 ml Volume of 0 2 reacted with hydrocarbon

= (100 - 18.75) = 81.25 ml Let the f o r m u l a of the hydroca rbon be CX¥LY

C H , + x O , - > X C 0 2 + ^ H 2 0

1 vol vo l x vol

12.5 ml 12.5 | x + ^ | m l 1 2 . 5 * ml

5 0 „ 115 Here, 12.5.x = 50

Again, 12.5| x + ^ ( = 81.25

12.5| 4 + ^ | = 81.25 = 10

Therefore, formula of the hydrocarbon = C4H I0.

48. (b) : PA = 2PB, dA = d, TA = 2TB, VA = 2VB, dB = ? P P

As, we know that —j— = —— d,7] a2T2

2 PR . - J j L dATA dBTB

dH = d. d x 2 Tb dBTB

49. (d) : By trial and error method, we can get x = 1 and y = 4.

50. (b)

W l - f c G - Most Powerful Books for CBSE-PMT/AIiMS/AFMC & Other PMT Exams

E X P L O R E R

JSSEBL

Assertion & Reason

Add Rs.35 for postage/handling charge if ordering a single book.

PostageGESlon order of more than one book.

Assertion & Reason

H u m a n Biolosr

M6UH MrPaRef

How to order Send a demand Draft in favour of

'MTG Books' payable at Delhi.

M T G BOOKS 503, Taj A p t , Ring Road, Near Safdarj img Hospital, New Delhi - 29. Tel.: 26194317, 26191601

CHEMISTRY TODAY | DECEMBER '05 47 47

42. (a) : Silver will be precipitated in 1 st group as silver chloride on addition of HCl.

43. (d): Because, the root mean square speed is inversely proportional to the square root of molecular weight. 44. (b): Presence of halogen in the alkyl chain enhances the acidic strength by increasing the ionisation of carboxylic group due to inductive effect.

45. (a) : Hydrolysis of PC13 -PC13 + 3HOH H3PO3 + 3HC1.

46. (c) : CaCOj = CaO + C0 2

(40 + 12 + 48) g 22.4 litres at STP 100 g CaCO, produces 22.4 litres C0 2 at STP

.-. 50 g CaCOj will produce 11.2 litres C0 2 at S.T.P. 47. (c) : Volume of hydrocarbon taken = 12.5 ml Volume of oxygen taken = 100 ml Volume of C0 2 produced = 50 ml Volume of oxygen left = (68.75 - 50.00) ml = 18.75 ml Volume of 0 2 reacted with hydrocarbon

= (100 - 18.75) = 81.25 ml Let the formula of the hydrocarbon be CVH,.

C rHv + • x C 0 2 + ^ H 2 0 2 2 2

1 vol i x + ^ I vo l X vol

12 .5 ml 12.5 x+ — \ m l 12.5A ml

Here, 12.5* = 50 5 0 , x = = 4

Again, 12.5| x + ^ | = 81.25

12.51 4 + —1 = 81.25

12.5

_y = 10

Therefore, formula of the hydrocarbon = C4H]0.

48. (b) : PA = 2PB, dA = d, Ta = 2Tb, Va = 2 Vb, dB = ? P P

As, we know that —— = —— d\T\ diTi

Pa _ PB 2PB PB

d

AT

A d

BT

B dx2Tg dBTB

=> dB = d. 49. (d) : By trial and error method, we can get x = 1 and y = 4. 50. (b)

Wl-fcG" Most Powerful Books for CBSE-PMT/AIIMS/AFMC & Other PMT Exams

CRACK THE PflE BBBtCAl TESfS

E X P L O R E R

Assertion & Reason

Assertion & Reason For CompetiUve Exams.

gg • rap*-, •

Rs.200 'li '" 1

How to order Send a demand Draft in favour of

'MTG Books' payable at Delhi.

Add Rs.35 for postage/handling charge if ordering a single book.

PostageUliiSil on order of more than one book.

MTG BOOKS 503, Taj A p t . , Ring Road, Near Safdarjung Hospital. New Delh i - 29. Tel.: 26194317, 26191601

CHEMISTRY TODAY | DECEMBER '05 47 47

SOLVED PAPER

KERALA PMT - 2005 1. The silver salt of an unknown monoacidic alkyne contains 67.08% silver. The structure of the alkyne is (a) H3C - CH2 - C = CH (b) H3C - C = CH (c) HC = CH (d) H3C - C = C - CH3

(e) either (c) or (d).

2. Match list I with list II and select the correct answer using the code given below the lists.

List I (mixture)

1. o- and p-nitrophenols 2. Naphthalene and urea 3. Heptane and octane 4. Benzene and nitro-

benzene

List II (method for separation)

A. Azeotropic distillation B. Steam distillation C. Simple distillation D. Sublimation

E. Gas chromatography Codes : (a) 1-A, 2-D, 3-E, 4-C (c) 1-B, 2-D, 3-E, 4-C (e) 1-E, 2-D, 3-C, 4-A.

(b) 1-B, 2-D, 3-C, 4-E (d) 1-C, 2-E, 3-D, 4-A

3. 0.4 g of a silver salt of a monobasic organic acid gave 0.26 g pure silver on ignition. The molecular weight of the acid is (atomic weight of silver = 108) (a) 58 (b) 72 (c) 90 (d) 104 (e) 124.

4. Consider the parallel reactions in the electrophilic addition of HBr to propene via the alternative pathways shown below.

path 1 (with peroxide) ^

Propene + HBr path 2 (without peroxide)

1-bromopropene + 2-bromopropene (4) (S)

Identify the wrong statement with reference to the above. (a) path 1 has predominance of (A) over (B) (b) path 2 has predominance of (B) over (A) (c) path 1 is in accordance with anti-Markownikoff's

rule (d) path 2 follows Markownikoff's rule (e) both the paths afford 50% yield of (A) and (B).

5. Relative stabilities of various alkenes represented as R2C = CR2, R2C = CHR, RCH = CHR (trans), RCH — CHR (cis) are in the increasing order of (a) R2C = CR2, R2C = CHR, RCH - CHR (trans),

RCH = CHR (cis) (b) R2C = CR2, R2C = CHR, RCH = CHR (cis),

RCH = CHR (trans) (c) RHC = CHR (trans), RCH = CHR (cis),

R2C = CHR, R2C = CR2

(d) RHC = CHR (cis), R,C = CR2, R2C = CHR, RCH = CHR (trans)

(e) R2C = CR2, R2C = CHR, RCH = CHR (trans), RHC = CHR (cis)

6. Gasoline with an octane number of 90 is equivalent in knocking characteristics to a mixture of heptane and isooctane of the following composition. (a) 20% heptane + 80% iso octane (b) 90% heptane + 10% iso octane (c) 80% heptane + 20% iso octane (d) 10% heptane + 90% iso octane (e) 5% heptane + 95% iso octane.

7. Consider the following carbocations. 1. C13C+ 2. CI,CH+ 3. C1CH2

+ 4. CH3+.

The stability sequence follows the order (a) 4 < 1 < 2 < 3 (b) 1 < 2 < 3 < 4 (c) 4 < 1 < 3 < 2 (d) 4 < 2 < 1 < 3 (e) 4 < 3 < 2 < 1

8. Which of the following is dehydrohalogenated readily at the highest rate?

Br (b) (a)

(c)

(e)

-Br

Br

Br

(d)

Br

9. Which one of the following statements about the nitrogroup in organic compounds is not correct?

189 CHEMISTRY TODAY | DECEMBER '05 47

(a) the N atom is sp2 hybridised (b) the N atom forms a N = O bond and a N —> O

bond (c) the two nitrogen-oxygen bond lengths are different (d) it exists in two canonical forms (e) the nitrogen atom carries a formal charge of +1.

10. Which one of the following molecules is achiral? (a) l-bromo-2-butene (b) 3-bromo-l-butene (c) 2,3-dihydroxy propanal (d) 2-hydroxypropanoic acid (e) 2-chlorobutane.

11. The simplest acyclic alkane containing a chiral atom has a vapour density of (a) 50 (b) 45 (c) 90 (d) 100 (e) 75.

12. Given below are conformations of butane. Which conformation is the most stable? (a) staggered : anti (b) methyl-hydrogen eclipse (c) hydrogen-hydrogen eclipse (d) staggered : Gauche (e) methyl-methyl eclipsed.

13. The raw materials for the commercial manufacture of DDT are (a) chlorobenzene and chloroform (b) chlorobenzene and chloromethane (c) chlorobenzene and chloral (d) chlorobenzene and iodoform (e) chlorobenzene and BHC.

14. Iodoform cannot be prepared by reacting which of the following with NaOH and iodine? (a) CHjOH (b) CH3CH2OH (c) CH3CHO (d) (CH3)2CO (e) (CH3)2CHOH.

15. Identify C in the following scheme.

CH3CH2CH2OH P C ' ? > A ALC- KQH > B

(i) conc. H 2 SQ 4 ^

(ii) H20, heat

(a) propyne (b) propene (c) propanol (d) propanone (e) propanal.

16. The reaction of o-chloroaniline with a mixture of HCl and NaN02 followed by cuprous bromide will give

17. Choose the correct acidity order of the following phenols. (a) />nitrophenol > p-chlorophenol > p-cresol > phenol (b) p-nitrophenol >p-cresol >p-chlorophenol > phenol (c) p-nitrophenol >p-chlorophenol > phenol > /?-cresol (d) p-nitrophenol > phenol >77-chlorophenol > /?-cresol (e) p-nitrophenol >p-cresol > phenol >p-chlorophenol.

18. If diazotisation of aniline is carried out at 30°C, the main organic product formed is (a) phenol (b) phenyl isocyanide (c) diphenyl amine (d) phenyl hydrazine (e) nitrosobenzene.

19. Give the monomers of nylon-66. (a) butadiene and acrylonitrile (b) ethylene glycol and terephthalic acid (c) hexamethylene diamine and adipic acid (d) melamine and formaldehyde (e) phenol and formaldehyde.

20. CH3CHO + HCHO <"1. NaOH J I C N ^ 3 heat H , 0

The structure of compound B is (a) C H , = C H - C H - C O O H

OH (b) CH, = CH - CH - OH

I CN

(c) CH3CH2-CH-COOH

OH CN I

(d) C H , - C H - C O O H (e) C H 3 - C - C O O H

OH OH

21. In DNA molecule, ^ + ^ ratio is 0.9. If the number G + C

of moles of cytosine in the molecule of DNA is 300,000. The number of moles of thymine in the molecule is (a) 240,000 .(b) 150,000 (c) 120,000 (d) 330,000 (e) 270,000

CHEMISTRY TODAY | DECEMBER '05 47 47

22. Nitration of aniline is achieved by (a) direct treatment with nitration mixture under reflux (b) using fuming HN0 3

(c) acetylation followed by nitration and subsequent hydrolysis

(d) KN0 3 + conc. HNOj (e) using dilute HN0 3 in ice.

23. The vulcanized rubber has (a) high water absorption, resistant to oxidation and

good elasticity (b) high water absorption, susceptible to oxidation and

no elasticity (c) high water absorption, susceptible to oxidation and

good elasticity (d) low water absorption, resistance to oxidation and

good elasticity (e) low water absorption, susceptible to oxidation and

no elasticity.

24. Match list I with list II and select the correct answer using the code given below the lists.

List I List II (chemotherapeutic (Examples)

agents) 1. Antibiotic A. Chloroxylenol 2. Antiseptic B. Chlorampheniramine 3. Analgesic C. Chloramphenicol 4. Tranquilliser D. Diclophenac sodium

E. Veronal. Codes: (a) 1-A, 2-C, 3-D, 4-E (b) 1-B, 2-A, 3-D, 4-E (c) 1-B, 2-C, 3-E, 4-D (d) 1-C, 2-A, 3-D, 4-E (e) 1-D, 2-B, 3-C, 4-A.

25. Which of the following is/are true? (i) sucrose is a non reducing agent (ii) glucose is oxidised by bromine water (iii) glucose rotates plane polarized light in clockwise

direction (iv) fructose is oxidised by bromine water. Select the correct answer using the coded given below, (a) (i), (ii), (iii) (b) (i), (ii) only (c) (ii), (iii) only (d) (i), (iv) only (e) all (i), (ii), (iii) and (iv).

26. Which of the following statements about carbon fibres are correct? 1. They can be made from viscose rayon. 2. They are used for making nose tips of missiles.

3. They have low thermal conductivity and used as insulators.

4. They are used for making components of bone plates. Select the correct answer using the codes given below, (a) 1 and 2 (b) 2 and 3 (c) 2 and 4 (d) 1, 2 and 3 (e) 1 ,2 and 4.

27. Match the lists I and II and pick the correct matching from the codes given below.

List I List II A. Peroxy acetyl nitrate 1. Waste incineration B. Polycyclic aromatic 2. Global warming

hydrocarbons C. Dioxins 3. Photochemical smog D. Indigo 4. Carcinogens E. IR active molecules 5. Vat dye. (a) A-3, B-4, C-l , D-5, E-2 (b) A-l , B-2, C-3, D-4, E-5 (c) A-3, B-5, C-l , D-2, E-4 (d) A-5, B-3, C-l , D-2, E-4 (e) A-2, B-3, C-5, D-4, E- l .

28. For the formation of 3.65 g of hydrogen chloride gas, what volumes of hydrogen gas and chlorine gas are required at N.T.P. conditions? (a) 1.12 lit., 1.12 lit. (b) 1.12 lit., 2.24 lit. (c) 3.65 lit., 1.83 lit. (d) 1 lit., 1 lit. (e) 3.55 lit. and 1.83 lit.

29. An alkaloid contains 17.28% of nitrogen and its molecular mass is 162. The number of nitrogen atoms present in one molecule of alkaloid is (a) five (b) four (c) three (d) two (e) one.

30. Which of the following sets of quantum numbers violates the rules of quantum mechanics? (a) n = 3, / = 2, m, - - 3 , ms = +1/2 (b) n = 3, / = 1, TM; = -1 , ms = -1 /2 (c) n = 4, / = 0, mi = 0, ms = +1/2 (d) n = 3, 1 = 2, m, = - 2 , ms = +1/2 (e) n = 5, I = 3, m, = 0, ms = -1 /2

31. In which of the following molecules the central atom does not follow the octet rule? (a) S0 2 (b) BF3 (C) H 2 0 (d) H2S (e) PC13.

32. Which one of the following statements is true?

191 CHEMISTRY TODAY | DECEMBER '05 47

(a) the dipole moment of NF3 is zero (b) the dipole moment of NF3 is less than NH3

(c) the dipole moment of NF3 is more than NH3

(d) the dipole moment of NF3 is equal to NH3

(e) the dipole moment of NH3 is zero.

33. The correct order of the lattice energies of the following ionic compounds is (a) NaCl > MgBr2 > CaO > A1203

(b) NaCl > CaO > MgBr2 > A1203

(c) A1203 > MgBr2 > CaO > NaCl (d) MgBr2 > A1203 > CaO > NaCl (e) A1203 > CaO > MgBr2 > NaCl.

34. A metal crystallises in a bcc lattice. Its unit cell edge length is about 300 pm and its molar mass about 50 g mol"'. What would be the density of the metal (in gem-3)? (a) 3.1 (b) 6.2 (c) 9.3 (d) 12.4 (e) 15.5

35. A crystalline solid has a cubic structure in which tungsten (W) atoms are located at the cubic corners of the unit cell, oxygen atoms at the cube edges and sodium atom at the cube center. The molecular formula of the compound is (a) Na2W03 (b) NaW0 4

(c) NaW03 (d) Na2W04

(e) NaW02 .

36. The rate of diffusion of methane is twice that of x. The molecular mass of x is (a) 16 (b) 32 (c) 80 (d) 64 (e) 128

37. Match list I with list II and select the correct answers using the codes given below. List I (successive List II ionisation energies) (elements)

JEt IE2 IEI (kJ mol-') 1. 2080 3963 6130 A. H 2. 520 7297 11810 B .L i 3. 9000 1758 14810 C. Be 4. 800 2428 3660 D. B

E. Ne Codes : (a) 1-C, 2-B, 3-D, 4-E (b) 1-C, 2-E, 3-B, 4-D (c) 1-D, 2-A, 3-B, 4-D (d) 1-E, 2-B, 3-C, 4-D (e) 1-E, 2-B, 3-D, 4-C.

38. Which of the following properties of interstitial hydrides is incorrect?

(a) they generally form non-stoichiometric species (b) the hydrogen dissolved in titanium improves its

mechanical properties (c) they give rise to powdered metals fit for fabrication (d) on thermal decomposition, they afford a source

of pure hydrogen (e) they can be used as hydrogenation catalysts.

39. The bond angle in water is (a) 120° (b) 109.5° (c) 107° (e) 90°.

(d) 104.5°

40. A substance A on heating gives a colourless gas. The residue is dissolved in water to form B. When excess C0 2 is bubbled through solution of B, C is formed. C on gentle heating re-forms A. The substance A is (a) calcium carbonate (b) sodium carbonate (c) potassium carbonate (d) calcium nitrate (e) sodium bicarbonate.

41. Which of the following statements about alkaline earth metals are correct? (i) Hydration energy of Sr2+ is greater than that of

Be2+

(ii) CaC03 decomposes at a higher temperature than BaC03

(iii) Ba(OH)2 is a stronger base than Mg(OH)2

(iv) SrS04 is less soluble in water than CaS04

Select the correct answer using the code given below, (a) (iv) only (b) (i) and (iii) (c) (i) and (iv) (d) (ii) and (iii) (e) (iii) and (iv).

42. Match the lists I and II and pick the correct matching from the codes given below.

List I List II (metal) (ore)

A. Aluminium 1. Bauxite B. Copper 2. Dolomite C. Potassium 3. Pyrites D. Magnesium 4. Pitchblende E. Radium 5. Sylvine (a) A-l , B-3, C-4, D-5, E-2 (b) A-l , B-3, C-4, D-2, E-5 (c) A-4, B-5, C-2, D-l , E-3 (d) A-4, B-2, C-l , D-5, E-3 (e) A-l , B-3, C-5, D-2, E-4.

43. Which of the following is not correct with regard to the structure of diborane? (a) four terminal hydrogen atoms and two bridged

hydrogen atoms

CHEMISTRY TODAY | DECEMBER '05 47 47

(b) the terminal hydrogen atoms and the boron atoms lie in a plane

(c) four two-centre bonds and two three-centre bonds (d) the two bridged hydrogen atoms lie above and below

the plane (e) the sp3 hybrid orbitals of the boron atoms have

one electron each.

44. Which one of the following statements about halogens is correct? (a) F2 has lower bond dissociation energy than Cl2. (b) The electron affinities are in the order F > Cl > Br > I. (c) HF is the strongest hydrohalic acid. (d) All halogens show variable oxidation states. (e) van der Waal's forces are the weakest in iodine.

45. Match the list I and II and pick the correct matching from the codes given below.

List 1 (substance) A. Helium 1. B. Argon 2.

List II (application) Chlorinating agent Inert atmosphere (Metallurgy) Cryogenics Advertising signs Dehydrating agent.

C. Neon 3. D. P4O10 4. E. PC15 5. (a) A-3, B-2, C- l , D-4, E-5 (b) A-3, B-4, C- l , D-5, E-2 (c) A-3, B-2, C-4, D-5, E-l (d) A-5, B-4, C-l , D-2, E-3 (e) A-5, B-2, C-l , D-3, E-4

46. The correct order of the number of unpaired electrons in the ions, Cu2+, Ni2+, Fe3+ and Cr3+ is (a) Cu2+ > Ni2+ > Cr3+ > Fe3+

(b) Cr3+ > Fe3+ > Ni2+ > Cu2+

(c) Fe3+ > Cr3+ > Cu2+ > Ni2+

(d) Ni2+ > Cu2+ > Fe3+ > Cr3+

(e) Fe3+ > Cr3+ > Ni2+ > Cu2+.

47. Which of the fol lowing forms colourless compounds? (a) Sc3+ (b) V3+ (c) Ti3+ (d) Cr3+

(e) Fe2+.

48. Find the magnetic moment of a divalent ion in aqueous solution if its atomic number is 25. (a) 3.9 B.M. (b) 4.9 B.M. (c) 5.9 B.M. (d) 6.9 B.M. (e) 7.9 B.M.

49. in the nuclear fusion ?H + ?H ,He , the masses

of ^H and jHe are 2.014 mu and 4.003 mu respectively.

The energy released per atom of helium formed is MeV.

(a) 16.76 (b) 26.38 (c) 13.26 (d) 9.31 (e) 23.275

50. In successive emission of p and a particles, how many (3 and a particle should be emitted for the natural (4n + 1 series) conversion of 94Pu241 to 92U233? (a) a , p (b) a , 2p (c) 2a, 3P (d) 2a, P (e) 2a, 20.

51. What will be the binding energy of 1S0, if the mass defect is 0.210 amu? (a) 1.89 x 1016 J mol"1 (b) 1.89 * 1015 J mol"1

(c) 1.89 x 1014 J mol'1 (d) 1.89 x 1013 J mol"1

(e) 1.89 x io12 J mol"1.

52. Match the list 1 and II and pick the correct matching from the codes given below.

List 1 List II

A. (dE/dV) 7. =0 B. JV=-AE C. AE=0 D. AG°

1. Isothermal process 2. -nFE° 3. Adiabatic reaction 4. van der Waal's gas

E. (dT/8P)H* 0 5. Ideal gas (a) A-2, B-l , C-4, D-5, E-3 (b) A-2, B-5, C- l , D-4, E-3 (c) A-3, B-l , C-2, D-5, E-4 (d) A-5, B-4, C- l , D-2, E-3 (e) A-5, B-3, C- l , D-2, E-4.

53. The heats of neutralisation of HCl with NH4OH and NaOH with CH3COOH are respectively -51.4 and -50.6 kJ eq'1. The heat of neutralisation of acetic acid with NH4OH will be (a) -44.6 kJ eq~' (b) -50.6 kJ eq"1

(c) -51.4 kJ eq"1 (d) -57.4 kJ eq"1

(e) -70.2 kJ eq"1

54. Five moles of a gas is put through a series of changes as shown t

graphically in a cyclic \ process. The process during i A -H> B, B C and C A respectively are (a) isochoric, isobaric,

isothermal

temperature

193 CHEMISTRY TODAY | DECEMBER '05 47

(b) isobaric, isochoric, isothermal (c) isothermal, isobaric, isochoric (d) isochoric, isothermal, isobaric (e) isobaric, isothermal, isochoric.

55. For the hypothetical reactions, the equilibrium constant (K) values are given.

A ^ B ; = 2.0 B C ; k2 = 4.0 C ^ D • k3 = 3.0

The equilibrium constant (X) for the reaction A ;=± D is (a) 48 (b) 6 (c) 2.7 (d) 12 (e) 24.

56. The Ka values of formic acid and acetic acid are respectively 1.77 x 10"4 and 1.75 * 10"5. The ratio of the acid strength of 0.1 N acids is (a) 10 (b) 3.178 (c) 0.3 (d) 0.1 (e) 100.

57. Equal volumes of the following Ca2+ and F" solutions are mixed. In which of the solutions will precipitation occur? (K,p of CaF2 = 1.7 x IO"10) 1. IO"2 M Ca2+ + IO"5 M F" 2. 10-3 M Ca2+ + 10"3 M F" 3. 10"4 M Ca 2 + + 10-2 M F-4. 10"2 M Ca 2 + + 10-3 M F" Select the correct answer using the codes given below, (a) in 4 only (b) in 1 and 2 (c) in 3 and 4 (d) in 2, 3 and 4 (e) in all.

58. Which of the following solutions are isotonic with one another? 1. 0.15 M urea 2. 0.05 M CaCl2

3. 0.1 M MgS0 4 4. 0.15 M glucose. Select the correct answer using the codes given below, (a) 1 and 4 (b) 2 and 3 (c) 1, 2 and 4 (d) 2, 3 and 4 (e) 1, 2, 3 and 4

59. 50 ml of 10 N H2S04 , 25 ml of 12 N HCI and 40 ml of 5 N HN0 3 are mixed and the volume of the mixture is made 1000 ml by adding water. The normality of the resul, j solution will be (a) I N (b) 2 N (c) 3 N (d) 4 N (e) 9 N.

60. Equal volumes of ethylene glycol (molar mass = 62) and water (molar mass = 18) are mixed. The depression

in freezing point of water is (given K} of water = 1.86 K moH kg and specific gravity of ethylene glycol is 1.11) (a) 0.0033 (b) 3.33 (c) 0.333 (d) 0.033 (e) 33.3

61. It costs Rs. 10 to electro deposit 1 g of Mg from a solution of MgS04 . The cost of electro depositing of 1 g of Al from A12(S04)3 solution at the same temperature is (atomic weight of Mg = 24, Al = 27) (a) Rs. 10 (b) Rs. 6.66 (c) Rs. 13.33 (d) Rs. 8.4 (e) Rs. 9.6

62. Identify the wrong statement. (a) The oxidation potential for E°2s°c F2/2F~ being the

highest of+2.85 V makes the halogen the strongest oxidizing agent.

(b) Li A1H4 is the best reducing agent in organic synthesis for E°25°c Li+/Li system is -3 .05 V.

(c) HC104 is a powerful reducing agent because the oxidation state of chlorine atom is as high as +7.

(d) A cotton piece dipped in spirit catches fire in Mn207

due to the highest oxidation state of the metal. (e) Manganese atom in Mn04" has a high +ve oxidation

state that allows charge transfer transition, imparting a dark violet colour.

63. How many coulombs of electricity are required for the reduction of 1 mol of Mn04~ to Mn2+? (a) 96500 C (b) 1.93 x 105 C (c) 4.83 x 105 C (d) 9.65 x 106 C (e) 5.62 x 105 C.

64. The experimental rate law for a reaction 2A + 2B —> products, is F<= CACB

m. If the concentrations of both^ and B are doubled, the rate of reaction increases by a factor of (a) VI (b) 2 (c) 272 (d) 4

(e) 4^2

65. What is the time required for a first order reaction to be 99% complete, compared to the time taken for the reaction to be 90% complete? (a) there is no change (b) time taken is double (c) the time taken is triple (d) the time required is half the initial value (e) the reaction is instantaneous.

66. During the decomposition of H 20 2 to give oxygen.

CHEMISTRY TODAY | DECEMBER '05 47 47

48 g 0 2 is formed per minute at a certain point of time. The rate of formation af water at this point is (a) 0.75 mol min 1 (b) 1.5 mol min 1

(c) 2.25 mol min"1 (d) 3.0 mol min"1

(e) 3.5 mol min -1 .

67. If a homo-geneous catalytic reaction can take place through three alternative ™ cs paths, as depicted 'H below, the ° catalytic efficiency of P, Q, R -representing the relative ease would be (a) P> Q> R (c) P> R> Q

Reaction coordinates

(b) Q> P > R (d) R>Q>P

(e) P = Q = R as the initial and final states are same.

68. 10"4 g of gelatin is required to be added to 100 cm3 of a standard gold solution to just prevent its precipitation by the addition of 1 cm3 of 10% NaCl solution to it. Hence, the gold number of gelatin in mg is (a) 10 (b) 1.0 (c) 0.1 (d) 0.01 (e) 0.001

69. Which one of the following will have the highest coagulating power for a ferric hydroxide sol? (a) NaCl (b) BaCl2

(c) K 2 Cr0 4 (d) K3[Fe(CN)6] (e) [K4[Fe(CN)6],

70. Match list I and II and pick the correct matching from the codes given below.

List I L i s t I I (complex) (structure and magnetic

moment) A. [Ag(CN)2]- 1. Square planar and

1.73 BM B. [CU(CN)4]3" 2. Linear and 0 C. [Fe(CN)s]3- 3. Octahedral and 0 D. [ C U ( N H 3 ) 4 ] 2 + 4 . Tetrahedral and 0

E. [Fe(CN)6]4- 5. Octahedral and 1.73 BM. (a) A-2, B-4, C-5, D- l , E-3 (b) A-5, B-4, C- l , D-3, E-2 (c) A- l , B-3, C-4, D-2, E-5 (d) A-4, B-5, C-2, D- l , E-3 (e) A-4, B-2, C-5, D-3, E- l .

71. The number of chloride ion/s produced by comple: tetraamminedichloroplatinum(IV)chloride in an aqueou solution is (a) four (b) two (c) one (d) three (e) none of these.

72. The blue colour obtained in the Lassaigne test is due to formation of the compound (a) Na4[Fe(CN)6] (b) NaCN (c) Fe3[Fe(CN)6]4 (d) Fe4[Fe(CN)6]3

(e) Fe4[Fe(CN)6],

S O L U T I O N S X

1. (a) : Let the structure of silver salt be R - C = C - Ag.

The mass of silver salt which contains 108 g of Ag

= - i ° ° - x ! 0 8 = 161 67.08

.-. R - (12 + 12 + 108) = 161 R = 161 - 132 = 29 or, R = C2H5

Thus the structure of alkyne is CH3CH2C = CH. 1.e. butyne-1. 2. (c)

3. (a) : Mass of silver salt = 0.4 g Mass of silver = 0.26 g Eq. mass of silver salt _ mass of silver salt

Eq. mass of silver mass of silver 0.4

Eq. mass of silver salt = —— xl08 0.26

0.4 E + 107 = ~ ~ r x 108

0.26 E = 59.15 (most probable answer is (a)).

4. (e) : In case of path 1: peroxide

Br I

CH3CH,CH2Br + CH3CHCH3

l-bromopropane 2-bromopropane (major product) (minor product)

In case of path 2:

CHXH — CH, + H+

t CH3CH2CH, ° carbocation (less stable)

Br~

/ CH3CHCH2 2° carbocation (more stable)

Br~

CH3CH,CH,Br 1-bromopropane (minor product)

' CH3CHBrCH3 2-bromopropane (major product)

195 CHEMISTRY TODAY | DECEMBER '05 47

5. (b) : A more substituted alkene is more stable than less substituted alkene owing to higher number of resonating forms. The following order of stability exists in the alkenes. R \ / R R \ / R

R / R R > yC=C

H R cis -H

Rv. / H

R

R \ / H H \ / H > = C < > > = C <

H H H

6. (d) : The octane number of a given sample may be defined as the percentage by volume of iso-octane present in a mixture of iso-octane and heptane which has the same knocking performance as the fuel itself. Octane number 90 means given sample has the knocking performance equivalent to a mixture containing 90% iso-octane and 10% heptane. 7. (b) : The stability of carbonium ions is influenced by both resonance and inductive effects. The groups like - N 0 2 and CI (electron attracting) which have -I-effect reduce the stability of carbonium ions.

CH3 > C1CH2 > C1,CH > C13C

8. ( d ) : • a Br - H B r .

r ^ r B r Among given compounds only gives the

most stable product.

M < / \ ^ The two nitrogen-oxygen bond lengths are not different as in canonical structures, there is only delocalisation of electrons without shifting any atoms.

OH I*

10. (d) : CH, - C - COOH

H *C atom is achiral as all the four valencies are satisfied by four different atoms or groups.

11. (a) : The simplest acyclic alkane containing a chirai H I*

carbon atom would be CH, - C - CH,CH,CH,. I CH,CH3

Molecular wt. of this compound = 1 2 x 7 + 1 x 1 6 = 100 m.wt. 100 c n

Its vapour density = —-— = =

12. ( a ) :

Relative energy

CH3

H ^ f ^ H CH3

Staggered or Anti 0.0 kJ mol"1

CH, CH, H

W T ^ H H

Gauche or Skew 3.35 kJ mol- 1

Relative Partially eclipsed Fully eclipsed

energy 12.13 kJ mol"1 15.06 kJ mol"1

As evident from their relative energies, the decreasing order of the stability of these conformation follows the sequence :

staggered > gauche > partially eclipsed > fully eclipsed.

13. (c) : CCl3CHO + H CI chloral

H - < Q > - C 1

dichlorodiphenyltrichloroethane (DDT)

14. (a): Methyl alcohol does not respond to the iodoform test. Iodoform test is exhibited by ethyl alcohol, acetaldehyde, acetone, methyl ketone and those alcohols which possess CH3CH(OH) - group.

PCI alc KOH 15. (c) : CH3CH2CH2OH ^CH3CH2CH,C1— >

CH3CH — CH?

(B)

H+HSO,

3 2 2 (A)

CH3 - CH - CH3

HSO, CH.CHCH,

3 | 3

OH propan-2-ol

16. (c) CI

NH,

N a N O , ^

HCI '

CI

N2cr

Cu,Br 2

HBr '

CI

Br

6 0 CHEMISTRY TODAY | DECEMBER '05 47

17. ( c ) : Presence o f electron attracting group, (e.g. - N 0 2 , - X, - N R 3

+ , - C N , - CHO, - COOH) on the benzene ring increases the acidity of phenol as it enables the ring to draw more electrons from the phenoxy oxygen and thus releasing easily the proton. On the other hand, presence o f electron releasing group (e.g. - CH3 , - C2H5 , - OCH3 , - N R 2 ) on the benzene ring decreases the acidity of phenol as it strengthens the negative charge on phenoxy oxygen and thus proton release becomes difficult. Thus cresols are less acidic than phenol.

OH OH OH OH

($)>(§)>(§)>(§) N02 Cl ' CH3

p-nitrophenol />-chlorophenol- phenol ^-cresol

18 . ( a ) : When benzene diazonium chloride solution is warmed, phenol is formed with evolution of nitrogen.

N2C1 OH

19. (c)

20. (a) : H - C - H + H-CH2CHO d i l N a O H ; >

O H 1 i—i

CH2 - c m - CHO IT > CH 2= CH - CHO I - A

OH • ~ . H C N > CH, CH - CH - COOH H 3 0 + 2 |

t (B) 21. (e) 22. (c) 23. (d) 24. (d) 25. (a) 26. (e) 27. (a) 28. (a) 29. (d) 30. (a) 31. (b) 32. (b) 33. (e) 34. (b) 35. (c) 36. (d) 37. (d) 38. (b) 39. (d) 40. (a) 41. (e) 42. (e) 43. (e) 44. (a) 45. (c) 46. (e) 47. (a) 48. (c) 49. (e) 50. (e) 51. (d) 52. (e) 53. (a) 54. (a) 55. (e) 56. (b) 57. (d) 58. (c) 59. (a) 60. (e) 61. (c) 62. (c) 63. (c) 64. (c) 65. (b) 66. (d) 67. (d) 68. (d) 69. (e) 70. (a) 71. (b) 72. (d)

For detailed answers, refer to MTG's K e r a l a P M T E x p l o r e r .

MTG Exemplary

Chemistry Teacher Award W e r e a l i z e t h a t t h e r e a r e s e v e r a l g r e a t s c i e n c e

a n d m a t h teachers w h o have d e v o t e d l y s h a p e d t h e f u t u r e s o f t h e i r s t u d e n t s . T h e i r c o n t r i b u t i o n s a r e b a r e l y r e c o g n i z e d a n d a p p r e c i a t e d . To h o n o u r s u c h d e d i c a t e d m a s t e r s a n d t o s h a r e t h e i r t e c h n i q u e s f o r t h e b e n e f i t o f o t h e r s t u d e n t s , w e h a v e d e c i d e d t o c o n s t i t u t e t h e m o n t h l y ' M T G E x e m p l a r y C h e m i s t r y T e a c h e r ' a w a r d .

T h e s tuden ts w h o h a v e b e n e f i t e d a r e t he best s o u r c e o f i n f o r m a t i o n f o r us a b o u t s u c h g r e a t t e a c h e r s . W e inv i te s tuden ts t o p r o p o s e t h e n a m e s o f t e a c h e r s f o r t h e a w a r d by w r i t i n g t o us, g i v i n g t h e f o l l o w i n g i n f o r m a t i o n .

1. S t u d e n t ' s n a m e , c l a s s , s c h o o l , c i ty , c o n t a c t a d d r e s s , p h o n e n u m b e r , e - m a i l .

2 . T e a c h e r ' s n a m e a n d c a r e e r d e t a i l s , c o n t a c t a d d r e s s , p h o n e n u m b e r , e - m a i l , s u b j e c t ( P / C / M / B ) , w h e r e d o e s he t e a c h , w h e n d i d l i e t e a c h y o u .

3 . N a r r a t e a n i n c i d e n t w h i c h d e m o n s t r a t e s t h e t e c h n i q u e s h e / s h e u s e s t o e x p a n d y o u r t h i n k i n g . . . ( 2 0 0 w o r d s ) .

4 . H o w d i d he i n f l u e n c e y o u r l i f e . ( 1 0 0 w o r d s )

S e n d y o u r e n t r i e s t o ..

M T G L e a r n i n g M e d i a (P) L td . 4 0 6 , Taj A p t . R i n g R o a d ,

N e w D e l h i - 1 1 0 0 2 9 e - m a i l : i n f o @ m t g . i n

Every m o n t h t h e s e l e c t e d e n t r y i n c l u d i n g t h e t e a c h e r ' s p r o f i l e w i l l b e p u b l i s h e d in M a t h e m a t i c s T o d a y , C h e m i s t r y T o d a y , P h y s i c s F o r Y o u a n d B i o l o g y T o d a y .

T h e t e a c h e r w i l l a l s o b e a w a r d e d a c e r t i f i c a t e , Rs. 1 1 0 0 c a s h a n d a l i f e - t i m e s u b s c r i p t i o n o f t h e m a g a z i n e .

CHEMISTRY TODAY | DECEMBER '05 47 47

Practice Paper for

West Bengal JEE 2006

5 o n " 5 2 3 r d a n d 24th< ^ A p r i l 2006,

1. Which of the following reacts with nitrous acid to form an alcohol?

(b) (CH3)2NH (d) C2H5X.

(a) CH3CH2NH2

(c) (CH3)3N

2. The reaction

RCH2CH2COOH red P/Br ,

RCHX'H - COOH I Br

is called (a) Reimer-Tiemann reaction (b) Hell Volhard Zelinsky reaction (c) Cannizzaro's reaction (d) Howarth's reaction.

3. When propanone is treated with PC15, it forms (a) trichloropropanone (b) hexachloropropanone (c) 2,2-dichloropropane (d) trichloropropanal.

4. Ketones are less reactive than aldehydes because (a) C — O group is less polar in ketones (b) of electromeric effect (c) of steric hindrance of the attacking reagent (d) none of these.

5. Hexamethylene tetramine is also called (a) benzoin (b) formalin (c) urotropin (d) none of these.

6. When but-3-en-2-ol reacts with aq. HBr, the product formed is (a) 3-bromobutan-l-ene (b) l-bromobut-2-ene (c) a mixture of both (a) and (b) (d) 2-bromobut-2-ene.

7. 1-chlorobutane on reaction with alcoholic potash gives (a) 1-butene (b) 1-butanol (c) 2-butene (d) 2-butanol.

8. Identify Z in the following series. _ T T HBr H2C — CH7

hydrolysis > Y- Z

(a) C2H5I (c) CHI3

6 2

(b) C2H5OH (d) CH3CHO.

9. Which of the following alkyl halides is hydrolysed by SN1 mechanism? (a) (CH3)2CHC1 (b) CH3CH2C1 (c) CH3CH2CH2C1 (d) (CH3)3CC1.

10. For the reaction

C2H5OH + HX ZnCl ,

C2H5X > the decreasing order of reactivity of halogen acid is (a) HI > HCI > HBr (b) HI > HBr > HCI (c) HCI > HBr > HI (d) HBr > HI > HCI

11. Some statements are given below. (A) Silicon is harder than carbon. (B) Silicon has vacant cf-orbital and carbon has not. (C) Carbon forms readily C - C bond but silicon does

not form Si - Si bond. (D) Silicon can displace carbon from Na2C03. Among the above, true statement(s) is/are (a) only B (b) only A and D (c) only B, C and D (d) only B and C.

12. Some statements are given below. A. Astatine is unstable. B. Formula of fluorite is CaF3. C. Bond dissociation energy of I2 < F2. D. Fluorine shows +1 oxidation state. Among the above, the incorrect statement is/are (a) only A (b) only B and D (c) only A, B and C (d) all of these.

13. Fluorine is a better oxidising agent than Br2 because of its (a) small size (b) more electron-electron attraction (c) non metallic nature (d) highest electronegativity.

14. Which of the following does not form polyhalide? (a) (c) 15. (a)

Cl2

h (b) (d)

Br2

F2.

Pick up incorrect statement regarding HF. It is used for making chlorofluorocarbons (CF2C12) used as refrigerating agents.

CHEMISTRY TODAY | DECEMBER '05 47

(b) It is used for removing silica from graphite. (c) Aqueous HF is used for etching glass. (d) HF is stored in glass bottles.

16. The crystal structure of sodium metal is (a) body centered cubic (b) face centered cubic (c) hexagonal (d) tetragonal.

17. Which of the following is the strongest oxidising agent? (a) silicon (b) phosphorus (c) sulphur (d) chlorine".

18. The first ionisation potential of Na, Mg, AI and Si are in the order (a) Na < Mg > AI < Si (b) Na > Mg > AI > Si (c) Na < Mg < AI < Si (d) Na > Mg > AI < Si.

19. C14 isotope in an old wood is one eighth present in fresh wood. If the half life of C14 is 5580 years, the age of wood in years is given by

2.303 log 8 2.303x5580, n (a) x (b) log8 v ; 0.693 5580 v ' 0.693 (c) 5580 log 8 (d) none of these.

20. The decay constant of a radioactive sample is X. The half life and mean life of the sample are respectively

1 • 2 (a) i n -

(c) A,ln2, — \

21. In the reaction: 4Be9 + projectile —> 4Be8 + 0 n \ the projectile is (a) a-particle (b) P-particle (c) positron (d) y-ray.

22. The instability of a nucleus is due to (a) high proton electron ratio (b) high proton neutron ratio (c) low proton electron ratio (d) low proton neutron ratio.

23. A colloidal state can be purified by (a) filtration (b) peptization (c) sedimentation (d) dialysis.

24. Which one of the following forms mono molecular layer? (a) chemisorption (b) chemical adsorption (c) Langmuir adsorption (d) all of these.

CHEMISTRY TODAY | DECEMBER '05 47

25. The substance which gets adsorbed on the surface of a solid is called (a) absorbate (b) adsorbent (c) miscelle (d) catalyst.

26. The salt hydrolysis of the salt of a strong acid and weak base is called (a) anionic hydrolysis (b) cationic hydrolysis (c) amphoteric hydrolysis (d) none of these.

27. Purification of NaCl by passage of HCl gas through brine is based on (a) common ion effect (b) distribution coefficient (c) Le Chatelier's principle (d) displacement law.

28. A mixture of a weak acid (say acetic acid) and its salt with a strong base (say sodium acetate) is a buffer solution. Which other pair of substances from the following may have a similar property? (a) HCl and NaCl (b) NaOH and NaN0 3

(c) KOH and KC1 (d) NH4OH and NH4C1.

29. Solubility product (K,P) (a) varies with temperature (b) does not change with temperature. (c) changes with the change in concentration of ions (d) both (a) and (c).

30. Conjugate base of a strong acid is (a) strong (b) weak (c) neutral (d) none of these.

31. Which of the following is not a Lewis base? (a) NH3 (b) O2" (c) H 2 0 (d) I+.

32. The pH of the solution containing 10 ml of 0.1 N NaOH and 10 ml of 0.05 N H2S04 would be (a) zero (b) 1 (c) greater than 7 (d) 7.

33. If 5 ml of M/100 HCl is added to 10 ml of M/100 HCl, the pH of the mixture will be (a) 1 (b) 2 (o) 3 (d) 4.

34. For equimolar solutions pH is more for (a) HCOOH (b) C6HsCOOH (c) CH3COOH (d) melonic acid.

35. Other things being equal, the life of a Daniel cell may be increased by

47

. 2 1 (b) lnr x (d) none of these.

(a) keeping temperature low (b) using large copper electrode (c) using large zinc electrode (d) decreasing concentration of Cu2+ ions.

36. The electrode potential of an electrode is (a) the potential applied to the electrode (b) the ionisation potential of the material of the electrode (c) the tendency of the electrode to lose or gain electrons

when it is in contact with its ions (d) the potential energy of the electrons in an electrode.

37. The difference of potential of two electrodes of a galvanic cell is called (a) potential difference (b) ionic difference (c) EMF (d) electrode difference.

38. The quantity of current required to produce 15 g of zinc in 200 minutes is (a) 2.67 A (b) 3.69 A (c) 1.67 A (d) none of these.

39. The current of 2.6 amperes was passed through CuS04 solution for 6 minutes and 20 seconds. The amount of copper deposited is (a) 0.3175 g (b) 0.0031 g (c) 6.35 g (d) 3.175 g.

40. In the electrolysis of the fused salt, the weight of the substance deposited on an electrode will not depend on (a) temperature of the bath (b) current intensity (c) time of electrolysis (d) electrochemical equivalent of ions.

41. A system is changed from state A to state B by one path and from B to A by another path. If AE, and AE2

are the correspondin'g changes in internal energy, then (a) A£, + A£2 = +ve (b) A£, + A£2 = -ve (c) AE[ + AE2 = 0 (d) none of these.

42. When ammonium chloride is dissolved in water, the solution becomes cold. The change is (a) endothermic (b) exothermic (c) supercooling (d) none of these.

43. Enthalpy change of a reaction does not depend upon (a) conditions of a reaction (b) initial and final concentrations (c) physical states of reactants and products (d) reaction path.

44. Select the correct order in the following. (a) 1 erg > 1 joule > 1 cal (b) 1 erg > 1 cal > 1 joule (c) 1 cal > 1 joule > 1 erg (d) 1 joule > 1 cal > 1 erg.

45. The heat of formation of a compound (a) is always positive (b) is always negative (c) may be positive or negative (d) is zero at any stage of the reaction.

46. Which of the following compounds has an electrovalent linkage? (a) CH4 (b) MgCl2

(c) SiCl4 (d) I2.

47. Indicate the nature of the bonding in CC14 and CaH2? (a) covalent in CC14 and electrovalent in CaH2

(b) electrovalent in both CC14 and CaH2

(c) covalent in both CCI4 and CaH2

(d) electrovalent in CC14 and covalent in CaH2.

48. Choose the incorrect statement. (a) o bond is weaker than n bond (b) TI bond is weaker than c bond (c) 71 bond is present along with a CT bond (d) G bond can be present alone.

49. Solid NaCl is a bad conductor of electricity because (a) in solid NaCl there are no free ions (b) it is covalent (c) in solid NaCl there is no movement of atoms (d) in solid NaCl there are no electrons.

50. Molten sodium chloride conducts electricity due to the presence of (a) free electrons (b) free ions (c) free molecules (d) atoms of sodium and chlorine.

ANSWERS

1. (a) 2. (b) 3. (c) 4. (c) 5. (c) 6. (c) 7. (c) 8. (c) 9. (d) 10. (b) 11. (c) 12. (b) 13. (d) 14. (d) 15. (d) 16. (c) 17. (d) 18. (a) 19. (b) 20. (b) 21. (d) 22. (d) 23. (d) 24. (d) 25. (a) 26. (b) 27. (a) 28. (d) 29. (a) 30. (b) 31. (d) 32. (c) 33. (b) 34. (c) 35. (a) 36. (c) 37. (a) 38. (b) 39. (a) 40. (a) 41. (c) 42. (a) 43. (d) 44. (c) 45. (b) 46. (b) 47. (a) 48. (a) 49. (c) 50. (b)

6 4 CHEMISTRY TODAY | DECEMBER '05 47

SOLVED PAPER

MANIPUR PMT - 2005 1. An organic compound Xon treatment with acidified K2Cr207 gives Y, which when treated with I2 + NaOH gives triiodomethane. The compound X is (a) CH3OH (b) CH3CHO (c) CH3CH(OH)CH3 (d) CH3COCH3.

2. 2-Acetoxybenzoic acid is an (a) antiseptic (b) antibiotic (c) antipyretic (d) antimalarial.

3. Buna-S is a polymer of (a) butadiene and styrene (b) adipic acid and urea (c) urea and formaldehyde (d) chloroprene.

4. The carbohydrate which can't be hydrolysed by the human digestive system is (a) starch (b) cellulose (c) glycogen (d) all of these.

5. Which of the following bases is found in RNA but not in DNA? (a) adenine (b) uracil (c) thymine (d) guanine.

6. Which of the following is an azo dye? (a) methyl orange (b) malachite green (c) indigo (d) alizarin.

7. At 300 K, the value o f Kc for the reaction l / 2 N , + 0 2 ^ N 0 2 is 2 x 1 o4. What will be the value of K'c for 2NOz ^ N2 + 20 2 ?

1 ( a ) V2 xlO2

(c) 2 x 1 0 4

(b)

(d) 4x10 s

1 x 104.

8. The rate of d i f fus ion of methane at a given temperature is twice that of X. The molar mass of X in g mol"1 is (a) 64 (b) 32 (c) 40 (d) 80.

9. Which of the following sets of quantum number is not possible?

(a) n = 3, / = 0, m = 0 (b) n = 3, / = 1, m = - 1 (c) n = 2, I = 0, m = - 1 (d) n = 2, I = 1, m = 0

10. Which pair of atomic numbers represents s-block elements? (a) 7, 15 (b) 6, 12 (c) 9, 17 (d) 3, 12

11. Which of the following has minimum dipole moment? (a) but-l-ene (b) cis but-2-ene (c) trans but-2-ene (d) 2-methylpropene.

12. For the reaction, C3H8 + 5 0 , w 3CO + 4H 2 0 (/), AH - AE at 2 0.0 constant temperature is (a) RT (b) -RT

(c) 3RT (d) - 3 R T .

13. Which of the following is not a redox reaction? (a) 2H2 + 0 2 2H 2 0 (b) CaC0 3 CaO + C 0 2

(c) Na + H 2 0 NaOH + 1/2 H, (d) MnCl3 -H> MnCl2 + 1/2 Cl2.

14. Which of the following anions is the weakest base? (a) C 2H 50" (b) CN" (c) CH3COO- (d) NO3-.

15. Ammonia gas can be dried over (a) CaCl2 (b) conc. H 2 S0 4

(c> P 2 0 5 (d) CaO.

16. Which of the following leaves no residue on heating? (a) CU(N03)2 (b) NaN0 3

( c ) P b ( N 0 3 ) 2 ( d ) N H 4 N 0 3 .

17. In which of the following there is no S - S bond? (a) S204

2" (b) S2032"

(c) S,062" (d) S207

2- .

18. If a = b / c and a = (3 = y = 90°, the crystal system is

47 CHEMISTRY TODAY | DECEMBER '05 47

(a) cubic (c) tetragonal

(b) rhombic (d) monoclinic.

19. 18 carat gold contains (a) 18% gold (b) 24% gold (c) 75% gold (d) 60% gold.

20. The number of water molecules present in a drop of water having a volume of 0.05 mL and density of 1.0 g mL"1 is (a) 6.022 x 1021 (b) 1.673 x IO21

(c) 6.022 x io22 (d) 1.084 x 1022.

21. In the nuclear reaction, 4Be9 (P, a ) X, X is (a) 2He4 (b) 3Li6

(c) 3Li7 (d) 4Be8.

22. The bond order of 02~ is (a) 0.5 (b) 1.0 (c) 1.5 (d) 2.5

23. Which of the following has the highest boiling point at 1 atm? (a) 0.1 M sucrose (b) 0.1 M NaCl (c) 0.1 M urea (d) 0.1 M BaCl2.

24. In the reaction X + Y —» Z, keeping [X] constant iffy] is doubled, the rate becomes double and keeping [K] constant if [X] is doubled, the rate becomes four times. The order of the reaction is (a) 0 (b) 1 (c) 2 (d) 3.

25. In the following sequence of reactions,

CH3CH2Br K 0 H ( a l c ) > X Y KOH(ALC^ Z

Z is (a) CH2 = CH2 (b) CH2BrCH2Br (c) CH = CH (d) CH3CH3.

A N S W E R S

1. (c) : X acidified K 2 C r 2 0 7

» y I , + NaOH

C H I , iodofonn test

Iodoform test is exhibited by ethyl alcohol, acetaldehyde, acetone, methyl ketones and those alcohols which possess CH3CH(OH) - group.

O acidified K , C r , 0 , II

CH3CH(OH)CH3 2 2 7 > CH3C - CH3

00

—2————> CHL

2. ( c ) : ff

OCCH, COOH ACTS a s a n a n t ipy r e t i c-

2-acetoxybenzoic acid (aspirin)

3. ( a ) : Buna-S rubber is a co-polymer of three moles of butadiene and one mole of styrene. In Buna-S, 'Bu' stands for butadiene, 'na' for sodium (Na) which is polymerizing agent and 'S ' stands for styrene.

CH = CH,

« C H 2 = C H - C H = C H 2 + « [ Q 1,3-butadiene

styrene

C H , - C H = C H - C H 2 - C H - C H 2 - \

gi Buna-S

4. (b) : Cellulose is insoluble in water and in most of the organic solvents.

5. (b) 6. ( a ) : The azo dyes contain one or more azo groups. - N = N - as the chromophore.

H O 3 S < ^ - N = N - ( o ) - N(CH3)2

methyl orange

1 7. (b) : - N 2 + 0 2 N02 ; KC = 2 x io4

For N0 2 — ^ N 2 + 0 2 , KC] = ^ R

Hence for 2NO, ;—^ N, + 202 ,

1 1 Kr< = (2x10 ) 4x10s

8. ( a ) : - = 2 = A4 M n

or, 2 = ,

or, 4 = -MY

16 M y =64

a 2

9. (c) : For n = 2, / = 0, 1 For / = 0, m = 0 (only one value).

10. (d) : In i-block elements, the last electron enters

CHEMISTRY TODAY | DECEMBER '05 47 47

the ns energy shell. The maximum capacity of ns energy shell is o f two electrons, thus these elements have valency shell configuration either ns1 or ns2.

11. (c) : Trans-homers have zero dipole moment and are non-polar.

1 2 . ( d ) : C 3 H 8 w + 5 0 2 ( r ) 3 C 0 2 ( g ) + 4 H 2 0 (/) An = 3 - (6) = - 3 . AH= AE + A nRT AH-AE = - 3 RT.

1 3 . ( b ) : C a C 0 3 CaO + C 0 2

This is an example o f decomposit ion reaction and there is no change in oxidation number. So it is not a redox reaction.

1 4 . ( d ) : Among the given options, H N 0 3 is the strongest acid, so its conjugate base N 0 3 ~ ion wil l be weakest base.

1 5 . ( d ) : For drying o f ammonia gas, the c o m m o n dehydrating agents like H 2 S 0 4 or CaCl2 or P 2 0 5 cannot be used as these react with NH 3 . However, quick lime, is used as it does not react with N H 3 but reacts readily with moisture.

CaO + H , 0 - > Ca(OH) 2

quick lime

1 6 . ( d ) : Alkali nitrates form nitrites and oxygen on heating.

2 N a N 0 3 - » 2 N a N 0 2 + 0 2

Nitrates o f alkaline earth metals and heavy metals d e c o m p o s e to g ive mixture o f nitrogen dioxide and oxygen along with a solid residue of metal oxide.

2 P b ( N 0 3 ) , 2PbO + 4 N O , + 0 2

2 C U ( N 0 3 ) 2 - > 2CuO + 4 N 0 2 + 0 2

A m m o n i u m nitrate decomposes to form nitrous oxide gas.

N H 4 N 0 3 N 2 0 + 2 H 2 0

9 0 _ II I!

1 7 . ( d ) : S 2 0 42 " : O - S - S - O "

O

S , 0 32 ~ : O - S - O "

O O II II

: o - s - s - c r II II o o

o o

S 2 0 72 " : O - S - O - S - O "

II II o o

18. (c) : For tetragonal : a = b * c , a = P = y = 90° , two equal edges, all right angles, five planes and f ive axes.

18 1 9 . (c) : 18 karat gold means ^ - x 100 = 75%.

18 karat gold means that 18 parts o f the jewelry is gold and the balance 6 parts is s o m e other metal or equal to about 75 percent gold plus 25 percent o f some other metal alloy.

2 0 . ( b ) : Mass = vo lume x density = 0 .05 x i.o = 0.05 gm

Number o f molecules in 18 g water = 6 .023 x 1023

6 . 0 2 3 x 1 0 " N o . of molecules in 0 .05 e water = x 0.05 6 18

= 0 .01673 x 1023 = 1.673 x 1021 .

2 1 . ( b ) : 4 Be 9 + ,H' 3Li6 + 2 He 4

This is an example o f p, a reaction.

22. (c) : 02" - ( a l J ) 2 (a* Is)2 (o2s)2 (a*2s)2 (o2px)2

(7t2Py)2 (n2Pz)2 (n*2Py)2 (n*2p.y 1 0 - 7 3 , c

Bond order = — - — = T = 1 - 5

2 2 2 3 . ( d ) : Boi l ing point is a col l igat ive property which depends upon the number o f molecules / ions . Among the given options, BaCl 2 has greater number o f ions, i.e. it has highest boi l ing point.

24. (d) : X + Y Z If K is doubled, rate becomes double i.e. with respect to Y, order o f reaction = 1. I f X is doubled, rate becomes four times i. e. with respect to X, order o f reaction = 2. Overall order o f reaction = 1 + 2 = 3.

H H I I

2 5 . (c) : H - C - C - H + KOH (alc.) >

H Br

Br Br

H H I I

H - C = C - H + HBr w

CH2 = CH2 CH, — CH, C H = C H X

2 2

Y

74 CHEMISTRY TODAY | DECEMBER '05 47

Genius RULES

AI! students preparing for PMT/PET examinations can participate in W f c t / G " Chem-Genius Contest. Answers marked only on the entry form of the magazine / photocopy of form will be accepted. More than one response to a question will be disqualified. Prizes

• 1st Prize-LG Mobile phone • 2nd Prize-Adidas Bag • 3rd Prize - MTG Books (worth Rs. BOO/-) The entries with maximum number of correct answers for three consecutive months (February '08 - April '08) will be awarded 1 st prize. 2nd and 3rd prize will be given to the next maximum scorers. In case of a tie, the winners wil l be decided through a lucky draw. The decision of the editor wil l be final and binding in all cases and will not be a matter for consideration of any court and no correspondence wil l be entertained. Name and photograph of the prize winners of this contest wil l be published in the June issue of 2008. MTG is not responsible for any postal delays, transit losses or mutilation of entries. Last Date The entries should reach on/before 31st March '08 to - WfcfcGr Chem-Genius Contest-3,406, Taj Apartment, Ring Road, Near Safdarjung Hospital, New Delhi-29. [Note : Enclosures include a passport size photograph and a photocopy of age proof.]

r. TI has how many significant number ? (a) 4 (b) zero (c) 3 (d) an infinite number

2. A 1.0 g projectile is shot from a gun with velocity 100 m/s. What is the de Broglie wavelength ?

1-34 cm (a) 3.33 x IO"34 cm (b) 6.63 x 10 (c) 3.33 x 10"33 m (d) 6.63 x 10~33 m

The third line in the Balmer series corresponds to an electronic transition between which Bohr orbits in hydrogen ? (a) 3 —> 2 (b) 4—>2 (c) 2 -s> 1 (d) 5 2

4. The dipole moment of HBr is 2.60 x IO3 0 cm, and the interatomic spacing is 1.41 A. What is the percent ionic character of HBr ? (a) 12.9% (b) 11.5% (c) 13.1% (d) 13.9%

5. A litre of milk weighs 1.032 kg. The butter fat it contains to the extent of 4.0% by volume has a density of 865 kg/m3. What is the density of the fat-free skimmed milk ? (a) 1080 g/cm3 (b) 1040 kg/m3

CHEMISTRY TODAY | MARCH '08 204

(c) 1080 kg/m3 (d) 1140 g/cm3

6. A gaseous compound is composed of 85.7% by mass carbon and 14.3% by mass hydrogen. Its density is 2.28 g/L at 300 K and 1.00 atm pressure. Determine the molecular formula of the compound. (a) C 2 H 2 (b) C 3H 4 ( C ) C 4 H 8 ( d ) C 4 H 4

7. The density of a 2.0 M solution of acetic acid (MW= 60) in water is 1.02 g/mL. Calculate the mole fraction of acetic acid. (a) 0.0038 (b) 0.0380 (c) 3.80 (d)4.12

A saturated solution of silver benzoate, AgOCOC6H5, has a pH of 8.63. Ka for benzoic acid is 6.5 x 10 5. Estimate the value of Ksp for silver benzoate. (a) 1.4 x io^ 3 (b) 1.4X10-2

(c) 1.96X10"2 (d) 1.96X10-3

, The activity of 30 mg of 247Cm is 2.8 nCi. Calculate the half-life of 247Cm. (a) 1.5 x 109 years (b) 2.2 x 109 years (c) 1.5 x 107 years (d) 1.9 x 108 years

10.Calculate the partial pressure of carbon monoxide

from the following data: CaCO — 3(j) > CaO(j) + C021\ Kp = 8 x l(H C0 2 ( f f ) + C w - > 2 C 0 f e ) , A , = 2 (a) 0.2 (b) 0.4 (c) 1.6 (d)4.

The orbital diagram in which aufbau principle is violated

w rmmif i w (d) (c)

11 t l t

11 1 t 1

1 11 1 1

11 11 11 1 '2 Which of the following pairs of compounds are

isoelectronic and isostructural ? (a) N0 3- C03

2" (b) N0 3- C103 J3 , (c) CO/-, S03 (d) N03 , S03

The oxidation number of sulphur in S8, S2F2 and H2S are (a) 0, +1 and - 2 (b) +2, +1 and - 2 (c) 0, +1 and +2 (d) -2, +1 and -2 Oxidation state of nitrogen is incorrectly given for

Compound Oxidation state (a) [Co(NH3)5Cl]Cl2 - 3 (b) NH2OH - 1 (c) (N2H5)2S04 +2 (d) Mg3N2 - 3 The metal that cannot be produced on reduction of its oxide by aluminium is

(d) all

(a) K (b) Mn (c) Cr (d) Fe 16. 1,3-butadiene has hybridization.

(a) sp (b) sp2 (c) sp3

17. At absolute zero (a) only para hydrogen exists (b) only ortho hydrogen exists (c) both para and ortho hydrogen exist (d) none of these

18. The nitride ion in lithium nitride is composed of (a) 7 protons +7 electrons (b) 10 protons +7 electrons (c) 7 protons +10 electrons (d) 10 protons +10 electrons

19. Which ion form hydroxide easily soluble in water ? (a) Zn2+ (b) Ba2+ (c) Mg2+ (d) Al3+

20. Al dissolves in molten NaOH with the formation of H2 and (a) sodium aluminate (Na3A103) (b) sodium metaaluminate (NaA102) (c) aluminium hydroxide (d) alumina

21. A salt X gives white precipitates with lead acetate solutions insoluble in hot water and nitric acid. The salt X most probably contains

PRIZE W I N N E R S WfcfcG- Chemgenius Contest-2

I s t Pr ize

Sahi l Makh i ja 2-A/14, NIT, Faridabad,

Haryana -121 001

I In d Pr ize r

G a u r a v S i n g h Near Saraswati Shishu Mandir,

Near Nirala Nagar.Rewa, M.P. 486002

I I I r d Pr ize

N i p u n Gupta Rajwada Complex, Dhar, M.P. 454 001

(Other Highly Commendable Contestants) • Ram Krishna Mishra (Sultanpur, U.P.) • Aniket Deb (Kota, Rajasthan)

• Monali Sahu (Balasore, Orissa) • Dipti Bisht (Lucknow, U.P.) • Tilottama Turkar (Navi Mumbai) "Heartiest Congratulations and Best

Wishes for bright future to all winners!' Prizes are being sent.

CHEMISTRY TODAY I MARCH '08 3 I

(a) Cl- (b) Ba2+ (c) S042" (d)C03

2" Which compound is formed when excess of KCN is added to aqueous solution of copper sulphate? (a) Na2C03 (b) CaC03

(c) MgC03 (d) NaHC03

The plot of log k versus 1/T of a reaction is linear with a (a) positive slope and zero intercept (b) positive slope and nonzero intercept (c) negative slope and zero intercept (d) negative slope and nonzero intercept. Which oxide of nitrogen is the most stable? (a) 2 N 0 2 f e ) ^ N 2 f e ) + 202 ( g ) ;

f = 7 . 6 x 1016 mol litre"1

(b) 2N2Ofe) — 2 N 2 f e ) + 0 2 ( g ) ; K= 5.3 x 1033 mol litre"1

(c) 2N205fe) 2N2(?) + 502 ( g ); K = 1.2 x 10 3 4 m o l l i t r e 1

( D ) 2 N 0 T E ) ^ N 2 ( G ) + 0 2 T E ) ;

K= 2.2 x IO30 mol litre"1. The active ingredient in aspirin is acetyl salicylic acid

O O n 11

0 - C - C H 3 O - C - C H J

r ^ r c o o H r r ^ r c o ° " + [or + H2O [OJ +H3O+

withXa = 2.75 x IO"9. The pHofthe solution obtained by dissolving two aspirin tablets (containing 0.32 g of acetyl salicylic acid in each tablet) in 250 ml of water is (a) 5.20 (b) 4.20 (c) 3.20 (d)7.20.

The hydroxide with highest solubility product is (a ) A l ( O H ) 3 ( b ) C O ( O H ) 2

(c ) C r ( O H ) 3 (d ) F e ( O H ) 3

Of the following reactions, only one is a redox reaction, identify it. (a) Ca(OH)2 + 2HC1 CaCl2 + 2H20 (b) BaCl2 + MgS04 - » BaS04 + MgCl2

(c) 2S2072 + 2 H 2 0 - ^ 4 S 0 4

2 - + 4H+

(d) Cu2S + 2FeO 2Cu + 2Fe + S02

According to Kohlrausch law, the limiting value of molar conductivity of an electrolyte A2B3 is (a) X \ 3 + +X°b2_ (b)

( c ) 2X°A3+ + 3 A . V " (d ) 2\°a2+ - 3 A . V -

n>. Actinium disintegration series starts with A and ends

at Z, A and Z are (a) A = 90Th232, Z = Pb-206 (b) ^ = 9 2U2 3 5 ,Z=Pb-207 (c) A = 92U238, Z = Pb-208 (d) A = 8 6AC 2 2 7 , Z = Bi-209

30. Pick out the corrcct statement. (a) A catalyst speeds up the reaction by increasing

the activation energy (b) A catalyst speeds up the reaction by decreasing

the activation energy keeping the reaction mechanism the same

(c) A catalyst speeds up the reaction by providing an alternate path of lower activation energy

(d) A catalyst alters the positions of equilibrium.

31. How many layers are adsorbed in chemical adsorption? (a) One (b) Two (c) Many (d)Zero.

32. Distribution of molecules with velocity is represented by the following curve. Velocity corresponding to point A is

« JW <«> M

33. Which of the following electrolytes is least effective in causing flocculation of ferric hydroxide sol? (a) K3Fe(CN)6 (b) K2Cr04

(c) KBr (d) K2S04

34. Which of the following double sulphate is not an alum ? (a) K2S04.A12(S04)3.24H20 (b) K2S04.Cr2(S04)3.24H20 (c) Na2S04.Fe2(S04)3.24H20 (d) CuS04.Al2(S04)3.24H20

35. The IUPAC name of CH2—CH - CH2 - C = C H is (a) Pent-l-en-4-yne (b) Pent-4-en-l-yne (c) Pent-2-en-5-yne (d) Pent-4-en-2-yne

36. Which of the following sets of quantum numbers is not possible ?

66 CHEMISTRY TOO AY MARCH '08

n I

(a) 5 1

(c) 5 1

0 2

0 \ (b) 3

(d) 3

2 - 3 ± 1 i 2 2 - 2 \

Formation of salicylic acid is by reaction (a) Kolbe's electrolytic reaction (b) Perkin reaction (c) Reimer-Tiemann reaction (d) Hoffmann's ammonolysis

38. Consider the general reaction A + B —> C + D. It is exothermic in forward direction. When we double the temperature by 10°C then the value of equilibrium constant will be (a) Halved (b) Doubled (c) Unchanged (d) Trippled

39. Which of the following denotes German silver ? (a) Cu + Al + Ni (b) Ni + Zn + Cr (c) Cu + Zn + Ni (d) Cu + Cr + Sn

40. Dry ice is a (a) KMn04

(b) K2S04-A12(S04)3-24H20 (d) Solid C02

(d) Sodium hexametaphosphate 41. Two solutions having pH 1 and 2, both are mixed,

then pH will be (a) 1.76 (b) 0.96 (c) 1.26 (d) 1.5

42. Carbon-carbon bond distance in benzene molecule is (a) 1.397 A (b) 1.54 A (c) 1.30 A (d) 2.397 A

43. Predict the final product B in the sequence of reaction:

60% H , S 0 4 H C = C H ,. ' 4>A- B H g S 0 4 " " N a O H

(a) CH3COONa (b) CH3COOH (c) CH3CHO

H I

(d) CH3 - C - CH2 - CHO

OH

44. The fatty acid that shows reducing property is (a) Ethanoic acid (b) Acetic acid (c) Formic acid (d) Oxalic acid

45. Assertion : In any ionic solid MX with Schottky defects, the number of positive and negative ions are same.

Reason : Equal number of cation and anion vacancies are present

(a) If both assertion and reason are correct and reason is the correct explanation of the

assertion (b) If both asertion and reason are correct, but reason

is not the correct explanantion of assertion (c) If assertion is correct, but reason is incorrect (d) If assertion is incorrect, but reason is correct

46. The compound C5HI3N is optically active and reacts with HONO to give C sHnOH. The compound is (a) N-methylbutanamine (b) 2-Aminopentane (c) 1 -Aminopentane (d) N,N'-Dimethylpropanamine

47. The active species which takes part in the sulphonation of benzene is (a) S03+ (b) S03 (d) S02+ (d)HS04"

48. The end product B of the reaction,

HCHO/HCl . N a A Ether ZNCL,

( . ( O X O )

(B) < ^ C H 2 - < @ >

w ( 0 ) - C H 2 - C H 2 ^ O )

(1) < o > CH2 — CH2C1

49. Which of the following compounds is formed when excess of benzene reacts with CH2C12 in presence of anhydrous A1C13?

(a)

(b)

(c)

-CH2C1

CI

CI

(d) < O ^ C H 2 - < g >

50. Which compound is prepared by the following reaction

Ni vessel Xe + 2F 2 6 7 3 K , 5 - 6 a t m

(a) XeF2

(c) XeF4

(b) XeF6

(d) XeOF2

CHEMISTRY TODAY | MARCH '08 207

j WB-JEE BHU CMC "CPMlTwianipal PMT AIIMS fBSE PMT

iH i i iMMi^mHt tHi iNH MGIMS t»MDT Bihar TNPCEE CET Karnataka PMT Haryana

An element has a body-centred cubic (bcc) structure with a cell edge of288 pm. The density of the element is 7.2 g/cm3. How many atoms are present in 208 g of the element? (a) 6.02 x 1024 atoms (b) 12.09 x 1023 atoms (c) 24.16 x io23 atoms (d) 29.88 x io24 atoms Calculate the mole fraction of ethylene glycol (C2H602) in a solution containing 20% of C2H602 by mass. (a) 0.0963 (b) 0.0932 (c) 0.068 (d) 0.68

Give the systematic name for —(—/ \ — i s (a) r-butylcyclohexylcyclopropane (b) 1 -/-butyl-4-cyclopropylcyclohexane (c) 2,2-dimethyl-3-(cyclopropylcyclohexyl)propane (d) 1 -cyclopropyl-4-/-butylcyclohexane. Nitrolium is a mixture of (a) Ca(CN)2 and graphite (b) CaC2 and graphite (c) CaCN2 and graphite (d) CaCN2 and N2

The first emission line in the atomic spectrum of hydrogen in the Balmer series appears at

(a) § cm"' (b) ^ c n f 1

, , , 9 R -i (d) M o m

7.

8.

The binding energy of an electron in the ground state of He atom is equal to E0 = 24.6 eV. The energy required to remove both the electron is (a) 24.6 eV (b)13.6eV (c) 79.0 eV (d) 54.4 eV

Styrene can be prepared from (a) Ethyl benzene (b) Cinnamic acid (c) 1-Phenylethanol (d) All of these

The analysis of a rock shows that the relative number of 206Pb and 238U atoms is ^ = 0.25 . If tm for the

reaction 238U —> 206U is 4 x 109 yrs, the age of rock (in years) is

(a) § § f ( 4 x l 0 > g | (b) a 6 9 3 2 ' 3 0 3 ( 4 x l 0 > g i

W H i ( 4 x l 0 9 ) l o g 4 (d) f f ( 4 x l 0 > g f 0.693v

What is the maximum possible concentration of Ni2+ ions in a solution which is 0.15 M in HCl and 0.1 M in H2S. Given that K„miS) = 2 x IO"21 and [S2_]h2S = 4 X IO - 2 1

(a) 0.65 M (b) 0.45 M (c) 0.10 M (d) 0.15 M NaOH can be used to separate (a) C u S , AS 2S 3 (b) AL(OH)3 , C r ( O H ) 3

(c) Both (a) and (b) (d) None of these Although aluminium has a high oxidation potential, it resists corrosion because of the formation of a tough, impervious surface coat of (a) A1N (b) A1203

(c) A1(N03)3 (d) A12(C03)3

The correct expression relating molarity (M), molality (m), density (d) and molar mass (M) of solute is „ M

M (a) T ^ M M 7 (b) ~

(c) m =

d-MM' d + MM'

(d)

' d + MM' d-MM'

M w M For the concentrated solution of a weak electrolyte AxBy the degree of dissociation is given by (a) a=^Keq!C(x+y) (b) a = ^ K e q C / x + y

(c) a= AT \Mx+y

xyyCx+y~

(d) a^Keq!xyC

In steam distillation of toluene, the pressure of toluene in vapour is (a) Equal to pressure of barometer (b) Less than pressure of barometer (c) Equal to vapour pressure of toluene in simple

distillation (d) More than vapour pressure of toluene in simple

distillation

0.025 g of starch was added to 10 ml of gold sol when 1 c.c. of 10% NaCl was added to this gold sol no precipitation occurs. What is the gold number of starch?

64 CHEMISTRY TODAY MARCH '08

(a) 0.025 (b) 0.25 (c) 2.5 (d) 25

Least melting point is shown by the compound (a) PbCl2 (b)SnCl4 (c) NaCl (d)AlCl3

Enthalpy change of heterolytic fission of C - CI bond is given as CH3 - CI ->> CH3 + CI" AHx °

CH3CH2CI CH3CH2 + c r AH2°

(CH3)2CHC1 (CH3)2CH + CI" Ah3°

(CH3)3CC1 (CH3)3C+ + CI- A H °

(a) AH; > AH; > AH; > AH; (b) AH; < AH; < AH; < AH; (c) AH; < AHI < AH; < AH; (d) AH; > AH; > AH; > AH;

2 is the equilibrium constant for the reaction at a constant temperature. What is the degree of dissociation for A2 or fi2?

(a) 0.2 (b) 0.5 W ^ W ] ^

For the concentration cell Zn|Zn2+(M,)|jZn2+(M2)|Zn, the value of AG will be negative if (a) Mx = M2 (b) M, > M2

(c) M1 < M2 (d) none of these

What is the pH of the mixture 100 ml IN HCI + 60 ml IN NaOH ? (a) 0.50 (b) 0.40 (c) 0.60 (d) 0.20

In P4Oio molecule the number of oxygen attached to each phosphorus atom is (a) 2 (b)3 ( c ) 4 (d)5

The percentage ionic character of a bond having 1.275 A its length and 1.03 D its dipole moment is (a) 10% (b) 15% (c) 16.83% (d) 18.8%

Which species has the maximum number of lone pair of electrons on the central atom ? (a) CIO3- (b) XeF4 (c) SF4 (d) [I3]"

Measurement of the amount of dry gas collected over water from volume of moist gas is based on (a) Gay Lussac's law (b) Dalton's law of partial pressure (c) Boyle's law (d) Avogadro's hypothesis

The standard free energy change AG° is related to equilibrium constant KP as ( a ) K p = e-«™r ( b ) ^ = z | p !

(c) Kp — R1VAAG° (d) AG = e'K"'RT

An amount of solid NH4HS is placed in a flask already containing ammonia gas at a certain temperature and 0.50 atm pressure. Ammonium hydrogen sulphide decomposes to yield NH3 and H2S gases in the flask. When the decomposition reaction reaches equilibrium, the total pressure in the flask rises to 0.84 atm. The equilibrium constant for NH4HS decomposition at this temperature is (a) 0.30 (b) 0.18 (c) 0.17 (d)0.11

For the chemical reaction: N2(g) + 3 H 2 f e ) ; = ^ 2NH3fe | at 500°C, the value of

KP is 1.44 x 10-5, when partial pressure is measured in atmosphere. The corresponding value of KC with concentration mol litre-1 is (a) 1.44 x 10 5/(0.082 x 500) 2

(b) 1.44 x 10-7(8.314 x 733)~2

(c) 1.44 x 10-5/(0.082 x 773)2

(d) 1.44 x 10-7(0.082 x 773) 2

Highest pH is given by (a) 0.1 M H2S04 (b) 0.1 M NaOH (c) 1 N NaOH (d) 1 N HCI

Which of the following metals give hydrogen with very dil. HN03? (a) Al (b) Mg (c) Au (d) Sn

Inert pair effect is predominant in (a) Si (b) Pb (c) Ge (d) Sn

Which of the following is not a protonic acid? (a) B(OH)3 (b) PO(OH)3

(c) SO(OH)2 (d) S02(0H)2

0.4 gm of an organic compound gave 0.188 g of silver bromide by a halogen estimation method. The percentage of bromine in the compound is (atomic mass o f A g = 108, Br = 80) (a) 39.8% (b) 46.0% (c) 20.0% (d) 40.0% The ozonolysis of (CH3)2C = C(CH3)2 followed by treatment with zinc and water will give (a) acetone (b) acetaldehyde and acetone (c) acetic acid (d) formaldehyde

Which of the following resonating structure of 1-methoxy-l, 3-butadiene is least stable ?

(a) CH2 - CH = CH— O - CH3

• (b) CH2 - C H = C H = 0 - C H 3

(c) CH2 - CH = C H = 0 - CH3

(d) CH2 - CH = CH = 0 - CH3

CHEMISTRY TODAY | MARCH '08 209

(b) CH2 = CH CH = O (d) C H 3 C H 2 0 - N = 0

PAN stands for (a) CH2 = O (c) CH,COONO,

II O

Allyl isocyanide has (a) 9 sigma and 4 pi bonds (b) 8 sigma and 5 pi bonds (c) 8 sigma, 3 pi and 4 non-bonded electrons (d) 9 sigma, 3 pi and 2 non-bonded electrons Intermolecular hydrogen bonding is strongest in (a) methylamine (b) phenol (c) formaldehyde (d) methanol

Which of the following compounds has isopropyl group ? (a) 2,2,3,3-tetramethylpentane (b) 2,2-dimethylpentane (c) 2,2,3-trimethylpentane (d) 2-methylpentane

C7H80 shows how many isomers? (a) 2 (b)3 ( c ) 4 (d) 5

The most stable conformation of 1,2-diphenylethane is

c 6 H 5

Arrange the following nucleophiles in the order of their nucleophilic strength (a) OH" > CH3COO- > OCH3- > C 6 H 5 0 "

( b ) C H J C O O " < C 6 H S O - < C H 3 O - < O H "

( c ) C 6 H 5 0 - < C H J C O O " < C H 3 O - < O H

( d ) C H J C O O - < C 6 H 5 0 " < O H - < C H 3 O -

A hydrocarbon of formula C 6 H 1 0 absorbs only one molecule of H2 upon catalytic hydrogenation. Upon ozonolysis, the hydrocarbon yields a compound of the following structure,

H O I II

o = c - CH 2 - CH 2 - CH 2 - CH 2 - C - H

The hydrocarbon is (a) cyclohexane (c) cyclohexene

(b) cyclohexyne (d) cyclobutane

Which of the following does not have chirai centre? CH,

(a) HO

HOOC COOH OH (b)

OH

( c ) X = \

CH,

CH2OH (d)

Br CH2CH3

The correct order of basicities of the following compounds is

, N H C H 3 - C ^

(1) N H 2

C H 3 - C H 2 - N H 2

(2)

o

C H 3 - C - N H 2 (CH3)2NH

( 3 ) " ( 4 )

(a) 2 > 1 > 3 > 4 (b) 1 > 3 > 2 > 4

(c) 3 > 1 > 2 > 4 (d) 1 > 2 > 3 > 4

Diabetes is detected by testing urine of the patient with (a) Tollen's reagent (b) Nessler's reagent (c) Fenton's reagent (d) Benedict's solution Oxidation of glucose is one of the most important reactions in a living cell. What is the number of ATP molecules generated in cells from one molecule of glucose? (a) 36 (b) 12 (c) 38 (d) 28 Match List I with List II and select the correct answer from the given codes.

List I List I I (a) First amino acid (i) Glycine

synthesised in lab (b) Sulphur containing amino (ii) Cysteine

acids

(c) Amino acids of highest iso- (iii) Methionine electric point

(d) Optically inactive amino (iv)Arginine acid a b e d a b e d

(a) (i) (ii) (iii) (iv) (b) (iii) (ii) (iv) (i) (c) (iii) (ii) (iv) (i) (d) (iv) (iii) (ii) (i) The monomer of dacron is/are :

C O O H

(a) HOCH2 - CH2OH and COOH

66 CHEMISTRY TOO AY MARCH '08

(b).

O

•COOH (c) HOCH2 - CH2OH and HOOC«

(d) F 2 C = C F 2

49. In the sequence of reaction

A—-—>5 — > C — ^ — ^ D k3 > k2 > kt, then the rate determining step of the reaction is (a) A B (b) B C (c (d )A-^D

50. Which is not a disproportionation reaction? ( a )

CHO CH2OH COO" | + OH > | + |

(b) COOH COO- COO"

(c) NaH + H 2 0 NaOH + H2

(d) 3CIO (aq) C103 (aq) + 2C1 -( ,

SOLUTIONS

1. (c) : Volume of the unit cell = (288 pm)3

= (288 x IO12 m) = (288 x 10 10 cm)3

= 2.39 x 10"23 cm3

Volume of 208 g of the element

= mass_= 2 0 8 g -28 .88 cm3

density 7 .2gem - 3

Number of unit cells in this volume

= 28.88 cm3

2.39 xlO"23 cm3 /unitcell = 12.08 x 1023 unit cells

Since each bcc cubic unit cell contains 2 atoms, therefore, the total number of atoms in 208 g = 2 (atoms / unit cell) x 12.08 x io23 unit cells = 24.16 x io23 atoms

2. (c): Assume that we have 100 g of solution (one can start with any amount of solution because the results obtained will be the same). Solution will contain 20 g of ethylene glycol and 80 g of water. Molar mass of C2H602 = 12 x 2 + 1 x 6 + 16 x 2

= 62 g mol"1

20 g Moles of C2H602 =

62 g mol : 0.322 mol

Moles of water = — — - = 4.444 mol 18 gmol

_ moles of C 2 H 6 0 2 Xglyco1" moles of C 2 H 6 0 2 + moles of H 2 0

0.322 mol

Similarly, x.

0.322 mol + 4.444 mol 4.444 mol

= 0.068

: 0.932 0.322 mol + 4.444 mol

Mole fraction of water can also be calculated as 1 - 0.068 = 0.932

2 2 4 A A l 3. ( d ) : ; root word is cyclohex,

suffix is ane and prefix is l-cyclopropyl-4-f-butyl. Therefore IUPACnameis l-cyclopropyl-4-/-butylcyclohexane.

4. (c)

5. (a) The first emission line in the atomic spectrum of hydrogen in the Balmer series will appear at

^ cm 1 . This can be calculated as

For Balmer series nl-2, n2 = 3, 4, 5

v (for first line) = ^ Q r - ^ " )

-(B) 6. (c) • The energy required to remove the first electron

is called binding energy of the atom which is given as 24.6 eV. When the first electron is removed, the remaining atom is a hydrogen like atom. Binding energy of such a hydrogen like atom

= Z2x 13.6 eV = 2 2 x 13.6 eV = 54.4 eV. So energy required to remove both the electrons is 24.6 eV + 54.4 eV = 79 eV.

CH2CH3

7- (d ) : ( O )

Ethyl benzene

C6H5CH = CHCOOH Cinnamic acid

C6H5CHOHCH3 > C6H5CH — CH2 + c o 2

1-phenylethanol

8. (a) ; 238U is disintegrated to form the end product 206pb

Pb = 0 2 5 U 1

Initially the no. of a _ U-238atoms present 1 + 0.25

At present the ratio

(a - x) No. of U-238 atoms at present

CHEMISTRY TODAY | MARCH '08 211

1

For first order reaction(decay of U-238 obeys first order kinetics)

, = 2303 ! a a n d * = M 2 3 t a(a-x) TU2

0.693 = 2.303 , _ a _ 4 x l 0 9 t S a-x

1.25 . 2.303 ,A „ l n 9M 5 = LOO = < = O 6 9 3 ( 4 X l 0 ) 1 ° g 4

9. (b ) : In a saturated solution of NiS

KsiKNiS) = [Ni2+][S2-]

2 x IO 21 = [Ni2+] x 4 x 10"21

[Ni2+] = 2 x l r ; ' = 0.5M 4 x 1 0 I

Maximum possible concentration in solution of Ni2+

would be 0.5 M. This value is not given in question. The highest possible value of [Ni2+] = 0.45 M as 0.65 M can not be permissible as ionic product then would exceed solubility product of NiS.

10. (c): As2S3 dissolves in NaOH whereas CuS does not. Similarly Al(OH)3 dissolves in NaOH but Cr(OH)3

does not.

Both (a) and (b) can be separated by NaOH. As2S3 + 6NaOH Na3As03 + NaAsS3 + 3H 2 0

sod. arsenite sod. thioarsenite Al(OH)3 + NaOH H> Na[Al(OH)4]

or NaA102 + 2H 2 0

11. (b) Al is easily oxidised to aluminium oxide as it has high oxidation potential. This coating of A1203

resists further oxidation of Al.

Molarity = (M), molality = (m), density = (d in kg/lit), volume - (F in lit.), mass of solvent = (fFkg), wt. of solute = w kg, molar mass = ( M in kg)

M ~ W V (i) w .... m = Ww

W (Wt. of solvent) = Wt. of solution - Wt. of solute

W- (Vd— MMV) (iii)

From equations (ii) and (iii) w fn =

M (Vd-MM'V) _ w

m~ M'V(d-MM') (iv)

The value of M i s put into eq. (iv) from eq. (i) M

m d-MM'

13. (c) : Ax By.

(1 - <x)C

xAt + yBx~

xaC yaC

K.= (xaC)' (yaCy (1 - a)C

a « 1 (1 - a ) = 1

KeqC = xxxax+*y>'Cx +y

a

a =

x+y K eq

X1 yyCx+y~1

K„„ xx yyCx+y

I x+y

14. (b ) : In steam distillation the atmospheric pressure would be equal to the sum of the vapour pressures of toluene and water vapour.

Patm — Ptoluene Pll20(g) P < P 1 toluene 1 atm

15. (d ) : Gold number is defined as the amount of milligram of protective colloid added to 10 ml gold sol so that its coagulation is just prevented by adding 1 ml of 10% NaCl solution.

Gold number of starch = 0.025 x 103 mg = 25

16. (b ) : SnCl4 has more covalent character as the polarisation of CI ions would be large by Sn4+ ion as the cation has high charge. Evidently m.p. of SnCl4

is lowest.

17. (a) : 3° carbonium ion is most stable whereas CH3+ is

least stable (high energy).

= A^products -^AHreactants

. ' .AH° is largest and AH4° is least.

The order would be

AHx° > AH2° > A#3° > A#4°

18. (c) : A2 + B 2 ^ = ^ 2 A B (1 - a ) (1 - a ) 2a

K=V501 = 2

( 1 - a ) 2

or 2 a + V2a = V2 a =

V2 + 2 1 + V 2

19. (c). Since AG = -ve, the cell reaction is spontaneous i.e. oxidation occurs at right hand electrode At LHS (anode) Zn Zn2+(M,) + 2e At RHS (cathode) Zn2+(M2) + 2e^Zn

68 CHEMISTRY TODAY | MARCH '08

cell reaction Zn2+(M2) —> Zn2+(M t) 20. (c): When 100 ml 1 N HCl and 60 ml IN NaOH were

mixed the resulting normality of acidic solution.

I^ac id^ac id- I^base^base N' =

N'--

Final Volume

1 x 1 0 0 - 1 x 6 0 _ 40 1 6 0 ' 160

1

pH = - log^- = + log4 = 0.6020

21. (c) :

Each P atom is linked with 4 oxygen atoms.

= 4.8 x IO-10 e.s.u. x 1.275 x 10'8cm

= 6.12 x 10~18 e.s.u. cm = 6.12 D

% Ionic character

_ Observed dipole moment Dipole moment for complete ionic character

_ 1.03

xlOO

6.12 X100 = 16.83%

23.(d) : I3 has maximum number of unpaired electrons.

F.VFT":

A - -xe:; (a) (b)

F ,F

• S : -IH F

(c)

I .

24. (b)

26. (d ) : Initial pressure 0 Pressure at equilibrium 0

I L (d) 25. (a)

NH4HS, :NH3(g ) + H2Sfe)

0.5 0.5 +p

0 P

0.5 +p+p = 0.84 atm 2p = 0.84 - 0.5 = 0.34 orp = 0.17 atm ^(H2S) = 0.17 atm, /?(NH3) = 0.5 + 0.17 = 0.67 Kp = /?(NH3) x /7(H2S) = 0.67 x 0.17 = 0.11 atm

Ang = 2 — (1 + 3) = —2 Kp= 1.44 x 10"5; Kp = Kr.(RT)A"g

Kp 1.44 xlO"5

A . -or (RT)A"g (0.082 X773)"2

= 1.44 x 10"5/(0.082 x 773)-2

It is most basic. 29. (b) 30. (b) 31. (a)

Mass of halogen in 0.4 g of compound 80 188

xO.188 = 0.08 g

% of B r = ^ x l 0 0 =20% 0.4

(1)0, 33. (a) ( C H 3 ) 2 C = C ( C H 3 ) 2 - Z n > H 2 0 .

(CH3)2CO + (CH3)2CO

The octets of all atoms in structures of options (a) and (b) are complete. In structure of option (d) electron deficiency of positively charged carbon is duly compensated by lone pair of electrons of adjacent oxygen atoms. Such deficiency is not compensated in structure of option (c).

Peroxyacyl nitrate 37. (d) 38. (d)

OH OH OH

39. (d)

CH2OH OCH3

Conformer(d) is staggered form which is most stable because in staggered form the phenyl group are far apart as possible causing minimum repulsion. Structure (a) is skewed and (b) and (c) are eclipsed forms.

Weaker the acid stronger will be its conjugate base, hence higher will be the nucleophilicity. C H 3 C O O H > C 6 H 5 O H > H 2 0 > C H 3 O H

Decreasing acidic strength

So, C H 3 C O O < C 6 H S O - < O H - < C H 3 0 (conjugate base) Increasing basic strength or nucleophilicity.

Since it absorbs only one H2 molecule, so it must have one double bond. Further, the product of ozonolysis contains six-carbon atoms, hence it must be cyclohexene.

CHEMISTRY TODAY | MARCH '08 213

H ,

Reduction

cyclohexene cyclohexane

Z n O

O

^ HCCH2CH2CH2CH2CH

43. (c) :

HO COOH \ * * /

,CH—HC. / HOOC

.OH

OH

Contains chirai carbon

CH,

y C H - Br CH^CH^

CH3 - CH = CHCH2OH does not have chirai carbon atom.

44. (b) : In case of CH,C , lone pair electrons X N H 2

of both the N are readily available which results in higher density of electrons on the central C-atom, 2°amine is stronger than 1° amine. Whereas acetamide is least basic because its lone pair is in conjugation of double bond undergo in resonance. Hence the correct order

C H , C : >NH

"NH,

O

• (CH3)2 NH > CH3CH2NH2> CH3CNH2

(1) (3) (2) (4) 45. (d) : Urine of diabetic person contains glucose

which is reduced by Benedict's solution as glucose is reducing sugar.

46. (c): C6H1206 + 602 +6H20->6C02+12H20+ 38 ATP 47. (b) 48. (c): Terylene or dacron is a condensation polymer of

ethylene glycol and terephthalic acid. COOH

CH0 - CHN I I OH OH

+ n «H20

— O - C H 2 C H 2 - O COOH

C

O O 49. (a): Slowest step is the rate determining step. 50. (c) : In the reaction

NaH + H 2 0 h> NaOH + H2 + 1 - 1 + 1 - 2 +1 - 2 + 1 0

H atom undergoes oxidation only. Thus this is not a disproportionation reaction because in disproportionation reaction same substance undergoes oxidation and reduction simultaneously, which is shown in all other options. •

* Based on latest pattern of AIEEE/I IT and PMT's"' The book is represented in such a way that each student would be able to solve any problem of Engineering as well as Medical Entrance Exam* Book covers complete syllabus in the form of concise t ex t * Improve your problem solving skills with CHECK YOUR CONCEPT boxes • Presentation makes Organic Chemistry interesting and easy® MCQ's of 3 different levels - Concept based, standard, numerical based with detailed solutions* A number of objective and subjective problems to enhance problem solving approach.

Most Powerful Book for

A I E E E / N T and all other Engg. Entrance Exams

CBSE-PMT/AIIMS/BHU/AFMC and all other Medical Entrance Exams

W - t & M T G B O O K S 503, Taj Apt., Ring Road Near Safdarjung Hospital, New Delhi - 29 Tel.: 26194317, 26191601 e-mail: [email protected] website : www.mtg.in

66 CHEMISTRY TOO AY MARCH '08

Model Test Paper

West Bengal JEE 2008 The pattern and the syllabus for WB JEE 2008 has been changed. Under the new pattern 80% questions will be multiple choice questions of 1 mark each. 20% questions (short answer type) will carry 2 marks each.

Time: 1 hour INSTRUCTIONS Question paper consists of two sections:

Section A contains multiple choice questions. Each question carries 1 mark. Section B contains short answer type questions. Each question carries 2 marks.

Max. Marks: 50

S E C T I O N - A

1 In Haber process, 30 litres of dihydrogen and 30 litres of dinitrogen were taken for reaction which yields only 50% of the expected product. What will be the composition of the gaseous mixture under the aforesaid condition in the end? (a) 20 litres NH3, 25 litres N2, 20 litres H2

(b) 10 litres NH3, 25 litres N2, 15 litres H2

(c) 20 litres NH3, 10 litres N2, 30 litres H2

(d) 20 litres NH3, 25 litres N2, 15 litres H2.

2. 4 g of copper was dissolved in concentrated nitric acid. The copper nitrate on strong heating gave 5 g of its oxide. The equivalent weight of copper is (a) 23 (b) 32 (c) 12 (d) 20.

3. In hydrogen atom, energy of the first excited state is - 3.4 eV. Then find out the kinetic energy of the same orbit of H-atom. (a) + 3.4 eV (b) + 6.8 eV (c) -13.6 eV (d) +13.6 eV.

4 Mass of ^ Li is 7.016005 amu. Mass of proton is 1.007277 amu and mass of neutron is 1.008665 amu. Mass defect of lithium nucleus in amu is (a) 0.04048 amu (b) 0.04050 amu (c) 0.04052 amu (d) 0.04055 amu

5 Consider the following nuclear reactions 2 3 8 9 2 M-

The number of neutrons in the element L is (a) 142 (b) 144 (c) 140 (d) 146

6 The statement that is not true for the long form of the periodic table is (a) it reflects the sequence of filling the electrons in

the order of sub-energy levels s,p, d and f . (b) it helps to predict the stable valency states of the

7.

elements (c) it reflects trends in physical and chemcial

properties of the elements (d) it helps to predict the relative ionicity of the bond

between any two elements.

One mole of magnesium in the vapour state absorbed 1200 kJ of energy. If the first and second ionization enthalpies of magnesium are 750 and 1450 kJ mol-1

respectively, the final composition of the mixture is (a) 69% Mg+, 31% Mg2+ (b) 59% Mg+, 41% Mg2+

(c) 49% Mg+, 51% Mg2+

(d) 29% Mg+, 71% Mg2+.

8. The dipole moment of HBr is 0.78 x IO18 esu cm and inter atomic spacing is 1.41 A . The % ionic character

.of HBr is (a) 7-5 (b) 11-7 (c) 15 (d) 27.

9. In P043- , the formal charge on each oxygen atom and

the P - O bond order respectively are (a) -0 .75 ,0 .6 (b) - 0.75, 1.0 (c) -0 .75 , 1.25 (d) —3.1,1.25

10.Which of the following has maximum root mean square velocity at the same temperature? (a) S02 (b) C0 2

(c) 0 2 (d) H2.

11.At constant volume and temperature conditions, the rates of diffusion DA and DB of gases A and B having densities pA and p^ are related by the expression

PA. (a) Da =

(c)

D1 .PA

PB. /

DB PA.

PB V ® J

(b) DA = D„

(d) d a = Db

f \"2

P* PA

2.Which expression is correct for the work done in adiabatic reversible expansion of an ideal gas?

CHEMISTRY TODAY | MARCH '08 215

(a ) nRT I n V2/VL

(c) PAV

(b) C^T2-T,) 2

(d) I PdV

13. The following equilibria are given N, + 3H, ^ N, + O, ^F

2NH3, KX

2NO, K2

H2 + V4 0 2 H 2 0, K3

The equilibrium constant of the reaction 2 NH, + — O 2 v- 2NO + 3H,0

in terms of KX, K2 and K3 is (a ) KXK2K3 (b ) KXK2IK3

(c ) K,K3VK2 ( d ) K2K3VK}.

14. Vapour density of PC15 is 10416 but when heated at 230°C its vapour density is reduced to 62. The degree of dissociation of PC15 at this temperature will be (a) 6-8% (b) 68% (c) 46% (d) 64%

15. The decomposition of N 20 4 to N 0 2 is carried out at 280°C in chloroform. When equilibrium is reached 0.2 mol of N 2 0 4 and 2 x l O 3 mol o f N 0 2 are present in 2 litre solution. The equilibrium constant for the reaction.

N 2 0 4 2N0 2 is (a) 1 x 10"3 (b) 2 x 10 3

(c) 1 x IO"5 (d) 2 x IO 5

16. The velocity of a reaction is doubled for every 10°C rise in temperature. If the temperature is raised by 50°C the reaction velocity increases by about (a) 12 times (b) 16 times (c) 32 times (d) 50 times.

17. A G.M. counter is used to study the radioactive process. In the absence of radioactive substance A, it counts 3 disintegration per second (dps). Initially in the presence of A, it records 23 dps and after 10 minutes 13 dps, (i) What does it count after 20 minutes? (ii) What is half-life of A? (a) 8 dps, 10 min (b) 5 dps, 10 min (c) 5 dps, 20 min (d) 5 dps, 5 min.

18. The rate of reaction CljC.CHO + NO —> CHCI3 + NO + CO is given by equation, Rate = &[Cl3C.CHO] [NO]. If concentration is expressed in moles/litre, the units of k are (a) litre2 mole-2 sec 1 (b) mole litre-1 sec-1

(c) litre mole 1 s ec 1 (d) sec-1.

19. If three Faradays of electricity is passed through the solutions of AgN03 , CuS04 and AUC13, the molar ratio of the cations deposited at the cathodes will be (a) 1 : 1 : 1 (b) 1 : 2 : 3 (c) 3 : 2 : 1 (d) 6 : 3 : 2.

20. Arsenic sulphide is a negative sol. The reagent with least precipitating power is (a) AICI3 (b) NaCl (c) CaF2 (d) Glucose

21.2Hg H g ^ , E° = +0.855 V Hg Hg2+, E° = +0.799 V Equilibrium constant for the reaction Hg + Hg2+ -> Hg2

2+ at 27°C is (a) 89 (b) 82.3 (c) 79 (d) none of these.

22. The precipitate of CaF2 (Ksp = 1.7 x 10 10) is obtained when equal volumes of the following are mixed (a) 1 0 - M C a 2 + + l O ^ M F -(b) 10- 2 MCa 2 + + IO-3 M F " (c) 10"5 M Ca2+ + 10"3 M F" (d) 10~3 M Ca2+ + 10~5 M F~.

23. A certain buffer solution contains equal concentration o fX" and EX. The KA for HAT is 10~8. The pH of the buffer is (a) 5 (b) 8 (c) 6 (d) 14.

24. Consider the following reaction, 5 H 20 2 + xC\02 + 20H- xCl" + >>02 + 6H 2 0 The reaction is balanced if (a) x=5,y = 2 (b)x = 2,y=5 (c) x = 4,y = 10 (d) x= 5,y~ 5.

25. In the following reaction sequence, c H B r CuCN/pyridine ^ ^ dil. H , S Q 4 ?

6 5 523 K b o i i the producty is (a) benzonitrile (b) benzene (c) benzoic acid (d) benzamide.

26. Which one of the following amines gives carbylamine reaction? (a) CH3CH2NHCH3 (b) C6H5N(CH3)2

(c) (CH3)2CHNH2 (d) (CH3)3N.

27. Which of the following carbocations is expected to be most stable?

NO,

(a)

NO,

CHEMISTRY TODAY | MARCH '08 216

N O , NO.

(d) I

28. Equivalent mass of oxidising agent in the reaction, S0 2 + 2H2S 3S + 2H2G is (a) 32 (b) 64 (c) 16 (d) 8.

29. The acid which gives phosphine on heating is (a) H3PO4 (b) HPO3 (c) H4P207 (d) H3PO3

30. Which of the following molecules show pn-pn bonding? (a) P4 (b)As4

(c) Sb4 (d) N2.

31. The correct order of increasing boiling point is (a) H2S, H20, H2Te, H2Se (b) H20, H2S, H2Se, H2Te (c) H2Te, H2Se, H2S, H 2 0 (d) H2S, H2Se, H2Te, H20.

32. Which reaction is not feasible ? (a) 2KI + Br2 —> 2KBr +12

(b) 2KBr +12 —» 2KI + Br2

(c) 2KBr + Cl2 —> 2KC1 + Br2

(d) 2H20 + 2F2 -» 4HF + 0 2

33. The most stable carbocation is A-

(a) C w " ) (b) |l o (c) (d) 0

34. Tautomerism is not exhibited by

(a) °

(b) / 7— CH = NOH

O

(c)

O (d) O

O O o

35. l-butyne can be converted into 1-bromo-l-butene by reacting it with which of the following reagent? (a) HBr (b) HBr and (C6H5COO)2

(c) Br2 and H202 (d) Br2 and CC14.

36. What mass of isobutylene is obtained from 37 g of

CHEMISTRY TODAY | MARCH '08

tertiary butyl alcohol by heating with 20% H2S04 at 363 K if the yield of the reaction is 65%? (a) 12.8 g (b) 18.2 g

(c) 21.8 g (d) 28.1 g.

37. Which of the following will be most readily dehydrated in acidic conditions ?

0 OH OH

(a) O

(b) O

(c) Y x (d) O H O H

38. The IUPAC name of CH3COCH(CH3)2 is (a) 3-methyl-2-butanone (b) Isopropyl methyl ketone (c) 2-methyl-3-butanone (d) 4-methyl isopropyl ketone

39. Cyclohexene reacts with cold dilute alkaline KMn04

to yield (a) cis-1,2-cyclohexanediol (b) trans-1,2-cyclohexanediol (c) cyclohexanone (d) hexane-l,6-dial

40. To obtain iodine, sea weeds are dried, burnt and extracted with hot water. The mother liquor left after the crystallization of certain salts is treated with (a) chloride (b) conc. H2S04

(c) conc. NaOH (d) Mn02 + conc. H2S04

SECTION B

41. (a) A sample of U-238 (half-life = 4.5 109 Y) ore is found to contain 23.8 g of U-238 and 20.6 g of Pb-206. Calculate the age of the ore.

(b) OF2 is called as oxygen di-fluoride and not written as F20, where as C120 is called as chlorine oxide and not written as OCl2, explain.

(c) The dipole-moment of HF is 2 debye. The H F bond length is 0.92 . Calculate the % of ionic character in this compound.

42. (a) 0.296 gm of a bivalent metal was deposited by the passage of 0.5 A current for 30 minutes through a solution of metal sulphate. Calculate atomic mass of the metal.

(b) The half-life period of a first order reaction is 30 minutes. Calculate the specific reaction rate of the reaction. What fraction of the reactant remains after 70 minutes?

(c) Find equivalent mass of MnO^ - in the following disproportionation reactions:

3MnO^~ + 4H+ -> Mn0 2 + 2Mn04 + 2H 2 0

43. (a) Explain, why a glowing splinter burns vigorously in N 2 0 than in oxygen?

(b) Why concentrated H2S04 or P205 can't be used for drying NH3 gas?

(c) Explain why brown solution of FeCl3 gets decolourised when H2S gas is passed through it?

44. (a) Which hydracid of halogen is most reactive towards ethanol? (HCI, HBr, HI)

(b) A compound C3H80 is inert to sodium and when heated with excess HI yields a mixture of methyl iodide and ethyl iodide. What is the structure and name of the compound?

(c) Identify A, B, C, D in the following reactions:—

C2H5OH PCL

A AgN02

B Sn/HCI > c

D<~ HNO,

45. (a) An organic compound C4H80 forms hydrazone derivative and responds to iodoform reaction, but does not reduce Tollen's reagent. Write the structural formula of the compound,

(b) Identify A, B, C, and D in the following reaction.

CH 3 -COOH PC15> A H2 > b dil. NaOH. Pd - BaS04 (Sulphur)

> c

(c) Arrange, HCHO, (CH3)3C-CHO, C6H5-CHO in decreasing order of reactivity towards Cannizzaro's reaction.

SOLUTIONS

1. (b) ;N2(g) + 3H2(g) ^ 2NH 3 fe) 1 L of N2 reacts with 3 L of H2 to form 2 L of NH3. Thus N2 is the limiting reactant. 10 L N2 will react with 30 L H2 to form 20 L NH3. As actual yield is 50% of the expected value, NH3

formed = 10 L, N2 reacted = 5 L, H2 reacted = 15 L.

2. (b): Cu = 4 g, CuO = 5 g. Oxygen = 1 g. Thus 1 g oxygen combines with Cu = 4 g

8 g oxygen will combine with Cu = 4 x 8 = 32g

Equivalent weight of Cu = 32.

3. (a) Energy in excited state is nothing but kinetic energy.

4. (a) :No. of protons in jLi = 3 and neutrons = 7 — 3 = 4.

Mass of 3 protons and 4 neutrons = 3 x 1.007277 + 4 x 1.008665 amu = 7.056491 amu Mass defect = 7.056491 - 7.016005

= 0.040486 amu.

(b)™M-> ™N + 2? He 230 L + 2+?P

Number of neutrons in L = 230 - 86 = 144.

6. (b) : It does not help to predict the stable valency states of the elements.

7. (a) :Mgfe)+ IE, H> Mg+fe) + e";

IE, = 750 kJ mol"1 ...(0 Mg+

fe) + IE2 Mg2+fe) + <r;

IE2 = 1450 kJ mol"1 ...(ii)

8. (b): Dipole moment of HBr when 100% ionic = 4-8 x 10-1" x 1-41 x IO-8 cm = 6-678 x 10"18 esu cm

Actual dipole moment = 0-78 x 10-18 esu cm

% ionic character = 0-78x10' •18

xl00 = l l -68 6-678x10 18

9. (c): Formal charge on each O-atom = total charge = Z 3 = _ 0 7 5 and

number of atoms 4 P - O bond order

total no. of bonds total no. of resonating structures 4

- = — = 1.25

10 . (d ) : f / = 3RT M

. At constant T, U <

As H2has least molecular mass, U will be maximum for H2.

i.e„ DA = Df

( V'2

P i PA

12. (b) :Work done in adiabatic reversible expansion of an ideal gas = CV(T2 - J7,).

n / j i g [ N H 3 l 2 K _ [NO]2

• ~ 2 - rxi irn i ' [N2][H2]3 [N2][02]

K3 = [H 2Q]

[H2] [02]'

Required : K = [N0]2[H20]3

[NH 3] 2[O 2] 5 / 2 '

Thus K = K2Kl

K,

14. (h) • a = D ~ d = 104.16-62 : d 62

: 0.68 = 68%

CHEMISTRY TODAY | MARCH '08 218

15. (c) : K = [N0 2 ] 2 _ ( 2 x 1 0 _ 3 / 2 ) 2 1(T

• = 10' -5 [ N 2 0 4 ] (0 .2 / 2 ) IO"1

16. (c) Temperature range Change in velocity of reaction

0 - 1 0 ° 2 times 10°-20° 22 times 20° - 30° 23 times 30° - 40° 24 times 40° - 50° 2s times

Therefore for 50° rise in temperature velocity increases by 25 = 32 times.

17. (b) In the absence of A, it counts 3 dps (zero-error). Therefore at the start = 23 - 3 = 20 after 10 min = 13 - 3 = 10 dps after 20 min = 5 dps (recorded = 8 dps) (50% fall in 10 min, Tso = 10 min).

1S. < c A s r = At[C13CCHO] [NO] k = r / [ C l 3 C C H O ] [ N O ]

. mole litre 1 sec" mole2 litre-2

„ - l = mol""1 litre1 sec" - l

19. (d) Ag+ + e Ag, Cu2+ + 2e -> Cu, Au3+ + 3e Au.

3F will deposit 3 moles of Ag, 3/2 moles of Cu and 1 mole of Au.

Mole ratio = 3 : 3 / 2 : 1 = 6 : 3 : 2 .

Glucose does not give cations to precipitate or coagulate negative As2S3 sol.

E° for the given reaction = 0.855-0.799 = 0.056 V

cell = ^ E t

, „ 0.056x2 , 0 „ 0 „ or, \ogK = = 1.8983 0.059

Kc = antilog 1.8983 = 79.12 ~ 79.

22. (b) For CaF2, Ksp = [Ca2+][F~]2

Precipitate forms only when ionic product exceeds the solubility product which is possible when [Ca++] = IO2 M and [ F ] = 10 3 M in that case Kip = 10~2 x ( I0 3)2= 10 8

i.e Kip>Ksp

23. (b): For acidic buffer

•: [Salt] = [Acid] pH = pKa = - log Ka = - loglO"8 = 8.

The two half reactions are : -1 o

Oxidation : 2H2 O2 -> O2 ...(i)

+4 -1 Reduction : C102 Cl ...(ii) To balance O.N. in Eqn. (i) add 2e~ to R.H.S. we have

H202-H> 0 2 + 2e~ ...(iii) To balance charge add 20H to the L.H.S., we have

H 2 0 2 + 20H~ —> 0 2 + 2e~ ^

To balance H and O atoms add 2H20 to the R.H.S., we have H 2 0 2 + 2 0 H " 0 2 + 2 H 2 0 + 2e~ (v)

This is the oxidation half equation. To balance O.N. in Eqn. (ii) add 5 e to L.H.S., we have

C102 + 5 er Cl" ...(vi) To balance charge, add 4 OH" to the R.H.S., we have

C102 + 5 e" ^ Cl- + 4 OH" ...(vii) To balance O and H atoms, add 2HzO to L.H.S., we have

C102 + 2 H 2 0 + 5 e" Cl" + 40H" ...(viii) This is the balanced reduction half equation. To cancel the number of electrons lost in Eqn. (v) and gained in Eqn. (viii) multiply Eqn. (v) by 5 and Eqn. (viii) by 2 and add, we have

H202 + 2 OH"-> 0 2 + 2H20 + 2e-] x 5 ClOo + 2H2Q + 5e~ -» C P + 4QH~] x 2

5 H 2 0 2 + 2C102 + 2 0 H " 2 C F + 5 0 2 + 6H 2 0

Thus x, the coefficient of C102 is 2 while y, the coefficient of Cl" is 5.

25. ( 0 : C 6 H 5 B r » ^ C 6 H 5 ™ dil. ll2SO

523 K

[C 6H 5 (OH) 3 ]

boil

> C6H5COOH benzoic acid (unstable)

26. (c) Only 1° amines give carbylamine reaction (CH3)2CHNH2 + CHC13 + 3KOH(alc.)

(CH3)2CHNC + 3KC1 + 3H20. Isopropyl carbylamine

27. (d) : Has no resonance structure in which the +ve charge moves to a carbon to which the electron-withdrawing - N0 2 is attached.

2 8 . I n this reaction S02 oxidises H2S to S, therefore, S02 acts as the oxidising agent. During oxidation four electrons are lost, i.e.

S0 2 + 4e- S + 202" Thus, eq. wt. of S02 = mol. wt./4 = 64/4 = 16.

76 CHEMISTRY TODAY MARCH '08

29. (d) 4 H3PO3 - A + 3H3PO4 + PH3.

30. (d) : In N2 molecule, one o-bond and two pn-pn bonds are present.

31. (d) :As the size increases from S to Te, the van der Waal's forces of attraction interacting the molecules H2M increase and hence the boiling point increases in that order. However, water has the highest boiling point because of inter molecular H-bonding. Thus, H2S has the lowest boiling point.

32. (hi ;I2 cannot oxidise Br~ to Br2.

33. (b) : Carbocation (a) is antiaromatic and hence is least stable. Carbocation (b), (c) and (d) are all secondary but (b) and (c) are aromatic. Further since (c) is more strained than (b), therefore, (b) is the most stable carbocation.

34. (a) Essential condition for tautomerism is the presence of an a-hydrogen on a saturated carbon attached to C = O, NO, N02 , etc. or should have a H-atompresent on aheteroatom (O, N, S, etc.) attached to a double bond. Thus, option (a) does not have a a-hydrogen on a saturated carbon and hence does not exhibit tautomerism.

35. (b):

CH,CH2C = CH + HBr (C6H5

COQ)2 CH3CH2CH=CHBr

36. (b) :

( C H 3 ) 3 C O H 2 0 % H 2 S 0 4 > ( C H 3 ) 2 C = C H 2 + H 2 O 3/3 74! 56 g

74 g of /-BuOH give isobutylene = 56 g 37 g of ?-BuOH will give isobutylene

= — x37 = 28 g 74 B

But the yield of the reaction = 65% Wt. of isobutylene actually formed _ 28x65 _ | g 2 g

100 37. (a) : Aldols, i.e., P-hydroxyaldehydes or

P-hydroxyketones readily undergo dehydration to form a, p-unsaturated aldehydes or ketones

O OH „ O OH2

P

i-Hydroxyketone

-IT, -H,0 O

H H

/ a , (3 -Unsatura ted k e t o n e

38. (a)

39. (a) :cw-l,2-cyclohexanediol.

40. (d) :2 Nal + Mn02 + 3 H2S04

2 NaHS04 + MnS04 + 2 H ,0 + I,.

41. (a): Disintegration occurs as : 9 2 U 2 3 8

82Pb206 + 8 2He4 + 6 Atoms of Pb present

20 6 ' • = 0.1 g atom = U decayed 206

23.8 Atoms of U present = - ().l g atm 238

JV= 0.1 g atom jV0 = U present + U decayed

= 0.1 +0.1 =0.2 g atom , _ 2 .303x4.5xl0 9 , 0 2

0.693 g l 0 0 . 1 t = 4.5 x 109 years.

(b) While writing the formula of a compound less electronegative element is written first, followed by more electronegative element and accordingly the name is written. As electronegativity values of CI, O, and F are 3, 3.5 and 4 respectively. So the compound of oxygen and fluorine is written as OF2, while compound of oxygen and chlorine is written as ci2o. (c) The theoretical dipole moment of HF

= 0.92 x 10~10 m x 1.6 x 10 I9C = 1.47 x IO"29 Coulomb, metre

1.47 xl0~2 9

t -30 debye = 4.40 debye

3.335x10" Observed dipole moment = 2 debye (given) Now % of ionic character in HF

observed dipole moment = : — , , X 1 0 0

theoretical dipole moment

= -£-x l00 = 45.45 4.4

42. (a! From Faraday's first law,

96500 ^ E _ IFx96500 _ 0.296x96500 _ 3 1

Ixt ~ 0.5x30x60 Hence equivalent mass of metal = 31.74 As valency of the metal = 2 So atomic mass of the metal = 31.74 x 2 = 63.48 (b) For a first order reaction

, [ = M 9 3 2 = M 9 3 2 = 0 . 0 2 3 l m i n - i ' 1 /2 30

Suppose the reaction is

CHEMISTRY TODAY | MARCH '08 77

A a

-> product 0 f = 0

t = 70 ( a - x ) x Fraction of the reactant remains unreacted

J a - x ) a

, T , 2.303, a Now, k, = log 1 t a-x

or 0.0231 = log 70 a-x

0-0231x70 = Q 7 0 2 1

a-x 2.303

or a-x

a-x

•• antilog 0.7021 = 5.036

1 _ a 5.036

0.1985 = 0.2

(c) 3 M n O f + 4H Mn0 2 + 2 M n 0 4 + 2H 2 0 In this disproportionation reaction the two half equations are: M n O ^ _ > MnC>4

(change in oxidation number = 1 ) . , „ , „ -.2- Formula weight

Equivalent mass of M n 0 4 =

Mn0 4 ~ Mn0 2 (change in oxidation number = 2)

Equivalent mass of MnO; 2- _ Formula weight

Now equivalent mass of M n 0 4 in the overall reaction undergoing disproportionation

FW FW 3 3

1 2 2 2

43. (a) • The temperature of a glowing splinter is enough to decompose nitrous oxide giving nitrogen and nascent oxygen. The nascent oxygen is more reactive than molecular oxygen, so a glowing splinter burns vigorously in the atmosphere of nitrous oxide. On the other hand molecular oxygen gives off nascent oxygen somewhat at higher temperature than N 2 0. (b) Ammonia is basic in nature. So it reacts with both concentrated H2S04 and P205 . Therefore these two substances can't be used for drying of NH3 gas.

2NH3 + H2S04 -> (NH4)2S04

6NH3 + P 20 5 + 3H 2 0 2(NH4)3P04

(c) H2S reduces ferric chloride to ferrous chloride. Therefore when H2S gas is passed through FeCl3

solution, its brown colour is discharged due to formation of ferrous chloride.

2FeCl, + H,S - > 2FeCl, + 2HC1 + Si

44. (a) : HI is most reactive towards ethanol even in presence of red phosphorus and alcohol is reduced to alkane. (b) The formula C3H80 represents, either the compound is an alcohol or ether, as it is saturated and contains one oxygen atom. But the compound is inert to sodium, so it is an ether, because only alcohol reacts with sodium metal. Again the ether when heated with HI yields a mixture of methyl iodide and ethyl iodide, so it is ethyl-methyl ether. C2H5 - O - CH3 + 2HI — (ethyl methyl ether) C2H5 - 1 + CH3 - I + H 2 0 (c) A = C2H5 - Cl; B = C2H5 - N0 2 ;

C = C 2 H 5 - N H 2

D = C2H5 - OH (ethyl alcohol)

45. ( a ) : C4HgO is unsaturated and contains one oxygen atom. Further it forms hydrazone derivative, so it may be aldehyde or ketone. As it responds to iodoform reaction but does not reduce Tollen's reagent, so it is 2-ketone. Hence the structural formula is :

CHc, - C - CH, - CH, (Butan-2-one) II O

(b) A = CH3COCl; B = CH3-CHO ; OH

C = CH3 - CH - CH2 - CHO D = CH3-CH = CH-CHO

(c) The decreasing order of reactivity towards Cannizzaro's reaction i s :

H - CHO > (CH3)3C - CHO > C6H5 - CHO • •

wtt&

BITSAT EXPLORER

Available at leading book-shops

throughout India.

Send D.D/M.O in favour of MTG Books.

Add Rs. 35 for postage.

• I I

EXPLORE

m m - M T G B O O K S 503, Taj Apt., Ring Road Near Safdarjung Hospital New Delhi -110 029 Tel.: 26191601,

66 CHEMISTRY TOO AY MARCH '08